FInal Year Surgery MCQs

Download as pdf or txt
Download as pdf or txt
You are on page 1of 483

Surgery MCQs

Surgery HBS MCQs

Surgery A
General surgery
1. All are true about keloids except:
• is a benign condition
• Is an extensive overgrowth of scar tissue
• Is the same as a hypertrophic scar*
• May occur on the ear lobes:
• Common is Africans
2. A surgical unit presented monthly report at the end of month. It was showing 35 clean cases
operated, 50 clean contaminated surgeries done and 5 dirty cases operated over in last
month. Which of the following statements about types of wounds are true?
• A 'clean-contaminated wound has an infection rate almost nil
• The infection rate in a 'clean wound' is less*
• There is not role of prophylactic antibiotics in clean surgery.
• Abscess surgery is classified in dirty wounds
• The wound after a biliary surgery is classified as contaminated!
3. Which of the following statements regarding lymphedema are true?
• Early treatment includes surgical drainage.
• Primary lymphedema occurs in more than 5 per cent or the population.
• Lymphedema often involves the muscle compartments.
• Fluid is relatively low in protein in lymphedema.
• Early treatment is usually successful*
4. A 27-year-old man is involved in a car crash while traveling in excess of 70 miles/hr. He sustains
an intra-abdominal injury and a fracture of the femur. The BP is 60/40 mm Hg, and the
hematocrit is 16%. There is likely to be a proportionately greater increase in blood flow to
which of the following?
• Heart*
• Kidneys
• Thyroid gland
• Skin
• Liver
5. A 34-year-old male has serum sodium of 114mEq/L. What amount of serum sodium is required
for correction of hyponatremia as daily requirement?
• 5mEq/kg/day
• 1mEq/kg/day*
• 10mEq/kg/day
• 7mEq/kg/day
• 3mEq/kg/day
6. A young male after road traffic accident has pulse 130/min, B.P 70 systolic with suspected blunt
trauma abdomen. There are cold peripheries on examination with no cyanosis? Which of the
following regarding blood pressure in shock are false?
• Hypotension is one of the first signs of shock.
• Elderly patients who are normally hypertensive may present with a 'normal' blood pressure.
• Blood pressure is decreased by reduction in stroke volume and peripheral vasoconstriction*
• Beta-blockers may prevent a tachycardiac response.
• Children and fit young adults are able to maintain blood pressure until the final stages of
shock.
7. A poly trauma case lands in emergency with pulse 120/min, B.P>P 90/60mmHg, bleeding from
leg wound. Initial resuscitation is best done by administration of which of the following?
• Ringer's lactate solution*
• D5 W
• 5% hydroxyethyl starch solution
• 5% plasma protein solution
• D5 W and 0.45% normal saline
8. A tunnel connecting two epithelial surfaces is:
• Duct
• Sinus tract
• Ulcer
• All of the above
• Fistula*
9. A 40 year diabetic developed an ulcerative lesion on forefoot dorsum, the lesion has necrotic
base with scanty pale granulation and blackish edge. The foot is not swollen with non-obvious
discharge. Which of the following best suits the description?
• Neuropathic ulcer
• Infective ulcer
• Malignant ulcer
• Traumatic ulcer
• Ischemic ulcer*
10. A boy 14 year age burnt in open space with accidental fire from his mobile battery catching
fire. He had both his arms burnt with blisters formed. His head, face and neck were also burnt.
What is the percent of body burnt in this patient?
• 27%*
• 9%
• 10%
• 18%
• 5%
11. Debridement means:
• Amputation
• Excising mm of skin from the edge of a wound
• Suturing of a wound
• Laying open all layers of a wound and excision of devitalized tissue*
• Not excising skin but excising all damaged muscle
12. A 40-year-old man who weighs 65 kg is being observed in the ICU. Twenty-four hours
postoperatively, he develops convulsions. His serum sodium is 118mEq/L. Appropriate
management includes which of the following?
• Emergency hemodialysis
• Administration of normal saline (0.9%)
• Administration of Lasix, 40 mg intravenously (IV)
• Administration of vasopressin
• Administration of hypertonic saline (3%)*
13. Which of the following is not a risk factor for lymphedema?
• Family history
• Varicose vein operation
• Air travel
• A Baker's cyst*
• Obesity
14. A burn patient had closed space burns. At the pre-hospital area of burn what should be
checked in patient by a rescuer?
• Don't open mouth of patient
• Put water on patient to stop burning*
• Cover patient with plastic sheet
• Check other associated injuries
• Don't remove clothes of patient
15. Disaster cause mass casualties which are of varied medical emergencies raising the need of
triage. Which of the following statement is true? triage,
• is carried at the same time as basic life support given*
• means treating all injured
• Triage implies selection of surgical procedure.
• patient's triage category cannot be reviewed.
• is carried out at the spot of disaster
16. A polytrauma case received in emergency with shock having a pulse 130/min, B.P 70 systolic,
cold peripheries is suspected to have intra-abdominal bleeding. On exploration there was
shattered spleen and 2.5 L hemoperitoneum. He received 3 Red Cell Concentrates and 6 fresh
frozen plasma. What does Fresh frozen plasma contain?
• Factor XII
• Coagulation factors*
• Both A & B
• Platelets
• Factor VIII & Fibrinogen
17. A 55 year with a recent history of myocardial infarction attends emergency with c/o left leg
pain with numbness, on examination the leg is cool, pale with paresthesia. Which of the
following is your most likely clinical diagnosis?
• Acute arterial ischemia*
• Acute lymphatic obstruction
• Sciatica left leg
• Compartment syndrome
• Deep vein thrombosis
18. Debridement is invariably needed in contaminated wounds, as the patient involved in high
impact road traffic accident sustaining lacerated and crushed injuries. Which of the following
statements regarding debridement is not true?
• All doubtful tissue should be removed.
• The wound thoroughly washed with antiseptic solution
• It involves definitive repair treatment*
• It frequently involves leaving the wound open.
• It is better done through a long incision.
19. A line joining the highest point of iliac crests passes through
• Lumbar vertebra 4*
• Lumbar vertebra 1
• Cervical vertebra 7
• Thoracic vertebra 12
• Lumbar vertebra 2
20. A young male received in a field hospital after a major earth quake had head injury with
extradural hematoma. He had GCS 8/15. Right femur open fracture with bleeding from
wound. Right sided pneumothorax. He was having intact airway. His pulse was 140/min and
B.P was 80 systolic. Regarding damage control surgery in a disaster management which things
can be performed in this patient in the field hospital?
• Tracheostomy
• Fixation of Right Femur
• Exploratory laparotomy
• Stop bleeding, stabilize fracture, resuscitate and thoracostomy, craniotomy*
• Right Thoracotomy
21. A 20 year old boy received multiple injuries in road accident. His pulse is 140/min & BP is 70
Systolic. He has active bleeding from open fracture femur. What is immediate step to prevent
further shock in emergency?
• Stop Hemorrhage*
• IV line with Crystalloids solutions
• Surgery
• Blood Transfusion
• Factor VIII & Fibrinogen
22. A 45 year old lady has burnt 25% of her body and weighs 55Kg. She has been brought to
emergency. Resuscitation is started with crystalloids. What is fluid volume given over 1st
24hrs in this patient?
• 3500ml
• 1500ml
• 500ml
• 2000ml
• 5500ml*
23. Most common mixture of gases for GA is:
• 02/N20 and Isoflurane*
• 02/Nitrogen and halothane
• Sevoflurane/02
• CO2/N20 and Isoflurane
• 02/CO2 and sevoflurane
24. A 35 year old lady was operated for a cystic lesion on sternum about 3 months ago. She later
had itching and swelling at the scar. On examination a 4 cm long and 3 cm wide hypertrophic
keloid is formed in this patient. Which of the following are useful in the treatment of keloid in
this patient?
• Excision and steroid injection*
• Tight garments
• Vitamin D preparations
• Chemicals
• Excision of scar only
25. A 30-year-old man is brought to the emergency department following a high-speed car
accident. He was the driver, and the windshield of the car was broken. On examination, he is
alert, awake, oriented, and in no respiratory distress. He is unable to move any of his four
extremities; however, his extremities are warm and pink. His vital signs on admission are HR
54 beats per minute and BP 70/40 mm Hg. What is the diagnosis?
• Neurogenic shock*
• Hemorrhagic shock
• Irreversible shock
• Septic shock
• Cardiogenic shock
26. A 35 year lady had notes performed on her for right ovarian cyst. Post operatively a house
surgeon didn't see any abdominal scar. Which of the following can be used as the entry point
in natural orifice transluminal surgery (NOTES)?
• Umbilicus
• Intestine
• Intraperitoneal & Retroperitoneum
• Inguinal deep ring
• Mouth, Vagina*
27. A patient of ischemic heart disease is taking aspirin. He has to undergo open cholecystectomy.
How many days before surgery the aspirin should be stopped?
• 5 days
• 1 day
• 14 days
• 7 days*
• 3 days
28. 30 year old lady presented with gall stones in OPD. Her Laparoscopic cholecystectomy was
performed. Which of the following are advantages of minimal access surgery?
• Good tactile feedback
• Decrease in wound size*
• Use of Methane gas for insufflation
• Less cost of procedure
• Increased postoperative pain
29. You are appointed in the surgical department of a field hospital, after disaster, a victim is
triaged for immediate surgery, the surgeon should best choose which of the following,
• Perform damage control surgery*
• Continue resuscitation in ER
• Transfer to specialist care center
• Perform definitive surgery
• Continue resuscitation in OT
30. A poly trauma patient received 5 units of whole blood during resuscitation and in operation
theatre while laparotomy was performed on him, which of the following is a complication of
massive blood transfusions?
• Hyperkalaemia
• Coagulopathy*
• Hypophosphotemia
• Hyponatremia
• Hypercalcaemia
31. Focused abdominal sonogram is done to primarily rule out which of the injuries:
• Abdominal Aorta*
• Cardiac tamponade
• Intestine
• Urethra
• Hepatic Injury
32. A 70-year-old man, who weighs 70 kg, is admitted with acute cholecystitis. His calculated daily
fluid requirement for maintenance is approximately which of the following?
• 2.5 L*
• 1L
• 4L
• 3L
• 2L
33. A 55-year-old man sustains numerous injuries involving the abdomen and lower extremities.
During the Intra- and postoperative periods, he is resuscitated with 10 L of Ringer's lactate
and 2 Units of packed red blood cells (RBC). After initial Improvement, he has severe dyspnea
on the second postoperative day. The most useful initial diagnostic test is which of the
following?
• Electrocardiogram
• Analysis of arterial blood gas
• Insertion of a central venous line
• Ventilation-perfusion scan*
• Computed tomography (CT) scan of abdomen
34. Following vein joins long saphenous vein before entering to femoral vein
• Deep pudendal vein
• Superficial epigastric vein
• Deep epigastric vein*
• Superficial pudendal vein
• Superficial circumfacial Iliac vein
35. Patient received in ER, road traffic accident, on examination he received injury over right side
of chest, has an open pneumothorax. How will you manage the case?
• Chest intubation
• Pack the wound and apply sticking on 3 sides*
• Primary closure of wound
• Secondary wound closure
• Pack the wound and apply sticking on 2 sides
36. Patient presented with a history of stab Injury to the left side of chest. Upon examination the
patient had blood pressure of 92/54 mmHg, a pulse of 56 beats/min, muffled heart sounds
and respiratory rate of 33 breaths/min. There is percussion note dull on the left side of chest
and a GCS of 11/15. The most likely underlying cause is:
• Tension pneumothorax
• Massive haemothorax
• Cardiac tamponade*
• Perforation of the ileum
• Splenic rupture
37. A young man who was involved in a road traffic accident a week ago had fracture shaft of right
femur within 24 hours he developed shortness of breath, tachycardia with deranged ABG's
and had changes in his ECG. At the time of arrival at emergency he was fully conscious and
oriented without any evidence of bleeding from nose and ears. The most likely cause of his
symptoms can be?
• Associated head injury.
• Compartment syndrome right thigh.
• Fat embolism*
• Hemothorax/Pneumothorax
• Extradural haematoma
38. Inferior thyroid artery is a branch arises from
• Internal carotid artery.
• External carotid artery
• Lingual artery
• Thyrocervical trunk*
• Facial artery
39. A 40 year old male underwent pelvic fracture due to road traffic accident, posterior urethral
Injury was suspected and confirmed on urethrogram, regarding the treatment deferred for 03
month because,
• For patient to void normally
• For urethral ends get closed
• For edema and hematoma get resolved*
• Patient wants to continue medical treatment
• Needs Foleys catheterization
40. Grade 1 Renal Trauma,
• Laceration >1.0 cm Renal cortex only. No urinary extravasation, Intact collecting system
• Main renal artery/vein Injury with contained hemorrhage
• Avulsion of renal hilum (pedicle) which devascularises kidney
• Hematoma, subcapsular, Non expanding, Parenchyma normal*
• Injury to hilum of kidney.
41. A 35 years old lady presented with swelling and discharge from scalp of two years duration.
She was diagnosed having Pott's Puffy Tumor of scalp, which is:
• Infected sebaceous cyst of scalp
• Subperiosteal abscess with osteomyelitis of skull*
• TB brain
• Sebaceous horn of the scalp
• Periosteal abscess with osteomyelitis
42. A 35 years old lady sustained burns to right arm front of chest and abdomen. Resuscitation
with ringer lactate was started, what is the best indicator for volume replacement?
• Normal blood pressure
• Low blood creatinine level
• Normal pulse rate
• Mental alertness
• Urinary output*
43. A 60 years old lady sustained burns to both arm front of chest and abdomen. What is the
burned surface area in this patient?
• 36%*
• 40%
• 50%
• 55%
• 60%
44. A 33 years old lady sustained burns to nostrils and upper airway passages in a close room.
What is the best Initial treatment in this patient?
• Ringer lactate infusion
• Normal saline Infusion
• Endotracheal intubation*
• Wound debridement
• Skin grafting
45. A 60 years old lady 70 kg of weight sustained 25 % burns. What will be his initial fluid
requirement for Initial 24 hours?
• 4000ml
• 5000ml
• 6000mml
• 7000ml*
• 8000 ml
46. A 50 years old male presented with multiple discharging sinuses in the axilla for the last 6
months. She was diagnosed to be having Hidradenitis suppurativa which is an infection of:
• Apocrine glands*
• Adenoids
• Hair follicles
• Middle ear
• Tonsils
47. A patient was diagnosed having Onychogryphosis which is:
• An absent nail
• A broken nail
• An infected nail
• An ingrowing nail
• An overgrown curved nail*
48. A 50 years old farmer presented with ulcerative lesion over the face for last 1 year. Incisional
biopsy confirmed it to be Rodent ulcer which is
• Aphthous ulcer
• Basal cell carcinoma*
• Melanoma
• Peptic ulcer
• Squamous cell carcinoma
49. A 20 years old patient had appendectomy 3 days back. He is complaining of pain swelling and
discharge from the wound. His temperature is 38 C What is best treatment option in this
patient
• Antibiotics
• Analgesics
• Proton pump inhibitor
• Open the wound stitches*
• Wound dressing
50. An 80-year-old man who is 6 days following open anterior resection for rectal carcinoma. The
patient reported seeing a gush of fluid from the laparotomy wound. You notice that the small
bowel is eviscerating from the wound. Following initial resuscitation, the next best step is to:
• Cover the small bowel with sterile saline-soaked gauze pieces*
• Secondary suture application after 3 days
• Administer intravenous cefuroxime 1.5 g
• Return the patient to theatre for deep-tension abdominal wall closure
• Apply vacuum-assisted closure therapy
51. Flow phase of metabolic response to injury is characterized by all expect:
• Tissue edema
• Increased basal metabolic rate
• Increased cardiac output,
• Decreased body temperature*
• Leukocytosis
52. A patient is recovering from major burns and is in catabolic stage. Loss of one gram of nitrogen
is contained in how much of grams of protein?
• 5.25 grams of protein
• 6.25 grams of protein*
• 7.25 grams of protein
• 8.25 grams of protein
• 9.25 grams of protein
53. In a 70 years old male patient with multiple long bones fracture Avoidable factors that
compound the response to injury are all except
• Continuing hemorrhage
• Hyperthermia*
• Tissue edema
• Tissue under perfusion
• Immobility
54. A patient sustained extensive laceration of right forearm which is contaminated and there is
lot of devitalized tissue in the wound. Best treatment option in this patient is
• Primary suturing alone
• Debridement with leave wound open*
• Debridement and suturing with prolene
• Dressing and elevation
• Amputation of the limb and elevation
55. A 25 years old male presents in emergency with H/O RTA, resulting in chest trauma. He Is
dyspneic & experiencing severe chest pain. His neck veins are engorged, percussion note is
hyper-resonant on left side along with absent breath sounds on the same side. Most likely
diagnosis is?
• Massive haemothorax
• Flail chest
• Tension pneumothorax*
• Ruptured diaphragm
• Simple ribs fracture
56. A flail segment of the chest occurs when;
• 2 or more ribs are broken at more than one site
• 3 or more ribs are broken at one site
• 3 or more ribs are broken at more than 2 sites*
• There is penetrating chest injury
• An emphysematous bulla of a lung, ruptures
57. Regarding FAST, following 4 windows(regions) of the abdomen are scanned for blood(free
fluid) EXCEPT;
• Pelvis
• Lt Lenarenal space
• Rt hepatorenal space
• Pericardium
• Paracolic gutters*
58. 19 years old male who was involved in a motor vehicle accident presented to ER he had
suffered poly trauma with long bone fractures, his BR is 70/30 mmHg with pulse of 122 bpm.
Immediate management includes?
• Send blood for cross-match and then transfuse type specific and crosshatched blood
• Start immediate fluid resuscitation with ringer lactate send cross-match and call for O-ive
blood from bank*
• Start dextrose saline and call for O+ive blood from bank
• Start O+ive blood transfusion
• Do not give any fluids and wait for blood
59. Which of the following are not potentially life-threatening injuries?
• An Aortic injury
• Tracheobronchial injuries
• Myocardial contusion
• Rupture of the diaphragm*
• Pulmonary contusions
60. Which of the following is considered positive in diagnostic peritoneal lavage;
• >500,00 red blood cells/ml
• >100 white blood cells/ml
• Visible food particles in the aspirate*
• A negative Gram stain test
• Straw colored aspirate
61. A man comes to emergency with stab Injury to the left flank. He has stable vitals. What would
be the next step in management?
• CECT*
• Diagnostic peritoneal lavage
• Laparotomy
• Laparoscopy
• FAST Scan
62. Which of the following statements regarding blunt injury are false
• The mechanisms are direct or indirect
• In indirect injury, associated injuries may be present and should be sought.
• Overt Injury should lead the clinician to look for a covert injury as well.
• Proper exposure is essential so as not to miss other Injuries.
• In chest injuries damage to abdominal organs is rare*
63. A 55-year-old woman gives a history of tiredness, aching, and a feeling of heaviness in the left
lower leg for the past 3 months. These symptoms are relieved by leg elevation. She is also
awakened frequently by calf and foot cramping, which is relieved by leg elevation, walking, or
massage. On physical examination there are superficial varicosities, non-pitting edema, and a
slightly painful, 2 cm. diameter superficial ulcer 5 cm. above and behind the left medial
malleolus. What is the most appropriate diagnosis?
• Isolated symptomatic varicose veins.
• Superficial lymphatic obstruction.
• Deep venous insufficiency*
• Arterial insufficiency.
• Incompetent perforating veins.
64. In a 55-year-old grocery store cashier with an 8-month history of leg edema increasing over
the course of a workday, associated with moderate to severe lower leg bursting pain, the
most appropriate investigative study or studies are:
• Doppler duplex ultrasound.*
• Brodie-Trendelenburg test.
• Ascending and descending phlebography.
• Measurement of ambulatory and resting foot venous pressure.
• Venous reflux plethysmography.
65. A 70-year-old woman develops a cold, painful, right leg. No pulses are palpable throughout
the right leg. She has several risk factors for atherosclerotic disease. How will a medical
student differentiate between acute and chronic ischemia?
• History of 40 pack-years of smoking
• History of intermittent claudication*
• History of thrombotic stroke
• Presence of femoro-popliteal bypass scar in the left leg
• Presence of foot pulses in the left leg
66. A 55-year-old man who smokes 40 cigarettes per day presents with a history of pain in his left
calf after walking 100 meters. The pain subsides with rest. He has no symptoms in his thigh or
buttock. He has a good volume femoral pulse but no popliteal or pedal pulses are palpable.
Neurological examination is normal. He undergoes duplex Doppler ultrasound. Which is the
single most likely finding of this investigation?
• Left iliac artery occlusion
• Left internal iliac artery stenosis
• Left posterior tibial artery occlusion
• Left profunda femoris artery stenosis
• Left superficial femoral artery occlusion*
67. Which of the following describes chronic leg ulcers?
• Cause of ulcers cannot be determined by their locations on the leg
• Venous ulcers are seldom located on the foot
• Arterial ulcers are seldom located on the leg*
• Leg ulcers affect diabetic patients less often than other patient groups
• Ulcer healing is Improved more significantly with balloon angioplasty than with arterial
bypass
68. A 72-year-old man underwent repair of an endovascular abdominal aneurysm 1 year back
collapses and complains of back and abdominal pain. His blood pressure is 90/40 mm Hg. The
patient denies a history of peptic ulcer or alcohol abuse. What is the most likely diagnosis?
• Aortoenteric fistula
• Bleeding duodenal ulcer
• Ruptured abdominal aortic*
• Acute Pancreatitis aneurysm.
• Diverticulitis of transverse coloni
69. 40 year man suffered from long standing peptic ulceration, started to have postprandial upper
abdominal distension and repeated projectile non- billious vomiting, O/E he is hypovolemic
with disturbance of electrolytes and acid base balance, the lab most likely reports as:
• Hypernatremia hyperkalemic alkalosis
• Hypernatremic hyperkalemic acidosis
• Hypornatremic hyperkalemic acidosis
• Hypornatremia hypokalemic alkalosis*
• Hypernatremia hypokalemic alkalosis
70. A 30-year-old man who weighs 60 kg has the following laboratory values: hemoglobin is 10
g/dl; serum sodium is 120mEq/L; serum potassium is 4mEq/L; serum chloride is 90mEq/L; and
serum CO2 content is 30mEq/L What is his sodium deficit approximately?
• 20mEq*
• 200m Eq
• 400mEq
• 720mEq
• 120mEq
71. A man fell from roof of single storey building. He was received in ER with severe shock. His
radial pulse was not palpable and he could not move his limbs. Patient was resuscitated with
IV fluids but not responding. What can be the cause of neurogenic shock?
• Septicemia and endotoxins
• Spinal nerve overflow
• Activation of cell and humoral immunity
• Parasympathetic outflow failure
• Sympathetic outflow failure*
72. Refeeding syndrome results in deficiency of which nutrients to deteriorate the patient
condition?
• Calcium, Vitamin D, Potassium
• Phosphate, Calcium, Vitamin D
• Magnesium, Calcium, Phosphate*
• Vitamin C, Sodium, Calcium
• Phosphate, Sodium, Magnesium
73. A baby 10kg body weight has vomiting for 1 day. Dehydration is obvious on examination. How
much fluid/hour are recommended in his case/ 24 hour?
• 20ml/kg/day
• 40ml/kg/day*
• 100ml/kg/day*
• 200ml/kg/day/250ml/kg/day
• 50ml/kg/day
74. A 20 year old boy hit by a car on road landed in emergency with severe pain right
hypochondrium. Pulse is 120/min. B.P is 80 systolic Abdomen has generalized tenderness with
guarding in right hypochondrium. What is the important investigation, you will order now in
ER?
• Blood CP
• FAST scan
• CT Abdomen with contrast*
• Barium meal
75. In liver trauma, what is grade II Injury?
• Subcapsular hematoma > 50% surface
• Hilar avulsion
• Laceration < 1cm deep
• Subcapsular hematoma 10-50% surface area*
76. In management of blunt Trauma liver following is true:
• CT scan is advised in unstable patient
• Conservative management advised in haemodynamically stable patients*
• For resuscitation ideal fluid is colloid infusion
• clotting profile is not necessary
77. 40 year man suffered from long standing peptic ulceration, started to have postprandial upper
abdominal distension and repeated projectile non- billious vomiting, O/E he is hypovolemic
with disturbance of electrolytes and acid base balance, the lab most likely reports as.
• Hypernatremia hyperkalemic alkalosis
• Hypernatremia hyperkalemic acidosis
• Hyponatremic hyperkalemic acidosis
• Hyponatremic hypokalemic alkalosis*
• Hypernatremia hypokalemic alkalosis
78. A young man sustained head Injury during a road traffic accident. His GCS Is 5/15. He needs
artificial ventilation. Which one Is definitive and secured method of airway management?
• Guedel's alrway
• Laryngeal mask airway
• Endotracheal Intubation
• Tracheostomy*
• Nasal airway
79. A 40 year female patient had sub-total thyroidectomy for adenomatous goiter, in this patient
physiological response to operative trauma In first 24 hour Includes,
• Retention of potassium
• Retention of sodium
• Retention of Calcium*
• Retention of magnesium
• Retention of bircarbonates
80. A 35 year female developed a painful neck swelling. She gives history of mild fever, received
multiple courses of antibiotics with no relief. On examination there is multinodular mildly
tender mass in carotid triangle, mostly firm with few soft areas. Which of the following is
most likely diagnosis? (HBS)
• Carotid body tumor
• Carotid artery aneurysm
• Metastatic lymphadenopathy
• Tuberculous lymphadenopathy*
• Branchial cyst
81. A 30 year old school teacher presented with history of dilated tortuous vein over medial
aspect of left leg. His venous Doppler revealed incompetence of left sapheno-femoral
junction. Which operation is best for this patient? (HBS)
• Trendlenberg operation*
• Injection sclerotherapy
• Gel foam injection
• Multiple Stab avulsions
82. You are summoned to disaster area after an earthquake as a member of health care provider
victims are retrieved from accident area, your best designated job at site is? (HBS)
• Assist in safe retrieval of victims
• Provide basic life support at safe site*
• Provide basic life support at accident site
• Provide advanced trauma life support
• Triage and referral to regional hospital
83. A patient sustained gunshot wound in left lower abdomen. There is no exit wound.
Exploratory laparotomy revealed injury to sigmoid colon. Which is the best treatment option
in this patient? (HBS)
• Primary repair
• Colostomy and mucous fistula*
• Conservative treatment
• Ligation of sigmoid colon
84. A 20 years old male patient sustained blunt abdominal trauma in a road traffic accident. In
emergency examination of patient revealed pulse 140/min, BP 90/60 mmHg. On Abdomen
examination there were multiple bruises over left lower chest. Patient failed to improve after
infusion of one liter of ringer lactate. Patient will be send for: (HBS)
• CT scan Abdomen*
• B MRI Abdomen
• Ultrasound Abdomen*
• OT for emergency Laparotomy
85. A 40 year old male underwent pelvic fracture due to road traffic accident; posterior urethral
injury was suspected and confirmed on urethrogram. The reason to defer treatment for 03
months is: (HBS)
• for patient to void normally
• for urethral ends to get closed
• for edema and hematoma to get resolved*
• None of the above
86. Which of the following is not a sign of tension pneumothorax? (HBS)
• Tracheal deviation
• Decreased breath sounds
• Respiratory distress with hypertension*
• Distended neck veins
87. An 18-year-old man is admitted to the emergency department shortly after road accident. His
pulse is barely palpable at a rate of 140 / minute, and BP is 60/0. Breathing is rapid and
shallow, aerating both lung fields. His abdomen is moderately distended with no audible
peristalsis. After rapid IV administration of 2 L lactated Ringer solution his pulse is 130 and BP
70/0. The next immediate step should be to: (HBS)
• Obtain cross-table lateral X-rays of cervical spine.
• Obtain head and abdominal CT scans.
• Obtain supine and lateral decubitus X-rays of the abdomen,
• Obtain an arch aortogram.
• Explore the abdomen*
88. A patient with penetrating injury to the chest should undergo thoracotomy it: (HBS)
• There is less than 100 mL blood which drains from the chest tube when placed.
• There is more than 200 ml/h of blood for 3 hours from the chest tube*
• There is documented lung injury on CT scan.
• Chest x-ray shows pneumothorax.
89. A 27year old pregnant female brought to emergency after trauma road accident. Her BP is
90/60 mmHg. Pulse is 90/min. What maneuver can help in raising her Blood pressure? (HBS)
• Manually displacing Uterus to right side to decrease pressure from Inferior vena cava.
• Log roll by 70 to 90 degree to right side
• Log roll to left side for decreasing pressure over Aorta
• Displacing Uterus to left side to decrease pressure from Inferior vena cava*
• Elevating foot ends
90. A 65 year old road construction laborer comes to out-patient department with a history of
mole on the cheek which has recently increased in size. On examination, the mole is 2cm in
size, has irregular margins. What is the most probable diagnosis? (HBS)
• Pigmented naevus
• Squamous cell carcinoma
• Basal cell carcinoma
• Malignant melanoma*
• Kaposi's sarcoma
91. A 25 year old female is evacuated from a building on fire after being trapped for 30 minutes.
She has singed nasal hair. Rest of the examination is unremarkable. What is the next step in
management? (HBS)
• Discharge the patient.
• Early elective intubation*
• Admit for observation
• Chest X-ray
• Bronchoscopy
92. In a 70 kg male patient with 40% burns, total intravenous fluid requirement in 24 hours is:
(HBS)
• 3200ml
• 5200ml
• 7200ml
• 9200ml
• 11200ml*
93. Which of the following is true for electric burns? (HBS)
• Airway intubation is mandatory
• Copious lavage helps in most cases
• Limb amputation should be done early
• Myocardium is not damaged
• Pacing interruption can cause cardiac arrest*
94. 27 years old soldier is referred from Gilgit after sustaining frostbite at Siachen 10 days ago On
examination, you see black discoloration of all toes in both feet with line of demarcation at
metatarsophalangeal joints. Which of the following is true in this patient? (HBS)
• Amputation can be considered*
• Prophylaxis against cardiac arrhythmias should be given
• Prophylaxis against clotting disorders should be given
• Rewarming the foot will reestablish blood supply*
• Warm fluids and Ibuprofen will be beneficial
95. 20 years old female presents in emergency with a wound just below the knee anteriorly,
sustained in a road traffic accident. There is loss of skin and extensive soft tissue injury with
bone exposure but no underlying fracture. What will be the most appropriate treatment in
this patient? (HBS)
• Coverage by skin graft
• Coverage by free muscle flap*
• Debridement and daily dressings
• Debridement and intravenous antibiotics
• Primary closure by tension sutures
96. A 30 years old male comes to the hospital with painful swelling of left arm. He injured his hand
3 days back with a rock. On examination he has an infected wound on the left palm with
marked tenderness, swelling and crepitus in the forearm. His temperature is 102F (normal:
98.6 F). What should be the next step in the management? (HBS)
• Give broad spectrum intravenous antibiotics
• Give tab augmentin 1gm twice a day
• Limb elevation and analgesics
• Take blood cultures and proceed according to the result
• Wound excision and broad spectrum intravenous antibiotics*
97. Which of the following is a premalignant lesion? (HBS)
• Actinic keratosis*
• Basal cell papilloma
• Blue naevus
• Mongolian spot
• Sebaceous cyst
98. A 60 year old smoker comes to emergency department with sudden onset of severe pain in his
right leg. On examination, no pulses are palpable and the leg is pale and cold. Left leg is
normal. What is the next step in management? (HBS)
• Angiography
• Echocardiography
• Asking the patient to stop smoking and reassess after 2 months
• Give heparin and shift to operating room*
• Thrombolytic therapy
99. Which of the following is the most common site involved in atherosclerotic occlusion.? (HBS)
• Common iliac artery
• Dorsalis pedis Artery
• Popliteal Artery
• Posterior tibial Artery
• Superficial femoral Artery*
100. A 70 year old female undergoes cholecystectomy. Next day she develops swelling and pain in
her right leg. On examination, the calf is swollen and tender. Distal pulses are palpable. What
is the next step in management? (HBS)
• Intravenous antibiotics
• Limb elevation and rest
• X-ray lower leg.
• Angiography
• Duplex ultrasound*
101. Which of the following is a high risk for developing deep venous thrombosis? (HBS)
• Moderate length procedure
• Chest surgery
• Bleeding disorders
• Obesity
• Previous deep venous thrombosis*
102. Which of the following organism causes Filariasis? (HBS)
• Wucheria bancrofti*
• Plasmodium falciparum
• Schistosoma
• Ascaris
• Helicobectar Pylori.
103. As a house officer in surgical ward you are asked to do saphenous cannulation for a burns
patient. The patient develops sudden onset of pain along the medial border of the
corresponding foot. Which nerve must have been accidentally ligated? (HBS)
• Sural nerve
• Deep peroneal nerve
• Saphenous nerve*
• Genicular nerve
• Medial Cutaneous nerve.
104. In a patient on anticoagulant therapy, the INR is maintained at: (HBS)
• 1.0-1.5
• 1.5-2.5
• 2.5-3.5*
• 3.5-4.5
• 4.5-5.5
105. A 60 years old male patient has chronic lymphedema of the right leg. Which of the following
complication can he develop? (HBS)
• Basal cell carcinoma A
• Marjolin Ulcer
• Pulmonary embolism
• Recurrent infections*
• Thinning of skin
106. A 47 years old male patient, having a long history of cigarette smoking comes to emergency
with gangrene of left foot. What is your most probable diagnosis? (HBS)
• Takayasu arteritis
• Giant cell arteritis
• Hypersensitivity angitis
• Thromboangiitis obliterans*
• Deep venous thrombosis
107. A patient presents with history of fall on outstretched hand and has pain and swelling at the
wrist joint. Which x ray will you advice to rule out fracture at wrist joint? (HBS)
• Xray wrist PA and Lat views.
• X ray hand AP and Lat views.
• Xray wrist AP view
• X ray wrist AP and lateral views*
• X ray wrist AP, Lat and Oblique Views
108. Which metal is applied in the walls of an Xray room for radiation protection? (HBS)
• Copper
• Iron
• Lead*
• Silver
• Steel
109. Which of the following is the gold standard for diagnosis of pelvic pathology? (HBS)
• Bone scan
• Computed tomography
• Contrast studies
• Magnetic resonance imaging*
• Xray Pelvis
110. For which of the following suspected condition, abdominal x ray is done as the first line
investigation? (HBS)
• Chronic pancreatitis
• Duodenal bleed
• Gastric bleed
• Intestinal obstruction*
• Renal colic.
111. Which of the following cancer is potentially curable without the need for surgical excision?
(HBS)
• Basal Cell carcinoma of skin*
• Invasive ductal carcinoma breast
• Follicular thyroid carcinoma
• Gastric carcinoma
• Colorectal carcinoma.
112. A 10 years old boy is brought to the emergency department with a deep cut on his thigh. He
is bleeding profusely. What should be the immediate management of the wound? (HBS)
• Apply pressure on the wound*
• Call vascular surgeon
• Explore the wound
• Tie a tourniquet above the wound
• Wash the wound with normal saline
113. A 45 year old male patient is brought in emergency after sustaining a blunt chest injury in a
road traffic accident. Which of the following signs will give you the suspicion of tension
pneumothorax? (HBS)
• Hypotension
• Increased respiratory rate
• Oliguria
• Shift of the trachea*
• Tachycardia
114. Which of the following is the most important initial step in managing a comatose head injury
patient? (HBS)
• Computed tomography brain.
• Glassgow Coma Scale scoring*
• Immediate referral to a neurosurgical unit
• Passing 2 large bore intravenous cannula
• Securing airway
115. A 35 year old male falls from a building. On primary survey he is alert and vitally stable but is
complaining of severe pain in lower back. Lower limb examination is unremarkable. Which of
the following investigations will confirm the suspected diagnosis? (HBS)
• Blood complete picture
• Serial xrays of spine*
• Ultrasound pelvis
• Urine analysis
• Xray erect abdomen
116. A patient sustains head injury. He opens his eyes to painful stimulus. He is moaning
periodically but does not follow commands. His hand reaches purposefully towards the
painful stimulus. What is his Glasgow coma scale? (HBS)
• 6
• 7
• 8
• 9*
• 10*
117. A 25 year old male patient sustains trauma to chest. He has 2 fractured ribs on the right
without any underlying lung injury. He is complaining of severe pain. Which of the following
will be most helpful in relieving his pain? (HBS)
• Chest physiotherapy
• Chest tube insertion
• Axillary Block
• Intercostal nerve block*
• Paracetamol
118. A 4 year old child is booked for herniotomy. For how long should the patient be kept nil by
mouth before surgery? (HBS)
• 2 hours
• 4 hours
• 6 hours*
• 8 hours
• 12 hours
119. A 37 year old - male car driver skids from the road at high speed and hits a tree. Which of the
following injuries, he is most likely to suffer because of deceleration? (HBS)
• Cardiac tamponade
• Liver laceration
• Mesenteric avulsion*
• Shoulder joint dislocation
• Splenic rupture
120. Which of the following is a criterion of a tidy wound? (HBS)
• Contaminated wound
• Devitalized tissues
• Crushed wound
• Minimal tissue loss*
• Primary repair is not possible
121. Which of the following indicates the criteria of Red color entry in triage categories? (HBS)
• Compound fractures
• Ruptured abdominal viscus
• Severe brain damage.
• Simple fractures
• Tension pneumothorax*
122. A 30 year old- male, is brought to emergency after road traffic accident following high speed
driving. There is a 5 cm laceration just above the right ear over the scalp. His GC is 7. No other
Injuries are seen on primary survey. He is hemodynamically stable. What is the next step in
the management of this patient? (HBS)
• Admit for observation
• Burr hole hematoma evacuation
• Computerized tomography brain*
• Intravenous mannitol
• X-ray skull
123. 27 year old- female, injures her leg in a high speed road traffic accident. On examination,
there is a large bruise over the left calf. The distal pulses are palpable. On X-rays no fractures
are detected. What is the best treatment plan for her? (HBS)
• Admit for observation*
• Angiography
• Discharge on analgesics
• Exploration and vascular repair
• Fasciotomy
124. An 18 year old- female, is brought to the emergency after a road traffic accident involving her
car with a motorcycle. Her blood pressure is 60 systolic when brought to the hospital, but
responds to fluid resuscitation. Her blood pressure now, is 100/65 with a pulse of 104/min.
She has bruising in the region of her left lower chest. Rest of the survey shows no other injury.
What is the next step in the management of this patient? (HBS)
• Computerized tomography of abdomen*
• Immediate laparotomy
• Diagnostic laparotomy
• Diagnostic peritoneal lavage
• X- ray erect abdomen
125. A 78 year old- male, presents with acute retention of urine, urinary tract infection and
increasing confusion. He is catheterized and treated with intravenous antibiotics, intravenous
fluids and is advised to drink plenty of fluids. The following day he becomes increasingly
drowsy, What is the cause of electrolyte imbalance in this patient? (HBS)
• Hypernatremia
• Hyperkalemia
• Hypoglycemia
• Hyponatremia*
• Hypokalemia
126. A 68 year old- male is brought in an unconscious state to the hospital. He was complaining of
back pain before collapsing suddenly. On examination, he looks pale. Pulse is 136/min with
50mm Hg systolic blood pressure. His abdomen is distended. He improves partly with fast
crystalloid infusion. What is the most likely type of shock in this patient? (HBS)
• Anaphylactic
• Cardiogenic
• Hypovolemic*
• Neurogenic
• Traumatic
127. A 52 years old- male, presents in surgical clinic with right sided inguinal swelling. He has
chronic renal failure secondary to glomerulonephritis. On examination, he has a right sided
reducible direct inguinal hernia. He also has mild ankle edema. His serum creatinine is
3.5mg/di ( Normal: 0.5-1.2 mg/dn) How will you proceed in this patient? (HBS)
• Cancel the surgery and advise to wear a truss.
• Check electrolytes and proceed with surgery.
• Dialyse the patient and proceed with surgery
• Postpone the surgery until cleared by nephrologist*
• Take high risk consent and proceed with surgery.
128. A 40 year old-male is brought to the emergency after a street fight 15 minutes ago. He was
punched hard in the epigastrium. His pulse is 80/min and blood pressure is 120/80. Abdominal
examination reveals mild tenderness in epigastric region. His blood complete picture shows a
hemoglobin of 9mg/dl (Normal: 14-18 mg/di). How will you manage this patient? (HBS)
• Does not need any treatment.
• Investigate further for anemia*
• Transfusion of packed red cell
• Urgent cross matched blood transfusion.
• Urgent" O" positive blood transfusion
129. A 20 year old female is evacuated from car accident site in shock. Her respiratory rate is
25/min. Examination of airway and chest is unremarkable. Her pulse is 115/min and blood
pressure is 110/70. Which type of shock is she most probably suffering from? (HBS)
• Anaphylactic shock
• Cardiogenic shock.
• Hypovolemic shock*
• Neurogenic shock
• Septic shock.
130. According to traditional classification of shock, how do you define class 3 shock as per
volume loss? (HBS)
• 1-5 %
• 5-15%
• 15-30%
• 30-40%*
• >40%
131. A 60 years old male patient undergoes left hemicolectomy for carcinoma of the sigmoid
colon. On third post-operative day he becomes listless and has slurred speech. His ECG shows
flattened T-waves. Which parenteral fluid will be most appropriate for this patient? (HBS)
• Blood transfusion.
• 5% Dextrose.
• 5% Dextrose with 2 Saline.
• Normal saline with KCL*
• Ringer lactate
132. A 35 years old female is received in emergency in hypovolemic shock. She receives multiple
blood transfusions during surgery and in the post-operative period. On 2nd post operative day
she develops perioral numbness and chevostek's sign is positive. Which treatment do you
choose for this patient? (HBS)
• Intravenous bicarbonate
• Intravenous calcium carbonate*
• Intravenous 25% dextrose
• Intravenous potassium chloride
• Intravenous 1.8% saline
133. A 32 year old- female with known Crohn's disease develops a fistula in the right iliac fossa.
Investigations reveal a low-output distal small bowel fistula. Which mode of nutrition is best
for this patient? (HBS)
• Elemental/ low residue diet*
• Feeding gastrostomy
• High protein enteral diet.
• Nasogastric tube feeding
• Total parenteral nutrition.
134. A 16 year old- boy sustains head injury after a road traffic accident. He is admitted to
neurosurgical intensive care unit. It is assessed that he may need nutritional support for a
week. What will be the best route for nutritional support in this patient? (HBS)
• Feeding gastrostomy
• Feeding jejunostomy
• Nasogastric tube feeding*
• Oral liquids with spoon
• Total parenteral nutrition
135. A 60 year old diabetic male patient undergoes cholecystectomy in emergency for empyema
gall bladder. Post operatively he develops pneumonia. He is likely to require ventilation in
intensive care for a number of days. Which type of feeding is most appropriate in this patient?
(HBS)
• Feeding gastrostomy*
• Feeding jejunostomy
• Nasogastric tube feeding*
• Oral liquids with spoon
• Total parenteral nutrition
136. 45 year old- male sustains a bullet injury leading to perforation of descending colon. Post
operatively he develops necrotizing fasciitis of the anterior abdominal wall. What is the
treatment of choice in this patient? (HBS)
• Incision and drainage
• Intravenous broad spectrum antibiotics
• Oral broad spectrum antibiotics
• Removal of stitches and daily dressings
• Wide excision and broad spectrum antibiotics*
137. Which one of the following is the most common cause of lymphedema worldwide? (HBS)
• Chaga's disease
• Filariasis*
• Secondary to radiotherapy
• Secondary to surgery
• Leptospirosis
138. A 70 years old male patient is taking broad spectrum antibiotics. On 5th day he develops
watery diarrhea and abdominal cramps. What is the best next step in management? (HBS)
• Abdominal CT scan
• Stool routine examination
• Stop antibiotics*
• Urgent colonoscopy
• Intravenous fluids
139. 30 years old male sanitary worker presents in surgical clinic with a painless lump in the neck
since 3 weeks. One day back he noticed some pus coming out of it. He also gives history of
generalized weakness and night sweats. What is your most probable diagnosis? (HBS)
• Acute lymphadenitis
• Cold abscess*
• Hodgkin's lymphoma
• Infected sebaceous cyst
• Metastatic carcinoma
140. Which one of the following is caused by Ascaris? (HBS)
• Hydatid Cyst
• Obstructive jaundice*
• Pruritis ani
• Schistosomiasis
• Splenomegaly
141. 60 year old diabetic patient is undergoing left below knee amputation. Which antibiotic will
you prescribe for prophylaxis? (HBS)
• Injection Benzylpenicillin 1.2 gm
• Injection Ceftriaxone 1 gm*
• Injection Ceftriaxone 2 gm
• Injection Co-amoxiclave 1.2 gm
• Injection metronidazole 500mg
142. In order to avoid post-operative wound infection, which of the following statements is true?
(HBS)
• Creme depilation should be used to remove hair 1 day before surgery
• Ensure that the patients temperature remains below 98F ( Normal -98.6F)
• Give intravenous antibiotics for 3 days and then switch to oral antibiotics
• Length of patient stay should be kept to a minimum*
• Patient should not take a bath for at least 10 days after surgery
143. 30 years old male patient is referred from district headquarter hospital with 15 days history
of a non-resolving mass in right iliac fossa despite treatment. Patient also has history of
bloody mucoid diarrhea, fever and abdominal pain. Which of the following is the best
management at this stage? (HBS)
• Colonic resection*
• Computed tomographic scan
• Injection Ceftriaxone
• Inj Metronidazole
• Ultrasound guided Aspiration
144. A 40 years old male patient comes to surgical clinic with 10 years history of a non-healing
ulcer on left forearm after sustaining a third degree burn. The ulcer has changed its
appearance in the last 2 months and now has a heaped up tissue growth around the edges.
Which of the following is the most appropriate next step in diagnosis? (HBS)
• Arteriogram
• Biopsy*
• Culture
• Doppler studies
• Venous pressure tracings
145. A taxi driver has been brought to A & E department after sustaining a head injury. He was
admitted for neuro-observation Feature which would cause concern during observation of
head injury patient is: (HBS)
• Bradycardia*
• Hypotension
• Photophobia
• Pyrexia
• Tachycardia
146. Following is NOT a life threatening chest injury to be ruled out in primary survey: (HBS)
• Tension pneumothorax
• Massive haemothorax
• Bronchial obstruction*
• Flail chest
• Open pneumothorax
147. You are assisting a Laparoscopic Cholecystectomy and your consultant asks you about the
most dangerous anomaly in the arterial supply of gall bladder?
• Cystic artery arising from gastroduodenal artery
• Accessory cystic artery
• Tortuous hepatic artery anterior to cystic duct.*
• Short cystic artery
• Cystic Artery lying anterior to cystic duct
148. Which of the following investigations is done to assess stomach in fluoroscopy?
• Barium swallow
• Barium enema
• Barium follow through
• Barium meal*
• Gastrograffin swallow.
149. Which of the following genes is implicated in the etiology of breast cancer?
• p53
• RET
• PTCH
• VHL
• BRCA1, BRCA2*
150. A 35 year old female is being investigated for anemia. After a normal upper and lower
gastrointestinal endoscopy the physician decides to check the small intestine. Which one of
the following will be the investigation of choice?
• Barium enema
• Barium follow through*
• Computed tomography
• Magnetic resonance imaging
• Barium swallow
151. Which of the following structures is protected while doing right hemicolectomy?
• Liver
• Pancreas
• Urinary bladder
• Duodenum*
• Common bile duct
152. Which one of the following arteries is an important landmark for identifying superior
parathyroid glands?
• External carotid Artery
• Common carotid Artery
• Inferior thyroid Artery*
• Superior thyroid Artery
• Subclavian Artery
153. In a burn patient, what is the best indicator for volume replacement:
• Normal blood pressure
• Low blood creatinine level
• Normal pulse rate
• Mental alertness
• Urinary output*
154. Hidradenitis suppurativa is an infection of:
• Apocrine glands*
• Adenoids
• Hair follicles
• Middle ear
• Tonsils
155. Lucid interval in a patient after a head injury typically is present in:
• Dural sinus haematoma
• Extra-dural haematoma*
• Intra-cerebral haematoma
• Scalp haematoma
• Sub-dural haematoma
156. A 65-year-old Australian presents with a painless, ulcerated lesion on his right cheek. The
lesion has been present for 1 year. Physical examination of the patient's neck reveals no
lymph node enlargement. The most likely diagnosis is:
• Melanoma
• Basal Cell Carcinoma*
• Squamous Cell Carcinoma
• Sebaceous cyst
• Anthrax
157. Stage 1 shock results after loss of following percentage of blood:
• 5%
• 10%
• 15%*
• 20%
• 25%
158. Main stay of treatment in lymphoedema is:
• Chemo-radiation
• Conservative*
• Radiotherapy
• Steroids
• Surgery
159. Which is the most common intra-abdominal organ injured with blunt trauma?
• a Colon
• b. Liver
• c. Small gut
• d. Spleen*
• e. Stomach
160. The initial maneuver to establish a clear airway in a trauma victim is:
• Jaw thrust
• Oropharyngeal airway
• Nasopharyngeal airway
• Removal of foreign body from mouth*
• Endotracheal intubation
161. A young man with automobile accident is having a tension pneumothorax. Immediate
treatment is:
• Chest inubation
• Intercostal block and strapping
• Needle thoracostomy*
• Endotracheal intubation and pleural aspiration
• Immediate thoracotomy
162. Select the most appropriate initial step in the management of a patient with multiple
injuries:
• Splinting of fractures
• Control of external hemorrhage
• Relief of tension pneumothorax
• Blood transfusion
• Maintenance of airway*
163. During the process of wound healing, collagen is laid down by:
• Eosinophils
• Basophils
• Fibroblasts*
• Neutrophils
• Platelets
164. Which of the following studies is most helpful in evaluating a patient's pulmonary status for a
Thoracic surgical procedure?
• ECG
• Chest X-ray
• History alone
• Lung functions test*
• ETT
165. 13.Malignant hyperthermia is a complication of:
• Atropine
• Pentothal
• Halothane
• Atracurium
• Succinylcholine*
166. ABI stands for:
• Ankle Brachial Index*
• Arm Body Index
• Airway Breathing Investigation
• Ankle Body Index
• Airway Blood Interface
167. 52. A patient with spontaneous eye opening, who is confused and localizes pain has a
Glasgow Coma Score (GCS) of:
• 5
• 7
• 9
• 11
• 13*
168. MRI is contraindicated if patient is having:
• Fever
• Cardiac pace maker*
• Plaster of Paris
• Leg edema
• Chest drain
169. Intravenous Urography is not done if a patient is having:
• Tachycardia
• Jaundice
• Renal failure*
• Haematuria
• Cough
170. 23. In laparoscopy surgery, gas used to create pneumo-peritoneum is:
• Carbon dioxide*
• Helium
• Hydrogen
• Nitrous oxide
• Oxygen
171. Cuffed endo-tracheal tube is not used in children as it leads to:
• Allergic reaction
• Bleeding
• Cough
• Cyanosis
• e Tracheal stenosis*
172. What is the difference between hypertrophic scars and keloids?
• Keloids are an overabundance of fibroplasia as a result of healing: hypertrophic scars are a
failure of collagen remodeling.
• Hypertrophic scars often regress over time, whereas keloids rarely regress.*
• c Hypertrophic scars are more common in darker pigmented ethnicities.
• Hypertrophic scars extend beyond the border of the original wound.
• e Keloids are very painful
173. 63. The presence of sulfur granules in a pus discharge indicates:
• Actinomycosis*
• Aspergellasis
• Brucellosis
• Mycoplasmosis
174. What type of nerve injury involves disruption of axonal continuity with preserved Schwann
cell sheath?
• Neurapraxia
• Axonotemesis*
• Neurotmesis
• Axonolysis
• Neurolysis
175. 60. Which type of collagen is most important in wound healing?
• Type I
• Type III*
• Type V
• Type VII
• Type XI
176. Signs of malignant transformation in a chronic wound include:
• Persistent granulation tissue with bleeding
• Everted wound edges*
• Non-healing after 2 weeks of therapy
• Distal edema
• Wound shrinkage
177. 31. Which one of the following is most radiosensitive tumour?
• Neuroblastoma*
• Renal cell carcinoma
• Gastric carcinoma
• Germ cell tumour
• Leiomyosarcoma
178. Earliest source of glycogen during starvation is:
• Blood
• Bone
• Liver*
• Muscle
• Spleen
179. 33. In prolonged starvation, main source of energy is:
• Liver glycogen
• Lipid stores*
• Muscle protein
• Muscle glycogen
• Inhaled oxygen only
180. Commonest presentation of arterial disease is:
• Acute limb ischemia
• Critical limb ischemia
• Gangrene
• Intermittent claudication*
• Venous thrombosis
181. 35.Commonest type of lymphedema is:
• Primary
• Secondary*
182. Axillary lymph nodes receive following percentage of lymphatics from breast:
• 15%
• 25%
• 50%
• 75%*
• 100%
183. 18.Parathyroid glands derive their blood supply from:
• Common carotid arteries
• Inferior thyroid arteries*
• Parathyroid arteries
• Superior thyroid arteries
• Tracheal arteries
184. 53. Massive transfusion protocols:
• Should include transfusion of fresh frozen plasma (FFP) and platelets in addition to packed
RBCs*
• Should only be initiated after blood typing, but cross match is not needed
• Should be initiated in patients with tachycardia despite administration of 3.5 L of crystalloid
fluids
• Should include testing for coagulopathies, present in 5% of patients requiring massive
transfusion
• Should include Vitamin K injection
185. 54. A patient with penetrating injury to the chest should undergo thoracotomy if:
• There is more than 500 mL of blood which drains from the chest tube when placed.
• There is more than 200 ml/h of blood for 3 hours from the chest tube.*
• There is an air leak that persists for >48 hours.
• There is documented lung injury on CT scan
• There is tension pneumothorax
186. 38. Lipodermatosclerosis is a feature of:
• Arterial disease
• Cardiac disease
• Hyperlipidemia
• Lymphatic disease
• e Venous disease*
187. Most common cause of leg ulcer is:
• Arterial
• Fungal
• Malignant
• Traumatic
• Venous*
188. 40. Slipping sign is a feature of
• Ganglion
• Lipoma*
• Melanoma
• Varicocele
• Neurofibroma
189. Compartment syndromes typically occurs in:
• Closed upper limb injuries
• Open upper limb injuries
• Closed lower limb injuries*
• Open lower limb injuries
190. 50. The first step in the evaluation and treatment of a patient with an infected insect bite on
the leg with cellulitis, bullae, thin grayish fluid draining from the wound, and pain out of
proportion to the physical is:
• Obtain C-reactive protein
• CT scan of the leg
• Magnetic resonance imaging (MRI) of the leg
• Surgical exploration*
• Observation only
191. A trauma patient arrives following a stab wound to the left chest with systolic blood pressure
(SBP) 85 mm Hg, which improves slightly with intravenous (IV) fluid resuscitation. Chest X-ray
demonstrates clear lung fields. What is the most appropriate next step?
• Computed tomography (C) scan of the chest
• Pelvic X-ray
• Focused abdominal sonography for trauma (FAST) examination*
192. 45. PAIR procedure is used for treatment of:
• Bone cyst
• Choledochal cyst
• Hydatid cyst*
• Ovarian cyst
• Sebaceous cyst
193. The most common cause for a transfusion reaction is :
• Air embolism
• Contaminated Blood
• Human error*
• Unusual circulating antibodies
• HIV
194. 44. Neurogenic shock is characterized by the presence of:
• Cool, moist skin
• Increased cardiac output
• Decreased peripheral vascular resistance*
• Decreased blood volume
• Increased peripheral vascular resistance
195. The best diagnostic imaging modality for identifying lower extremity occlusive disease is:
• MR angiography
• CT angiography
• Doppler ultrasound
• Contrast angiography*
• Radioneucleotide scan
196. An 8-year-old boy is brought to the emergency room after accidentally touching a hot iron
with his forearm. On examination, the burned area has weeping blisters and is very tender to
the touch. What is the burn depth?
• First degree
• Second degree*
• Third degree
• Forth degree
• Fifth degree
197. 58.Steroids impair wound healing by:
• Decreasing angiogenesis and macrophage migration*
• Decreasing platelet plug integrity
• Increasing release of lysosomal enzymes
• Increasing fibrinolysis
• Increasing vasoconstriction
198. 37. Commonest site of neuropathic ulcer is:
• Sole of foot*
• Heel of foot
• Dorsum of foot
• Fore foot
• Tip of toes
199. Which is NOT a component of systemic inflammatory response syndrome (SIRS)?
• Temperature
• White blood cell (WBC) count
• Blood pressure*
• Heart rate
• Respiratory rate
200. 48. The best method for hair removal from an operative field is :
• Shaving the night before
• Depilating the night before surgery
• Shaving in the operating room
• Using hair clippers in the operating room*
• Shaving a day before
201. During a laparoscopic appendectomy, a large bowel injury was caused during trochar
placement with spillage of bowel contents into the abdomen. What class of surgical wound is
this?
• Class I (clean)
• Class II (clean/contaminated)
• Class III (contaminated)*
• Class IV (dirty)
202. 26.Following airway can be used in a conscious patient:
• Endotracheal tube
• Oropharyngeal airway
• Laryngeal mask airway
• Nasopharyngeal airway*
• Tracheostomy tube
203. 27.Following suture is a monofilament:
• Catgut
• Cotton
• Prolene*
• Silk
• Vicryl
204. Standard temperature for autoclave is Celsius:
• 101
• 111
• 121*
• 131
• 141
205. 29. Samples for routine histopathology are immediately placed in following fixative, to
preserve morphology:
• 10 % Alcohol
• 10% Formaldehyde*
• 10 % Glucose
• 10% Mannitol
• 10% Saline
206. 5-Fluorouracil acts by:
• Interfering with mitosis
• Interfering with DNA synthesis*
• Direct damage to DNA
207. 64. A 3-mm basal cell carcinoma (BCC) of the skin should be treated with:
• Biopsy and gross total excision
• Dermatologic laser vaporization
• Excision with 2- to 4-mm normal margin*
• Electrodessication
• e Observation only
208. 65.The primary risk factor for the development of cutaneous squamous cell carcinoma (SCC)
is:
• a UV exposure*
• Cigarette smoking
• Chemical agents
• Chranic, non healing wounds
• Trauma
209. 55. A 22-year-old man is brought to the emergency room after a house fire. He has burns
around his mouth and his voice is hoarse with laboured breathing. What is the most
appropriate next step in management?
• Immediate endotracheal intubation*
• Examination of oral cavity and pharynx, with fiberoptic laryngoscope if available.
• Place on supplemental oxygen.
• Placement of two large-bore intravenous (IV) catheters with fluid resuscitation.
• Chest intubation
210. 56. What percentage burn does a patient have who has suffered burns to right lower limb
(circumferential), right upper limb (circumferential), and the anterior trunk?
• 18%
• 36%
• 45%*
• 50%
• 70%
211. 45.A 70-kg male patient presents to ED following a stab wound to the abdomen. He is
hypotensive, markedly tachycardic, and appears confused. What percent of blood volume has
he lost?
• 5%
• 15%
• 25%
• 35%
• 55%*
212. 46. Which constellation of clinical findings is suggestive of cardiac tamponade?
• Hypotension, wide pulse pressure, tachycardia
• Tachycardia, hypotension, prominent neck veins
• Hypotension, wide pulse pressure, prominent neck veins
• Hypotension, muffled heart sounds, prominent neck veins*
• Hypertension, muffled heart sounds, prominent neck veins
213. A 15-year-old man is knocked off his cycle and hits his head on a stone. He is brought into
hospital with a fluctuating score on the Glasgow Coma Scale. He underwent a CT scan of skull
which shows a biconvex-shaped haematoma. What is les most likely diagnoses?
• Extradural hematoma*
• Intra cerebral hematoma
• Subdural harmatorna
• d. Hydrocephalus
• e Subirachnoid haemorrhage
214. A 45-year-old man presents to the emergency department with a history of coffee ground
vomiting He also reports that for 2 days his stool has appeared darker than usual Which of the
following gives the most sensitive guide as to the severity of Ins gastrointestinal
haemorrhage?
• Hemoglobin
• Systolic blood pressure
• Pulse rate*
• Volume of vomitus melaena
• Lying and standing blood pressure
215. A 31-year-old man is admitted following an assault outside a cinema. During the fight he was
hut by a blunt object across the side of the head. On admission his Glasgow Coma Scale score
is initially 12/15 but falls to 8/15 during his evaluation. The decision is taken to perform a
computed tomography head scan, which identifies a biconvex-shaped space-occupying lesion
within the cranial vault. The diagnosis is
• Extradural haematoma*
• Subdural faematorna
• Subarachnoid haemorrhage
• Cerebral contusion
• Intracerebral haemorrhage
216. (……).The sooner is relocated by you in AE our afterwards you noticed winning of the scapula
Which nerve is likely to be damaged?
• Axillary nerve
• Median nerve Redial serve
• Long thoracic nerve of Bell*
• Ulnar nerve

Anesthesia
1. Most commonly used IV anesthetic is:
• Diazepam
• Morphine
• Atracurium
• Propofol*
• Ketamine
2. 3 basic monitoring during anesthesia are:
• ECG, Temperature and Pulse Oximeter
• ECG, Pulse Oximeter and BP*
• ECG, Capnogram
• Pulse Oximeter, Respiratory Rate and BP
• Respiratory Rate, Heart Rate and BP
3. Most common drug used for Spinal anesthesia is:
• Lidocaine
• Fentinyl
• Ketamine
• Bupicaine*
• Diazepam
4. A 40 year female is planned for laparoscopic cholecystectomy under G/A. A routine airway
assessment before undergoing anesthesia Includes:
• Bronchoscopy
• Cervical spine X-ray
• Mallampati grading*
• MRI Cervical spine
• BMI calculation
5. There is a 65 years old male in ICU. The patient is in critical condition and is not expected to
survive beyond the next 24 hours without surgery, suspected ruptured abdominal aortic
aneurysm, Anesthetist is asked to evaluate the patient, This patient fall in which ASA grading.
• II
• III
• IV
• V*
• VI
6. Most cardio toxic local anesthetic is:
• Lignocaine
• Tetracaine
• Cocaine
• Bupivacaine*
• Mepivacaine
7. A surgeon is planning to do laparoscopic cholecystectomy under G/A. Anesthetist wants to
measure End tidal CO₂. End tidal CO₂ monitoring under general anesthesia is done with:
• ABG
• Arterial line
• Capnography*
• Pulse oximetry
• CVP line
8. A full stomach patient is being scheduled for emergency appendectomy. Anesthetist wants to
use depolarizing muscle relaxant for intubation. Which one is depolarizing muscle relaxant?:
• Atracurium
• Pancuronlum bromide
• Rocuronium
• Succinylcholine*
• Vecuron
9. Most commonly used Inhalational anesthetic agent in general anesthesia is
• Halothane
• Isoflurane
• Desfiurane*
• Sevoflurane
• Chloroform
10. Most of the elective cesarean sections are performed under
• Spinal anesthesia*
• Epidural anesthesia
• Local anesthesia
• General anesthesia
• Caudal anesthesia
11. Most commonly used IV anesthetic is: (HBS)
• Ketamine
• Morphine
• Propofol*
• Diazepam
12. Most common drug used for Spinal anesthesia is: (HBS)
• Bupivacaine*
• Lidocaine
• Diazepam
• Fentanyl
13. Most common site of spinal anesthesia is: (HBS)
• Thoracic region
• Cervical region
• Lumbar region*
• Sacral region
14. Most common anesthesia used for elective C- Section is: (HBS)
• GA
• Spinal anesthesia*
• IV Sedation
• Epidural anesthesia
15. Most common mixture of gases for GA is: (HBS)
• O2/Nitrogen and halothane
• O2/N2O and Isoflurane*
• O2/CO2 and sevoflurane
• CO2/N2O and Isoflurane
16. A 60 year old female patient undergoes open cholecystectomy under spinal anesthesia. Which
one of the following is the most serious complication that can occur in this patient? (HBS)
• Cardiac arrest*
• Headache
• Hypotension
• Needle breakage
• Urinary retention
17. Which type of anesthesia is ideal for a patient undergoing surgery of the humerus? (HBS)
• Bier block*
• General anesthesia
• Local anesthesia
• Ring block
• Spinal anesthesia
18. What is the dose of Injection lignocaine for giving local anesthesia? (HBS)
• 1-3 mg/kg body weight
• 3-5 mg/kg body weight*
• 5-7 mg/kg body weight
• 7-9mg/kg body weight
• 9-11 mg/kg body weight.
19. Which of the following investigation is mandatory in a patient receiving general anesthesia?
(HBS)
• Blood complete picture*
• Echocardiography
• Liver function test
• Renal function test
• Serum electrolytes
20. During anaesthesia, Succinylcholine injection is used for:
• Induction
• Maintenance
• Reversal
• Relaxation*
• Analgesia
21. Lignocain with Adrenalin is contraindicated in:
• Finger*
• Arm
• Scalp
• Palm
• Sole

Sendup Surgery A HBS 2022


1. A 20 years old male sustained incised wound over right forearm. He was taken to nearby
hospital. What is the best method to treat this patient?
• Primary suturing*
• Secondary suturing
• Delayed primary suturing
• Leaving the wound open
• Debridement only
2. A 45 years old lady sustained lacerated wounds on both thighs in a road traffic accident. She
was taken to the operation theatre after thoroughly cleaning the wounds what will be next
best step of treatment
• Primary suturing
• Secondary suturing
• Delayed primary suturing
• Leaving the wound open
• Debridement and leaving wound open*
3. A 30 years old male presented with ugly scar over 3. sternum one year after injury. Scar has
gone beyond the original injury area. What is the best diagnosis option in this patient?
• Keloid
• Hypertrophic Scar*
• Atrophic Scar
• weak scar
• Normal Scar
4. All are treatment options in Keloid expect
• compression stocking
• Local steroid
• Radiotherapy
• Excision
• Chemotherapy*
5. A 40 years male had open appendectomy three days back. Now complains of fever pain and
discharge from the wound. What is the best treatment option in this patient?
• Antibiotics
• Analgesics
• Antipyretics
• Open the wound sutures*
• Daily dressing
6. A 20 years old patient had Hernioplasty 3 days back. He is complaining of pain swelling and
discharge from the wound. His temperature is 38c. What is the diagnosis in this patient
• Surgical site infection*
• Hematoma
• Seroma
• Normal wound
• Lymph collection
7. A 60-year-old man who is 6 days following open anterior resection for rectal carcinoma. The
patient reported seeing a gush of fluid from the laparotomy wound. You notice that the small
bowel is eviscerating from the wound. Following initial resuscitation, the next best step is to
• Cover the small bowel with a sterile saline-soaked gauze
• Call for senior help
• Administer intravenous cefuroxime 1.5 g
• Return the patient to theatre for deep-tension abdominal wall closure*
• Apply vacuum-assisted closure therapy
8. Regarding wound healing, the inflammatory phase begins?
• immediately after wounding *
• one day after wounding
• two days after wounding
• three days after wounding
• four days after wounding
9. Regarding wound healing the proliferative phase of wound healing starts
• Immediately after wounding
• one day after wounding
• two days after wounding
• three days after wounding*
• four days after wounding
10. A 20 years old lady sustained untidy wounds over the lower limbs. This type of wounds is?
• Clean
• Incised
• Seldom tissue loss
• Contaminated devitalized tissue*
• Healthy tissue
11. Paraplegic patients are prone to develop pressure sores. Commonest site of bed sore in these
patients is
• ischium*
• Greater trochanter
• Sacrum
• Heel
• Malleolus
12. 12 A 60 years old diabetic patient developed necrotizing fasciitis of abdominal wall following
laparotomy for gangrenous small intestine. Best treatment option in this patient is?
• Incision and drainage
• Antiseptic dressing
• Repeated debridement*
• Broad spectrum antibiotics
• Cleaning with normal saline
13. 11 A 30 years old male sustained multiple gunshot wounds over right thigh. After initial
resuscitation he was taken to operation theatre. His best treatment option is
• Antiseptic dressing
• Primary repair of nerves
• Debridement and leaving wound open*
• Debridement and primary closure of wound
• & Primary repair of colon
14. 140 years old male sustained gunshot wound abdomen resulting in injury to sigmoid colon.
Best treatment option after initial resuscitation is
• Primary repair of tendons
• Colostomy and mucous fistula*
• Transverse colostomy only
• Ileostomy
• Jejunostomy
15. 15. A 20 years old male sustains multiple stab wounds in a 30cms treatment option in this
patient is? A Resection of 30cms segment and End to End anastomosis
• segment of mid jejunum Best*
• Repair of all individual stab wounds
• Jejunostomy
• Ileostomy
• colostomy
16. Features of malignant transformations of tissue are all except
• Evade apoptosis
• Acquire angiogenic competence
• Acquire ability to invade
• Acquire ability not to disseminate and implant *
• Evocation of inflammation
17. 17. Human papillomavirus (HPV) is associated with which of the following tumor?
• Cervical cancer*
• Kaposi's sarcoma
• Lymphomas
• Germ cell tumors
• Bladder cancer
18. 18.Which of the following is the best test for screening for carcinoma of colon?
• Occult blood test in stool*
• Ultrasound abdomen
• Blood complete picture
• CT Scan abdomen
• MRI Abdomen
19. 19. Tissue diagnosis in malignant disease can be confirmed by which of the following method?
• Ultrasound
• MRI
• CT SCAN
• X ray
• FNAC*
20. 20. A 70 years old lady presented with history of 1cm lump in right breast which is fixed to skin
What is T stage of this patient
• T1
• T2
• T3
• T4*
• TX
21. 21. A patient is involved in a road traffic accident. He suffers a left femur fracture with head
trauma. He is resuscitated at the scene and stabilized. Upon transfer to the ER, his blood
pressure gradually decreases and then suddenly drops to 60/40. He has a splint on the left leg.
What is the most likely cause of his decreased blood pressure?
• Head trauma
• Abdominal organ rupture*
• Rib fracture
• Cardiac contusion
• Rupture urethra
22. 22. A 71-year-old male is seen in the emergency room after a motor vehicle accident with
severe, sharp chest pain radiating to his back. His blood pressure is 190/100 mmHg and his
pulse is 100 beats per minute. Chest x-ray reveals a widened mediastinum. Before more can
be done, the patient dies. At autopsy, he has a massive amount of blood in the pericardial sac.
What is the most likely diagnosis?
• Pneumothorax
• Ascending aortic dissection*
• Uremic pericarditis
• Rupture of pericardial cyst
• Hemothorax
23. 23. A patient is profoundly dyspneic, tachycardiac, and hypotensive after sustaining a stab
wound to the left chest. Physical examination reveals decreased breath sounds on the side of
the wound, but the cardiac examination is normal. Examination reveals crepitus and soft
tissue swelling on the anterio chest wall and neck. He is awake and complains of pain at the
wound site. After the patency of the airways is assured, what is the next step in this patient's
management?
• Immediate intravenous access
• Orotracheal intubation
• Chest x-ray
• Left needle thoracostomy*
• Chest intubation
24. 24. What is the most common result of laceration of the middle meningeal artery?
• Subarachnoid hemorrhage
• Extradural hematoma *
• Subdural hematoma
• Intraparenchymal hemorrhage
• Subcutaneous hemorrhage
25. 25. Which of the following is not included in the Glasgow coma scale?
• Verbal response
• Motor response
• Eye response
• Pain response*
• None of the above
26. 26. Emergency decompression of the brain with a subdural hematoma should be evacuated if
there is a midline shift of what amount?
• 1mm
• 2 mm
• 3 mm
• 5 mm
• 10 mm*
27. 27. What percentage of patients with thoracic trauma will require thoracotomy?
• 10%-15% *
• 20%-25%
• 30% -40%
• 45%-50%
• 30-45%
28. 28. A 28-year-old male was injured in a motorcycle accident in which he was not wearing a
helmet. On admission to the emergency room he was in severe respiratory distress and
hypotensive (blood pressure 80/40 mm. Hg), and appeared cyanotic. He was bleeding
profusely from the nose and had an obviously open femur fracture with exposed bone. Breath
sounds were decreased on the right side of the chest. The initial management priority should
be:
• Control of hemorrhage with anterior and posterior nasal packing.
• Tube thoracostomy in the right hemithorax.
• Endotracheal intubation with in-line cervical traction. *
• Obtain intravenous access and begin emergency type O blood transfusions.
• Obtain cross-table cervical spine film and chest film.
29. 29 Trauma deaths most commonly occur at three distinct time periods after injury. Which of
the following statement is best concerning the time pattern of trauma mortality?
• Only 10% of trauma deaths occur within seconds or minutes of the injury
• A second mortality peak occurs within hours of injury with deaths in this time period being
markedly reduced with the development of trauma and rapid transport systems*
• Death one day to weeks after the injury are almost entirely due to infection and multiple
organ failure
• Late mortality in trauma patients, occurring days to weeks after the injury, has not been
affected by better trauma delivery systems
• Deaths in acute phase are rare and only occur in minor injuries
30. 30. A 22-year-old woman presents to the emergency department with a chief complaint of
severe left upper quadrant (LUQ) pain after being punched by her husband. Her blood
pressure is 110/70 mm Hg, pulse is 100 bpm, and respiration rate is 24 breaths per minute.
The best means to establish diagnosis which of the following?
• FAST*
• Physical examination
• CT of the abdomen
• Peritoneal lavage
• Upper gastrointestinal (Gl) series
31. Which of the following statement regarding lymphoedema is true?
• Primary lymphoedema occurs in more than 5 per cent or the population.
• Early treatment is usually successful. *
• Early treatment includes surgical drainage
• Fluid is relatively low in protein in lymphoedema.
• Lymphoedema often involves the muscle compartments.
32. 33. A 55-year-old woman gives a history of tiredness, aching, and a feeling of heaviness in the
left lower leg for the past 3 months. These symptoms are relieved leg elevation. She is also
awakened frequently by calf and foot cramping, which is relieved by leg elevation, walking, or
massage. On physical examination there are superficial varicosities, non-pitting edema, and a
slightly painful, 21 cm. diameter superficial ulcer 5 cm. above and behind the left medial
malleolus. What is the most appropriate diagnosis?
• Isolated symptomatic varicose veins.
• Superficial lymphatic obstruction.
• Deep venous insufficiency*
• Arterial insufficiency.
• Incompetent perforating veins.
33. 34. In a 55-year-old grocery store cashier with an 8-month history of leg edema increasing over
the course of a work day, associated with moderate to severe lower leg bursting pain, the
most appropriate investigative is
• Doppler duplex ultrasound*
• Brodie-Trendelenburg test.
• Ascending and descending phlebography.
• Measurement of ambulatory and resting foot venous pressure.
• Venous reflux plethysmography.
34. 35. A 61 year old male patient is admitted to A&E with sudden onset of a painful, cold, white
right leg. His radial pulse rate is 86 b/m and its rhythm follows no discernible pattern
throughout 30 seconds of palpation. Abdominal examination is normal. No pulses are
palpable in the right leg and ankle Doppler signals are absent. An ECG confirms the arrhythmia
but shows no signs of acute ischemia. Which is the single most likely diagnosis?
• Abdominal aortic aneurysm
• Aorto-iliac dissection
• Atrial fibrillation*
• DVT
• MI
35. 36. A 70-year-old woman develops a cold, painful, right leg. No pulses are palpable throughout
the right leg. She has several risk factors for atherosclerotic disease. A medical student asks
how best to differentiate between acute and chronic ischemia, Which is the single most
appropriate advice?
• History of 40 pack-years of smoking
• History of intermittent claudication
• History of thrombotic stroke
• Presence of femoro-popliteal bypass scar in the left leg
• Presence of foot pulses in the left leg *
36. 65-year-old with a 40-pack-year smoking history and type 2 diabetes present with cramp-like
pain in her right calf after walking 500 meters, relieved by rest. Her symptoms are aggravated
by walking up steep hills. Which is the single most appropriate management?
• Amputation
• Diagnostic angiography
• Endovascular stent
• Modification of risk factors*
• Reassure and follow up in 6 months
37. 38. A 55-year-old a history of man who smokes 40 cigarettes per day presents with pain in his
left calf after walking 100 meters The pain subsides with rest. He has no symptoms in his thigh
or buttock He has goo volume femoral pulse but no popliteal or pedal pulses are palpable.
Neurological examination is normal. He undergoes duplex Doppler ultrasound. Which is the
single most likely finding of this investigation?
• Left common iliac artery occlusion
• Left internal iliac artery stenosis
• Left posterior tibial artery occlusion
• Left profunda femoris artery stenosis
• Left superficial femoral artery occlusion*
38. 39. Abdominal aortic aneurysm was detected coincidentally on ultrasound examination of 68-
year-old man. He was referred to the vascular clinic. The patient is nervous about the
diagnosis, researching it on the Internet and has several questions. Which single statement is
correct?
• Abdominal aortic aneurysms are associated with tobacco smoking. hypertension, family
history, and diabetes mellitus
• Abdominal aortic aneurysms are considered for treatment by surgical or endovascular repair
when they reach a size of 25.5cm, in a patient fit for intervention *
• Abdominal aortic aneurysms most commonly involve the aorta at the level of the renal
arteries and below.
• Abdominal aortic aneurysms occur in 10% of the population aged over 65
• Abdominal CT scanning aortic aneurysm screening undertaken is in the UK
39. 40. Which of the following is not an independent risk factor for the development of coronary
and using peripheral atherosclerosis?
• Smoking
• Hypercholesterolemia
• Diabetes Mellitus *
• Hypertension
• Hypercoagulable conditions
40. 41. Which of the following describes chronic leg ulcers?
• Cause of ulcers cannot be determined by their locations on the leg
• Venous ulcers are seldom located on the foot*
• Arterial ulcers are seldom located on the leg
• Leg ulcers affect diabetic patients less often than other patient groups
• Ulcer healing is improved more significantly with balloon angioplasty than with arterial
bypass
41. 42. Which of the following characteristics of leg from venous insufficiency or lymphedema is
true?
• Edema forms when hydrostatic pressure in the interstitum is higher than that in the
lymphatics or venules
• Venous insufficiency causes pigmentation and hypertrophic changes in the skin over the
lymphedema ankle and results in late lymphedema with fibrosis *
• Lymphedema can be diagnosed by ultrasound imaging
• Operative intervention can treat venous insufficiency and is commonly used for
lymphedema
• Lymphedema may be pitting in form
42. 44. Fasciotomy should be performed in patients with which of the following signs or
symptoms?
• Tense fullness of the compartment in an otherwise asymptomatic patient
• Extremity ischemia for longer than 4 hours
• Progressively worsening neurologic signs after revascularization *
• Traumatic injuries to the popliteal artery
• Compartmental pressure higher than 15 mm Hg and unreliable findings on physical
examination
43. 45. A 72-year-old man underwent repair of an endovascular abdominal aneurysm 1 year back
collapses and complains of back and abdominal pain. His blood pressure is 90/40 mm Hg. The
patient denies a history of peptic ulcer or alcohol abuse. What is the most likely diagnosis?
• Aortoenteric fistula
• Bleeding duodenal ulcer
• Ruptured abdominal aortic aneurysm (AAA)*
• Pancreatitis
• Diverticulitis
44. 46. A 20-year-old female employed on the butcher's counter complains that her fingers
become white, then blue and cold at work. When she warms her hands under the hot tap they
become acutely painful developing a deep red color. Which is the single most appropriate
management?
• Oral prednisolone
• Request transfer from her current job to a different role*
• Lumbar sympathectomy
• Oral prostacyclin
• Nifedipine 5mg three times daily increasing to 20mg three times daily who worker,
45. A 43-year-old office worker, who has had a swollen left leg since her early teens has aching in
the calf at rest and has developed a 4cm shallow ulcer, with an erythematous base, above her
lateral malleolus what is cause of the disease?
• Aorto-iliac arterial disease
• Cellulitis
• Deep vein thrombosis
• Lipodermatosclerosis
• Lymphoedema *
46. 48. Which of the following is the greatest risk factor for developing an aortic aneurysm?
• Age
• History of Smoking*
• History of Urolithiasis
• Metabolic Syndrome
• Male Se Sex
47. 49. Which technique should be used to open the airway in a trauma patient suspected cervical
with spine injury?
• Tracheostomy
• Jaw thrust*
• Endotracheal intubation
• Head-tilt chin-lift
• Nasopharyngeal airway.
48. 50. A young man involved in a car crash accident was driving without a seat belt. He was
brought to the Emergency Dept in an ambulance. On arrival his B.P was 90/60 mmHg & pulse
rate of 120/min. On abdominal palpation, he had tenderness in the upper abdomen more on
the left side. There was no rebound tenderness, guarding or rigidity. As an Emergency doctor
your next plan of action will be:
• Prepare for Emergency Laparotomy
• FAST*
• Admit for observation
• Diagnostic Peritoneal Lavage
• CT scan of Abdomen & pelvis.
49. 51. A 10 year old boy, falls from a bike. The bike handle hits his abdomen. He is brought by his
parents to the hospital with complain of abdominal pain. On examination, his abdomen is soft
with mild tenderness. On the insistence of parents, he undergoes CT scan abdomen with IV &
oral contrast, which reports normal. He is discharged back home. Next day he is reports back
with vomiting after oral intake & distended abdomen. The next step in his management will
be:
• Admit for observation only
• Keep NPO & on I.V Fluids
• Inj. Ondansetron (Zofran) to stop vomiting
• Repeat CT Scan abdomen*
• Exploratory Laparotomy
50. 52. During a fight, a 40 year old man is hit in the abdomen by a steel bar. He reports to
Emergency Dept. where a Diagnostic Peritoneal Lavage (DPL) is performed. Which of the
following results of DPL is an indication of Exploratory Laparotomy?
• RBCs count of around 75000/cubic mm in the aspirate
• 10 ml of serous fluid
• Amylase level of around 150 IU/dl
• Bile aspirate*
• WBC count of around 400/cubic mm
51. 53. Which one of the following is an indication for Damage Control Surgery in a trauma
patient:
• A small intestinal perforation due to blunt trauma
• Major artery injury accessible for repair
• Coagulopathy leading to diffuse uncontrolled bleeding*
• Abdominal aortic injury
• Retroperitoneal hematoma
52. 54. Which is the TRUE statement about flail chest:
• Paradoxical motion of the chest wall causes respiratory failure.
• Initial chest x-ray can predict pulmonary contusion
• With the chest wall stabilized operatively, the duration of intubation is reduced if there is no
pulmonary contusion. *
• Patient should receive aggressive fluid resuscitation.
• All patients with flail chest need endotracheal intubation.
53. 55. A young man is involved in a road traffic accident. On arrival in Emergency Dept his BP was
70 systolic and pulse was 130/min low volume thin thready. On examination he had a
distended abdomen with clinical evidence of fracture of pelvis. For immediate resuscitation,
he was infused two litres of Ringer's Lactate but his vitals did not improve. The next step in
the management would be:
• Exploratory Laparotomy *
• CT Scan of Abdomen & Pelvis
• Angioembolization
• Focused Assessment with Ultrasound
• Plain X-ray Abdomen in supine position
54. 56. During Diagnostic Peritoneal Lavage (DPL), there is no need to put in one litre of warm
Normal Saline if the aspirate is:
• Frank bile
• Gastrointestinal contents like vegetable fibres
• Gross blood more than 10 ml
• All of the above*
• A & C only
55. 57. Which structures are located in the retroperitoneal space?
• Abdominal Aorta & Inf. Vena Cava
• Duodenum
• Kidneys and ureters
• All of the above*
• B and C
56. 58. Which of the following injuries of genitourinary system usually do not cause haematuria?
• Disruption of the renal pedicle
• Thrombosis of Renal Artery*
• Urethral injury
• All of the above
• A and B
57. 59. A 35 year old female back passenger on a motor bike had a road traffic accident. She
presented in Emergency Dept. with complain of pain in the neck. She also had stridor on
inspiration. She received oxygen inhalation with a mask but her oxygen saturation remained
below 90%. The next step in her management should be:
• Immediate tracheostomy
• Chest tube insertion on both sides
• Establish an airway while maintaining Inline immobilization *
• Immediate cervical spine x-rays while receiving 100% oxygen
• Apply traction to cervical spine.
58. 60. A nurse in Intensive Care Unit was asked to monitor a patient of head injury with Glasgow
Coma Scale. She was doing it correctly except for:
• Checking limb extension to pain
• Checking sensory loss *
• Checking conscious Level
• Checking eye opening
• Checking flexion in response to pain
59. 61. In Accident & Emergency Dept. to control arterial bleeding in an open wound on the thigh
of a 40 year old male with trauma, following methods were correctly used except for:
• Manual tourniquet
• Arteriography
• Manual pressure*
• Pneumatic tourniquet
• Pressure dressing
60. 62. Regarding Crush Syndrome the FALSE statement is
• It can lead to acute renal failure
• Occurs with sharp penetrating injuries*
• Causes traumatic rhabdomyolysis
• Myoglobin is released from cell death
• Muscle ischemia is a feature.
61. 63. In a trauma patient, what is FALSE about x-ray examination of skeletal injuries?
• Should be done during the Primary Survey*
• Depends on the mechanism of injury
• Depends on the patient's initial clinical findings
• Is appropriate during the Secondary Survey
• Depends on the hemodynamic status of the patient
62. 64 A 20 year old male patient of trauma involved in a road traffic accident presented in
Emergency Department with injuries to head & lower limb. He was able to walk but with a
limp. There was ecchymosis around his right eye & blurring of vision. X-ray reported a hair line
fracture in the skull orbital region. His Glasgow Coma Scale was
• 16
• 15*
• 20
• 14
• 12
63. 65. A 10-year-old boy was run over by a motor bike. He was brought to the Emergency Deptt
in respiratory distress. He complained of pain in his right chest. On examination, he was
hypotensive & tachypnoeic with tachycardia. His trachea was deviated towards the left side
with distended neck veins on the right side. He had normal breath sounds on the left side but
absent on the right side. The most likely diagnosis is
• Hemopneumothorax
• Open pneumothorax
• Hemothorax
• Tension pneumothorax*
• Multiple rib fractures with pneumothorax
64. 66. A 55 year old man was riding a bicycle when he was hit on the right chest by the bamboo
of a tonga, which entered his chest. In addition to severe pain in the chest, he became
tachypnoeic with difficulty in breathing. He was picked up by the ambulance. The trained
medic correctly:
• Stitched the wound edges & covered with dressing
• Applied sticking plaster to support the broken ribs
• Covered the wound with dressing & crepe bandage all around
• Applied occlusive dressing taped only on three sides*
• Packed wound tightly with antiseptic dressing to prevent leakage of air.
65. 67. In a motor vehicle crash, the lower rim of the steering wheel hit the abdomen of the 50-
year-old woman driver. Stable in the Emergency Dept. she was admitted for observation &
conservative management. At night she vomited twice & next morning she developed
abdominal pain with distension. The vitals were stable. Abdomen was tense, tender & rigid
with no bowel sounds. The most likely cause
• Liver trauma
• Splenic injury
• Small bowel perforation *
• Retroperitoneal hematoma
• Haemorrhage from mesenteric tear
66. 68. A 25-year-old construction worker fell from a height of 5 meters on the pavement. He was
brought to the hospital emergency dept fully conscious. He was hypotensive. On examination,
he had scrotal hematoma & blood at the urethral meatus, swelling & bruising around the
tender pelvis with a laceration in the perineum. The most appropriate early measure is
• Application of pelvic binder *
• Pass a urethral catheter
• Stitch the laceration in the perineum
• Drain the scrotal hematoma
• Give I.V antibiotic
67. A 2 year old child is has a history of fall from a height with an injury to the head. He was
breathing spontaneously but with a partially obstructed airway. To optimize his airway:
• Placed flat on the spine board facing up
• Neck should be flexed
• Insert oral airway only if he is conscious.
• Use jaw-thrust manoeuvre with bimanual inline spinal restriction*
• The airway should be inserted backward & rotating it 180 degree
68. 70. A 30 year old labourer loading boxes in a pickup truck when it accidently backed into him.
In the Emergency Dept. he had upper torso, facial and arm plethora with petechiae. His
clinical picture is most likely be due to:
• Traumatic diaphragmatic injury
• Subcutaneous emphysema
• Traumatic asphyxia*
• Fracture of the sternum
• Tension pneumothorax
69. 71. A 23-year-old mountaineer slipped & fell 10 meters down on the rocks. With a suspected
spine injury, he was picked up by the Ambulance paramedics & placed supine on a spine
board with paddings & straps. On arrival in the Emergency Dept. he was stable. He should be:
• Safe to keep on the rigid spine board for 6-8 hours
• Shifted to the hospital bed by lifting by 2 porters
• Kept strapped on the spine board till assessed by the Neurosurgeon.
• Sent directly to the Radiology Dept. for x-rays
• Logrolled by 4 persons for examination of the spine*
70. 72. A 15 year old girl got 30% scalds on her body from boiling water. In the Emergency Dept.
I.V access was established. The ED doctor should start her with:
• Blood Transfusion
• Warm Ringer's Lactate Solution. *
• 10% Dextrose solution)
• 5% Dextrose Saline solution
• Normal saline cooled before infusion
71. 73. A 40 year old man is admitted in the Burn Unit with 50% superficial & partial thickness
burns. He is receiving I/V Fluids. To monitor adequate fluid intake, the doctor should instruct
the Nurse on duty to:
• Check his B.P 2 hourly
• Follow the I.V Fluid regimen strictly
• Send blood sample for Serum Electrolytes 6 hourly
• Feel his tongue for signs of dehydration
• Monitor Urine output*
72. 74. A healthy 25 year old man sustains chest injury in a road traffic accident. X-ray in the
hospital reported hemothorax. On passing a chest tube about 500 ml of blood was drained.
His vital signs will show
• Normal pulse pressure*
• Increased heart rate
• Low urine output
• High respiratory rate
• low blood pressure
73. 75. An 8 year old child is being monitored in Intensive Care Unit after trauma. A return to
hemodynamic normality is indicated by
• Cold extremities
• Decreased pulse-pressure
• Pale skin colour
• Return of peripheral pulses*
• Cloudy sensorium
74. 75 A 14 year old boy fell down the stairs & is brought to the emergency with an obvious
deformity indicating fracture of the thigh bone. The most important thing to do in Emergency
Dept is
• Send patient for x-rays
• Apply skin traction
• Maintain IV line
• Give Injection for Analgesia
• Neurovascular assessment of the limb*
75. 77. An 20 year old man insisted on driving a car alone despite his family telling him no to. He
drove the car carefully but still hit another car. What was not the contributing risk factor in
this accident:
• Slower reaction time
• Larger blind spot
• Limited cervical mobility
• Driving during day time. *
• Decreased hearing
76. 78. A plater cast was applied for a 35 year old man for an undisplaced fracture of left tibia,
Discharged next day, he reported to OPD after one week with severe pain in his left leg with
altered sensation in the foot. On examination, the leg was tense & had severe pain on
dorsiflexion of the foot. Peripheral pulses were heard with the hand held Doppler. The most
likely diagnosis is
• Limb ischemia
• Tight plaster
• Compartment syndrome*
• Displacement of fracture
• Nerve compression
77. 79. A 52 year old patient presented with a swelling at the angle of the mouth on the right side.
There is significant deformity of the external ear, making it appear to have been pulled back.
There is no history of fever, body aches or trauma. What is the most probable diagnosis?
• Dermoid Cyst
• Lipoma
• Mumps Pleomorphic
• Adenoma of the Parotid*
• Parotid Abscess
78. 80.A 13 year old boy had a history of fall 2 weeks ago. There is a scar mark just above the
sternal notch. The wound appears to have been healing well initially, but it currently
deformed. It appears to be extending beyond the edges of the wound What is the most
probable diagnosis?
• Hypertrophic scar
• Keloid*
• Abrasion
• Hematoma
• Contracture
79. 81. A 14 year old male patient presented with a 2cm x 2cm swelling on the back. The swelling
is oval, raised and the surrounding skin is normal in color. There is a punctum seen on the
swelling that is oozing out cheesy material which is also foul smelling. There is no tenderness
or fever. What is your most probable diagnosis?
• Lipoma
• Hematoma
• Dermoid Cyst
• Sebaceous cyst *
• Abscess
80. In which layer of the skin, does the above lesion reside?
• Epidermis
• Dermis*
• Hypodermis
• Subcutaneous tissue
• Muscular layer
81. What is the best mode of treatment for the above mentioned patient:
• Incision and Drainage
• Application of regular dressings
• Surgical Excision under
• A Surgical excision under LA*
• Observation
82. 4. An 8 months old infant presented with small, hard, keratin retention cysts distributed all
over the chest and the back. There is no history of fever, fits or sun exposure. What is your
most probable diagnosis?
• Epidermal
• Lentigo
• Milia *
• Naevi
• Impetigo
83. 5. A 44 year old female presented with multiple coffee colored macules of variable sizes
distributed all over her body. She is a known case of Neuro-Fibromatosis type 1 What is your
most probable diagnosis?
• Spindle cell naevus
• Purpura Fulminans
• Verruca Vulgaris
• Café-au-lait spots *
• Ephelis
84. 6. A 65 year old smoker, presented with an irregular lesion on the edge of the nose, more
towards the left side. The lesion has been present for more than 2 years, and has gradually
increased in size over time. The edges appear to be rolled. What is the most likely diagnosis?
• Basal Cell Carcinoma*
• Squamous cell carcinoma
• Melanoma
• Marjolin Ulcer
• Hemangiomata
85. 7. A 34 year old Australian surfer presented with an irregular lesion on the left forearm. The
lesion is raised, cauliflower like and has grown significantly over a period of 5 months. What is
your most probable Diagnosis?
• Basal Cell Carcinoma
• Squamous cell carcinoma*
• Sun burn
• Marjolin Ulcer
• Lipomatosis
86. B.A 56 year old model presented with a lesion on the neck which is concerning her. She
initially it, thinking it was a mole, but over a period of time, the lesion has gotten darker and
the ignored it, edges have gotten irregular. The lesion as per the patient is now twice as big
and raised as well.What is your most probable diagnosis?
• Basal Cell Carcinoma
• Squamous cell carcinoma
• Melanoma*
• Cysts
• Hemartoma
• E Neurofibroma
87. A patient presented with a history of burn on the torso which occurred over 10 years ago The
wound is not healing and appears to have formed an ulcerative lesion?
• Basal Cell Carcinoma
• Squamous cell carcinoma
• Naevus
• Marjolin Ulcer*
• Adenoma
88. An 87 year old male smoker, presented to the OPD with a raised, dark, regular lesion on the
neck. A diagnosis of melanoma is made. What is the specific type of melanoma in this case?
• Superficial spreading Melanoma
• Nodular melanoma*
• Lentigo maligna melanoma
• Acral Lentigious melanoma
• Desmoplastic melanoma
89. 91. A nodular melanoma is diagnosed in a 76 year old male patient on the scalp. A decision to
remove the lesion is taken. What is the most important factor in regards to the prognosis in
this patient?
• Age of the patient
• Site of the lesion
• Depth of the lesion*
• Color of the lesion
• None of the above
90. 92 A 14year old patient presented with a history of burns 3 months ago on the left forearm.
The wound had developed in to a contracture and is irregular in shape limiting complete range
of movement What is the best mode of treatment?
• Free flap
• Release of contracture. *
• Physiotherapy
• Skin grafting
• Skin expansion
91. 93. A 22 year old soccer player attained an injury to the left heel. There is a defect of 3cm x
3cm and the calcaneum is exposed. What is the best mode of treatment?
• Free flap
• Advancement flap
• Islanded pedicle flap*
• Skin grafting
• Skin expansion
92. 94. A 33 year old female presented with 2nd degree burn that occurred 3 months ago. It
appears that the wound has healed but it is lacking the superficial skin layer. The defect is
around 10cm x 13 cm What is the best mode of treatment?
• Free flap
• Advancement flap.
• Islanded pedicle flap
• Skin grafting*
• Skin expansion
93. 95. A man presented with a defect on the scalp, which is due to removal of a previous
pigmented cancer. The wound is healthy. The defect is oval and around 4cm x 4cm. What is
the best way to cover the defect?
• Free flap
• Advancement flap
• Islanded pedicle flap
• Rotational flap*
• Skin expansion
94. A patient has stage 2 breast cancer. She is going to undergo Radical Mastectomy in a week.
She wishes a reconstruction of the area in the same surgery. Which is the best option?
• Skin grafting
• Advancement flap
• Latissimus dorsi free flap
• Islanded pedicle flap *
• Skin expansion
95. 22 year old male presented with a history of pain in the jaw while eating. There is a swelling in
the region of the angle of the mandible. He also complains of fever for the last 3 days. What is
the most probable diagnosis?
• Dermoid Cyst
• Lipoma
• Sebaceous cyst
• Pleomorphic Adenoma of the Parotid
• Submandibular lymph node*
96. A 43 year old female presented with a swelling on the arm. The swelling is around 4cm x 4cm,
soft, non-tender and is not fixed to the skin. There is no visible punctum. Slip sign is positive.
What is your most likely diagnosis?
• Dermoid Cyst
• Lipoma*
• Mumps
• Pleomorphic Adenoma of the Parotid
• Submandibular lymph node
97. A patient underwent minimal invasive surgery. What site can be used for NOTES?
• Umbilicus
• Nose
• Urethra
• Vaginal*
• Throat
98. A male presented with trauma right hypochondrium and was received with no pulse and
Feeble brachial pulses. His abdomen was generalized tender and rigid. After resuscitation
which investigation may help in diagnosis?
• Contrast enhanced CT abdomen
• X ray abdomen
• MRI scan abdomen*
• Laparoscopy
99. A boy had left supracondylar fracture humerus. He has no palpable pulses. What should be
ideal management?
• Reduce fracture till pulse comes
• POP cast of the limb
• Open reduction & Internal fixation *
• Closed reduction & internal fixation
100. An old lady fell in bathroom with her arm extended. When received in emergency she had.
wrist drop left hand. There is bruise in axilla. What structure is damaged?
• Ulnar nerve
• Radial Artery
• Brachial plexus*
• Axillary nerve
101. A man received multiple stab Injuries abdomen and chest. He had acute abdomen and
presented in shock in emergency. On exploration was ruptured descending colon at 2 points
with intervening gangrenous part. There was stomach perforation and ileal perforation, Small
left kidney laceration. What management should be done?
• Repair all perforations & laceration
• Exteriorize the lleum after other structure repair and nephrectomy
• Exteriorize the colon after other structure repair with repair of kidney
• Primary repair of colon and exteriorize ileum
• Resect the segment of colon and exteriorize it after repairing other injuries *
102. A man was anesthetized for thyroidectomy. He had short neck. What is commonly used way
for intubation?
• Nasal intubation
• Oral intubation
• Small Endotracheal tube
• Guide wire guided intubation
• Fiberoptic intubation *
103. The anesthesia of choice for perianal fistulectomy is?
• Ketamine
• Spinal anesthesia
• Saddle block*
• Caudal block
• Epidural anesthesia
104. A patient with right diabetic foot and gangrenous big toe was due for amputation of big toe:
What type of anesthesia should be given?
• Ankle block
• Baer's block
• Spinal anesthesia
• Local ring block *
• I/V anesthesia
105. A patient presented with history of tight knee osteoarthritis. There is fever 3 days and joint
fluctuation positive at knee confirmed on knee tap test. What has happened?
• A Fluid around knee
• Bursitis of knee
• Septic arthritis*
• Cellulitis of knee
• Cruciate ligament inflammation
106. A female with chronic pain right hip landed with fall in home. Hip is externally rotated and
found on X ray. Subsequently she developed AVN. What can be the cause of fracture neck of
femur AVN in this case?
• Femoral artery and medial circumflex
• Artery of ligamentum teres. *
• Circumflex humoral artery
• Branch of popliteal artery
• Artery of the trochanter
107. A male had fracture right tibia and fibula with right foot drop. To confirm nerve damage what
investigation should be done?
• Vasculography
• Electromyography:
• Nerve conduction study *
• Clinical examination only
• Nerve stimulation
108. A man has to undergo esophagectomy for cancer. Anesthetist wants to keep patient pain
free. What can be done for this patient?
• Give Bupicaine injection
• Put Spinal block
• Give Epidural block*
• Celiac block
• Local wound anesthesia
109. A 3 year boy fell from roof top. He had left femur shaft fracture. Distal neurovascular bundin
is intact. What is the management?
• A Apply skin traction
• Apply POP cast till thigh
• Apply back slab to thigh
• Give Hip Spica to patient *
• Only manage with pin traction.
110. A man came with 3 fingers lacerated and tendons incised. What is his management?
• Repair the laceration and send him to plastic surgeon
• Repair tendon in emergency with vicryl
• Admit and repair tendons with prolene. *
• Leave the wounds open for washing dirt
• Repair tendons with silk suture
111. A woman who had fracture of right femur after a trivial injury came in emergency X ray of
bone showed lytic lesion. Her breast examination showed advanced CA right breast. What
should be done for patient
• Skin traction to femur and surgery on breast at earliest
• Biopsy of CA breast and shift to oncologist
• Operate femur and breast at same time
• Manage breast and femur for palliation
• Fix femur and send patient after biopsy for palliation*
112. In electrolyte imbalance in which insulin drug is required for treatment patient?
• Hypokalemia
• Shift of oxyhemoglobin dissociation to the right
• Hyperkalemia*
• Hypercalcemia
• Hyperchloremia
113. Inferior thyroid artery is a branch arises from
• Internal carotid artery..
• External carotid artery
• Lingual artery
• Thyrocervical trunk*
• Facial artery
114. 137. A 60 years of old lady sustained burn to both arm front of chest and abdomen. What is
the burned surface area in this patient
• 36%*
• 40%
• 50%
• 55%
• 60%
115. 118 A 60 years old lady 70 kg of weight sustained 25% burns. What will be his initial fluid
requirement for initial 24 hours
• 4000m
• 5000ml
• 6000ml
• 7000ml*
• 8000 ml
116. 119 A patient was diagnosed having Onychogryphosis which is
• An absent nail
• A broken nail
• An infected nail)
• An Ingrowing nail
• An overgrown curved nail*
117. 120 A 20 years old patient had appendectomy 3 days back. He is complaining of pain swelling
and discharge from the wound. His temperature is 38. What is best treatment option in this
patient
• Antibiotics
• Analgesics
• Proton pump inhibitor
• Open the wound stitches*
• Wound dressing
Surgery B
Musculoskeletal System
1. A wrestler was brought to A & E department as he was unable to move left shoulder joint.
Commonest dislocation of shoulder joint is:
• Lateral
• Anterior*
• Superior
• Posterior
• Inferior
2. 90 years old male with fracture neck of femur complete separation of fragments on lateral view
of the x ray. He is the candidate for following treatment options:
• Hemiarthroplasty and bed rest
• Conservative management with skin traction
• Fracture fixation with screw and side plate (DHS).
• Fracture fixation with screw and immediate partial weight bearing.
• Total hip replacement within 24 hours and mobilization next day*
3. Long thoracic nerve damage causes paralysis of:
• Serratus anterior*
• Deltoid
• Pectoralis Minor
• Pectoralis major
• Brachioradialis
4. Uneducated parents are worried about mid-thigh swelling of their 1 week old baby with H/O
home delivery. What can be the most likely cause of swelling?
• latrogenic fracture shaft of femur during delivery*
• Osteogenic tumor
• Congenital anomaly of the femur
• Soft tissue hematoma due to vascular injury
• Acute bone infection
5. A middle aged man reports to orthopedic outdoor with a swelling over posterior aspect of right
elbow joint for the last 3 years. He was diagnosed to be having Olecranon bursitis, which is
also known is:
• Golfer's elbow
• Clergyman's elbow
• Tennis elbow
• Student's elbow*
• Cricket elbow
6. Brachial artery injury most commonly occurs due to:
• Distal forarem fracture
• Upper arm fracture
• None
• Proximal radius fracture
• Supracondylar fracture humerus*
7. Complication of anterior shoulder dislocation include all except:
• Radial nerve injury*
• Axillary nerve injury
• Fracture greater tuberosity of humerus
• Recurrent dislocation
• Fracture neck of humerus
8. 16 year old boy was operated in March 2021 to fix his mid-shaft fracture left humerus, post
operatively he noticed that he cannot dorsiflex his left wrist joint. During surgery tourniquet
was applied but none of the nerve was explored, which of the following nerves could have
been damaged during surgical procedure resulting into left wrist drop?
• Anterior Interosseous nerve.
• Ulnar nerve
• Musculocutaneous nerve.
• Posterior Interosseous nerve.
• Radial nerve*
9. A 60 year old lady who fell down on an out stretched hand came to ER with a complain of pain
and swelling in right wrist joint, clinically she had dinner fork deformity of right wrist joint
with intact neurovascular status. Which one of the following imaging studies is the most
informative method to the injury?
• MRI of right forearm.
• CT Scan with 3D reconstruction.
• Simple biplane X-Ray of right wrist joint*
• Bone scan.
• Ultrasound soft tissue
10. A new born baby was brought by their parents in orthopedic OPD with a swelling in the left
thigh with 48 hour duration which of the following can be the most likely cause of this
swelling?
• Hematoma.
• Congenital malignant bone tumor.
• Compartment syndrome.
• Birth trauma resulting into fracture shaft of femur.
• Child abuse
11. Which one of the following definitive diagnostic modality is helpful in the management of
musculoskeletal tumor?
• Simple Bi plane X-Ray of the involved region.
• MRI of the involved region.
• C.T scan and Bone scan of the involved region.
• Biopsy of the lesion with histopathology of the specimen*
• soft tissue
12. Young patients presenting without history trauma but having a high grade temperature
swelling and reluctance to move the limb with raised TLC and ESR usually have which
following diseases
• Osteogenic sarcoma of one of the long bone.
• Acute osteomyelitis of the metaphyseal region of the long bone*
• Ewing's tumor.
• Ricketts (Due to vitamin D deficiency).
• Chronic painful bone
13. Fracture clavicle is usually managed by which of the following modalities of treatment?
• 90% fractures of clavicle are treated conservatively
• Every fracture of clavicle should be fixed by ORIF
• Fracture of clavicle managed by figure of 8 bandage is still the method of choice
• None of the above
• Intramedullary injection
14. Degenerative osteoarthritis of the large joint is common in the following major joint of the
body:
• Shoulder joint and Acromioclavicular
• Ankle joint and distal tibiofibular joint
• Tibiofemoral joint and patellofemoral joint (knee joint)
• Elbow joint and proximal radio-ulnar joint.
• Hip joint acetabular area
15. Nonunion of the long bones is common in
• Fracture of the shaft (diaphysis)*
• Fracture of the metaphyseal region.
• Fracture of the growth plate of the long bones
• Fracture of the epiphyseal area of the long bone
• Fracture distal end of bone
16. Ewing's sarcoma of the bone in teenage patients resembles with.
• Chronic Osteomyelitis and Osteomalacia.
• Osteoporosis and Osteoarthritis.
• Acute Osteomyelitis and Osteogenic Sarcoma*
• Fresh displaced fracture of the long bones.
• TB of the bone
17. What is the most common injury is an ankle sprain?
• Fracture base of fifth metatarsal
• Deltoid ligament injury.
• Ankle fracture.
• Anterior talofibular ligament injury*
• Triangular ligament at ankle
18. What is the most common fracture around the elbow joint in children?
• Monteggia fracture dislocation
• Supracondylar fracture humerus*
• Fracture neck of radius
• Fracture lateral epicondyle of humerus
• Fracture of ulna
19. Fracture shaft of femur in children under 12 years of age is usually managed by which one of
the following modality of treatments?
• Closed intramedullary interlocking nail
• Dynamic compression plating
• Fracture can be stabilized with K wires
• Hip Spica involving one and half leg
• Skin traction & Spica*
20. The most common dislocation of the Hip joint in young adults is:
• Posterior dislocation of Hip*
• Posterior dislocation with fracture acetabulum
• Central fracture dislocation Hip.
• Interior dislocation with fracture neck of femur
• Lateral dislocation with tear of nerve
21. The early (within 24-72 hours) most common complication of the fracture shaft of femur in
young adults is?
• Acute osteomyelitis.
• Hematoma formation.
• Compartment syndrome.
• Fat embolism.*
• Vascular thrombosis
22. Closed undisplaced fracture of the shaft of the humerus is best treated by:
• Closed reduction and shoulder spica
• Intramedullary nail*
• Plate and screw fixation
• U-shaped plaster cast & sling
• Skeletal traction
23. The most common site of fracture of clavicle is:
• Medial end
• Lateral end
• Mid clavicle*
• Junction of Medial two third and Lateral third
• Junction of Lateral two third and Medial third
24. Commonest complication of untreated Extra-capsular fracture neck of femur is:
• Non union
• Malunion
• Avascular necrosis*
• Delayed union
• Pulmonary embolism
25. A line joining the highest point of iliac crests passes through
• Lumbar vertebra 1
• Lumbar vertebra 2
• Lumbar vertebra 4*
• Thoracic vertebra 12
• Sacral vertebra 1
26. The most preferred treatment fracture neck of femur in a young patient is:
• Hemiorthroplasty
• Total Hip Replacement
• Conservative treatment
• Closed reduction & internal fixation*
• Open reduction & internal fixation
27. The most common injury In 7 year old due to fall on outstretched and
• Dislocation of shoulder
• Colle's fracture*
• Clavicle fracture
• Soft tissue injury
• Supracondylar fracture of humerus
28. A 76 year old female with history of fall 2hrs back presented in emergency with pain in the
groin that get worse when attending doctor tried to rotate it. Her leg appears shorter than
uninjured leg and externally rotated. The most probable diagnosis is:
• Fracture neck of femur*
• Hip dislocation
• Per-trochanteric fracture of femur
• Sciatic nerve injury
• Fracture femur
29. Fracture lateral condyle of the humerus is a common injury In children. What is the ideal for a
displaced fracture lateral condyle of humerus in a 7 year old child?
• Open reduction & immobilization
• Cast immobilization
• Closed reduction & plaster immobilization
• Open reduction & internal fixation*
• Excision of fractured fragment
30. A 38 year old male presenting in emergency with history of road traffic accident. He has
deformity of arm with inability to extend his wrist. His x-ray arm shows:
• Fracture shaft of humerus
• Fracture clavicle
• Elbow dislocation*
• Shoulder dislocation
• Normal study
31. For a 12 year old girl with a displaced, closed fracture of olecranon and no neuromuscular
deficit, the treatment is:
• Above elbow cast in extension
• Skeletal traction
• Open reduction & internal fixation
• Excision of olecranon fragment
• Above elbow cast in elbow flexed*
32. An 8 year old child, sustained supracondylar fracture of humerus, underwent closed reduction
& percutaneous pin fixation. His arm was Immobilized in flexion. Later on, he developed
severe pain on extension of his fingers. to: Next step In management is
• Remove the splint & extend the elbow
• Measure compartment pressure*
• Doppler's ultrasound
• Arteriogram
• Fasciotomy of forearm
33. 70 yr old lady presented in OPD with h/o fall on outstretched hand one day before and
complain of pain, swelling and dinner fork deformity of Rt wrist. X-ray wrist confirmed
fracture. Most likely diagnosis is: (HBS)
• Displaced fracture scaphoid
• Colle's fracture*
• Smith fracture
• Lunate fracture dislocation
34. A 51 yr old lady sustained a comminuted fracture of radial head. Initially she was splinted in
back slab. 10 days later, she presented in OPD, what treatment option is recommended for
her: (HBS)
• Excision of radial head
• External fixator
• Open reduction and internal fixation*
• Continue with splint
35. A 42 yr old male presents with fracture of distal third radius and tenderness at distal
radioulnar joint. The fracture is named as: (HBS)
• Smith fracture
• Colle's fracture
• Galeazzi fracture*
• Monteggia fracture
36. A 4 yr old male child presented in emergency with h/o fall. He has pain in Rt shoulder. X ray
revealed displaced fracture of clavicle. What is the preferred treatment? (HBS)
• Closed reduction and K wire fixation
• Sling immobilization*
• Open reduction and plating
• Open reduction and suture fixation
37. A 17 yr old university student with intact neurovascular Rt upper limb underwent surgery
(DCP) for fracture Rt humeral shaft. She was worried postoperatively due to weak grip and
inability to dorsiflex the wrist. Most likely the reason is (HBS)
• Combined injury to median and radial nerve
• Injury to median, ulnar and radial nerve
• Isolated injury to median nerve
• Isolated injury to radial nerve*
38. Monteggia fracture dislocation is: (HBS)
• Fracture shaft proximal ulna with radiohumeral joint dislocation*
• Fracture proximal radius with proximal radioulnar joint dislocation
• Fracture coranoid process with elbow dislocation
• Fracture shaft ulna with ulnohumeral joint dislocation
39. Supracondylar fracture of humerus in children is: (HBS)
• K wire fixation is recommended X in undisplaced fracture
• A common fracture in this age group*
• Not associated with compartment syndrome of forearm
• Is always treated conservatively
40. A 64 yr old woman fell from horse while riding. She developed pain in her shoulder and
difficulty in moving her limb. X-ray show fracture proximal humerus. Her ideal treatment is:
(HBS)
• Immobilization with U slab
• Closed reduction
• Cuff and collar sling
• Open reduction and internal fixation*
41. 19 year old boy had fracture right Femur and tibia 10 days before. He was operated within 24
hours. Both fractures were fixed with DCP He became unconscious post operatively. Which
one is the most likely cause: (HBS)
• Associated head injury
• Complication of spinal anesthesia
• FAT embolism*
• Encephalitis
• Drug reaction
42. Open fracture shaft of femur will be managed with following protocol, which is not true of this
protocol? (HBS)
• Wound debridement under aseptic conditions.
• Fracture stabilization with external fixator.
• Take swab for C/S and start antibiotic
• Stitch wound immediately after debridement*
• If required manage wound with VAC dressing.
43. Most appropriate treatment for close fracture shaft long bones is: (HBS)
• MUA + cast immobilization
• Immobilization with traction and counter traction
• Close I/M Interlocking nailing*
• Dynamic compression plating (DCP)
• Immobilization with external fixator.
44. Popliteal artery injury at the level of distal femur is commonly due to: (HBS)
• Pressure of distal fragment of supra condylar fracture femur*
• Tight plaster
• Skeletal traction, pin passing through distal femur
• Pressure of proximal fragment of the femur
• Hematoma
45. 80 year old lean, thin lady presented in OPD on wheelchair with history of stumble while
washing hands in the washroom basin. She is unable to put weight on her left lower limb. X-
ray facility is not available due to technical reasons. She is on bisphosphonate for last 10
years. What is most likely provisional diagnosis? (HBS)
• Hip dislocation
• Fracture left superior and inferior pubic rami.
• Soft tissue injury around hip.
• Fracture neck of femur.
• Inter trochanteric fracture left Hip*
46. Common due to anterior dislocation of hip (HBS)
• Patient presents with foot drop*
• NCS is not helpful in diagnosis of type of nerve injury
• Nerve repair should be done in type 2 injury
• Wait and see is the policy to manage in every case.
47. 15 year old boy presented with history of pain, swelling left thigh. He was running high grade
fever and refused examination of the limb due to severe pain. Before doing further
assessment the provisional diagnosis maybe: (HBS)
• Ewings sarcoma
• Osteogenic sarcoma
• Acute osteomyelitis*
• Chronic osteomyelitis
• Soft tissue hematoma.
48. Most common site of fracture in mandible is? (HBS)
• The neck of the condyle*
• The angle of the mandible
• The region of the canine tooth.
• Mid of Mandible
• At the level of incisors
49. An abscess is a collection of pus. Brodie's abscess involves: (HBS)
• Bone*
• Brain
• Breast
• Liver
• Lung
50. The commonest bone tumor is: (HBS)
• Chondroma
• Fibroma
• Metastatis*
• Osteoma
• Sarcoma
51. A five year old boy is having high grade fever and swelling of right leg for the last two days.
Acute osteomyelitis in children usually begins at: (HBS)
• Diaphysis
• Epiphysis
• Metaphysis*
• Periosteum
• Synovium
52. Commonest dislocation of hip joint is: (HBS)
• Anterior
• Inferior
• Lateral
• Posterior*
• Superior
53. The commonest location of osteogenic sarcoma is: (HBS)
• Around wrist
• Around elbow
• Around hip
• Around knee*
• Around ankle
54. A 'claw hand' is usually associated with injury to which of the following nerves?
• a Axillary nerve
• b Musculocutaneous nerve
• c. Radial nerve
• d. Median nerve
• e Ulnar nerve*
55. Which one of the following does NOT occur as a systemic manifestation of inflammatory
bowel disease.
• & Ankylosing spondylitis
• b. Proderma gangrenosum
• Scleritis
• Osteoporosis*
• Sclerosing cholangitis

GIT (Upper & Lower)


1. A 65 yrs old lady presented with history of rectal bleeding tenesmus and weight loss of 6
months duration. Digital rectal examination revealed growth in the lower rectum 4cm from
anal verge. Rectal biopsy confirms it to be adenocarcinoma rectum. On CT scan stage is
T2NOMO Tumor. Which is the best treatment option?
• Conservative treatment
• Sub mucous resection
• Chemoradiation
• Anterior resection
• Abdominopreaneal resection*
2. According to TNM classification, carcinoma of colon, which has crossed muscularis propria is:
• T2*
• T4
• TO
• T3
• T1
3. Mallory-Weiss syndrome is:
• Type of nephrotic syndrome
• Esophageal cardiac tear*
• Congenital cardiac anomalies
• Congenital diaphragmatic hernia
• Prominent umbilical veins
4. Stomach is supplied by all the following arteries except
• Left gastric artery
• Short gastric arteries
• Right gastric artery
• Right gastro-epiploic artery
• Right hepatic artery*
5. Which of the following malignancies can be cured by radiotherapy alone?
• Lymphoma
• Breast carcinoma
• Squamous cell carcinoma*
• Leukemia
• Esophageal carcinoma
6. The medical student presented a case of 70 year male with h/o persistent painless deep
jaundice for 2 months without pyrexia, palpable gall bladder in RUQ, the ultrasound scan does
not reveal gall stones, he implies this clinical feature as:
• Murphy's sign
• Courvoisier's law*
• Balance' sign
• Reynold pentad sign
• Charcoat's sign
7. A lady with gall stones is having biliary colic. Her pain would be radiating towards:
• Left iliac fossa
• Right Iliac Fossa
• Right shoulder*
• Umbilicus
• Hypogastrium
8. A 20 years old male patient presented with history of headache palpitation and sweating of one
month duration. His serum catecholamine level is raised. Which is the best test to localize
pheochromocytoma in this patient?
• CT Scan Abdomen
• MIBG scan
• MRI Abdomen*
• Ultrasound Abdomen
• X-ray Abdomen
9. Porcelain gall bladder is:
• Strawberry shaped speckles in gall bladder
• Premalignant condition of gall bladder
• Calcium oxalate stones*
• Cholesterol crystals in gall bladder
• Benign condition of gall bladder
10. A 40 year obese lady presents with recurrent attacks of right upper abdominal pain, the pain is
of short duration spreads in upper abdomen and comes after fatty foods, her clinical
examination is unremarkable, your likely clinical diagnosis is?
• Biliary colic*
• Acute Cholecystitis
• Acute Pancreatitis
• Sub-acute hepatitis
• Chronic gastritis
11. Alvarado score is used to diagnose:
• Cholecystitis
• Terminal ileitis
• Pancreatitis
• Appendicitis*
• Hepatitis
12. 45 years old female underwent routine cholecystectomy 3 days ago, brought to ER with
increasing abdominal pain and constipation. On examination she looks unwell, jaundiced and
tender abdomen with green color discharge from wound. Her vitals are stable. Your most
probable clinical diagnosis would be:
• Biliary leakage*
• Acute cholangitis
• Bowel perforation
• Portal vein injury
• Wound dehiscence
13. Left sided Virchow's lymph nodes are important in relation with:
• Carcinoma spine
• Carcinoma anal canal
• Carcinoma stomach*
• Carcinoma prostate
• Carcinoma bone
14. A middle aged smoker has undergone upper Gl endoscopy for chronic dyspepsia. Endoscopist
has taken multiple biopsies from a growth in the antrum. Gastric carcinoma is associated with
blood group:
• AB
• A*
• O
• Both A & O
• B
15. Following is NOT a complication of acute appendicitis
• Appendicular mass
• Adenocarcinoma appendix*
• Pelvic abscess
• Pylephlebitis
• Appendicular abscess
16. In acute appendicitis patient is feeling pain in right iliac fossa while palpating on left iliac fossa.
What is this sign called?
• Kocher's sign
• Rovsing's sign*
• Obturator's sign
• Cough sign
• Kehr's sign
17. The most common cause of small intestinal obstruction is:
• Malignancy
• Crohn's disease
• Adhesions*
• Tuberculosis
• Hernia
18. Hippocratic facies is associated with:
• Diffuse peritonitis*
• Dehydration
• Renal failure
• Burns
• Bell's palsy
19. Murphys' sign is present in:
• Pyelonephritis
• Peritonitis
• Ac Cholecystitis*
• Appendicitis
• Gastritis
20. 45 years old female with long history of pain right hypochondrium presents in emergency with
sudden colicky abdominal pain, distension, vomiting and constipation. X ray plain abdomen
shows dilated small bowel loops with air in biliary tract and radio opaque shadow in RIF. Most
probable diagnosis would be:
• Perforated peptic ulcer
• Acute pancreatitis
• Acute cholangitis
• Choledochal cyst
• Gallstone ileus*
21. Familial adenomatous polyposis
• More than 100 polyps on colonoscopy is diagnostic*
• It only affects colon
• Autosomal recessive disease
• Surgery should be done before age of 18 years
• Conversion to malignancy is less likely
22. 20 years male patient presented with history of dysphagia of 1 year duration his barium
swallow revealed bird beak appearance. What is the likely diagnosis in this patient?
• Cardia Achalasia*
• Barret's Esophagus
• Hiatus Hernia
• Corkscrew Esophagus
• Carcinoma Esophagus
23. A 27-year-old man presents to the emergency department with a 6-hour history of upper
abdominal pain radiating to the back and associated vomiting. On examination, he has
marked epigastric tenderness and you notice a bluish discoloration around his umbilicus. His
heart rate is 118/ min and blood pressure 108/76 mmHg. Which of the following blood test
would be most useful in identifying the diagnosis
• Amylase*
• C-reactive protein
• Hemoglobin
• Sodium
• Urea
24. A 64-year-old woman presents with a 2-day history of increasing left side abdominal pain with
fever. On examination, she has localized peritonism in the left iliac fossa. Her blood tests
reveal a raised white cell count and C-reactive protein. Which of the following is the most
likely diagnosis?
• Constipation
• Diverticular disease
• Diverticulitis*
• Diverticulosis
• Irritable bowel syndrome
25. A 72-year-old woman presents to the emergency department with a 5-hour history of
abdominal pain. bloating and vomiting. On examination, the abdomen is distended and
auscultation reveals intermittent high-pitched sounds. An abdominal X-ray shows air in the
biliary tree. What is the most likely
• Emphysematous cholecystitis
• Gallstone ileus
• Paralytic ileus
• Perforated gallbladder
• Pyogenic cholecystitis
26. A 40-year-old woman presents to the emergency department with a 4-hour history of right
upper abdominal pain and fever. She tells you that she has previously been investigated for
abdominal pain and was found to have gallstones. She says that she has never felt this unwell
with the pain before. On examination, she is markedly jaundiced with a temperature of 39°C
and pulse rate of 130/min. There is localized peritonism in the right upper abdomen. Which of
the following complications of gallstones has dis patient presented with?
• Acute cholecystitis
• Ascending cholangitis
• Biliary colic
• Carcinoma of the gallbladder
• Gallstone ileus
27. A 33-year-old man presents with a 2-hour history of upper abdominal pain. On examination,
he is sweating and his abdomen is rigid. His examination revealed heart rate 106/min and
blood pressure 98/58 mmHg, His x ray chest shows free air under right hemi diaphragm.
Which of the following is the most likely cause of his symptoms?
• Epigastric hernia incarceration
• Gastric carcinoma
• Gastritis
• Hiatus hernia
• Perforated peptic ulcer
28. A 54-year-old woman has been having episodes of epigastric pain associated with food. She is
referred for an endoscopy which shows gastric ulceration. A rapid urease test performed on a
biopsy sample is positive. Which of the following would you use to treat this patient?
• Augmentin and metronidazole
• Omeprazole, clarithromycin and metronidazole*
• Omeprazole, prednisolone and Augmentin
• Omeprazole, ibuprofen and clarithromycin
• Ranitidine
29. A 73 year old mas presents to emergency department with sudden pain right (..) radiating to
the abdomen. He has a past medical history of hypertension and have gallstones. On
examination, he is afebrile, hemodynamically stable and tender at the abdomen. Blood tests
show leukocytosis. What investigation would you perform
• CT abdomen
• Intravenous urogram
• Laparotomy
• MRI spine
• Ultrasound abdomen
30. A 54-year-old man presents with a 2-day history of upper abdominal and shoulder tip pain in
(..) side. He also complains of difficulty breathing and episodes of fever and sweats. He
underwent laparotomy 2 weeks ago following a perforated appendix. What is the most likely
diagnosis
• Appendix abscess
• Cholecystitis
• Diverticular abscess
• Polycystic liver disease
• Subphrenic abscess
31. A 24-year-old man, who has been suffering from intermittent fresh bleeding per rectum,
present to the emergency department with a 6-hour history of right-sided abdominal pain,
fever and nausea. On examination, he has tenderness and guarding in the right iliac fossa. His
temperature is 38.2°C. What is the most likely diagnosis?
• Appendicitis*
• Hemorrhoids
• Meckel diverticulitis
• Renal colic
• B. Shigella infection
32. A 38-year old woman who has a history of intermittent right upper quadrant pain associated
with eating, attends the emergency department with a 3-day history of severe, unremitting
abdominal pain and jaundice. Ultrasound scan shows an 8mm stone in her common bile duct.
Which of the following would be most suitable management?
• Cholecystectomy
• ERCP*
• Fragmentation of the stone
• Laparotomy
• MRCP
33. A 34-year-old man presents to the emergency department with a 6-hour history of abdominal
pain and vomiting. On examination, the abdomen is distended and tympanic. He tells you that
for the last 6 months he has been having episodes of cramping right-sided abdominal pain
with intermittent episodes of blood in his stools. Which of the following complications has
resulted in his presentation today?
• Abscess formation
• Fistula formation
• Primary sclerosing cholangitis
• Stricture formation
• Toxic megacolon
34. 27-year-old man presents with fevers and right lower abdominal pain following a 1-week
history of bloody diarrhoea. He has recently returned from holiday in the northern areas but
has previously been fit and well On examination, he has a tender palpable liver Which of the
following is the most likely cause of his symptoms
• Acute cholecystitis
• Amoebic liver abscess
• Fitz-Hugh-Curtis syndrome
• Polycystic liver
• Pyogenic liver abscess
35. An 8-year-old boy is brought into the emergency department by his father with a 1-day history
of fever, right-sided abdominal pain and diarrhoea. On examination, he has a temperature of
39°C and is tender, but not guarding, in the right iliac fossa What is the most likely diagnosis?
• Appendicitis
• Crohn disease
• Coeliac disease
• Gastroenteritis
• Mesenteric adenitis*
36. 69-year-old man presents to the GP with a 2-month history of epigastric pain. He also feels full
after eating small amounts of food and has lost 10 Kg weight over the last 2 months.
Examination is unremarkable. Which of the following is the most likely diagnosis?
• Duodenal ulcer
• Gastric cancer*
• Gastric ulcer
• Menetrier disease
• Pernicious anemia.
37. A 35-year-old woman presents to the emergency department with a 1-day history of colicky
central abdominal pain and vomiting. There is history of absolute constipation. She had an
operation on her abdomen 4 years back. On examination, she looks unwell and moderately
dehydrated. The abdomen is distended and tender with hyperactive bowel sounds. Which of
the following is most likely to have caused her symptoms?
• Adhesions
• Colorectal tumor
• Constipation
• Gallstone ileus
• Paralytic leus
38. A 74 year-old man presents to the emergency department with a 6-bour history of colicky
lower abdominal pain accompanied by gross abdominal distension. He has had similar
episodes in the past that have been relieved by opening his bowels. Today he is not even able
to pass flatus. Abdominal X-ray shows a grossly dilated loop of large bowel. Which of the
following is the most likely diagnosis?
• Adhesions*
• Fecal impaction
• Intussusception
• Sigmoid volvulus
• Strangulated inguinal hernia
39. A 37-years-old woman presents to the GP with a 3-week history of frequent, loss motion (…)
and complains of muscle weakness. A subsequent colonoscopy demonstrates a 15cm and
mass growth in the rectum that has multiple projections. The remainder of the colon was
unremarkable
• Adenomatous polyp
• Colorectal carcinoma
• Infective colitis
• Pseudomembranous colitis
• Villous adenoma
40. A 45-year-old woman presents to the emergency department with a 12-hour history of
worsening right upper abdominal pain and vomiting. On examination, Murphy's sign is
positive in the right upper quadrant. Her temperature is 38.1°C. Which of the following is
initial treatment options?
• Cholecystectomy*
• Intravenous antibiotics
• Intravenous steroids
• Intravenous proton pump inhibitor
• Percutaneous drainage of the gallbladder
41. A 30 years female patient presented with pain in the abdomen which started in the umbilical
region but within few hours shifted to right lower abdomen. There is tenderness over the
McBurney’s point What is the likely diagnosis?
• Acute pancreatitis
• Acute appendicitis*
• Acute pyelonephritis
• Acute salpingitis
• Perforated peptic ulcer
42. A 55 years male patient presented with 12 hours history of pain abdomen, vomiting and
constipation. Her abdomen on examination is distended and bowel sounds are exaggerated.
What is the best initial Investigation to diagnose the disease?
• U/S abdomen
• MRI Abdomen
• Colonoscopy
• Enteroscopy
• X-ray Abdomen erect and supine
43. A 45 years old woman has been diagnosed with gallstones. She opts to have an open
cholecystectomy which of the following surgical incision would be most appropriate?
• Gridiron
• Lanz
• Right kochar's*
• Right paramedian
• Rooftop
44. A 56-year-old man is brought to hospital by his wife. He has been vomiting fresh blood since
earlier that morning. She tells you he has a long history of alcohol abuse and drinks at least a
bottle a day. Which of the following is the most likely cause of his symptoms?
• Dieulafoy lesions
• Duodenal ulcer
• Gastric ulcer
• Oesophageal varices*
• Esophagitis
45. A 43-year-old woman presents with bright red fresh rectal bleeding and intermittent rectal
discomfort. She has recently noticed a mass in her rectum. On examination, a non-tender
lump is seen emerging from the rectum. It cannot be reduced. Which of the following would
be the most appropriate treatment?
• Anal dilatation
• Banding
• Hemorrhoidectomy*
• Injection sclerotherapy
• No intervention required
46. A 6-month-old boy is brought into the emergency department by his parents. He appears to
have been having paroxysms of intense abdominal pain since that morning, described by his
parents as episodes of crying and drawing of his legs up to the abdomen. This evening he has
passed some mucus like blood per rectum. His temperature is 36.8°C. Which of the following
would be the best treatment option in the first instance?
• Barium enema
• Continued observation
• Intravenous antibiotics*
• Laparotomy
• Scrotal exploration
47. A 64-year-old woman presents to the GP complaining of pain and difficulty in swallowing. It is
becoming progressively worse and now she complains it is even hard to drink water. On
examination, she appears cachexic. Which of the following is the most likely diagnosis?
• Oesophageal candidiasis
• Oesophageal carcinoma*
• Oesophageal spasm
• Oesophageal stricture
• Oesophageal web.
48. A 56-year-old woman presents to the GP complaining of episodes of severe retrosternal chest
pain after eating, associated with a bitter taste in her mouth. She has no difficulty or pain
when swallowing. Examination is unremarkable. Which of the following investigations would
be most useful in establishing the cause of her symptoms?
• 24-hour lower oesophageal pH
• Barium swallow
• Erect chest X-ray
• Manometry
• Upper gastrointestinal endoscopy*
49. A 36-year-old man presents to the surgical outpatient clinic with a 3-weekhistory of soiled
underwear. He is known to have Crohn's disease. On examination, att opening is seen at the
posterior margin of the anus which is discharging feculent material, Subsequent imaging
demonstrates an anal fistula that tracks through the puborectalis muscle. What would be the
most appropriate management of this condition?
• Diltiazem ointment
• Excision of the fistula*
• Laying open of the fistula tract
• Lord procedure
• Seton insertion
50. A 33-year-old woman presents to the surgical outpatient clinic with increasing dysphagia for
solids and liquids, retrosternal chest pain on eating, regurgitation of undigested food and
weight loss. Oesophageal and gastric biopsy at endoscopy was normal. Oesophageal
manometry showed impaired relaxation of the lower oesophageal sphincter with barium
swallow having dilated proximal oesophagus with distal narrowing. Which of the following
surgical procedures would be indicated?
• Billroth I
• Billroth II
• Heller myotomy*
• Ramstedt procedure
• Whipple procedure
51. A 36-year-old man complains of a 6-month history of intermittent pain in the rectum which is
unrelated to stools. The pain tends to occur at night. He denies passing any mucus or blood
per rectum and examination is unremarkable. Which of the following is the most likely
diagnosis?
• Anal fissure
• Anal fistula
• Perianal abscess
• Perianal haematoma
• Proctalgía fugax*
52. A 73-year-old woman presents to the surgical outpatient clinic with a 3-week history of rectal
bleeding, a sensation of incomplete emptying despite defecation and rectal discomfort. She is
found to have a tumor 2 cm from the anal margin. Which of the following would be the most
appropriate operative intervention?
• Abdominoperineal resection*
• Anterior resection
• Hartmann procedure
• Left hemicolectomy
• Right hemicolectomy
53. A 30 years old male patient presented with hematemesis after severe bouts of vomiting Upper
GI endoscopy revealed longitudinal mucosal tears in esophagus near gastric junction. What is
the likely diagnosis in this patient?
• Boerhaave's esophagus
• Mallory Weiss tears*
• Carcinoma esophagus
• Cardia achalasia
• Reflux esophagitis
54. A girl presented with history of pain and swelling of abdomen. She passed red currant jelly
stools and examination shows a sausage shaped mass near paraumbilical region. What is
likely diagnosis?
• Small intestinal obstruction
• Meckel's diverticulum
• Tumor of gut
• Tumor of stomach
• Intussusception of bowel*
55. An elderly man presented with a history of fall. He is conscious and oriented, following
commands and speaking fluently. He complains of pain in the left hypochondrium. Suddenly
his blood pressure drops and he loses consciousness. What is the most probable diagnosis:
• Hypoglycemia
• Subarachnoid Hematoma
• Epidural Hematoma*
• Splenic rupture
• Intestinal perforation
56. An 80-year-old woman with a known history of femoral hernia is admitted to the hospital
because of strangulation of the hernia. There is a tender swelling in the right femoral region
immediately below and lateral to the pubic tubercle. She has had multiple bowel movements
without relief of symptoms. What is the most likely diagnosis?
• Lymphadenitis
• Diverticulitis
• Volvulus
• Richter's hernia*
• Gastroenteritis
57. A 65-year-old female on her routine examination was noted to have a pulsatile abdominal
mass. She has been otherwise healthy with history of hypertension with no other history,
except family history of father dying of ruptured abdominal aortic aneurysm. What are the
acceptable reasons to operate on abdominal aortic aneurysms in 65-year-old female with 5-
cm infra-renal aneurysm?
• Presence of aneurysm
• Aneurysm with intramural thrombus
• Asymptomatic aneurysm 5.5 cm*
• Associated 2-cm iliac aneurysm
• Patient with splenic artery aneurysm 1.5 cm
58. All of the following are complications of peptic ulcer surgery except;
• Duodenal stump blowout*
• Dumping.
• Diarrhea.
• Delayed gastric emptying.
• Steatorrhoea.
59. Which of the following conditions is associated with the development of esophageal
carcinoma?
• Nutcracker spasm esophagus
• Tracheo-esophageal fistula
• Plummer-Vinson syndrome*
• Zenker diverticula
• None of above
60. Metabolic complications of subtotal gastrectomy with Billroth I or Billroth II reconstruction
include
• Hypothyroidism*
• Anemia.
• Reactive hypoglycemia.
• Dumping syndrome.
• Metabolic bone disease.
61. Which of the following statement is correct for diagnostic study for esophageal carcinoma?
• CT scan is not useful
• A barium swallow is an unnecessary test
• Bronchoscopy should be performed*
• Bone and brain scans should not be obtained
• Endoscopic ultrasound is not necessary
62. Which of the following statement is the best treatment option for palliation therapy of
esophageal carcinoma?
• Radiation therapy*
• Esophageal Stenting
• Endoscopic laser fulguration
• surgical bypass
63. Which of the following statements about the segmental anatomy of the liver is true?
• Segments are subdivisions in the German system.
• Segments are determined primarily by the hepatic bile drainage.
• There are VII segments for hepatic resection.
• Segments are important for understanding the topographic anatomy*
64. The following is true regarding liver blood supply.
• Hepatic artery supplies the same amount of blood as the portal vein.
• Portal vein provides more blood to the bile ducts than the hepatic artery*
• Detoxification is done in liver from portal vein blood.
• Hepatic artery supplies no blood to hepatic metastases
65. Which of the following therapy is most accurate for pyogenic hepatic abscess?
• Antibiotics alone.
• open surgical drainage.
• treat the abscess as well as the underlying source*
• Percutaneous drainage.
66. 45 year man suffered from long standing peptic acid disease, started to have repeated
vomiting and malnutrition with weight loss, the commonly requested investigation Is,
• Gastric 24 hour pH monitoring
• CT scan abdomen*
• Esophageal manometry
• Barium swallow study
• Gastrointestinal endoscopy
67. A 43 year smoker who had been on H-2 blockers presents with projectile non -bilious vomiting
for few weeks containing food residue, O/E be is dehydrated, abdomen Is distended with
positive succussion splash, your most likely clinical diagnosis Is
• Subacute intestinal obstruction
• Chronic colonic obstruction
• Celiac disease
• Gastric outlet obstruction*
• Achalasia of esophagus
68. A 45 year lady presents in ER with right upper abdominal pain and vomiting, with h/o fatty
intolerance she is pyrexial, not jaundiced, murphy's sign is positive, the most likely diagnosis
• Acute pancreatitis
• Acute cholecystitis*
• Acute hepatitis
• Acute gastro enteritis
• Acute appendicitis
69. A Junior doctor clinically diagnosed a 50 Year patient as a case of obstructive jaundice, this
laboratory report is expected
• Decreased serum conjugated bilirubin
• Increased serum unconjugated bilirubin
• Raised serum urobilinogen
• Increased serum alkaline phosphatase*
• Low serum gamma GTP
70. A 50 year farmer presents with continues dull pain in Right upper abdomen for 6 weeks,
examination reveals palpable mass arising from right liver lobe, lab shows eosinophilia and Ct
scan shows right lobe lesion with floating membrane, what is your most likely diagnosis,
• Amoebic liver abscess
• Acute hepatitis
• Heamangioma liver
• Mucoceole gall bladder
• Hydatid cyst*
71. A 28 years old man develops shivering fever, jaundice and pain in upper abdomen. Ultrasound
shows dilated common bile duct with Gall bladder stones. Serum amylase is normal but
billrubin and alkaline phosphatase is raised. The likely diagnosis is
• Acute cholecystitis
• Bile duct tumor
• Acute cholangitis*
• Acute pancreatitis
• Perforated peptic ulcer
72. A 50 years old woman has severe abdominal pain and jaundice, she is apyrexial, stable and
abdominal examination is unremarkable, labs show normal CBC. LFT reveals raised Serum ALP
but normal transaminases. What is the most appropriate next step of management
• Treat conservatively
• CT scan abdomen
• ERCP*
• MRCP
• Upper Gl endoscopy
73. A 30 year male found to have an incidental finding of a liver lesion on ultrasound reported as
benign, the commonest benign hepatic lesion is,
• Adenoma*
• Focal nodular hyperplasia
• Haemangioma
• Lymphoma
• Lipoma
74. 70 year male presented in surgical outdoor as worsening abdominal pain and distension with
weight loss. He has history of chronic hepatitis, o/E he is jaundiced, with a hard Right Upper
Quadrant mass, lab reports AFP Is raised and ultrasound examination reveals liver lesion, His
most likely diagnosis is
• Fibrolamellar carcinoma
• Hepatic nodular hyperplasia
• Hepatocellular carcinoma*
• Intrahepatic cholangiocarcinoma
• Liver cyst
75. A 50 year old Male patient presented with history of hematemesis of 6 hours duration, there
is history of weight loss and anorexia for the last 6 months. On examination patient is pale,
pulse is 120/min. BP 90/60. Select one best Investigation in this patient
• Barium studies upper GIT
• Upper GI Endoscopy*
• CT scan abdomen
• U/S scan Abdomen
• Endoluminal ultrasound
76. A 30 year old male patient presented with history of difficulty in swallowing of 2 years
duration, there is bird beak appearance on Barium swallow. Most likely diagnosis in this
patient is
• Zinkers diverticulum
• Cardia achalasia*
• Carcinoma esophagus
• Hiatal hernia
• Barrett's esophagus
77. A 45 year old female presented with history of central abdominal pain radiating to back
nausea vomiting of 2 days duration. There is previous history of biliary colics. Her serum
amylase is 2300 IU. Most likely cause of her acute pancreatitis is:
• Autoimmune pancreatitis
• Alcoholic pancreatitis
• Drug Induced pancreatitis
• Gall stone pancreatitis*
• Idiopathic pancreatitis
78. A 50 year old male known case of gallstones disease presented with history of pain In right
hypochondrium. She is pyrexial and tender RUQ. USS shows features of acute cholecystitis.
Best treatment option In this patient Is
• Cholecystectomy*
• Radical Cholecystectomy
• Cholecystostomy
• cholecystectomy with CBD exploration
• Treat conservatively
79. The preferred treatment for carcinoma of the gallbladder is T1N0M0: (HBS)
• None of the above
• Chemotherapy
• Radical resection that includes gallbladder in continuity with the right hepatic lobe and
regional lymph node dissection.
• Simple Cholecystectomy*
80. A 40 years lady presented with history of hematemesis after a bout of severe vomiting.
Emergency Endoscopy revealed tear in lower Esophagus mucosa. What is the best treatment
options? (HBS)
• Cold Saline lavage
• Operative treatment
• Conservative treatment
• Injection sclerotherapy*
81. In patient of appendicitis, patient feels pain at right illiac while palpating at left iliac fossa
what is name of this sign? (HBS)
• Murphy's sign
• Kehr's sign
• Pointing sign
• Psoas sign
• Rovsing's sign*
82. Stomach is supplied by all the following arteries except (HBS)
• Right gastroepiploic artery
• Right hepatic artery*
• Right gastric artery
• Left gastric artery
• Short gastric arteries
83. A 40 years old alcoholic male patient readmitted in the hospital with pain and a palpable non
pulsatile mass in epigastrium, after 4 weeks of pancreatitis attack. He is apyrexial. There is
persistent hyperamylasemia. Most likely diagnosis is: (HBS)
• Pancreatic carcinoma
• Panoreatic abscess
• Abdominal aneurysm
• Pancreatic pseudocyst*
• Pancreatic ascites
84. A 45 year old male patient presented with history of dyspepsia of 2 years duration. He has
developed repeated vomiting which is non-bilious for the last 2 months. His examination
revealed succussion splash. What is the most likely diagnosis: (HBS)
• Duodenal Stenosis
• CA Stomach
• Duodenum CA
• Fibrotic Stricture pyloris*
85. A 40 years old patient presented with history of dyspepsia of 4 years duration. His Upper GI
endoscopy revealed ulcer in first part of duodenum. Patient fails to respond to medical
treatment. What is the best surgical treatment option: (HBS)
• Billroth 1 gastrectomy
• Billroth 2 gastrectomy
• Highly selective vagotomy and pyloroplasty*
• Pyloroplasty
• Truncal Vagotomy
86. A 40 year obese lady presents with recurrent attacks of right upper abdominal pain, the pain is
of short duration spreads in upper abdomen and comes after fatty foods, her clinical
examination is unremarkable, your likely clinical diagnosis is: (HBS)
• Acute Cholecystitis
• Subacute hepatitis
• Chronic gastritis
• Chronic pancreatitis
• Biliary colic*
87. A 70 years male patient presented with history of progressive dysphagia of 3 months duration
There is history of anorexia and weight loss for last 2 months. His Endoscopy revealed growth
in thoracic Esophagus. Select one best investigation to stage the tumor. (HBS)
• X-Ray chest
• CT Scan Chest and neck
• Ultrasound chest
• CT Scan chest
• CT Scan Neck Chest and Abdomen*
88. True statement about the surgical management of patients with acute cholecystitis include:
(HBS)
• Antibiotic therapy should be initiated as soon as the diagnosis is made*
• Dissection of the gallbladder is easy in every case
• A per-operative cholangiogram should be done in every patient
• Operation should be performed in X all patients as soon as the diagnosis is made.
89. A 30 year old female patient arrived in emergency room with complain of abdominal pain.
vomiting for 2 days. she has not passed stool yet, abdomen is distended with mild tenderness.
What is first diagnostic investigation? (HBS)
• Ct scan abdomen
• Serum Electrolytes
• MRI
• X-ray abdomen*
• Ultrasound abdomen
90. 10 month old baby was brought to the hospital by mother complaining of screaming of baby
and drawing her legs up. and passing red currant jelly stool. what is the diagnosis? (HBS)
• Volvulus
• Intestinal obstruction
• Inflammatory bowel disease
• Intussusception*
• Enteritis
91. A 60 year old male patient presented with history of jaundice high colored urine clay colored
stool and itching of 3 months duration. His serum bilirubin is 70 micro mol, alkaline
phosphatase is 1200 iu. Upper GI Endoscopy revealed peri ampullary growth. What is the best
treatment option: (HBS)
• Endoscopic Resection
• Gastrectomy
• Dudenectomy
• Whipple operation [whipple/pancreaticoduodenectomy (pylorus preserving)]*
92. A 65 y old male patient visited OPD with complain of BLEEDING PER RECTUM and black
colored stool for 6 months, blackish stool at finger seen on digital rectal examination, double
contrast barium enema shows Apple core lesion, what is your diagnosis? (HBS)
• Hemorrhoids
• Bleeding peptic ulcer
• Carcinoma of colon*
• Tuberculosis of intestine
• Rectal polyps
93. Which of the following is the most common cause of obstructive jaundice in patients with
chronic pancreatitis? (HBS)
• Pancreatic pseudocyst formation
• Adenocarcinoma of the head of the pancreas
• Fibrotic stricture of the common bile duct*
• Choledocholithiasis
94. A 80 male patient presented with history of dyspepsia hematemesis weight loss anorexia of 4
months duration. Endoscopy revealed fungating growth in prepyloric region. Staging of tumor
is T1N0M0. What is best surgical treatment option? (HBS)
• Partial Proximal Gastrectomy
• Radical Gastrectomy
• Partial Distal Gastrectomy*
• Total Gastrectomy
95. Echinococcosis liver disease caused by Echinococcus granulosus: (HBS)
• Is not endemic to parts of Europe and United States.
• Is a neoplasm.
• Is more deadly than in its Echinococcus multilocularis form.
• Is usually curable by resection*
96. A 30 year male patient known dyspeptic presented with history of sudden onset of epigastric
pain of 6 hour duration. His pulse is 120/min BP 100/70 mmHg. Examination of the Abdomen
revealed board like rigidity. What is the likely diagnosis? (HBS)
• Acute Pyelonephritis
• Acute Pancreatitis
• Acute Cholecystitis
• Acute Appendicitis
• Perforated peptic ulcer*
97. Which of the following statement characterizes amoebic abscess? (HBS)
• Patients with amoebic abscess tend to be older than those with pyogenic abscess
• Mortality is higher than that for similarly located pyogenic abscesses
• The treatment of amoebic abscess is primarily medical*
98. 65 years hypertensive man attends surgical OPD the medical student on examination finds
mass in epigastrium, he describes it non tender pulsatile mass measuring approx. 5cm
showing expansile pulsation. What is your most probable clinical diagnosis? (HBS)
• Para-aortic lymphadenitis
• Normal aortic pulsation
• Abdominal aortic aneurysm*
• Thrombosed inferior vena cava
• Gastric tumor
99. A 70 years male patient presented with history of weakness lethargy malaise and exertional
dyspnea of 5 months duration. His examination of abdomen revealed a non tender mass in
right iliac fossa. His Hemoglobin is 7 gm. Which of the following investigation is best to stage
the tumor? (HBS)
• CT Scan Abdomen with contrast*
• MRI Abdomen
• Endoluminal ultrasound
• Ultrasound Abdomen
100. You are assisting a Laparoscopic Cholecystectomy and your consultant asks you about the
most dangerous anomaly in the arterial supply of gall bladder? (HBS)
• Accessory cystic artery
• Cystic Artery lying anterior to cystic duct
• Cystic artery arising from gastroduodenal artery
• Short cystic artery
• Tortuous hepatic artery anterior to cystic duct*
101. Which of the following structures is protected while doing right hemicolectomy? (HBS)
• Common bile duct
• Duodenum*
• Liver
• Pancreas
• Urinary bladder
102. A 35 year old female is being investigated for anemia. After a normal upper and lower
gastrointestinal endoscopy the physician decides to check the small intestine. Which one of
the following will be the investigation of choice? (HBS)
• Barium enema
• Barium follow through*
• Computed tomography
• Barium swallow
• Magnetic resonance imaging
103. Which of the following investigations is done to assess stomach in fluoroscopy? (HBS)
• Barium enema
• Barium follow through
• Barium meal*
• Barium swallow
• Gastrograffin swallow.
104. A 5 year boy has been brought to hospital with six months history of off and on fresh
bleeding per rectum. He is most likely to be having: (HBS)
• Anal fissure
• Hemorrhoids
• Rectal carcinoma
• Rectal polyp*
• Rectal prolapse
105. A middle aged female is having a reducible swelling over left groin for the past four months.
The most common hernia in females is: (HBS)
• Direct inguinal hernia
• Femoral hernia
• Indirect inguinal hernia*
• Obturator hernia
• Umbilical hernia
106. Duodenal diverticulum is commonest in (HBS)
• First & second parts
• Second third parts*
• Third & fourth parts
• First & fourth parts
• First & third parts
107. Foramen of Morgagni is present in: (HBS)
• Bone
• Brain
• Nose
• Diaphragm*
• Skull
108. A 2 months old baby boy is going to be operated for congenital pyloric stenosis. He would be
presenting with which of the following biochemical abnormality: (HBS)
• Acidosis
• Respiratory Alkalosis
• Hypernatremia
• Hyporchloraemic alkylosis*
• Paradoxical alkalosis
109. In case of Littre's hernia, the hernia sac contains: (HBS)
• Appendix
• Meckel's diverticulum*
• Testis
• Ovary
• Bladder
110. The commonest site of aneurysm is: (HBS)
• Ascending aorta
• Descending aorta
• Abdominal aorta*
• Common carotid
• Renal artery
111. The commonest site of perforated duodenal ulcer is: (HBS)
• First part*
• Second part
• Third part
• Fourth part
• Duodeno-jejunal junction
112. A 70 year old male smoker is having progressive dysphagia for which he has under gone
endoscopy, which has revealed a growth in proximal esophagus. Biopsy is likely to show:
(HBS)
• Adenocarcinoma
• Sarcoma
• Squamous cell carcinoma*
• Transitional cell carcinoma
• Anaplastic carcinoma
113. A 40 years old man has had recurrent symptoms suggestive of peptic ulcer disease for 4
years. Gastroscopy reveals an ulcer located in the greater curvature of the stomach and
mucosal biopsy reveals H. Pylori with a positive rapid urease test (RUT). What is true about H.
Pylori? (HBS)
• It can be confirmed by serologic tests in the blood
• It is protective against gastric carcinoma
• It is associated with chronic gastritis*
• It causes gastric ulcer but not a duodenal ulcer
• Treated with a single antibiotic
114. Duodenal diverticulum is commonest in:
• First & second parts
• Second& third parts*
• Third& fourth parts
• First& fourth parts
• First& third parts
115. In case of Litter’s hernia, the hernia sac contains:
• Appendix
• Meckel’s diverticulum*
• Testis
• Ovary
• Bladder
116. Haman papillomavirus (HPV) can cause which of the following malignancy?
• Carcinoma of breast
• Carcinoma of anal canal*
• Carcinoma of esophagus
• Bladder carcinoma
• Gastric carcinoma
117. During a cholecystectory, the cystic artery must be located and ligated. This artery most
commonly arises from the
• Gastroduodenal artery
• Hepatic artery proper
• Right hepatic artery*
• Left hepatic artery
• Superior pancreatico-duodenal artery
118. 27. A 29-year-old patient presents with a short history of right upper quadrant pain. She is
jaundiced with dark urine and pale stool. She has a fever of 38.9 °C Abdominal examination
gives no suggestion of a palpable gallbladder. The diagnosis is
• Ascending cholangitis
• b Acute cholecystitis
• Biliary colic with bile duct obstruction*
• Pancreatitis
• Mirizzi's syndrome
119. 29 A patient was admitted for acute pancreatitis secondary to alcohol misuse. He was
treated conservatively and discharged for outpatient follow-up. He presents to your clinic
after 8 weeks complaining of continued abdominal pain in the epigastrium radiating into the
back Serum amylase is 200 IU/L. On examination he is tender over the epigastrium. Blood
pressure, pulse rate and temperature are all unremarkable. The diagnosis is
• A Pancreatic pseudocyste*
• & Cholecystitis e Chronic pancreatitis
• d. Recurrent acute pancreatitis
• e Pancreatic abscess
120. A man undergoes an open inguinal hernia repair. During the procedure the spermatic cord is
visualized What structures does this contain?
• 1 Dartos muscle
• b Femoral branch of the genitofemoral nerve
• io inguinal nerve
• Inferior epigastric artery
• Pampiniform plexus*
121. Gastrointestinal consequences of major burns include which of the following?
• Splanchnic vasodilation
• Acute gastric dilatation
• Cushing's ulcers*
• Teminal ileal hyperplasia
• e Mechanical bowel obstruction
122. Splenectomy patients are at particular risk of overwhelming sepsis from which organism?
• Anaerobic bacteria
• Bacteroides fragilis
• Haemophilus influenza*
• Fungi
• Staphylococcus aureus
123. Which of the following is the most common site for colonic carcinoma?
• Rection
• Sigmoid colon*
• e Cecum and ascending colon
• d Descending colon
• e Transverse colon
124. 38. The most common cause of acute intestinal bowel obstruction is:
• Adhesions*
• Intestinal parasites
• Volvulus
• Hermia
• Neoplasia
125. A 55-year-old with a 2-year history of dyspepsia, is brought to the emergency department
following a sudden onset of severe epigastric pain. The pain is made ware en movement. On
examination, the patient is cold sweating profusely and taking shallon breaths. The abdomen
is rigid and bowel sounds are A plain film chest radiograph reveals free air under the
diaphragm. The most likely diagnosis is
• a Perforated appendicitis
• Arute cholecystitis
• Acute pancreatitis
• d. Myocardial infarction
• Perforated peptic ulcer*
126. A patient presents with a short history of perfuse, projectile vomiting without bile staining
He has a history of peptic ulceration and chronic dyspepsia and has noticed increased bloating
over the preceding 9 months. On examination there is distension in the epigastric region and a
succession splash. The abdominal radiograph shows a grossly distended stomach and
collapsed bowel. The most likely canse is
• Carcinoma of the esophagus
• Carcinomas of the head of pancreas
• Fibrotic stricture*
• 4 Compression by malignant nodes
• e Chronic pancreatitis
127. A 18-old patient presents with a 5-day history of abdominal pain and e Oration bowel sounds
are present and the abdomen is soft with no rebound tenderness A mass in the right ilac fossa
is palpable. Abdominal CT confirms the diagnosis of an appendicular mass with an ed abscess.
The patient is started on metronidazole and admitted for observation and conservative
management. After 2 days the mass has not reduced for and the temperature remains raised
The next stage in management is:
• Continues metronidazole for further 14 days a cefuroxime
• Proceed to appendicectomy
• Drainage of Appendicular abscess*
• Colonoscopy
• Laparoscopy
128. A patient with a previous anal abscess presents with persistent perianal discharge and
discomfort. On examination a sinus sis identifiable at the 6 o'clock position with the patient in
the lithotomy position. What procedure the surgeon is most likely to perform?
• Satin insertion
• colostomy
• Open exploration of tract*
• I&D
• Lateral Syphinterotony
129. A 58-year-old patient presents with a 6-week history of increasing difficulty in swallowing He
first noticed problems when eating meat which became stuck behind his heart, but this
gradually began to include other foods. The patient is currently worried because he is now
struggling with thick fluids and has noticed some involuntary weight loss. What is the most
appropriate first investigation?
• a Staging computed tomography
• b Barium meal
• e Upper gastrointestinal endoscopy*
• d. Barium swallow
• Electrocardiography
130. A 22-year-old female patient presents with a 6-week history of bloody motions. She has
noticed significant weight loss over the preceding 6 weeks with increasing lethargy and
fatigue. She has previously had constipation and admits to regular laxative use What is the
most likely diagnosis?
• a Bowel cancer
• imitable bowel syndrome e Diverticular disease
• d. Inflammatory bowel disease*
• e Anal fissure
131. Which of the following is NOT an option when treating an anal fissure?
• Propranolol*
• botulinum toxin A injections
• Glycerol tractate cream
• Diacream cream
• e Lautives
132. Which of the following in NOT an associated with acute appendicitis?
• Murphy’s sign*
• Cope sign
• o. Pisces sign
133. 15. What statement is true of Meckel's diverticulum is present in 20 per cent of the
population
• Arises from the mesenteric border of the jejunum
• May contain heterotrophic pancreas*
• is present only in males
• is a diverticulum of the bladder
134. A patient is known to have exposure to beta-naphthylamine. What is he/she at increased of
developing
• Small-cell lung carcinoma
• Bladder cancer*
• Breast cancer
• Chemical premonitis e Lymphoma
135. 40 A woman arrives at the emergency department with unbearable tease right iliac fossa
pain Earlier that day, she was experiencing on and off moderate pain in the umbilical area
which gradually moved over to the right iliac fossa Associated symptoms include anorexia,
nausea and vomiting. On examination, the patient is pyrexial and there is rebound tenderness
and guarding over the right ilac forsa. A beta-human chorionic gonadotrophin test is negative.
What should you do next?
• Send the patient to the emergency operating theatre for an Appendicectomy *
• Alert the obstetrics and gynaecology team, suspecting that she may have a ruptured ectopic
pregnancy
• Manage the patient medically in the emergency department
• Order an ultrasound scan of the abdomen
• Send the patient for a plain film radiograph of the abdomen
136. Radiotherapy can be the sole treatment for which of the following cancers?
• 1 Adenocarcinoma of the oesophagus
• Rectal carcinoma
• Anal cancer*
• Gastric carcinoma
• e Phyllodes breast tumor

Urogenital System
1. A 50 year old female patient is having painful micturition for last one month. There is poor
stream of urine and she strains on micturition. What is the likely cause of this condition?
• Ureteric stone
• Trauma urethra
• Ureter Tumor
• kidney stone
• Stricture Urethra*
2. The investigation of choice for stricture urethra is:
• Retrograde Urethrogram*
• CT scan urethra
• CT KUB
• Ultra sound Abdomen KUB
• Cystogram
3. A 67-year-old man is referred to the urology clinic with painless hematuria. There are no other
significant findings in his history and examination. He undergoes a cystoscopy which reveals a
transitional cell carcinoma. Which of the following is not a risk factor for bladder cancer?
• Smoking
• Schistosomiasis*
• Cyclophosphamide
• Renal stones
• Aniline dye
4. A 71-year-old gentleman is referred by his GP to the urology outpatient clinic with a 3-4 month
history of painless hematuria, increased frequency, difficulty in initiating micturition, and loss
of weight. He also complains of generalized tiredness and occasional palpitations. He smokes
16-20 cigarettes a day. Prior to his retirement, he worked in a petrochemical industry. On
examination, he appears pale and anemic Abdominal examination is unremarkable. Per rectal
examination reveals a normal prostate gland. What is the most likely diagnosis?
• Renal cell carcinoma
• Squamous cell carcinoma of the renal pelvis
• Angiomyolipoma
• Renal tuberculosis
• Carcinoma of the bladder
5. A 30-year-old man presents with severe right loin pain radiating to his groin. He undergoes a CT
scan of a his kidneys, ureters and bladder, and is diagnosed with a ureteric stone. At which
anatomical site may the stone have been impacted?
• Pelvi-ureteric junction
• Point at which the iliac vessels cross the ureter
• Vesicoureteric junction
• All of the above
• None of the above.
6. Which of the following statements is false?
• The testes in men should be the same size
• It is best to examine a man's genitalia in a warm room
• The normal prostate is the size of a walnut
• A hydrocele feels like a bag of worms
• Sometime for undescended testis scrotal sac is not developed
7. The most common cause of painless hematuria in a patient older than 50 years is:
Renal calculi
• Infection
• Benign prostatic enlargement
• Bladder cancer*
• Trauma
• Renal stones
8. Acute pyelonephritis is the most likely diagnosis in a patient with:
• Bacteria and pyuria
• Chills, fever and flank pain*
• Focal scar in renal cortex
• Delayed renal function
• Vesico ureteric reflux
9. The most common cause of STD [sexually transmitted disease) in United States America is.
• Gonorrhea*
• Chlamydia
• Syphilis
• Chancroid
• Ureaplasma
10. All of the following are typical of lower urinary tract symptoms except
• Urgency
• Frequency
• Nocturia
• Dysuria*
• Weak urinary stream
11. The most common bacteria associated with UTI
• Klebsiella
• Staphylococcus saprophyticus
• Proteus mirabilis
• E Coli*
• Pseudomonas
12. CA prostate arises from?
• Transitional zone of prostate
• Central zone of prostate
• Peripheral zone of prostate*
• A and B
• B and C
13. A 55-year-old man presents to the emergency department with increasing itching and upper
discomfort. His wife has noticed that he is looking "yellow", On examination, there is a non-
tender mass in the right upper quadrant. What is the most likely diagnosis?
• Cholangiocarcinoma
• Gallstones*
• Hemolysis
• Hepatitis
• Pancreatic carcinoma
14. A 24-year-old woman presents to the emergency department complaining of a 2-day history of
lower abdominal pain with associated fool-smelling vaginal discharge. On examination, she is
tender and guarding in both the suprapubic region and the right iliac fossa. A urine dipstick
shows leukocytes and protein. What is the most likely diagnosis?
• Appendicitis
• Ectopic pregnancy
• Mittelschmerz
• Pelvic inflammatory disease*
• Urinary tract infection
15. A 90-year-old retired GP incidentally discovers a swelling in his right Inguino scrotal area. Self
examination reveals a semi-firm lump that does not transilliuminate, Cough impulse is there.
What is the most likely diagnosis of this lump?
• Epididymal cyst
• Inguino scrotal hernia*
• Hydrocele
• Neoplasm of the vas deferens
• Testicular seminoma
16. What is the most common cause of abdominal pain associated with visible hematuria?
• Benign prostatic enlargement
• Urinary tract injury*
• Urinary tract infection
• Urinary tract calculi
• Bladder cancer
17. The most likely bacteria associated with UTI?
• Klebsiella
• Staphylococcus saprophyticus
• Proteus mirabilis
• E Coli*
• Pseudomonas
18. A 50 year old female presented with hematuria with no history of trauma, fever or pain. CT
KUB with contrast shows a solid lesion in Right Kidney 12cm involving perl-nephric fat but not
Gerota Fascia. What is best treatment?
• Radiotherapy
• Chemotherapy
• Nephrectomy
• Radical Nephrectomy*
• Partial Nephrectomy
19. A 24 year old female has presented with pain In left flank. There is no fever. Urine shows 3-5
RBCS. Ultrasound shows a 3cm clear fluid filled area in renal cortex on Left side. What is
Diagnosis?
• Renal Calculus
• Pyelonephritis
• Pyonephrosis
• Simple Renal Cyst*
• Renal Cell CA
20. Most common bacteria in UTI is
• Enterococcus
• Pseudomonas aeruginosa
• Escherichia Coli*
• Klebsiella pneumonia
• Staphylococcus aureus
21. A 45 year old male presented with history of hematuria off and on for last 3 months and pain
in left flank. There is an echogenic shadow in Left Renal Pelvis with hydronephrosis on left
side on ultrasound. What is Diagnosis?
• Renal Cell CA*
• Polycystic Kidneys
• Hemophilia
• Congenital PUJ Obstruction
• Renal Calculus
22. A 25 year old female has presented with pain in flank and fever with rigors. There is
tenderness in Right Renal area. There is no radio opaque shadow In X-ray KUB. What is
Diagnosis?
• Renal Calculus
• Acute Pyelonephritis*
• Pyonephrosis
• Renal Cell CA
• Polycystic Kidneys
23. Composition of Renal Stones: (HBS)
• Pure calcium phosphate 85%
• Cysteine stone 20%
• Uric acid 80%
• Calcium oxalate 85%*
24. Carcinoma prostate originate in (HBS)
• Transition zone
• Central zone
• Peripheral zone*
• Capsule of prostate
25. Grade 1 Renal Trauma: (HBS)
• Laceration ,> 1.0 cm ,Renal cortex only ,No urinary extravasation Intact collecting system
• Hematoma, Subcapsular, Non expanding, Parenchyma normal*
• Main renal artery/vein injury with contained hemorrhage
• Avulsion of renal hilum (pedicle) which devascularizes kidney
26. What is recurrent UTI? (HBS)
• Bacterial persistence
• Infection by different organisms
• An interval of at least 6 months between infections
• >2 infections in 6 months, or 3 within 12 months*
27. The main diagnostic investigation of UTI is: (HBS)
• Microscopy
• X ray KUB
• Ultrasound
• Microscopy and culture of urine samples*
28. A patient came to you with history of severe pain at Right hemiscrotum with nausea and
vomiting. Ultrasound scrotum shows normal testis, on examination there was "Blue dot sign"
02-3 mm on the upper pole of testis. In which of the following scenario is this sign seen? (HBS)
• Extravaginal torsion
• Intravaginal torsion
• Testicular torsion
• Torsion of appendix of testis*
29. A 70 year old patient came with history of gross hematuria with clots, blood CP shows low
hemoglobin and renal profile is normal. He underwent TURBT for a mass 3x4cm at left
postero-lateral wall. On histopathology tumor turned out to be superficial bladder cancer.
What is the stage? (HBS)
• T4
• T2
• Tis*
• M1
30. A cyclist was run over by a tractor on his lower trunk. The commonest injury associated with
fractured pelvis is: (HBS)
• Colonic injury
• Ruptured urethra*
• Sciatic nerve injury
• Small gut tear
• Torn ureter
31. Following is a translucent swelling in the scrotum: (HBS)
• Hydrocele*
• Inguinal hernia in a baby
• Testicular tumor
• Testicular torsion
• Varicocele
32. The most common complication in an elderly patient, following hemorrhoidectomy is? (HBS)
• Fecal impaction
• Bleeding
• Urinary retention*
• Infection
• Anal stenosis
33. Treatment of renal adenocarcinoma is: (HBS)
• Chemotherapy
• Simple Nephrectomy
• Partial nephrectomy
• Radical nephrectomy*
• Radiation
34. Raised PSA is suggestive of: (HBS)
• Carcinoma pancreas
• Carcinoma liver
• Carcinoma prostate*
• Carcinoma ovary
• Carcinoma rectum
35. A 12-year-old is admitted to the emergency department with sudden onset of severe right
testicular and lower abdominal pain during athletic training. He has had one episode of
vermiting and constantly feels nauseous. On examination, the patient sweating and in
unbearable pain. There is marked tenderness and swelling of the right testicle which is
observed to be lying horizontally. What is the most appropriate next step in this patient's
management?
• Order a CT scan of abdomen
• Send the patient immediately for emergency surgical exploration of the scrotum
• Perform urine dipstick
• d Manage the patient with analgesia and observe
• Obtain a second opinion from your senior colleague, who will only be able to see the patient
in an hour

Head, Neck & Thorax/ Endocrines


1. Pseudo-cyst pancreas is one of the complications of acute pancreatitis. It should be operated
after an interval of:
• 6 days
• 6 weeks*
• 6 years
• 6 hours
• 6 months
2. A 32 years old male has under gone thyroidectomy for solitary thyroid nodule in the morning.
The house officer was called to see this patient who is in severe respiratory distress with a
swelling over neck. Emergency treatment of post thyroidectomy bleed is:
• Opening up of wound and exploration*
• Blood transfusion
• Pressure bandage
• Transamine injection
• Endotracheal intubation
3. Which of the following cancers are associated with MEN type 2b syndrome?
• Malignant Gl tumors
• Medullary carcinoma of thyroid*
• Mucosal carcinoma
• Adrenal carcinoma
• Basal cell carcinoma
4. A 21 year old boy presented in OPD with history of swelling in floor of mouth transparent in
appearance, painless and similar swelling below the chin is seen. On aspiration clear fluid
came out. What is most likely diagnosis?
• Ludwig's Angina
• Tumor of sublingual gland
• Ranula
• Mucous retention cyst
• Plunging Ranula of sublingual salivary glands*
5. The pancreas occupies a retroperitoneal position in the upper abdomen. Which statement is
correct?
• Uncinate process of the pancreas extends posterior to the inferior vena cava.
• Splenic artery lies in lower border of pancreas
• Portal vein from superior mesenteric and splenic vein forms posterior to neck of the
pancreas.*
• Tail of the pancreas extends to the right of the aorta, toward the kidney hilum.
• Head of the pancreas is supplied by arterial blood from the celiac axis only
6. A 60 year man presents with facial lump in front of left ear, the lump non tender, hard and
irregular with impalpable edges, he was also found to have left facial palsy and Palpable
lymph node in submandibular area. Which of the following is your likely diagnosis?
• Mandibular osteoma
• Mucoepidermid carcinoma*
• Facial neuroma
• Pleomorphic adenoma
• Warthin's tumor
7. A 52 year old lady has reported with a huge multinodular goiter with respiratory distress.
Tracheal obstruction in her is tested clinically by:
• Mobius test
• Berry's test
• Joffroy's test
• Nafziger test
• Kocher's test*
8. 55 years male presented in ER with severe abdominal pain radiating to back and vomiting after
a bout of alcohol intake, on examination he was dehydrated, the abdominal was severely
tender and distended. X ray plain abdomen shows dilated duodenal loops with no free air. Lab
reported WBC: 15000 pcm, serum AST& ALT: normal and serum amylase is 3500 IU. Most
probable diagnosis would be:
• Acute pancreatitis*
• Esophageal rupture
• Gallstone ileus
• Perforated peptic ulcer
• Acute cholangitis
9. Which of the following is the most common cause of obstructive jaundice in patients with
chronic pancreatitis?
• Pancreatic pseudo cyst formation
• Cholelithiasis
• Choledocholithiasis
• Fibrotic stricture of the common bile duct*
• Adenocarcinoma of the head of the pancreas
10. A consultant asks a house officer during a ward round about Multiple Endocrine Neoplasia 1
Syndrome. Following is not a component of MEN 1:
• Pheochromocytoma*
• Parathyroid Hyperplasia
• Pituitary Adenoma
• None of above
• Pancreatic islet cell tumors
11. 45 year developed a pre-auricular swelling and diagnosed to have pleomorphic adenoma, the
surgeon advised him surgery, which of the following surgical procedure is the most
appropriate.
• Superficial parotidectomy*
• Enucleation
• Radical parotidectomy
• Wide excision
• Total parotidectomy
12. Frey's syndrome follows:
• SMR for DNS surgery
• Parotidectomy*
• Thyroidectomy
• Mandibulectomy
• Cataract surgery
13. A 10-year child brought to ER with high grade fever and rapidly growing neck swelling. It is
large, hot, and tender with brawny induration and is fluctuant. Which of following is the best
management option?
• Start a course of antibiotics*
• Perform incision and drainage
• Excise the swelling
• Treat conservatively
• Perform CT scan
14. A medical student has noticed a 3x3 cm nodule in thyroid isthmus. FNAC shows adenomatous
goiter. She is undergoing surgery. Minimum operation for solitary thyroid nodule is:
• Isthmectomy
• Total lobectomy plus isthmectomy*
• Subtotal thyroidectomy
• Total thyroidectomy
• Excision of nodule
15. Pre-tibial Myxoedema occurs in:
• Thyrotoxicosis*
• Lymph-edema
• Nephrotic syndrome
• Chronic venous insufficiency
• Chronic liver disease:
16. A 55 years old executive is having sudden onset of pain and swelling of thyroid with
palpitations. He is diagnosed to be having De Quervain's thyroiditis, which is:
• Fungal thyroiditis
• Autoimmune thyroiditis
• Viral thyroiditis*
• Bacterial thyroiditis
• Tuberculous thyroiditis
17. A 35 year old male presented with welling right submandibular region. There is painful
swelling during meal and no history of fever or pus ooze. What can be most likely cause?
• Sialadenitis
• Sialolithiasis*
• Pleomorphic adenoma
• Stricture of duct
• Tumor in duct
18. Cranial nerves at risk of damage during submandibular dissection are?
• The lingual nerve
• Cervical nerve
• Temporal nerve
• Jugular nerve
• Submandibular nerve*
19. A 45 year old lady underwent subtotal thyroidectomy. What amount of tissue is to be left in
each lobe?
• 4g on each side
• 8g on each side*
• 6g on each side
• 2g on each side
• 1g on each side
20. A female patient presented with history of solitary toxic thyroid nodule. What is most likely
treatment advised to the patient?
• Anti-thyroid drugs lifelong
• Surgery of the nodule*
• Anti-thyroid drugs for 6months
• Anti-thyroid drugs & surgery
• Radiotherapy
21. A patient presented with off & on history of abdominal cramps, body aches and pains,
diarrhea and mood swings. After finding raised calcium levels, the specialist advised the
patient to undergo a scan most likely
• MIBG Scan
• PET Scan
• CT Scan
• Sestamibi Scan*
• MRI Scan
22. A patient of secondary hyperparathyroidism was put on cinacalcet drug and became normal.
For what purpose these drugs are used?
• Lowering calcium*
• Increasing calcium
• Regulates secretion of PTH
• Regulate vitamin D secretion
• Increase Serum PTH level
23. At the age of 46, an accountant has developed hoarseness due to an inoperable cancer of the
left upper lung lobe. He has smoked heavily since the age of 14. Which of the following
features of cancer of the lung indicates distant spread?
• Hypercalcemia
• Cushing-like syndrome
• Gynecomastia
• Syndrome of inappropriate secretion of antidiuretic hormone (SIADH)
• Brachial plexus lesion (Pancoast's syndrome)*
24. A 26-year-old woman with a known history of chronic alcoholism is admitted to the hospital
with severe abdominal pain due to acute pancreatitis. The serum and urinary amylase levels
are normal. On the day following admission to the hospital, there is no improvement, and she
has a mild cough and slight dyspnea. What is the most likely complication?
• Pulmonary atelectasis*
• Bronchitis
• Pulmonary embolus
• Afferent loop syndrome
• Pneumonia
25. A 40 years old alcoholic male patient admitted in the hospital with plan and a palpable non-
pulsatile soft mass in epigastrium, after 4 weeks of pancreatitis attack. He is apyretic and
stable, there Is persistent hyper-amylasemia. Most likely diagnosis is
• Abdominal aortic aneurysm
• Pancreatic ascites
• Pancreatic carcinoma
• Pancreatic pseudocyst*
• Pancreatic abscess
26. In acute pancreatitis serum amylase is raised Initially but drops back to normal in
• 02 - 06 hrs
• 06-12 hrs
• 48-72 hrs
• 100-120 hrs*
• 120-150 hrs
27. A 42-year-old male develops acute pancreatitis. 3 weeks after onset of symptoms, CT Scan of
the abdomen reveals a pancreatic phlegmon and associated pseudocyst. Which of the
following would decrease the likelihood of spontaneous resolution of the pseudocyst? (HBS)
• No Diffuse calcification of the pancreatic gland
• Location in the pancreatic tail
• Single cavity
• Size greater than 5 cm*
28. A 50-year-old man develops acute pancreatitis due to alcohol abuse. Hyperamylasemia
resolves by the third day after admission. By the eighth hospital day. the patient is noted to
have recurrent fever (38.5°C), progressive leukocytosis (18.500 WBC/mm3). and tachypnea.
The most appropriate management includes which as the next step: (HBS)
• ERCP with sphincterotomy and placement of biliary stent
• CT guided aspiration of peripancreatic fluid collections*
• Laparotomy with pancreatic debridement
• Intravenous amphotericin B
29. Best initial supportive measure for patients with mild pancreatitis is: (HBS)
• Intravenous fluid and electrolyte therapy*
• Withholding of analgesics to allow serial abdominal examinations.
• Nasogastric decompression
• Subcutaneous octreotide therapy.
30. An X-ray of young female presented with swelling right submandibular region for one month
shows stone in duct of gland. What type of stones is common in this gland duct? (HBS)
• Cholesterol
• Phosphate
• Oxalate and phosphate
• None of the above
• Vitamin*
31. A patient with opens eye on verbal command, who is confused and localizes pain has a
Glasgow Coma Score (GCS) of: (HBS)
• 9
• 11
• 12*
• 14
32. Which one of the following arteries is an important landmark for identifying superior
parathyroid glands? (HBS)
• Common carotid Artery
• External carotid Artery
• Inferior thyroid Artery*
• Superior thyroid Artery
• Subclavian Artery
33. What is raised in acute pancreatitis? (HBS)
• Serum alkaline phosphatase
• Serum Calcium
• Serum amylase*
• Serum albumin
• Serum ALT
34. Pain abdomen is one of the commonest reasons for a surgical referral Non-surgical causes of
acute abdominal pain may include which of the following? (HBS)
• Diabetic ketoacidosis*
• Hypoglycemia
• Hyperthyroidism
• Hypothyroidism
• Paget's disease
35. Ludwig's angina involves: (HBS)
• Face*
• Heart
• Nose
• Palate
• Tonsil
36. Which of the following is associated with hyperparathyroidism?
• a Paravertebral ossification
• h Peptic ulcer*
• Retinal haemorrhages
• Aortic stenosis
• Hourse voice
37. 14 In a 65-year-old women with a retrosternal goitre discovered on a routine chest X tay,
which of the following clinical sign can be present?
• & Tracheal deviation
• b. Berry's sign
• e Pemberton's sign*
• d Stellwag's sign
• Pretibial mykedema
38. The commonest malignancy of thyroid is:
• Anaplastic carcinoma
• Follicular carcinoma*
• Hurthle cell carcinoma
• Medullary carcinoma
• Papillary carcinoma
39. Thyrotoxicosis is characterized by which of the following?
• Weight gain
• Proximal myopathy*
• Exophthalmos
• Slow tendon reflexes
40. All of the following may cause a right iliac fossa mass that is palpable on abdominal
examination, except
• caecum
• & Ulcerative colins
• Tuberculosis
• Appendicitis
• e Ovarian cancer*
Breast
1. Which of the following conditions is associated with increased risk of breast cancer?
• Severe hyperplasia.
• Papillomatosis.
• Fibroadenoma
• Atypical hyperplasia*
• Fibrocystic mastopathy.
2. A 33-year-old woman is referred with nipple discharge bilateral breasts. What can best suggest
the reason for her discharge?
• Milky breast discharge would be expected two year after discontinuation of breast feeding
• Yellow discharge is not always due to infection
• Unilateral galactorrhea is suggestive of an underlying endocrinopathy.
• Serous discharge is usually seen in underlying breast cancer
• Every blood stained nipple discharge is not cancer*
3. Sentinel lymph node in CA Breast is:
• 1st draining lymph node for tumor*
• Supraclavicular lymph node
• Rotor's lymph node:
• Internal mammary lymph node
• Posterior axillary lymph node
4. In a patient with fibroadenoma of breast
• Almost always caused by trauma to breast
• Treatment is wide local excision
• Commonly occur in lactating mothers
• Metastasis through lymphatic's is common
• Confirmation by triple assessment is mandatory*
5. Mondor's disease is
• Chondritis of costal cartilage chest
• Pectus Excavatum Chest Wall
• Eczema of the nipple of the breast
• Type of thrombophlebitis particularly affecting breast veins*
• Lymphedema of arm & Chest Wall
6. Which of the followings method of biopsy gives maximum information on histology for
carcinoma breast?
• Trucut biopsy*
• Ultrasound guided FNAC
• Closed biopsy
• Fine needle aspiration cytology
• Skin biopsy
7. A 25 year old lady presented with a lump left breast upper outer quadrant for last 3 months. On
examination there is a small lump 2x2cm in size and mobile, firm. There are no lymph nodes
palpable. What investigation will further help in diagnosing the lump?
• MRI scan Left Breast
• Mammography
• CT Scan Chest
• Ultrasound Breast*
• Chest X ray
8. Which of the following conditions is associated with increased risk of breast cancer?
• Fibrocystic mastopathy
• Fibroadenoma
• Duct Ectasia
• Papillomatosis
• Paget's disease*
9. A 21-year-old woman presents with an asymptomatic breast mass. Which of the following will
help in her diagnosis?
• Mammography will play an important role in diagnosing the lesion
• Ultrasonography is often useful in the differential diagnosis of this lesion*
• The mass should always be CT scanned
• The lesion should not be biopsied
• FNAC may be sufficient
10. A 40 year old lady presented with history of blood stained nipple discharge from right breast
for 6 months. There is no lump palpable in breast on examination. What is true regarding
intra-ductal papilloma?
• This lesion is the most common cause of bloody nipple discharge*
• Serous discharge is unlikely
• Ductography is not diagnostic in a non-palpable lesion
• It is considered premalignant.
• It can be a dangerous lesion
11. A 33-year-old woman is referred with nipple discharge bilateral breasts. What can best
suggest the reason for her discharge?
• Unilateral galactorrhea is suggestive
• Serous discharge is usually seen in underlying breast cancer
• Every blood stained nipple discharge should be evaluated of an underlying endocrinopathy
with cytology stained*
• Milky breast discharge would be expected one year after discontinuation of breast feeding
• Greenish yellow discharge should not be given antibiotics
12. A 35 year old female presented with painless lump left breast upper outer quadrant >5cm size.
The lump is hard in consistency, not adherent to skin or underlying tissues. There are hard
mobile left axillary What is lymph the nodes palpable. No bony pains. Trucut biopsy of lump
shows invasive ductal carcinoma. the stage of tumor?
• T2,N2,M1
• T3, N1,M0*
• T3,N3,M0
• T4.N2, Mx
• T1, N1,M0
13. A 35-year-old woman, who is currently breast-feeding her firstborn child, develops an
erythematous and inflamed fluctuant area on breast examination about 5cm in size.
Tenderness is positive. What is her best management?
• Hot sponging & conservative management
• Put a drain inside the area
• Open surgical incision & drainage*
• Only aspiration under ultrasound guidance
• Antibiotics and follow up with ultrasound
14. What is the management for CA right beast of a 38 year patient which is early stage T2, N1,
and MO?
• Radical Mastectomy*
• Neo-adjuvant chemotherapy
• Modified radical mastectomy*
• Hormone therapy
• Conservative
15. What treatment is of benefit in managing mastalgia associated with fibrocystic breast disease?
• Caffeine compounds
• Smoking
• Vitamin D
• Vitamin E*
• Tramadol
16. A 15 year old boy had bilateral breast swellings. He said to notice it 6 months ago. Now he
wants to them removed surgically. What procedure should be opted for him?
• Modified radical mastectomy each
• Simple lumpectomy both sides
• Radical excision both sides
• Excision biopsy both sides
• Bilateral Subcutaneous mastectomy*
17. A 33-year-old woman is referred with nipple discharge bilateral breasts. What can best
suggest the reason for her discharge? (HBS)
• Unilateral galactorrhea is suggestive of an underlying endocrinopathy
• Serous discharge is usually seen in underlying breast cancer
• Every blood stained nipple discharge should be evaluated with cytology*
• Milky breast discharge would be expected one year after discontinuation of breast feeding
18. A 35-year-old woman, who is currently breast-feeding her firstborn child, develops an
erythematous and inflamed fluctuant area on breast examination. What is her best
management? (HBS)
• Hot sponging
• Open surgical drainage is indicated*
• Breast-feeding should be discontinued
• If the inflammatory process does not completely respond, a biopsy is not indicated
• Regarding sciatic nerve injury
19. What is the best management for CA breast of a 38 year patient with locally advanced, ER, PR
receptor positive tumor stage T3, N2, M0? (HBS)
• Radical Mastectomy
• Neo-adjuvant chemo-MRM-hormone therapy*
• Modified radical mastectomy
• Hormone therapy
• None of above
20. Which of the following genes is implicated in the etiology of breast cancer? (HBS)
• BRCA1, BRCA2*
• p53
• PTCH
• RET
• VHL
21. Virchow's lymph nodes are important in relation with: (HBS)
• Carcinoma breast*
• Carcinoma prostate
• Carcinoma anal canal
• Carcinoma bone
• Carcinoma testes
22. Which one of the following is the malignant condition of breast? (HBS)
• Cystosarcoma Phylloides
• Duct ectasia
• Fibroadenoma
• Mondor's disease
• Paget's Disease*
23. A 44 year old lady, mother of three children, reports to breast clinic with six months history of
blood stained discharge from her left nipple. Clinical examination is unremarkable. She is most
likely to be having: (HBS)
• Duct adenoma
• Duct carcinoma
• Duct ectasia
• Duct fibroma
• Duct papilloma*
24. Carcinoma of the breast is the commonest tumor in females. Following is not a component of
Triple assessment (HBS)
• Clinical examination
• Mammogram
• Sonomammogram
• FNAC
• X ray chest*
25. Greenish nipple discharge is a feature of: (HBS)
• Duct carcinoma
• Duct papilloma
• Duct ectasia*
• Duct sarcoma
• Duct abscess
26. Monder's Disease of breast involves: (HBS)
• Arteries
• Veins*
• Nerves
• Ducts
• Cooper's ligaments
27. A worried 23-year-old woman presents with painless lump in the right breast of one month
duration On examination a mobile, discrete, well-defined, non-tender, firm I champ is found.
There is no lymphadenopathy. The most likely diagnosis
• Breast cyst
• Lipoma
• Fibroadenoma*
• Sebaceous cyst
• Carcinoma of the breast
28. A 16-year-old man diagnosed with invasive ductal carcinoma of the right breast stage TINMO
The most appropriate treatment option for this patient is?
• Chemotherapy
• Wide local excision*
• Mastectomy postoperative radiotherapy
• d. Palliative care programme
• Radical mastectomy

Nervous system
1. Patient presented with history of trauma to the head. He was a low GCS of 9/15. The air entry
on both sides is clear and the abdomen is soft. His blood pressure is 210/130 mmHg and the
right pupil is dilated. What is the most appropriate investigation?
• Ultrasound of the abdomen
• X-ray of the skull
• CT Scan Brain*
• MRI of the brain
• ABG's
2. Which condition is commonly managed by surgical intervention?
• Pituitary prolactinoma
• Multiple cerebral metastases
• Trigeminal Neuralgia
• Myelomeningocele*
• Arachnoid cyst
3. Which of the following is a common complication of cranial Neurosurgery?
• Brain abscess
• Meningitis
• Hematoma*
• Subdural Empyema
• Tuberculous abscess
4. A young patient sustains blunt trauma to his right knee that results in acute thrombosis of his
popliteal artery. Which tissue is most sensitive to ischemia?
• Muscle
• Nerve*
• Skin
• Fat
• Bone
5. Which of the following is ASSOCIATED with secondary brain injury?
• Hypoxia*
• Hypothermia
• Hyperkalaemia
• Hyponatremia
• Hypercalcemia

Heart & great Vessels


1. A 68 year old male underwent cardiac bypass surgery. Regarding grafts used in bypass which of
the following is false?
• Short saphenous vein graft is used
• Great saphenous vein graft is used
• Radial artery graft is used
• Femoral artery graft is used*
• Left internal mammary artery is used

Pediatrics Surgery
1. A 10-year-old boy presents to the Emergency Department with a 24-hour history of progressive
right testicular pain. Examination reveals normal lying testes and some scrotal erythema and
swelling. The right testes, epididymis, and spermatic cord are tender, and elevation leads to
some relief of the pain. What is the most likely diagnosis in this patient?
• Testicular torsion
• Testicular tumor
• Epididymo-orchitis*
• Hydrocele
• Idiopathic scrotal edema
2. An 8-month-old boy is taken to see the GP by his anxious mother, who is worried that his right
testis does not always seem to be present within his scrotum. Which of the following
statements is true of undescended testes?
• Inability to palpate the testes is an indication for laparoscopy*
• One-quarter of undescended testes complete their descent in the first year of life
• Surgical exploration and fixation should be performed in the neonatal period to allow for
easier repair of the defect
• Undescended testes are associated with a reduced risk of testicular malignancy
• Undescended testes are associated with normal fertility
3. A mother brings her 5-week-old son to the pediatric surgery outpatient clinic. He is having
episodes of forceful vomiting after feeds for the last 2 weeks. She says her son appear
lethargic. Examination of the child reveals mild dehydration and the presence of a smooth,
firm, non-tender mass in the right upper quadrant of the abdomen. Blood tests are sent. What
biochemical abnormalities would you expect to find?
• Hyperchloraemic, respiratory alkalosis
• Hypernatraemic, hyperkalaemic, metabolic alkalosis
• Hypochloraemic, hyperkalaemic, metabolic alkalosis
• Hypochloraemic, hypokalaemic, metabolic alkalosis*
• Hyponatraemic, metabolic acidosis
4. A 1-day-old baby girl is having repeated bile-stained vomiting. Her mother was noted to have
polyhydramnios during the pregnancy. An abdominal X-ray reveals gas in the stomach and
first part of the duodenum, but nowhere else in the abdomen. What is the most likely
diagnosis?
• Choledochal cyst
• Duodenal atresia*
• Esophageal atresia
• Pyloric stenosis
• Renal agenesis
5. A 7-year-old boy is brought to the emergency department by his father, having told him that he
had swallowed a five rupees coin earlier that day. The child has not experienced any
symptoms and on examination is well. Which of the following investigations would you
perform in the first instance?
• Barium swallow
• Laryngoscopy
• Neck, chest and abdominal X-rays*
• No investigation required
• Upper gastrointestinal endoscopy
6. A 3-year-old boy is complaining of intermittent abdominal pain. His mother feels that his
abdomen is distended. On examination, there is a large mass on the left side of the abdomen
which does not cross the midline. What is the most likely diagnosis?
• Hepatoblastoma
• Lipoma
• Nephroblastoma*
• Neuroblastoma
• Polycystic kidney
7. A 4 year old boy presented with swelling abdomen and chronic constipation since birth. Mother
often uses glycerine suppositories for constipation relief of the child. He is having distended
large bowel shadow X ray abdomen. What further test will confirm his diagnosis?
• Colonoscopy
• Surgery
• CT Abdomen
• MRI Scan
• Rectal biopsy*
8. A 2 week old infant has non bilious vomiting with a palpable olive like mass in right upper
abdomen. She was diagnosed as a case of Infantile Hypertrophic Pyloric Stenosis. What is the
ideal procedure done for correction?
• Heller's myotomy
• Ramstedt's pyloromyotomy*
• Gastrojejunostomy
• Duodenostomy
• Gastrostomy
9. A 14-year-old boy is seen by a pediatric cardiologist because of increasing shortness of breath.
Studies reveal increased pulmonary vascular resistance, left axis on Electrocardiogram (ECG),
and mitral regurgitation murmur. What is the most likely diagnosis?
• Ostium primum defect
• Tetralogy of Fallot
• Right aortic arch
• Ostium secundum defect
• Atrioventricular canal*
10. 10 year old boy presented with face and perineum burns came to opd. There were second
degree burns with swelling of scrotum. Where this patient should be managed?
• Plastic surgery unit
• General surgery ward
• Burn unit ward*
• At home
• Specialized Intensive care
11. A new born baby boy vomits whenever he is given feed. Which is TRUE regarding congenital
pyloric stenosis: (HBS)
• Bilious vomiting
• Common in baby girls
• Projectile vomiting*
• Heller's myotomy is curative
• Baby is acidotic
12. An 11 year old boy is having sudden onset of severe pain in right testis. Commonest cause of
torsion of testis in children is: (HBS)
• Associated testicular tumor
• Congenital hydrocele
• High investment of tunica vaginalis*
• Maldescent of testis
• Trauma
13. An 11 year old boy is having sudden onset of severe pain in right testis. Investigation of choice
to rule out torsion of testis is: (HBS)
• Angiography
• CT scan
• Doppler ultrasound*
• MRI
• Venography
14. A 13 year old boy is diagnosed to be having branchial cyst on right side of neck. Branchial cleft
remnants most often present with which of the following clinical problem? (HBS)
• Airway obstruction
• Hemorrhage
• Infection*
• Malignant degeneration
• Pain
15. Following is NOT a part of Tetralogy of Fallot: (HBS)
• VSD
• ASD*
• Overriding aorta
• Pulmonary stenosis
• Right ventricular hypertrophy
16. Following is NOT a feature of cardiac tamponade: (HBS)
• Engorged neck veins
• Hypotension
• Tracheal shift*
• Muffled heart sound
• Normal breath sounds

Sendup Surgery B HBS 2022


1. A 69-year-old man is admitted to the emergency department with an acute UGI hemorrhage
following a bout of repeated vomiting. Fiberoptic gastroscopy reveals three linear mucosal
tears at the GE junction. What is the diagnosis?
• Reflux esophagitis with ulceration
• Barrett's esophagus
• Carcinoma of the esophagus
• D Mallory-Weiss tear*
• E. Scleroderma
2. A 52-year-old gastroenterologist suffers from intermittent dysphagia attributed to the
presence of a lower esophageal stricture. The doctor's condition is characterized by which of
the following?
• A full thickness scar in the upper esophagus
• B. Symptoms of mild-to-moderate dysphagia*
• C A low incidence in men
• D. The absence of a sliding hiatal hernia in most case
• E. The need for anti reflux surgery at an early stage
3. A 46-year-old man has a long history of heart burn (GERD). His barium study shows an
irregular, ulcerated area in the lower third of his esophagus. There is marked mucosal
disruption and overhanging edges. What is the most likely diagnosis?
• Sliding hiatal hernia GERD
• Paraoesophageal hernia.
• Benign esophageal stricture c
• Squamous carcinoma of the esophagus
• Adenocarcinoma arising in a Barrett's esophagus*
4. A 46-year-old man present with dysphagia of recent onset. His esophagogram shows a lesion
in the lower third of his esophagus. Biopsy of the lesion shows adenocarcinoma. His general
medical condition is excellent, and his metastatic workup is negative. What should his
management involve?
• A Chemotherapy
• B. Radiation therapy
• C. Insertion of a stent to improve swallowing
• D Surgical resection of the esophagus*
• E. Combination of chemotherapy and radiation therapy
5. A 25-year-old man arrives in the emergency department in respiratory distress following a
motor vehicle collision. A chest x-ray shows abdominal viscera in the left thorax. What is the
most likely diagnosis?
• A Traumatic rupture of the diaphragm*
• B. Sliding esophageal hernia.
• C. Short esophagus with intrathoracic stomach
• D. Rupture of the esophagus
• E. Bochdalek hernia
6. A patient has been diagnosed with achalasia. He refused surgery initially, preferring to try non
operative therapy. He tried life style modification, calcium channel blockers, botulin toxin
injection, and endoscopic pneumatic dilatation. None of the treatments alleviated his
symptoms. What is his surgical option?
• Esophagectomy
• Surgical esophagomyotomy proximal to the LES
• Modified Heller myotomy and partial fundoplication*
• Repeat pneumatic dilation using pressures of loop
• Nissen fundoplication
7. A 35-year-old man has known ulcerative colitis. Which of the following is an indication for
total proctocolectomy?
• Occasional bouts of colic and diarrhea
• Sclerosing cholangitis
• Toxic megacolon*
• Arthritis
• Iron deficiency anemia
8. Complications of diverticulitis include:
• A Carcinoma of the colon
• Extraintestinal manifestations such as arthritis, iritis, and skin rashes
• Fistulisation to adjacent bladder, with insueing colovesical fistula*
• D. Artheriovenous fistulae of the intestine
• E. Sclerosing cholangitis
9. A patient's CT scan reveals diverticulitis confined to the sigmoid colon. There is no associated
pericolic abscess. What is best course of treatment?
• Bowel rest, nasogastric suction, IV fluids, and broad spectrum antibiotics*
• Urgent surgical resection
• Steroids
• Diverting colostomy
• Ileostomy
10. A 42-year-old woman is admitted to the emergency department with severe colicky pain,
vomiting, and abdominal distention. She has not passed stools or flatus for 48 hours. X-rays of
the abdomen confirm the presence of small bowel obstruction. What is the most likely cause
of small-bowel obstruction in this patient?
• Adenocarcinoma
• Adhesions*
• Crohn's disease
• Ulcerative colitis of
• Gallstone ileus.
11. An 80-year-old woman with a known history of femoral hernia is admitted to the hospital
because of strangulation of the hernia. There is a tender swelling in the right femoral region
immediately below and lateral to the pubic tubercle. She has had multiple bowel movements
without relief of symptoms. What is the most likely diagnosis?
• Lymphadenitis
• Diverticulitis
• Volvulus
• Richter's hernia*
• Gastroenteritis
12. Why is distal resection of small intestine, as compared to proximal resection, poorly
tolerated?
• Transit time in the ileum is slower than that in the jejunum*
• Transit time in the jejunum is slower than that in the ileum.
• The greater bulk of food is absorbed in the ileum.
• Water absorption is mainly in the ileum
• All minerals are absorbed preferentially in the ileum.
13. A 28-year-old man is admitted to the emergency department complaining of pain in the
umbilical region that moves to the right iliac fossa. Which is a corroborative sign of acute
appendicitis?
• Referred pain in the right side with pressure on the left (Rovsing)sign*
• Increase of pain with testiculair elevation
• Relief of pain in lower abdomen with extension of thigh
• Relief of pain in lower abdomen with internal rotation of right thigh
• Hyper anesthesia in the right lower abdomen
14. A 48-year-old woman develops colon cancer. She is known to have a long history of ulcerative
colitis. In ulcerative colitis, which of the following is a characteristic of colon cancer?
• A Occurs more frequently than in the rest of the population*
• 8. more likely to occur when the ulcerative disease is confined to the left colon
• Occurs equally in the right and left side.
• D. Has a synchronous carcinoma in 4-5% of cases.
• E. Has an excellent prognosis because of physician awareness.
15. A pathology specimen indicates that synchronous lesions are present. Which of the following
statements are true regarding colon cancer with synchronous lesions?
• Cancer occurs in 20% of patients.
• Benign lesions occur in 20-30%.
• C Malignant lesions are usually adjacent to the primary cancer.
• Benign lesions are usually adjacent to the primary cancer,
• E. Lesions occur much less frequently than metachronous lesions*
16. The patient requests information from her surgeon as to her subsequent prognosis. She is
informed that the prognosis for colon and rectal cancer is favorably affected by which of the
following?
• Minimal serosal extension
• Minimal lymph node involvement
• Confinement to the mucosa*
• Right-sided obstructing lesions
• Elevated carcinoembryonic antigen (CEA) levels
17. An 83-year-old man is diagnosed on colonoscopy to have cancer of the colon. He refuses
surgical intervention and after a 6-month follow up period is admitted to the emergency
department with marked anemia and weight loss. Carcinoma of the colon is most likely to
obstruct if found in the
• A Cecum*
• B. Ascending colon
• C. Descending colon
• D. Rectum
• E. Transverse colon
18. A43-year-old man is seen in his physician's office for severe pain in the perineum. Examination
reveals exquisite tenderness in the area to the right side of the anal verge due to a perianal
abscess. Rectal examination is refused. What should be the next step in management?
• A Drainage of the abscess in the office under local anesthesia*
• B. Excision of the vertical fold of Morgagni
• Drainage under general anesthesia and immediate colonoscopy.
• D. CT scan of the abdomen
• E. Insertion of a rectal tube.
19. A 65-year-old woman with a history of chronic constipation is transferred from a nursing
home because of abdominal pain and marked abdominal distention. On examination, her
abdomen is found to be distended and tender in the left side of abdomen. What is the most
likely diagnosis?
• Appendicitis
• B Carcinoma of the colon
• Volvulus of the sigmoid colon*
• D. Volvulus of the cecum
• E. Small-bowel obstruction
20. A 68-year-old female is known to have had surgery several years previously for a bowel lesion.
Her surgeon had told her that she suffers from the blind loop syndrome. In which condition
can one anticipate the blind loop syndrome to occur?
• Intestinal bypass*
• Vesicocolic fistula
• Duodenal ulcer disease
• Multiple polyposis of the colon
• Anteriovenous fistula of the colon
21. 33-year-old woman is noted to have a Meckel's diverticulum when she undergoes an
emergency appendectomy. The diverticulum is approximately 60 cm from the ileocecal valve
and measures 2-3 cm in length, What is the most common complication of Meckel's
diverticulum among adults?
• Bleeding
• B. Perforation
• C. Intestinal obstruction*
• D. Ulceration
• E. Carcinoma
22. To confirm the diagnosis of ischemic colitis, what test should be requested?
• Selective angiogram of inferior mesenteric artery
• Angiogram of superior and inferior mesenteric arteries
• CT scan of the abdomen
• D. Barium enema after 2 weeks
• E. Barium enema as soon as possible*
23. A 68-year-old man presents with crampy abdominal pain and distention with vomiting.
Findings on physical examination are positive for healed abdominal scars. X-rays reveal
multiple gas fluid levels. The WBC count is 12,000. What is the most likely diagnosis?
• Small-bowel intestinal obstruction due to adhesions*
• 8. Hernia
• Appendicitis
• D. Inflammatory bowel disease
• E. Gallstones and ascites
24. A 74-year-old woman complains of vomiting and intermittent colicky abdominal pain. X rays
reveal fluid levels and air in the biliary tree. What is the likely cause?
• Abdominal adhesions
• Gallstone ileus*
• Carcinoma of the right colon
• Abdominal lymphosarcoma
• Previous choledochoduodenostomy
25. A 64-year-old woman presents with a strangulated femoral hernia. At operation, what is the
criterion used to determine the viability of a loop of bowel?
• Increased peristalsis
• Absent arterial pulsation*
• Venous engorgement
• Intraoperative CT scan
• Serum amylase
26. Patient of age 18yrs has of pain in the right illac fossa since three days, also complaint of
diarrhea a diagnosis of acute appendicitis made. From history what you think the position of
appendix?
• A Retrocaecal
• B. Paraileal
• C Pelvic*
• D. Sub hepatic
• E. Retroileal
27. Appendicular artery commonly arises from?
• Anterior branch of ileocolic artery
• Right colic artery.
• lleal arteries
• Posterior branch of ileocolic artery*
• Caecal artery
28. Middle rectal artery arises from?
• Superior mesenteric artery Inferior mesenteric artery
• C Internal pudendal artery
• D. Sigmoidal artery
• E. Internal iliac artery*
29. Advantages of laparoscopic appendectomy includes:
• It can be performed safely with minimal morbidity compared to open technique.
• Length of hospital stay is longer than with open technique
• Procedure cost is less than with open technique*
• Return to full feeding is less than with open technique.
• Wound complication rate is greater with open technique
30. The test with the highest diagnostic yield for detecting a colovesical fistula is:
• Barium enema.
• B. Colonoscopy.
• C. Computed tomography (CT).
• D. Cystography*
• E. Cystoscopy.
31. Which one of following features in a salivary gland tumor is not of advanced disease?
• Facial nerve weakness
• Rapid enlargement of the swelling
• Induration and/or ulceration of the overlying skin
• Cervical node enlargement
• Local Small swelling*
32. A boy came with swelling of left parotid gland 2nd time with high grade fever, chills & rigors.
There is difficulty in swallowing & eating for 4 days. He is diagnosed with recurrent Parotitis.
The x ray shows punctate Sialectasis that is also called?
• Cluster of grapes appearance
• Snowstorm appearance*
• Beaded appearance
• D Microdots appearance
• Membranes appearance
33. A patient presented with an ulcer in the mouth. The doctor on duty advised the patient biopsy
of the lesion. Which of the following lesion is dangerous to become mouth cancer?
• Ranula & Plunging Ranula
• Stáladenitis & Sialolithiasis
• Aphthous ulcer
• Proliferative Verrucous Leukoplakia*
• E. Acute Oral Candidiasis
34. (34) A 35 year old lady with multinodular goitre came for elective thyroid surgery. The
counseling for choice of surgery she should be undergoing depends on all except which of the
following risk post surgery?
• Risk of Recurrent goitre
• B Life Long hypothyroidism
• Recurrent Laryngeal Nerve Damage
• Risk of Hypo-parathyroidism
• Toxic Nodular Goitre*
35. 35 A 60 year old presented with diarrhea off & on and the doctor found a large hard goitre.
The FNAC showed malignant goitre. Which malignant goitre is associated with diarrhea in a
patient?
• Lymphoma
• Follicular carcinoma
• Papillary carcinoma
• Medullary carcinoma*
• Anaplastic carcinoma
36. 36. A boy presented with diarrhea, raised blood pressure, bone pains and cramps abdomen
with depression. His workup shows that he has MEN2A. Which of the following is a part of this
syndrome?
• Medullary thyroid cancer, Parathyroid & pancreatic tumor
• Pituitary tumor, pancreatic tumor & Pheochromocytoma
• Medullary thyroid cancer & Pheochromocytoma
• Parathyroid adenoma, Pheochromocytoma & Medullary thyroid cancer* -
• Prolactinoma, Pheochromocytoma, insulinoma
37. 37. Pancreas functions both as exocrine & endocrine gland. In a patient of insulinoma,
Whipple's triad has which of these components?
• Symptoms of Hyperglycemia, fluctuating hypoglycemia <2.8mmol & improved symptoms on
taking sugar
• Symptoms of hypoglycemia after fasting or exercise, plasma glucose levels <2.8mmol &
improved symptoms on iv glucose*
• Inflammation of pancreas with sweating on exercise, low sugar and improved on taking
sugar
• Symptoms of Hyperglycemia, >5.8mmol glucose & insulin is given to reduce the levels
• Symptoms of hypoglycemia, single tumor of pancreas & improved symptoms on iv glucose
38. 38. Which one of the following is not the prognostic factor in breast cancer patients?
• Metastasis
• Size of tumor
• e Tumor DNA in blood
• Lymph nodes
• (E Numbers of tumors*
39. 39. Patey mastectomy is indicated for which one of the following?
• A Fungating Tumor
• Recurrent Nodule
• Tumor near nipple*
• Metastatic disease
• Small Tumors
40. 40. A patient with recurrent CA breast came to OPD with skin of chest wall infiltrated with
tumor. What is the condition called?
• Lymphedema
• Peau d' Orange
• Cancer-en-cuirasse*
• Metastatic cancer
• Lymphangitis
41. 41. A 35 year old female presented with left breast upper outer quadrant lump that is 1.5cm
in size, hard and there are no palpable axillary lymph nodes. Biopsy showed it to be CA left
breast. For axilla surgeon opts for sentinel lymph node biopsy during surgery. Where is he
going to inject dye?
• In the breast lump
• Breast skin and axilla
• In axilla only.
• Around nipple and axilla*
• Around breast lump
42. 42.45 year peri-menopausal old lady had stage 3C cancer of right breast and received
chemotherapy cycles. Her cancer is ER, PR positive what hormonal therapy is recommended
her case?
• Anastrazole
• Herceptin
• Tamoxifen*
• Lapitinab
• Danazol
43. 43. A male presented with increased frequency of urine, poor stream of urine and straining on
passing urine. There is h/o fever off & on with no pain and hematuria. He has undergone
prostatectomy 5 years back. What can be the cause of new symptoms?
• Recurrent growth of prostate.
• 8 Stricture urethra post infection*
• Trauma urethra
• Stone in urinary tract
• CA Bladder
44. 44. A 2year old boy presented with swelling right scrotum which is reduced after night's sleep.
On examination there is no cough impulse. Fluctuation is positive. What next investigation
should be done for this patient?
• Xray pelvis
• Blood CP
• USG scrotum*
• Doppler scan
• Fluid C/S
45. 45. A male patient with urethral stricture developed pain and fever. He was advised workup
for level of stricture. What is first diagnostic investigation that will be useful in this case?
• Intravenous Urogram
• CT Scan KUB
• Retrograde Urethrogram*
• Cystourethroscopy
• X Ray KUB
46. 46. A male patient fell on a tree branch. On examination he had blood on meatus of urethra.
Abdomen showed palpable bladder. What should a surgeon on duty do for urine retention?
• Catheterize the patient immediately
• Aspirate the bladder with syringe
• Suprapubic catheter*
• Send patient on reassurance
• Admit patient & Observe
47. 47. A male patient 25 year old presented with left inguinal swelling. The swelling is hard and
no well developed scrotal sac is seen on left side. Patient is having bone pains and tender
spine with cough. What is likely reason of this swelling?
• Left inguinal hernia
• (8.) Left undescended testis -
• Left Lipoma of the spermatic cord
• Seminoma of the left testis*
• Torsion of the left testis
48. 48. A 15 year boy developed sudden pain in right testis for 4 hours. He has 2 episodes of
vomiting and is tossing with pain in emergency room. On examination there is severe
tenderness right se fying transversely. What is the management in this case?
• A Right orchidectomy
• De-rotate the testis
• Right orchidopexy
• B/L orchidopexy*
• Lift the testis for pain relief
49. 49. A female presented with left flank pain and hematuria off & on. She had been diagnosed
with left PUJ obstruction due to 2cm stone. What can be the management of choice?
• Uretroscopy & stone retrieval
• Left pyelolithotomy*
• Left pyeloplasty with pyelolithotomy
• Left percutaneous nephrolithotomy
• (E.) Left ESWL
50. 50. 70 year male presented with lower urinary tract symptoms for last 3 years. On DRE he has
hard prostate. His PSA is raised. What is the most commonly involved site in case of
metastasis?
• Inguinal Lymph nodes.
• Lumbar Vertebra*
• Lungs
• Liver
• Rectum
51. 51. A female with single episode of painless hematuria was diagnosed with right renal cell
carcinoma. What is the criterion of partial nephrectomy in this case?
• Tumor involving full kidney
• Tumor near hilum
• Multifocal disease
• Tumor in one pole*
• Renal vein thrombosis
52. 52. A boy presented with pain right lumbar region radiating from loin to groin. He has history
of passage of stones multiple times in the past. Now he has history of severe urethral pain and
on examination there is palpable urinary bladder with stone.at meatus. What is the definite
management of patient?
• Suprapubic catherisation for urine retention
• Break stone at meatus and remove piece meal
• Pass Foley catheter per urethra for retention
• Meatotomy of urethra and stone removal*
• Dilate the urethra and let the stone pass on itself
53. 53. A toxic multinodular patient is put on list for near total thyroidectomy. A house officer on
duty asks the nurse to stop the anti-thyroid drugs on night and continue beta blockers on the
day of surgery. Why the beta-blocker is still advised and anti-thyroid drug stopped?
• Beta-blocker is needed to control heart beat and other drug doesn't do that -
• Circulating hormone necessitate beta blocker to be used and anti-thyroid drug don't act on
that
• C. Anti-thyroid acts on vascularity of gland & beta-blocker is needed for T3 hormone
• Anti-thyroid is the drug needed for remaining thyroid but not the beta-blocker
• Since gland removed to antithyroid is stopped but betablocker is needed for T3 hormone till
its half-life*
54. 54. A young female presents with painful neck swelling moving with degtition. There is fever
and tenderness positive in the swelling. There is sweating and tachycardia with pain in throat.
What management option will cure this patient?
• Anti-thyroid carbimarole
• Bata blocker thrice a day
• Thyroxine 2 tablets
• Prednisolone for 7 days*
• Panadol for 5 days
55. 55. Post thyroidectomy patient complaints of spasm bilateral hands on 2 past op day. Surgeon
on duty does her Chvostek's sign that is positive. What management should be done
immediately for the patient?
• A Ask patient to take deep breaths in bag
• injection 10% calcium gluconate*
• Oral Calcium supplements
• Physiotherapy of hands
• Check her thyroid profile
56. 56. A 40 year old female presented in OPD with abdominal pain and altered behavior from
past few months. She had renal transplant 1 year ago. Her lab investigations show raised
serum calcium and parathyroid hormone but decreased serum phosphate. What is the most
likely diagnosis?
• Primary hyperparathyroidism
• 8. Secondary hyperparathyroidism
• C. Tertiary hyperparathyroidism*
• Parathyroid carcinoma
• Parathyroid Adenoma
57. 57. A 45 year old female presented in surgical OPD with multiple duct ectasia with greenish
discharge from right breast for 2 months. The ideal surgery for this presentation is?
• Right microdochectomy
• Conservative management with antibiotics
• Excision of nipple areola
• Mastectomy right breast
• Hadfield's surgery right breast*
58. 58. A 28 year old lady presented to surgical OPD with complaint of 2cm lump is her left breast.
On examination the lump was soft, mobile with no axillary lymph node involvement. What
investigation will be confirmatory in her case?
• Immediate core needle biopsy
• Breast mammogram*
• Magnetic Resonance Imaging
• Ultrasound breast
• Fine needle aspiration cytology
59. 59. A lactating mother with bilateral crack nipples presented with severe pain right breast and
swelling She had fever with rigors and chills. There is no redness or fluctuation on examination
of right breast but tenderness is positive with swelling. What is the management?
• A Cold sponging & milk evacuation
• B. Incision & drainage, antibiotics
• Aspiration of pus & antibiotics
• IV antibiotics, pump evacuation*
• Pump evacuation & reassurance
60. 60. A 16 year old boy presented in opd with pain right upper abdomen and history of
headache, sweating and raised blood pressure. His B.P was 170/100mmHg. What is the
investigation for confirming the disease?
• CT Scan abdomen
• BMRI Scan Abdomen.
• Positron emission tomography scan
• MIBG scintigraphy scan*
• Sestamibi scintigraphy scan.
61. 61. Gastro intestinal stromal tumors (GIST) account for,
• 1% of all GI tract tumors*
• B. 5% of Gl tumors
• C. 10 % of GI tumors
• 15% of all Gl tumors
• 12% of all Gl tumors
62. 62. Zolinger-Ellison syndrome is due to over secretion of,
• Glucagon
• Pepsinogen
• Amylase
• Gastrin*
• Lipase
63. 63. A 12 years old boy presents in ER with H/O fall from bicycle. He was hit by bicycle handle
in the epigastrium. He has features of injury to some retroperitoneal organ. Which organ is
most likely to get injured?
• Gallbladder
• Right lobe of liver
• Pancreas
• 2nd part of duodenum*
• 3rd part of duodenum
64. 64. Following are Surgical complications of peptic ulcer disease of Gastric antrum EXCEPT;
• Perforation
• Bleeding
• Gastric outlet obstruction
• Diverticulae formation *
• Acute peritonitis
65. 65. Options for treating patients with acute massive upper GI bleed due to esophageal varices
include following EXCEPT,
• I/V fluids like Ringers Lactate
• Blood & blood products transfusion
• Upper Gl endoscopy & banding
• D Insertion of Sangstaken- Blakemore tube
• Insertion of (SEMS) self-expanding metal stents*
66. 66. Classical finding on Barium in a patient with CA esophagus is,
• Rat tail appearance of esophagus
• Bird beak deformity of esophagus
• C Apple core sign*
• Corkscrew esophagus
• Cullen's sign
67. 67. Calot's triangle is important in identifying
• Rt. Hepatic duct
• Cystic duct
• C Optic artery*
• Cystic node of Land
• R. Hepatic artery
68. 68 A middle aged male, chronic alcoholic presents in emergency with jaundice, fever with
chills & pain in right hypochondrium of 5 days duration, most probable diagnosis is
• Acute cholecystitis
• Acute pyogenic liver abscess
• Cholangos*
• Acute viral hepatitis
• E Acute pancreatitis
69. 09. A 55 years old male presents with 2 weeks H/O worsening jaundice, clay stools & anorexia.
O/E he is deeply jaundiced, dehydrated & a palpable mass in Rt hypochondrium. His 5
bilirubin & ALP is markedly raised. The most likely diagnosis is
• A Stone in C807
• 15 Acute viral hepatitis
• CA head of pancreas*
• Worms in biliary channels
• Biliary atresia
70. 70. Following are the complications of Gallstones EXCEPT
• Obstructive jaundice
• 8. Acute pancreatitis
• Empyema of Gallbladder
• Pseudo pancreatic cyst*
• L. Mucocele of Gallbladder
71. 71. An 8 weeks old infant presents with obstructive jaundice. Best investigation for
investigating Billary atresla in infants is:
• Ultrasound
• CT scan
• Oral cholecystogram
• DISIDA scan*
• EMRCP
72. 72. A 48 years old woman is referred to ER with severe right upper quadrant pain & fever. O/E
a palpable tender mass is found with local peritonism. What is the most likely diagnosis?
• A Acute cholecystitis
• 8. Empyema Gallbladder*
• C Acute pancreatitis
• Biliary colic
• Biliary peritonitis
73. 73. A 55 years old lady presents with a cystic non tender mass in epigastrium. She was treated
conservatively for acute pancreatitis due to impacted Gallstone in CBD, 3 weeks back. She has
no other complaints. What is most likely diagnosis?
• Pyogenic liver abscess of left lobe of liver
• Pseudo pancreatic cyst*
• Pancreatic necrosis
• Benign pancreatic cyst
• Hemangioma of liver
74. 74. A 41 years old male presents with acute onset of epigastric pain. His serum amylase was
1851 & he was diagnosed with Acute Pancreatitis. Which of the following parameters is
included in the modified Glasgow criteria for acute pancreatitis?
• Alkaline phosphatase
• B. C reactive protein
• C. Serum creatinine
• PaCO2
• Blood glucose*
75. 75. A 55 years old male who's known diabetic & chronic alcoholic, presents with severe diffuse
abdominal pain of 6 hours duration, His abdominal examination is unremarkable except for
discoloration on his/periumbilical & Lumbar regions. His Lab results are still awaited.
Considering his clinical findings what is the most likely diagnosis?
• Acute intestinal obstruction
• Acute pyelonephritis
• Acute pancreatitis*
• Acute cholecystitis
• Mesenteric ischemia
76. 76. During splenectomy which organ injury should be avoided?
• A Tail of pancreas*
• 8. Left colonic flexure
• Greater curvature of stomach
• Both a & b
• None of above
77. 77. A patient underwent splenectomy due to sickle cell disease. His recovery was uneventful
after surgery. After a month he again develops signs & symptoms of sickle cell disease. O/E
there is a palpable mass left hypochondrium. Ultrasound of abdomen was advised. What is
the most probable diagnosis?
• Sub phrenic abscess
• Accessory spleen*
• Post op ileus
• Lt hydronephrosis
• None of the above
78. 78. Following are the absolute indications for splenectomy EXCEPT?
• ITP
• Malaria*
• C Grade 4 splenic trauma
• En block resection for Gastrectomy
• Sickle cell anemia
79. 79. A 40 years old female on oral contraceptives comes to ER with Ultrasound report
performed for Gallstones. Report shows a well-defined 2 x 2 cm mass in right lobe of liver &
gallstones. what is most likely diagnosis,
• A Focal nodular hyperplasia
• Hepatic adenoma*
• Hepatocellular carcinoma
• Hemangioma
• Hydatid cyst of liver
80. 80. Tumor marker for Hepatocellular Carcinoma is?
• bHCG
• Carcinoma embryonic antigen
• Alpha fetoprotein*
• CA 125
• CA 19-9
81. 81. Contents of Hepatoduodenal ligament is?
• CBD*
• Superior mesenteric artery
• Rt. gastro epiploic artery
• Gastroduodenal artery
• Cystic artery
82. 82. Most common metabolic abnormality in Gastric outlet obstruction is,
• Hyperkalemia
• Hypophosphatemia
• Hyperchloremic metabolic acidosis
• Hyporchloremic metabolic alkalosis*
• Compensatory acidosis
83. 83. Most common site for Gastrinoma is?
• Passaro's triangle*
• Body of stomach
• Tail of pancreas
• Lower one third of esophagus of
• 4th part of duodenum
84. 84. Following procedures can be performed for CBD stones EXCEPT
• Dormia extraction through ERCP
• ERCP & sphincterotomy
• CUltrasound breakage of stone through ERCP
• Open choledocholithotomy
• Hepaticojejunostomy*
85. 85. A 35 years old lady underwent Laparoscopic cholecystectomy 5 days back. She had
uneventful recovery & was discharged on 1st post-operative day. Now she presented in ER
with pain RHC, anorexia & low grade fever. Her ultrasound examination reveals 500 ml of fluid
collection in Morrison's pouch & pelvis. What would be next line of action after maintaining
I/V line & other supportive treatment? She is haemodynamically stable.
• Exploratory laparotomy.
• Conservative treatment like antibiotics & analgesia
• Urgent diagnostic laparoscopy
• Percutaneous drainage under ultrasound guidance & observe*
• Placement of biliary stent through ERCP
86. 86. A 15 years old boy sustained head injury in motorbike accident 6 hours ago. His
neurological t examination revealed that his eye opening was to vocal commands, he was
confused but able to answer questions & he could localize pain. He was haemodynamically
stable. Considering his above mentioned examination, what is his GCS score?
• 12*
• 10
• 8
• 11
• 6
87. 87. Following are signs of raised intracranial pressure EXCEPT,
• Vomiting
• B Papilledema
• Headache
• Blurred vision
• Anosmia*
88. 88. A 5 years old male child sustained head injury. His GCS is 13/15 with stable vitals.
Neurosurgeon has ordered to keep the patient nil per oral. What maintenance fluids you will
order for this patient?
• 5% Dextrose water
• Hypertonic saline
• Dextrose saline
• 0.9 % normal saline*
• 25% Dextrose water
89. 89. Normal intracranial pressure in adults is,
• 10-15 mmHg*
• 15-20 mmHg
• C 2-5 mmHg
• 20-25 mmHg
• None of the above
90. 90. A young batsman had a history of ball hit on his left tample. He lost his conscious level for
2minutes. Then he walked his way to the changing room where he collapsed. His colleagues
found seizures on left half of body. Neurosurgeon immediately operated upon him in
emergency theatre after arrival. What is the diagnosis?
• Subdural hematoma
• B Extradural hematom*
• Subarachnoid hematoma
• Diffuse pontine hematoma
• Meningeal hematoma
91. 91. Which of the following is most likely to be injured in case of left sided impact from road
traffic accident?
• Splenic rupture*
• B. Duodenal rupture
• Popliteal artery disruption
• Posterior dislocation of shoulder
• Posterior dislocation of hip
92. 92. Most likely indication for damage control surgery in trauma is:
• Stable hemodynamics*
• No hypoxemia
• Urine output >1ml/kg/hour
• Temperature <34 degree Celsius
• Normal coagulation
93. 93. 25 years old male comes to emergency department with history of fall on out stretched
hand half hour ago while playing soccer. On Examination he is haemodynamically stable.
There is 4 cm swollen tender area at right wrist. He is unable to flex or extend his wrist.
Further examination reveals dinner fork deformity right wrist. Most likely diagnosis is?
• A Colles' fracture*
• B. Smith fracture
• C. Jones fracture
• First metacarpophallengeal joint fracture
• Scaphoid bone fracture
94. 94, 30 years old squash player is rushed to emergency department accompanied by his
physiotherapist with history of as if kicked in the heel & feeling something go while smashing.
On examination there is a palpable gap in the lower one third of calf. Likely diagnosis is:
• A Achilles tendan rupture*
• B Fracture lower one third of tibia
• C. Calcaneal fracture
• Tibial nerve injury
• Deep peroneal nerve injury
95. 95. Regarding lungs anatomical differences what is true?
• Right lung is divided into two lobes by a single oblique fissure.
• Right lung has a shorter and more vertical bronchus*
• Left lung is divided into three lobes by an oblique & horizontal fissure.
• Left lung has a shorter, wider & more vertical main bronchus.
• Right lung has a medial curvature for heart.
96. 96. 30 years old young male known smoker for the last ten years presented to accident &
emergency department with complaint of sudden right sided chest pain exacerbated by deep
breathing associated with shortness of breath for last 6 hours. On examination he is
tachycardiac oxygen saturation 85% at room air, absent breath sounds on right side with
hyper-resonant percussion note on the same side. Most likely diagnosis
• Secondary pneumothorax
• B. Open pneumothorax
• C. Primary pneumothorax*
• Acute exacerbation of asthma
• Subcutaneous emphysema
97. 97.56 years old male known case of pleural TB comes to you with shortness of breath. Chest
Xray shows massive right sided pleural effusion with collapsed right lung. What is the
management?
• A Needle thoracostomy
• B. Right sided chest drain with underwater seal*
• C Median sternotomy
• Open thoracotomy
• Video assisted thoracoscopic surgery (VATS).
98. 98. The most common cause of mesothelioma is:
• Asbestosis*
• Pneumoconiosis
• Cigarette smoking
• Alcohol
• Histoplasmosis
99. 99. 60 years old known smoker is brought to OPD with contrast enhanced CT scan chest which
shows large mass involving right main bronchus. What is the gold standard investigation for
diagnosis?
• Ultrasound chest
• Indirect laryngoscopy
• CT Scan chest
• Chest x ray PA view
• Bronchoscopy with biopsy of mass*
100. 100. 15 years old boy is brought to OPD by her parents complaining gradually
deforming anterior chest wall noticed initially 2 years ago. On examination the boy has a
pigeon like anterior chest. Patient has no other symptoms. What is this condition called?
• Kyphoscoliosis
• Pectus Excavatum
• Costochondritis
• Pectus carinatum*
• Thoracic intervertebral disc prolapse
101. 101. Most common vessel used in coronary artery bypass graft( CABG) surgery is:
• Radial artery
• Left internal mammary artery
• Great saphenous vein*
• Short saphenous vein
• Femoral vein
102. 102. Regarding coronary arteries.
• Left main coronary artery arises from aortic root*
• Right coronary artery arises from aortic root
• Left anterior descending artery is the least often bypassed vessel in coronary artery bypass
graft(CABG) surgery
• Lateral circumflex artery is most often bypassed during CABG C
• Left coronary continues as interventricular artery
103. 103. Gold standard investigation for imaging heart yesals to
• Echocardiography
• CT scan heart
• Cardiac enzymes
• Coronary angiography*
• Angioplasty
104. 104. Which of t of the following is the cause of Eisenmenger syndrome?
• Ventricular septal defect*
• Ischemic heart disease
• Rheumatic heart disease.
• Mitral stenosis
• Paravalvular leak
105. 105. 6 months old baby brought to emergency department with complaint of acute
abdominal gain. On examination the baby is severely dehydrated with massive abdominal
distension. All peripheral veins are collapsed. Central venous access is attempted but failed.
Next suitable site for venous access is?
• A External jugular vein
• 8 Femoral vein
• C Great sephanous vein
• Axillary vein
• Intraosseous cannulation*
106. 106. One year old baby is brought to emergency with complaint of left groin swelling
noticed one month ago. On there is a 23cm irreducible firm swelling in left inguinal area.
There is underdevelopment of left side of scrotum with absent left testicle. What is the
probable diagnosis?
• A Left epididymoorchitis
• Left sided testicular torsion
• Left sided cryptorchidism*
• Hydrocele
• Varicocele
107. 107. Which of the following is not an indication of circumcision?
• Phimosis
• Recurrent balanoposthitis
• Religious tradition
• Recurrent urinary tract infections
• Hypospadias*
108. 108. Five weeks old infant is brought to pediatric emergency with complaint of
projectile non bilious vomiting for the last 3 days. The baby seems hungry. On examination he
is dehydrated & vomited twice in A & E department. USG abdomen shows thickened pylorus.
What is the likely diagnosis?
• Acute intestinal obstruction
• B Infantile hypertrophic pyloric stenosis*
• C Intestinal intussusception
• Gastroesophageal reflux
• Acute pancreatitis.
109. 109. Eive weeks old infant is brought to pediatric emergency with complaint of
projectile non bilious vomiting for the last 3 days. The baby seems hungry. On examination he
is dehydrated & vomited twice in A & E department. Diagnosis of pyloric stenosis was made.
What is the appropriate definitive management?
• A Ramstedt's pyloromyotomy*
• Exploratory laparotomy & adhesiolysis
• Duodenal bypass
• Conservative management
• Distal Gastrectomy
110. 110. One year old child is brought to pediatric emergency with complaint of colicky
abdominal pain with bilious vomiting for 2 days. The child appears well between episodes of
pain. Pain is associated with currant jelly stools. On examination there is a sausage shaped
mass in right upper quadrant. Digital rectal examination reveals blood. What is likely
diagnosis?
• Infantile pyloric stenosis
• B Intussusception*
• Ischemic colitis
• Acute dysentery
• Caecal volvulus
111. 111.Eight months old infant is brought to pediatric OPD with history of on &
off.constipation for three months. On examination there is mild distension of abdomen.
Histopathology of rectal biopsy reveals absent ganglia in the biopsy specimen. What is the
underlying condition?
• Hirschprung disease*
• Ogilvie's syndrome
• Constipation
• Rectal polyp
• Anal atresia
• PRRE
112. 112.Six months old infant is brought to OPD with complaint of right sided testicular
swelling which increases in size when sits up or crawls, disappears on lying down. On
examination there is a reducible brilliantly transilluminant swelling in right side of scrotum.
Most like cause is?
• Hydrocele*
• 8. Testicular tumor
• Varicocele
• Undescended testis
• Epididymal cyst
113. 113. Testes start descending from which site during intra-embryonic life:
• A Lumbar area*
• Pelvis
• Hypochondrium
• Thorax
• Superficial inguinal pouch
114. 114. 4 months old infant has his external urethral opening on the ventral aspect of
shaft of penis. What best describes this statement?
• A Epispadias
• B. Hypospadiasis*
• C Phimosis
• Chordee
• Paraphimosis
115. 115. 20 years old ballet dancer falls on floor suddenly while dancing. She was brought
to accident & emergency department. The patient complains of pain & swelling left ankle. On
examination there is mild swelling around left ankle associated with redness. Movements at
ankle are restricted & painful. X ray left ankle is normal. What is the most likely diagnosis?
• Achilles tendon rupture
• B Left ankle sprain*
• Fracture left fibula
• Deep peroneal nerve injury
• Tibial nerve injury
116. 116. 40 years old overweight diabetic pat presents to OPD with complaint of tingling &
numbness in lateral three & half fingers of right hand. The pain is worse at night & reduces
when she hangs her hand out of bed. Most likely initial diagnosis is?
• Ulnar nerve compression
• Radial nerve compression
• C Median nerve compression*
• Brachial plexus compression
• Digital nerve compression
117. 117. 40 years old overweight diabetic patient comes to OPD with complaint of tingling
& numbness in lateral three & half fingers of right hand. The pain is worse at night & reduces
when she hangs her hand out of bed. What is the appropriate investigation to diagnose the
underlying condition?
• A Nerve conduction study*
• Electromyogram
• X ray right hand
• Doppler upper limb
• HbA1C
118. 118. 40 years old overweight diabetic patient presents to OPD with complaint of
tingling & numbness in lateral three & half fingers of right hand. The pain is worse at night &
reduces when she hangs her hand out of bed. Nerve conduction study shows compression of
median at carpel tunnel right wrist. What is the definitive treatment of this condition?
• Splint right wrist
• Analgesics
• Steroid injection
• Median nerve resection & graft placement
• Median nerve decompression*
119. 119. 30 years old unmarried female comes to OPD with complaint of gradually
growing swelling on dorsal surface of left wrist. There is 2*2cm smooth, fluctuant & brilliantly
transilluminant swelling at right wrist. What is the likely diagnosis?
• A Lipoma
• B. Sebaceous cyst
• C. Ganglion*
• Dermoid cyst
• Cutaneous Hemangioma
120. 120. 30 years old unmarried female comes to OPD with complaint of gradually
growing swelling on dorsal surface of left wrist. There is 2*2cm smooth, fluctuant & brilliantly
transilluminant swelling at right wrist. Diagnosis of ganglion right wrist is made. What is the
surgical treatment?
• Reassurance
• B Excision*
• C. Intralesional steroid injection
• Analgesics
• Incision & drainage
Rawal Surgery MCQs

Paper A
1. A patient has presented with acute abdomen. Which of the following investigations is the first
to be advised?
• Plain x-ray abdomen
• Barium meal
• CT scan
• Laparoscopy
2. Best imaging investigation for a stress fracture is:
• X-ray
• CT scan
• Bone scan
• MRI
3. Which of the following investigations will be required before elective surgery in a 45 years old
patient?
• Blood sugar
• Chest x-ray
• ECG
• All of the above
4. A male patient of 50 years complains of dyspepsia of recent origin. Which of the following
investigations will be your first choice?
• Stool examination for occult blood
• Barium follow through examination
• Upper GI endoscopy
• CT scan abdomen
5. Risk of postoperative infections can be minimized by following measures EXCEPT:
• Good patient preparation
• Good surgical technique
• Antibiotic prophylaxis
• Sterilization of surgeon’s hands
6. One gram of nitrogen is equivalent to how much protein?
• 6.25 gm
• 9.25 gm
• 13 gm
• 16 gm
7. What is meant by debridement?
• Excising one mm of skin from edge of wound
• Not excising skin but excising all damaged muscles
• Partial amputation
• Laying open all layers of a wound and excision of devitalized tissue
8. Metronidazole is active against:
• Anaerobes
• Clostridia
• Peptostreptococci
• All of the above
9. Antibiotic prophylaxis before elective surgery is provided with:
• Single low dose of broad spectrum antibiotic
• Multiple low doses of broad spectrum antibiotic
• Single high dose of broad spectrum antibiotic
• Multiple high doses of broad spectrum antibiotic
10. A patient presents with a discharging ulcer on right side of neck. The edgesof the ulcer are
undermined. What is the most probable diagnosis?
• Rodent ulcer
• Tuberculosis
• Nonspecific ulcer
• Epithelioma
11. Shock can occur in burn patients if the burn area exceeds this much percentage of body
surface area (BSA):
• 15 percent
• 25 percent
• 35 percent
• 45 percent
12. The leading cause of trauma during first 40 years of life is:
• Falls
• Acts of violence
• Traffic accidents
• Sports injuries
13. Open reduction and internal fixation (ORIF) of a fracture achieves:
• Accurate reduction
• Stable fixation
• Stronger fixation
• All of these
14. Which nerve is involved in Carpal tunnel syndrome?
• Ulnar
• Radial
• Median
• Lateral cutaneous nerve
15. In a healthy 70 kg male:
• Total body water measures approximately 40 liters
• Extracellular fluid volume is about 8 liters
• Plasma volume measures about 5 liters
• Resting energy expenditure decreases after major surgery
16. The pH of extracellular fluid :
• Is maintained in health between 7.4 and 7.8
• Is increased in hypovolemic shock
• Increases abruptly after cardiac arrest
• Should be monitored in a critically ill patient
17. Potassium deficiency should be suspected
• In paralytic ileus
• When patient’s reflexes are exaggerated
• If there is ‘tenting’ of T waves in patient’s ECG
• In renal failure
18. Extracellular fluid losses may be extensive in following clinical settings EXCEPT:
• Acute intestinal obstruction
• Hepatic coma
• Acute peritonitis
• Acute pancreatitis
19. Total parentral nutrition (TPN):
• Should deliver about 2500 calories/day to an adult
• Should deliver at least 30 gm nitrogen (180 gm protein)/day
• Is free of complications in today’s settings
• Provides better nutritional support than the enteral route
20. In hypovolemic shock:
• The extremities are warm and sweating
• The central venous pressure (CVP) is low
• There is always a source of bleeding
• Urine output is not affected
21. Cleft palate may be associated with following abnormalities:
• Otitis media and deafness
• Dental anomalies
• Speech impairment
• All of them
22. Cystic hygroma can occur at all of the following sites EXCEPT:
• Neck
• Head
• Limbs
• Inguinal region
23. A young female patient presents with diffuse goiter. Which of the following statements is
correct?
• It may be an early stage of endemic goiter
• Thyroid disease is more common in females
• Thyroid scan is not indicated in simple goiter
• All of these statements are true
24. Tetany may be a complication of thyroid surgery. In most cases it is latent tetany. What is true
about latent tetany?
• Carpopedal spasm is diagnostic
• It is picked by Chevostic’s sign and Trousseau’s sign.
• Parathyroid hormone level is increased
• Serum calcium is increased
25. Following is true about FNAC of thyroid EXCEPT:
• It is safe and reliable
• It is the least invasive investigation for diagnosis of breast cancer
• It can differentiate between follicular carcinoma and follicular adenoma
• It is highly accurate
26. Which of the following terms is most suitable to describe a tract that connects an epithelial
surface with an abnormal cavity lined with epithelium?
• Fistula
• Sinus
• Tunnel
• Inter-calated duct
27. The following organs are part of lymphatic system except:
• Spleen
• Liver
• Thymus
• Tonsils
28. Following are some anatomical facts about oesophagus. Which of these is INCORRECT?
• It is 25 cm long
• It extends from cricopharyngeal sphincter to cruri of diaphragm
• It is lined by squamous epithelium
• Its upper part has striated muscles
29. The liver has how many segments?
• 2
• 4
• 6
• 8
30. What is WRONG about hepatitis C?
• One percent blood donors re HCV positive
• Acute hepatitis C proceeds to cirrhosis in 20 % cases
• Chances of developing hepatic malignancy are negligible
• Deteriorating HCV encephalopathy is an indication for liver transplantation
31. Diabetic patients are high risk for surgical procedures because of:
• Poor wound healing
• Increased chances of wound infection
• Increased risk of MI and CVA
• All of the above
32. Water is lost from body through following routes EXCEPT:
• Skin and Respiratory tract
• Saliva
• Urine
• Faeces
33. The blood collection bags contain anticoagulant solution. This solution contains following
EXCEPT:
• Calcium
• Citrate
• Phosphate
• Dextrose
34. What is true about 5% dextrose saline solution?
• It is isotonic
• It is hypotonic
• It is hypertonic
• It is the best solution for use in shock
35. Which is the best dressing for a wound?
• Cotton gauze
• Skin
• Silver foil
• Aerosol spray
36. Following is true about keloid EXCEPT:
• It is an extensive overgrowth of scar tissue
• It is common in Africans
• It is a benign condition
• It is also called hypertrophic scar
37. Following are reservoir of HIV infection EXCEPT:
• Lymphocytes
• Macrophages
• RBC’s
• Neural cells
38. The wall of a true cyst is lined by:
• Epithelium
• Endothelium
• Granulation tissue
• Mesothelium
39. Transfer of tissues between two genetically identical individuals is called:
• Allograft
• Autograft
• Isograft
• Xenograft
40. The palm of a patient is approximately equivalent to how much of BSA (body surface area)?
• 1 percent
• 2 percent
• 3 percent
• 4 percent
41. The bone densitometry of spine, hips, and other bones is best done by following investigation:
• CT Scan
• Ultrasound
• Dual-energy x-ray absorption (DEXA) scan
• MRI
42. The bone infarct in acute osteomyelitis is called:
• Involucrum
• Sequestrum
• Cloaca
• Sinus
43. Glasgow coma scale applies:
• Eye opening
• Best verbal response
• Best motor response
• All of these
44. What percentage of lymph from breast is drained by the cervical lymph nodes?
• 50 percent
• 65 percent
• 100 percent
• These nodes do not directly receive lymph from breast
45. Following is true for an incised wound EXCEPT:
• It has minimal tissue damage
• Healing is completed by laying down of collagen fibers
• It heals with secondary intention
• It may lead to formation of keloid
46. Hypovolemic shock can occur in following settings EXCEPT:
• Generalized peritonitis
• Acute intestinal obstruction
• Gastric outlet obstruction
• Massive pulmonary embolism
47. Septic shock:
• Can only be caused by Gram negative bacteria
• Carries a favourable prognosis
• Is particularly associated with infective complications of gastrointestinal and genitourinary
systems
• Can be most effectively treated with antibiotics
48. In a blood transfusion reaction, following statements are correct EXCEPT:
• It may be due to an incompatibility of the recipient serum and donor’s cells
• Is manifested by thrombophlebitis of the infusion site
• It occurs within first 30 minutes of transfusion
• Must be suspected when the patient having blood transfusion complains of loin pain
49. Massive blood transfusion may be complicated by:
• Hypokalemia
• Hypercalcemia
• Leukopenia (deficiency of WBCs)
• Coagulopathy
50. In estimation of the area of burn, following statements are correct EXCEPT:
• When extensive, the estimation is best undertaken once patient has been resuscitated
• Provides important prognostic information
• It is an important factor for estimation of fluid requirements in such cases
• It is based on the ‘Rule of nine’
51. A clean incised skin wound:
• Commences epithelialization after 8-10 days
• Regains the full strength of a normal skin in about 10 week
• Regains its strength as a result of fibrolast activity
• Cannot undergo contraction
52. Staphylococcal infections:
• Do not cause cellulitis
• Do not produce septicemia
• Do not give rise to fever
• May be air borne infections
53. What is TRUE about tetanus?
• May have an incubation period of over 10 days
• Can be prevented by immediate administration of tetanus toxoid
• It is more common after scalp lacerations than wounds elsewhere on the body
• It is usually associated with stupor or coma
54. Only one of the following statements about ‘bed sores’ (Decubitus ulcer) is correct. Which
one?
• They can be prevented by sheepskin blankets
• Can be prevented by changing the patient’s position 4-6 times a day
• They occur only over the sacrum
• They may heal spontaneously with suitable care
55. The process of wound healing is delayed by following EXCEPT:
• Deficiency of vitamin C
• Vasodilator drugs
• Malnutrition
• Uremia
56. Water depletion in a patient cannot be caused by which of the following conditions?
• Diabetes inspidis
• Head injury
• Peripheral oedema
• Reduced intake
57. The body responds to surgical trauma by alterations in water and electrolyte balance. Which
of the following is NOT part of this phenomenon?
• Retention of water
• Retention of sodium
• Water loss due to sweating caused by fever
• Release of steroids and catecholamines
58. What is INCORRECT about vasovagal shock?
• Blood pressure is low.
• Pulse is bounding in character
• Patient has tachycardia
• It is due to expansion of vascular space
59. Metabolic acidosis can occur due to following reasons EXCEPT:
• Episode of diarrhoea
• Repeated vomiting
• High output fistula
• Renal failure
60. Which of the following statements about septic shock is INCORRECT?
• It is caused by endotoxins released by bacteria.
• It is associated with overwhelming infections
• Antibodies against bacteria have some role in causing this shock
• Cold variety represents advanced stage of this type of shock.
61. What is WRONG about an acute abscess?
• It is caused by pyogenic bacteria
• It may develop few days after an infection
• It can be treated by antibiotics alone
• It is associated with necrosis of tissue
62. A patient has presented with a long standing discharging sinus on chest wall. Following are
some causes of persistent sinus discharge. Which one is most UNLIKELY in this case?
• Infection with resistant microorganisms
• Tuberculosis of underlying rib
• Some foreign body in deeper tissues of the sinus
• Crohn’s disease
63. Tetanus is one of the serious complications of a wound. What is INCORRECT about this
condition?
• It can develop within few days after injury.
• It can be prevented by active or passive immunization.
• Patient should be kept in isolation because it can spread from one person to other.
• Penicillin is the drug of choice.
64. Which of the following statements is INCORRECT about second degree burns?
• They are very painful.
• They are due to partial loss of skin layers.
• They are pale in appearance.
• They are associated with significant fluid loss.
65. A patient has developed shock after trauma due to an accident. Which of the following
solutions has NO ROLE in treating hypovolemic shock in such a patients?
• Blood transfusion
• Ringer’s lactate solution
• Fresh frozen plasma (FFP)
• Colloids solution
66. An informed consent contains following information EXCEPT:
• Name of the patient
• Name of the operating surgeon
• Nature of the operation
• Diagnosis of the patient
67. Following is true about AIDS, EXCEPT:
• It is associated with lymphocyte deficiency and malfunction.
• Patient may have lymphadenopathy.
• Patient is at high risk for developing perianal fistula.
• Risk of developing various malignancies is increased manifolds.
68. Which of the following complications may NOT be related to blood transfusion?
• Incompatibility reaction
• Septic shock
• Transmission of viral disease
• Coagulation failure
69. What is the treatment for overdose of Heparin?
• Platelet transfusion
• Cryoprecipitate
• FFP transfusion
• Intravenous protamine sulphate administration
70. A dilated pupil is an important sign of head injury. What is the underlying cause?
• Anoxia
• Increased intracranial pressure
• Damage to the optic nerve
• Compression of 3rd cranial nerve
71. Anaerobic infections of wounds are liable to occur in following settings EXCEPT:
• Wide spread tissue damage
• Immuno-compromised status
• Leaving the wounds open after exploration
• Diabetic foot
72. TPN is often used in clinical settings. Which of the following statements is most appropriate
about TPN?
• It is preferred over enteral route
• It is indicated when patients is kept nil by mouth for longer than 2 days
• Carbohydrates, lipids and proteins should be given in a balanced formulation
• It should not be given by a peripheral vein
73. What is true about haemorrhage?
• Venous haemorrhage is bright red in colour
• Bleeding during surgery is reactionary haemorrhage
• Bleeding due to blunt injury of abdomen is concealed haemorrhage
• Continuous ooze of bright red blood is secondary haemorrhage
74. A patient has developed infection of laparotomy wound. The dressing is found stained with
bluish green discharge. Which of the following organisms is most likely cause?
• Clostridium welchii
• Pseudomonas aerogenosa
• Bacteroids fragilis
• Staphylococcus aureus
75. Following is NOT a possible factor for development of a tumour:
• Familial
• Virus
• Over eating, especially high protein diet
• Smoking
76. Sutures applied for the first time to a wound after a delay of few days are:
• Retention sutures
• Secondary sutures
• Delayed primary sutures
• Non absorbable suture
77. Following are acute infections, EXCEPT:
• Cellulitis
• Viral hepatitis
• Boils
• Tuberculosis
78. Which of the following is not a PHYSICAL means of sterilization?
• Steam
• Ethylene oxide gas
• Autoclave
• Radiation
79. Which of the following is a specific infection?
• Carbuncle
• Leprosy
• Gas gangrene
• Acute appendicitis
80. The average daily loss of fluid from skin and lungs in temperate climate is normally in the
range of:
• 350-500 ml
• 500-750 ml
• 750-1000 ml
• 1000-1500 ml
81. Sterilization is a process used to clear the instruments of:
• Bacteria
• Spores
• Viruses
• All of the above
82. Antibodies are formed by:
• Fibroblasts
• Giant cells
• T-lymphocytes
• B-lymphocytes
83. What is INCORRECT about Vicryl suture?
• It is absorbable suture.
• It gives rise to formation of stitch granuloma.
• It is used in intestinal anastomosis.
• It is synthetic in nature
84. Following is true for second degree burns EXCEPT:
• They are very painful.
• They heal from left over skin cells
• They are due to partial loss of skin layers.
• They are pale in appearance.
85. Which of the following incisions gives good looking scar?
• Transverse incision
• Midline incision.
• Incisions along Langer’s lines
• Vertical incision
86. A patient has developed injection abscess over right deltoid region. What is the best option for
treatment?
• Drain the abscess, start appropriate antibiotics, and close the wound.
• Drain the abscess, start appropriate antibiotics, and pack the wound daily.
• Keep him on broad-spectrum antibiotics only
• Massage the area with antiseptic creams
87. Passive immunity is available for which of the following diseases?
• Polio
• Tetanus
• Mumps
• Appendicitis
88. What is the HOLDING TIME for chromic catgut suture?
• 2-3 days
• One week
• Two weeks
• One month
89. Which one of the following is NOT a pre-malignant condition?
• Bowen’s disease
• Solar keratosis
• Marjolin’s ulcer
• Leukoplakia
90. Which instrument is used for putting endo-tracheal tube (ETT) to a patient?
• Oesophagoscope
• Laryngoscope
• Gastroscope
• It needs no instrument
91. How is scrubbing done at the patient before operation?
• Only incision site is scrubbed
• From periphery to incision site
• From incision site to periphery
• The whole body is scrubbed
92. Severity of tissue damage in wounds caused by bullets principally depends on:
• Size of the bullet
• Shape of the bullet
• Velocity of the bullet
• Weight of the bullet
93. What is CORRECT about a hypertrophic scar?
• It is same as a keloid
• It requires radiotherapy for cure
• It is a raised red scar that persists for 6 months before regressing
• It occurs only on the abdomen
94. What you mean by SECONDARY sutures?
• Skin closure of a wound 6 hours after surgery
• Skin closure of a wound one day after surgery
• Suturing with double-knot technique
• Suturing a wound again after control of wound infection
95. In blood bank, blood for transfusion is stored at what temperature?
• 20oC
• Zero degree C to 4oC
• + 2oC to + 6oC
• +10oC to + 14oC
96. Hepatitis B can be transmitted through transfusion of contaminated blood. How much time is
required to develop infection?
• One week
• Six weeks
• Three months
• Six months
97. The parotid gland is a major salivary gland in the body. How much of the parotid tumours are
likely to be malignant?
• About 20%
• About 40%
• Upto 60%
• All parotid tumours are malignant in nature
98. The term VENOUS PUMP refers to:
• A part of auto-transfusion apparatus
• An apparatus used for rapid transfusion of blood
• The musculofascial anatomy and physiology of the calf
• The system of venous valves in IVC
99. Wound healing is impaired by following EXCEPT:
• Marked anemia
• Haematoma formation
• Hyperthyroidism
• Steroids
100. A man bled excessively during prostatectomy and required blood transfusion. What should
be remembered before requesting for blood transfusion in such cases?
• Hypothermia is indicated if cryoglobulin is found
• Blood group AB is universal donor
• Cross matching should be done before dextran or colloid administration
• Serum of recipient stored for one week is best for testing
101. A 16 year old male with a history of severe hemophilia A is undergoing an elective inguinal
hernia repair .Which of the following is the best option for preventing or treating bleeding
complication in the setting of this disease
• Whole Blood
• Cryoprecipitate
• FFPs
• Factor 1X
102. The most sensitive test for the diagnosis of acute pancreatitis is
• Serum Amylase
• Serum Lipase
• Serum Albumin
• Serum LDH
103. Which of the following may result in increased incidence of wound infection
• Asthma
• Pneumonia
• Ischemic Heart Disease
• Diabetes
104. A 70 kg male presents with a 30 percent burn ,according to Parkland formula the fluid to be
replaced is
• 7000 ml
• 7400 ml
• 8000 ml
• 8400 ml
105. A 24 year old married female presents with pain in right lower abdomen.On examination her
pulse is 130/min and Blood Pressure is 60/40mmHg.There is generalized tenderness in
abdomen
106. What is the most likely diagnosis?
• Mittleshemerz
• Acute Appendicitis
• Torsion of ovarian cyst
• Rup tured Pregnancy
107. Myocardial depression seen in ischemic reperfusion injury is due to buildup of which of the
following
• HCO3
• K+
• Na+
• Mg++
108. Obstructive Shock may be caused by
• Hemothorax
• Cardiac temponade
• Pneumothorax
• Cardiac Myopathy
109. A teenage boy falls from his bicycle after being struck by a truck. On arrival in emergency
ward he is awake and alert. X-rays chest shows air fluid level in lower lung field. What is the
treatment?
• Tube thoracostomy
• Thoracotomy
• Endotracheal intubation
• Tracheostomy
110. Regarding scars,which one of the following statements is true?
• Hypertrophic scar extends beyond the original wound.
• Keloid scar does not extend beyond the original wound.
• Pathophysiology of both keloid and hypertrophic scar is excess deposition of collagen fibres
• Steroids have no role in its management.
111. A 70 years old male presents with gastric outlet obstruction secondary to inoperable
carcinoma of stomach which is the most appropriate method of providing nutrition
• Feeding jujenostomy
• Gastrostomy
• PEG
• NG feeding
112. An elderly male presents in the surgical OPD with high grade fever, cough, and dyspnea. On
chest X-rays there is a loculated air fluid level in the upper lobe of right lung. What is the
likely lesion?
• Empyema
• Lung Abscess
• Pneumonia
• Carcinoma Lung
113. A 24 year old male presents in Emergency Department after being stabbed with a knife. On
examination there is a 6 X 5 cm lacerated wound in periumblical region. He remains
hemodynamically unstable even after resuscitation. What is the most appropriate
management option?
• CT scan abdomen
• Intra Venous fluids and observe
• Laprotomy
• Diagnostic Peritoneal lavage
114. A 25 year old female presents after a fall from the second floor. She is confused, opens her
eyes to verbal command and localizes to pain. Her Glasgow Coma Score is
• 9
• 10
• 11
• 12
115. Following is a cause of adynamic intestinal obstruction
• Volvulus
• Intussception
• Adhesions
• Paralytic ileus
116. Prognosis in head injury is best given by
• Glasgow coma score
• Age of patient
• Mode of injury
• CT head
117. A 70 year old diabetic lady has below knee amputation for peripheral vascular disease. Post
operatively the wound becomes infected with excessive pain, crepitus, and brownish exudate.
The most likely organism is
• Staphylococcus aureus
• Bacteroides
• Streptococcus pyogenes
• Clostridium perferinges
118. A 40 years old male presented to the emergency department with history of road traffic
accident. She is brought unconscious and head injury is suspected. What is the single most
important investigation in this patient to diagnose head injury?
• X-ray skull
• CT scan brain
• Intracranial pressure monitoring
• MRI brain
119. A sixty five years old male has presented with acute onset of pain, swelling and erythema of
the left knee. He denies any previous history of trauma. The differential diagnosis includes
gout and septic arthritis. Which of the following is the best study to differentiate between
gout and septic arthritis?
• White blood cell count
• X ray of the knee joint
• Magnetic resonance imaging
• Evaluation of the synovial fluid
120. The most common malignant bone tumor in younger age is:
• Osteosarcoma
• Chondrosarcoma
• Metastases
• Ewing’s sarcoma
121. A forty years old male has injured his right shoulder while playing. There is immediate pain
and deformity. A dislocation of shoulder joint is suspected. Which type of shoulder dislocation
is most common?
• Posterior
• Anterior
• Superior
• Lateral
122. A 40 years old female has presented to the emergency with tachycardia, fever and confusion.
Workup revealed markedly elevated T3 and T4 levels. She is diagnosed with thyroid storm.
What is the most appropriate next step in management?
• Emergency total thyroidectomy
• Emergency subtotal thyroidectomy
• Emergency radiation to the neck
• Administration of fluids, antithyroid drugs, beta blockers, iodine solution and steroids
123. Considering hyperparathyroidism, which of the following patients should undergo
parathyroidectomy?
• A 62 years old asymptomatic woman
• A 40 year old asymptomatic man
• A 42 year old woman with kidney stones
• A 60 year old woman with mildly decreased bone mineral density
124. A 60 years old lady has presented to the outpatient department with history of a chronic
lesion over the right hand. She also gives history of radiation treatment for hand eczema.
Biopsy shows squamous cell carcinoma. Which of the following is true regarding this lesion?
• It is more malignant than basal cell carcinoma
• It is rarely associated with chronic sun exposure
• It does not metastasize to regional lymph nodes
• It should be treated by radiation therapy
125. A 40 years old pregnant woman complains of pain in wrist and parasthesia of her hand. She
has a history of trauma 1 week back.You suspect carpal tunnel syndrome. Which of the
following statements is true?
• It is rarely traumatic
• May be associated with pregnancy
• caused by compression of ulnar nerve
• Surgery is never required to treat this disease
126. A 60 year old man is concerned about a white patch in his oral mucosa. You diagnose
leukoplakia. Proper management of this lesion includes:
• Excisional biopsy
• Application of topical antibiotics
• Low dose radiation therapy
• Avoidance of alcohol and strict oral hygiene
127. A newborn male infant is brought to outpatient department because they worry about his
unilateral cleft lip and palate. Which of the following is true?
• This child certainly has other anomalies as well
• Speech therapy is an important part of rehabilitation
• Lip repair should be done at 2 years
• Palate repair should be done at 5 months
128. A 42 year old woman has a history of 3rd degree burn to her right arm in childhood. Now she
has come with an ulcerating lesion over the burn scar for 6 months. She states that this lesion
appeared after she had a fall and wound over the previous burn scar. Biopsy will most likely
reveal:
• Basal cell carcinoma
• Squamous cell carcinoma
• Keloid
• Malignant melanoma
129. A 50 year old man complains of calf pain and swelling following a tumor surgery 24 hour
back. Duplex scan confirms Deep Vein Thrombosis. Which of the following statements is true?
• She will not have long term complications
• Can be treated with pneumatic compression devices
• Can be treated with antibiotics and analgesics
• Patient is at increased risk of pulmonary embolism
130. A 35 year old male has presented to the outpatient department with history of painful right
neck swelling for 6 months. He gives history of weight loss and evening pyrexia. On
examination there are matted cervical lymph nodes. His ESR is 70mm/hr. Biopsy of this
swelling will most likely reveal:
• Acute suppurative inflammation
• Squamous cell carcinoma
• Thyroid neoplasm
• Chronic granulomatous inflammation
131. Regarding thyroglossal duct cyst, which of the following statements is true?
• Arises from first pharyngeal pouch
• Is an acquired condition
• Definitive treatment is called Systrunk operation
• Is a pre-malignant condition
132. Calcium absorption occurs in
• The body of the stomach
• The antrum of the stomach
• The duodenum
• The proximal / Jejunum
133. The most commonly used imaging method for diagnosis of acute cholecystitis is:
• CT of the abdomen
• Ultrasonography of the gallbladder
• Oral cholecystogram
• Radionuclide (HIDA) scan of the gallbladder
134. Which of the following statements about achalasia is correct?
• In most cases in North America, the cause is a parasitic infestation by Trypanosoma cruzi
• Chest pain and regurgitation are the usual symptoms
• Distal-third esophageal adenocarcinomas may occur is as many
• as 20% of patients within 10 years of diagnosis
• Endoscopic botulinum toxin injection of the LES, pneumatic dilatation, and
esophagomyotomy provide highly effective curative therapy for achalasia
135. A 50 years old male, with a 2 year history of duodenal ulceration, develops sudden severe
epigastric pain 4 hours prior to evaluation. Physical examination reveals normal temperature
o F, pulse 80, BP 125/90, diminished bowel sounds, and abdominal muscular rigidity. An
upright chest X-ray reveals pneumoperitoneum. At laparotomy, an anterior perforation in the
first portion of the duodenum is observed. Optimal treatment would include:
• Omental patch of the perforation followed by truncal vagotomy and antrectomy after 8
weeks
• Omental patch of the perforation by chronic cimetidine administration
• Omental patch of the perforation plus proximal gastric vagotomy
• Omental patch of the perforation only
136. A 36 year old woman complains of a 3 month history of blood distained from the left nipple.
At examination a small nodule is found deep to the areola careful palpation of the nipple
areolar complex results in blood arrearing at the 3 O’clock position. Mammogram findings are
normal. What is the likeliest Diagnosis?
• Intraductal papilloma
• Breast cyst
• Intraductal carcinoma
• Fat necrosis
137. the lower end of the CBD. Clinical examination wound most likely rveal which of the
following?
• Enlarged gallbladder
• Shrunken gallbladder
• Enlarged pancreas
• Palpable tumor
138. 36 years old man, after being hit by a car, presented to ER with hypotension. On
examination, there is tenderness and bruising over his left lateral chest below the nipple. An
ultrasound examination is performed and reveals free fluid in the abdomen. What is the most
likely organ to have been injured in this patient?
• Liver
• Kidney
• Spleen
• Intestine
139. Sloping edge is characteristic of:
• Septic ulcer
• Syphilitic ulcer
• Carcinomatous ulcer
• Tubercular ulcer
• venous ulcer
140. A tunnel connecting two epithelial surfaces is:
• Sinus tract
• Fistula
• Duct
• All of the above
• Tract
141. In evaluation of a patient for bleeding disorder, the single most useful test is
• Bleeding time
• Clotting time
• Partial thromboplastin time
• Platelet count
• Prothombin time
142. Debridement means:
• Excising I mm of skin from the edge of a wound
• Not excising skin but excising all damaged muscle
• Amputation
• Laying open all layers of a wound and excision of devitalized tissue
• Suturing of a wound
143. All are true about keloids except:
• Is an extensive overgrowth of scar tissue
• Common is Africans
• Is a benign condition
• May occur on the ear lobes
• Is the same as a hypertorophic scar
144. Type of haemorrhage during operation
• Primary haemorrhage
• Reactionary haemorrhage
• Secondary haemorrhage
• External haemorrhage
• Internal haemorrhage
145. Normal central venous pressure is:
• 0-5cm saline
• 5-8cm saline
• 8-12cm saline
• 12-15cm saline
• e 15-20 cm saline
146. Focused abdominal sonogram in abdominal trauma is done to primarily rule out all except:
• Cardiac tamponade
• Hepatic Injury
• Splenic injury
• Renal injury
• mesenteric vascular injury
147. Compartment syndrome is common in:
• Upper arm
• Fore arm
• Thigh
• Foot
• Leg
148. Complication of anterior shoulder dislocation include all except:
• Axillary nerve injury
• Fracture neck/greater tuberosity of humerus
• Radial nerve injury
• Recurrent dislocation
• Axillary artery injury
149. The common form of elbow dislocation is:
• Anterior
• Posterior
• Poster lateral
• Anteromedial
• Lateral
150. Which type of fracture of humerus usually damages radial nerve:
• Fracture proximal third
• Fracture of middle third
• Spiral fracture of distal third
• Inter condylar fracture
• Fracture of greater tuberosity of humerus
151. The most common cause of shoulder pain is due to disease tendon of:
• Supraspinatus
• Infraspinatus
• Teres Major
• Biceps
• Teres minor
152. Unstable pelvic fractures are notorious for:
• Damage of rectum
• Torrential bleeding
• Malunion
• Noununion
• Urinary bladder injury
153. Long thoracic nerve damage causes paralysis of:
• Deltoid
• Brachioradials
• Serratus anterior
• Pectoralis major
• Pectoris minor
154. 80%of all salivary stones occur in:
• Parotid
• Submandibular
• Submaxillary
• Sublingual
• Minor salivary glands
155. The most common salivary tumour is:
• Pleomorphic adenoma
• Adenolymphoma
• Epidermoid carcinoma
• Adenocarcinoma
• Aanaplastic carcinoma
156. Spot the wrong statement:
• 75% of salivary neoplasms arise in parotid
• 80% of parotid tumours are benign
• 80% of benign parotid tumours are pleomorphic adenoma
• All are correct
• None is correct
157. Adenolymphoas are common in:
• Parotid gland
• Submandibular gland
• Minor salivary gland
• sublingual gland
• All of the above
158. Malignant parotid tumour is characterized by:
• Pain
• Hard and fixed mass
• Paralysis of facial muscles
• Progressive increase in size
• All of the above
159. Pain in the arm due to cervical rib is caused by:
• Compression of D1 root,
• Compression of C7 root
• Muscle ischaemia
• Compression of lower cord of brachial plexus
• All of the above
160. The primary growth commonly metastasizing to cervical lymph nodes are all except:
• Nasopharyngeal cancer
• Cancer in sinuses, laryngopharynx and vallecula
• Breast cancer
• Thyroid cancer
• Tongue cancer
161. The axillary lymph nodes receive what percentage of lymph from breast
• 50%
• 75%
• 90%
• 100%
162. A blood stained nipple discharge is commonly caused by:
• Duct ectasia
• Duct papilloma
• Duct carcinoma
• None of the above
• Blockage of lactiferous ducts
163. Male breast cancer is due to excess of:
• Androgen
• Estrogen
• Both androgen and estrogen
• Not related to hormones
• Progesteron
164. Cleft palate repair is ideal at:
• 6 months
• 6-18 months
• 12-14 months
• 2 -4 years
• 6-8 years
165. A lateral palatal swelling can be due to:
• Malignancy of maxillary sinus
• Palatal salivary gland tumour
• Neurofibroma of greater palatine nerve
• Squamous carcinoma of palate
• Any of the above
166. 28 Submandibular gland is related to following expect
• ingual nerve
• hypoglossal nerve
• facial artery
• facial vein
• facial nerve
167. Characteristics of cystic hygroma include all except:
• Develops from jugular lymphatics
• Brilliantly translucent
• Typically occupies the middle third of neck
• Enlarges when the child cries
• is multiloculated
168. The thyroid stimulating antibody is:
• IgG
• IgM
• IgA
• IgD
• Ig E
169. Q$1.At surgery for a right inguinal hernia, a 72-year old man is found to have a hernia sac
that is not independent of the bowel wall. The cecum forms part of the wall of the sac. Such a
hernia is properly referred to as which of the following?
• Incarcerated
• Irreducible
• Sliding*
• Richter's
• Interstitial
170. Q#2.In repair of a femoral hernia, the structure most vulnerable to major Injury lies:
• Medially
• Lateraly*
• Anteriorly
• Posteriorly
• Superficially
• B
171. Q#3.A 56-year-old man is scheduled to have a left indirect hernia repaired. He is
asymptomatic. Before surgical treatment, he should have which of the following?
• Rectal examination alone.
• B Rectal examination and sigmoidoscopy*
• Barum enema'
• Colonoscopy
• Intravenous pyelogram
172. Q#4.A 42-year-old man presents with cancer of the left testis. To exclude lymphatic
metastasis, which is the site that should be Initially examined?
• Vertical chain of inguinal glands
• Horizontal chain of inguinal glands
• Retrorectal glands
• Para-aortic glands*
• Obturator nodes
173. Q#5. A 42-year-old woman involved in a traffic accident presents to the emergency room
complaining of flank pain and gross hematuria, she is Hemodynamically stable. The next step
in management is: B
• Exploratory laparotomy
• Open lavage and, if positive, immediate laparotomy*
• Immobilization of the pelvis
• Computed axial tomography (CAT) scan with the use of intravenous contrast
• Skeletal traction
174. Q#6. During evaluation of the cause of varicocele in year-old man, attention is method of
drainage of the left testicular vein, which which of the following?
• Lefi adrenal vein
• Left renal vein
• Left inferior mesenteric vein
• Inferior vena cava (IVC) *
• Left inferior epigastric vein
175. Q#7. À 64-year-old male is admitted to the emergency department following a car accident.
His pulse is 94 bpm, blood pressure 95/60 mm Hg, and HCT 30%. Severe hematuria is evident.
Following resuscitation, his blood pressure is elevated to 120/80 mm Hg. A CT scan reveals
extensive contusion confined to the left kidney and perirenal fat. His blood pressure declines
to 80/40 mm Hg, and urgent laparotomy is performedvia?
• Through a left Blank incision
• BThrough a midline abdominal incision*
• Through an Gibson incision
• Through a thoracoabdominal incision
• Through an inguinal incision
176. Q#8. A 56-year-old male has history of leg pain at rest. Patient also has history of severe
coronary artery diseases. He cannot walk two flights of steps without getting short of breath.
He underwent evaluation and was noted to have complete aortoillac occlusive disease. He
heeds surgery. Which one of the following options is acceptable?
• Aortobililiac bypass
• Aortobifemoral bypass
• Aortoiliac angioplasty and stent placement*
• Axillobifemoral bypass
• Axilloiliac
177. Q#9. A middle-aged man is found to have a small pulsating mass at the level of the umbilicus
during a routine abdominal examination. What is the best initial test to establish the
diagnosis?
• Aoriography
• Ultrasound*
• Computed tomography (CT).
• Magnetic resonance imaging (MRI)
• Plaih films of the abdomen.
178. Q#10. Four days after undergoing hysterectomy, a 30. year-old woman develops phlegmasia
cerulean dolens over the right lower extremity. What is the most appropriate treatment?
• Bed rest and elevation
• (B)-Systemic heparinization
• Venous thrombectomy
• Prophylactic vena caval filter *
• Local urokinase infusion
179. Q#11. A21-year-old woman is referred to your office because of multiple lower extremity
varicose veins. She has large varicosities in the distribution of the long saphenous vein. What
is the next step in management?
• A ligation and stripping operation
• Ligation of both the long and short saphenous system
• Sclerotherapy
• Duplex evaluation along with clinical correlation as an essential initial step*
• Compression stockings and anticoagulation therapy
180. Q#12:A years old male patient was diagnosed case right submandibular gland tumor and was
operated days back. Now the ward he complains absence sensations the
submentairegion.What complication occurred?
• A Facial nerve injury
• Lingual nerve injury
• CHypoglossal nerve injury
• Injury to nerve mylohyoid*
• Marginal mandibular nerve injury
181. Q#13. A 72-year-old woman falls at home after an episode of dizziness. She had been
complaining of low-back pain..for 3. days before the fall. In the emergency department, she is
hypotensive and has cold, clammy extremities. A pulsating mass is palpable on abdominal
examination. Following resuscitation, the next step in the management should involve which
of the following?
• Peritoneal lavage.
• immediate abdominal exploration*
• CT scan of the abdomen
• Abdominal aortogram..
• Abdominal ultrasound
182. Q#14. A young patient sustains blunt trauma to his right knee that results in acute
thrombosis of his popliteal artery. Which tissue is most sensitive to Ischemia?
• Muscle
• (B Nerve*
• Skin
• Fat
• Bone
183. Q#15. Ahomeless elderly man is brought to the emergency department after sustaining
frostbite to bath feet What is the most appropriate immediate management?
• (Slow rewarming at room temperature
• B) Amputation of the gangrenous toes
• Rapid rewarming with warm water*
• Rapid rewarming with hot water or dry heat.
• Thorough debridement of blisters and devitalized tissue
184. Q#16. 1. A 45-year-old man complains of burning epigastric pain that wakes him up at night.
The pain is relieved by eating or using over-the counter antacids and H2 blockers. Diagnosis is
best confirmed by which of the following?
• Urea breath test
• Serum gastrin levels
• Barium meal examination
• Upper endoscopy
• Upper endoscopy and biopsy*
185. Q#17. Ahealthy 75-year-old man bleeds from a duddenal úcter Medical management and
endoscopic measures fall to stop the bleeding. What is the next step in management?
• Continued transfusion of 8 U of blood
• Administration of norepinephrine
• Oversewing of the bleeding point
• Oversewing of the bleeding point, vagotomy andpyloroplasty*
• Hepatic artery ligation.
186. Q#18: A 64-year-old man develops increasing dysphagia over many months. A barium
swallow is performed. What is the most likely cause of his clinical presentation?
• Carcinoma of the esophagus*
• Achalasia
• Sliding hiatal hernia
• Paraesophageal hernia
• Esophageal diverticulum
187. $19: A63-year-old woman from Norway is visiting ne United States. She presents with
dysphagia. On endoscopy, an esophageal web is identified and the diagnosis of Plummer-
Vinson syndrome is established. What would be the next step in management?
• Esophagostomy
• Dilatation of the web and iron therapy*
• Esophagectomy
• Gastric bypass of the esophagus
• Cortisone
188. Q#20:A 24 year old male presents in Emergency Department after being stabbed with a knife.
On examination there is a 6 X 5 cm lacerated wound in periumblical region. He remains
hemodynamically unstable even after resuscitation. What is the most appropriate
management option?
• CT scan abdomen
• Intra Venous fluids and observe
• Laprotomy
• Diagnostic Peritoneal lavage*
• E Diagnostic Laproscopy
189. Q#21: A 60-year-old man presents with excruciating chest pain. The pain follows an episode
of violent vomiting that occurred after a heavy meal. Subcutaneous emphysema was noted in
the neck. X-rays shows air in the medlastinum and neck, and a fluid level in the left pleural
cavity. What is the most likely diagnosis?.
• Perforated duodenal ulcer.
• Spontaneous rupture of the esophagus*
• Spontaneous pneumothorax
• Inferior wall myocardial infarction
• Dissecting aortic aneurysm
190. Q#22: A69-year-old man is admitted to the emergency department with an acute UGI
hemorrhage following a bout of repeated vomiting. Fiber opticgastroscopy reveals three linear
mucosal tears at the GE junction. What is the diagnosis?
• Reflux esophagitis with ulceration
• Barrett's esophagus
• Carcinoma of the esophagus
• Mallory-Weiss tear*
• Scleroderma
191. Q#23: A '25-year-öld man arrives in the emergency department in respiratory distress
following a motor Vehicle collisión..'A chest x-ray shows abdominal viscera in the left thorax.
What is the most likely diagnosis?
• A Traumatic rupture of the diaphragm*
• Sliding esophageal hernia
• Shortiesophagus with intrathoracic stomach
• Rupture of the, esophagus
• Bochdalek hernia
192. Q#24: A 44-year-old patient develops a mass on the anterior abdominal wall. He. notes that
the mass has gradually Increased in size over the last 3 months. On examination, the lesion is
a 5 x 8 cm mass in the left iliac fossa and hypogastrium. Which test will establish whether the
tumor is arising from the abdominal wall or the abdominal cavity?
• Needle biospy
• Ability to elicit a cough impulse
• Transillumination
• Examination of the mass with the patient in a prone position*
• Examination of the mass with the patient instructed to attempt sitting up
193. Q#25:A 26-year-old man is diagnosed with adenocaroinoma of the stomach. He wants to
know what could have caused him to develop this conditions He does an internet search.
Which of the following is a risk factor for developing gastric cancer?.
• Exposure to ionizing radiation
• Blood group B
• A diet high in fiber
• H. pylori infection*
• North American descent
194. Q#26: A50-year-old vehicle collision suffers multiple trauma. He admitted the Intensive care
unit. After days of hospital admission he bleeds massively from the stomach. the probable
cause?
• Duodenal ulcer
• (SYHiatal hernia
• Mallory-Weiss tear
• Erosive gastritis*
195. Q#28:A40-year-old lawyer comes into your office after seeing some information on the
Internetrelating to breast cancer. Which of the following factors has not shown to increase a
woman's riskfor breast cancer?
• AVSmoking
• Previous history of benign breast biopsies*
• (C) Atypiaseen on pathology from previous breast biopsy
• First-degree relative with history of breast cance
• Increasing age
196. A 56-year-old male is burned while sleeping in his home. His right upper and lower extremity
and the anterior aspect of the upper chest have extensive second-degree burns.
• Q#29: A second-degree burn is characterized by which of the following?
• Coagulative necrosis extending to subcutaneous fat
• Pearly white appearance
• Anaesthetic
• Erythema and bullae formation*
• Requires immediate skin grafting
197. Q#30: A40-year-old lawyer comes into your office after seeing some information on the
Internet relating to breast cancer. Which of the following factors has not shown to increase a
woman's risk for breast cancer?
• Smoking
• Previous history of benign breast biopsies*
• Atypia seen on pathology from previous breast biopsy
• First-degree relative with history of breast cancer
• (E). Increasing age
198. Q#31: A43-year-old teacher underwent left parotidectomy awakening-from surgery, paralysis
of the left lower lip was observed. This complication was most likely due to injury to which of
the following:
• Parotid duct
• (BVFacial nerve temporal branch
• Facial nerve cervical branch*
• Facial nerve - main trunk
• Platysma muşcle:
199. Q#32: A 60-year-old diabetic man is admitted to the hospital with a diagnosis of acute
cholecystitis. The WBC count is 28,000, and a plain film of the abdomen and CT scan show
evidence of Intramural gas in the gallbladder. What is the most likely diagnosis?
• A Emphysematous gallbladder*
• Acalculous Cholecystitis
• Cholangiohepatitis
• Sclerosing cholangitis
• Gallstone ileus
200. Q#33: Splenectomy: is often indicated in the management of which of the following?
• Hereditary Spherocytosis *
• (B) Hereditary neurofibromatosis
• (C) Aplastic anemia
• Phéochromocytoma
• Hashimoto's disease
201. Q34. A 62-year-old postal officer develops minimalurinary symptoms. His PSA level is
elevated and continues to increase during a 6-month period of observation. The next step in
evaluation, if transrectal ultrasound (TRUS) prostate biopsy (Fig. 9-1) were positive for
adenocarcinoma of prostate, would be:
• Refer to oncologist for chemotherapy
• Metastatic evaluation including CT and bone scans*
• Repeat PSA and biospy
• Evaluation by radiation oncologist
• Start hormonal ablation treatment
202. #3 A 27 year old man presents in the ER after a high speed motor vehicle collision with chest
pain and marked respiratory distress. On physical examination, he is hypotensive with
distended neck veins and absence of breath sounds in the left chest. Which of the following is
the proper initial treatment?
• Intubation
• Chest X-ray
• Pericardiocentesis
• Chest decompression with a needle*
• E.: Emergent thoracotomy
203. Q36. During a workup for infertility, a 34-year-old man is noted to have a solid tumor in the
anterior aspect of his right testis. What is the most likely diagnosis?
• Torsion of the testis
• Cyst of the epididymis
• Lipoma of the cord*
• (DYCancer of the testis
• Epididymo-orchitis
204. Q37. An Otherwise Healthy, 30-Year-Old Man Is Brought To The Emergency Department After
Being Thrown Off The Back Of A Motorcycle. During The Assessment, Blood Is Noted At The
Urethral Meatus, Which Of The Following: Statement Is True?
• A foley catheter should be inserted immediately.
• Dislocation of the sacroiliac joint is usually associated with fracture of the pubic ramus
separation of the symphysis. *
• Operi lavage is a useful indication for the need to perform laparotomy.
• Fracture of the coccyx requires surgical excision in most patients: a
205. Q38. A63-year-old man undergoes a peripheral vascular procedure under general anesthesia.
A decrease in urine formation and excretion are noted. Decreased urine flow under general
anesthesia occurs because of which of the following?
• Vasopressin
• Aldosterone suppression*
• Depression of glucocorticoid
• Depression of thyroid function
• Specific effect of anesthesia on renal tubules
206. Q39. A middle-aged. man undergoes a left belowknee amputation for left-foot gangrene
secondary to arterial occlusive disease. Which of the following statements is true after the
belowknee amputation?
• There is less efficient function than after a through knee amputation
• Stump prognosis can be judged by transcutaneous oxygert monitoring*
• Poor prognosis is inevitable if Doppler fails to record a pulse at that level.
• The fibula and tibia are of equal length.
• The level of transection is 5 cm above the medial malleolus.
207. Q41: A 78-year-old woman undergoes an uncomplicated minor surgical procedure under
local anesthesia. At the completion of the operation, she suddenly develops pallor, sweating,
bradycardia, hypotension, abdominal pain, and gastic, "distension. What is the next stem in
management?
• Rapid infusion of 3 L' of Ringer's lactate
• (B), Digoxin
• Insertion of a nașogastric tube
• Morphine
• Neostigmine*
208. Q40, A70-year-old man with a long-standing history of diabetes develops gangrene of the
right second toe. What is true of his diabetic foot?
• (A Borsalispedals and posterior tibia arteries are always absent.
• (B).-Gangrene of the toe always requires urgent below-knee amputation.
• Arterial reconstruction is invariably required.
• His right femoral artery is most probably occluded or stenosed.
• Trophic ulcers are sharply demarcated. *
209. Q42. A 40-year-old man is involved in a car crash, presenting with blood pressure of 80 mm
Hg. The patient is found to have subdural hematoma and supracondylar fracture of the left
femur FAST shows fluid within the abdomen. He is taken to the OR, where intra-abdominal
bleeding, is controlled, and the subdural hematoma is drained. The femur fracture (Fig. 12-1)
should be treated by which of the following?
• Long-leg cast
• Steinmann pin insertion and traction
• Operative reduction and internal reduction
• Aspiration of knee joint
• Operative reduction with internal fixation*
210. 043. A newborn boy was examined to exclude Congenital dislocation of the hip (CDH). Which
of the following tests is relative to the management of CDH?
• The diagnosis should be established between 2 and years of age:
• Abduction of the flexed hip causes a click (Ortolani's Sign) (Fig: 12-5): *
• Abduction of the hip 'is not limited.
• Apparent lengthening of the thigh with the hip and knee flexed may be seen.
• Open reduction usually is required
211. 645 A70-year-dld man has had a long-term "bowlegged" condition but recently his right knee
has become warm, swollen, and tender. He reports no recent trauma and gets no relief with
rest or Tylenol (paracetamol). He is otherwise in good health and takes no medication. X-
raysshow arthritis of the knee. Which would be the best treatment?
• (A Bed rest, anti-inflammatory agents, analgesics, and a knee brace*
• Use of a cane for ambulaling, restriction of knee bending'activities, and implementation of
Muscle-strengthening exercises
• intra-articular steroid injection, bed rest, and analgesics
• Long-leg casiland crutches for 3 weeks, analgesics. and anti-inflammatory agents
• Urgent surgical correction
212. Q44. A7-year-old boy falls, off his bicycle, landing on the left elbow. He presents to the
emergency room with massive, tense swelling of the elbow with painful and restricted elbow
motion. X-rays show a displaced fracture of the distal end of the humerus. Which of the
following is the most serious complication of this fracture?
• Nonunion of fracture fragments
• Nonunion of fracture fragments with deformity
• 6) Disruption of the growth plate at the distal end of the bumerus
• Forearm compartment syndrome (Volkmann's ischemia) *
• Ankylosis of the elbow joint
213. Q46. A 40-year-old woman was involved in a car crash. She was unconscious for 5 minutes. X-
ray Revealed a depressed fracture in the frontal region. Which of the following statements is
true of skull fracture?
• It always requires surgical exploration.
• It is compound if multiple.
• It requires burr holes if compound.
• On the anterior cranial fossa, it may produce rhinorrhea*
• It requires steroid administration.
214. Q47. A 35-year-old professional dancer presents with a well defined, tense, smooth mass in
the upper 'Quter quadrant of the left breast. She states that the mass, becomes larger just
before onset of her periods. Aspiration yields a clear yellow fluid and the mass disappears:
The most likely diagnosis is; !..
• Fibro adenoma is a cyst.
• Fibrocystic disease of the breast*
• Carcinoma in a cyst..
• Lipoma.
• Galactocele.
215. Q49. A67-year-old woman is evaluated for obstructive jaundice. The cholangiographic
findings indicate that she has a cancer of the lower end of the CBD. Clinical examination would
most likely reveal which of the following?
• A Enlarged gallbladder*
• (B) Shrunken gallbladder
• (C) Enlarged pancreas
• Shrunken pancreas
• Palpable tumor
216. 50. A 40-year-old man underwent laparoscopic cholecystectomy 2 years earlier. He remains
asymptomatic until 1 week before admission, when he complains of RUQ pain and jaundice.
He develops a fever and has several rigor attacks on the day of admission. An ultrasound
confirms the presence of gallstones in the distal CBD. The patient is given antibiotics. Which of
the following should be undertaken as the next step in therapy?
• Should be discharged home under observation
• Should be observed in the hospital
• Undergo surgical exploration of the CBD
• ERCP with sphincterotomy and stone removal*
• Anticoagulants
217. Which of the following is the composition of Normal Saline?
• 120 meq/L of NaCl
• b- 135 meq/L of NaCl
• 138 meq/L of NaCl
• d. 145 meq/1. of NaCI
• e-154 meq/l. of NaCl*
218. Calculate of the following gives a good-looking acar:
• Transverse incision.
• Midline incision.
• Incisions along Langer's lines. *
• Vertical incisions.
• Horizontal incisions.
219. While taking the blood pressure of a post-thyroidectomy patient you notice te-hand going
into a spasm, which of the following is likely to be responsible for this?
• Hypercalcemia
• b- Hypocalcemia*
• Hypokalemia
• d Hypermagnesimea
• e Hypomagnesimea
220. Excessive 25 years old male is brought in emergency department after a road traffic accident.
He has crush injuries of lower limbs with skin avulsions and plenty of debris and dead tissue.
What is the most appropriate initial management of this type of wound?
• Suturing the wound
• Immediate vacuum assisted skin closure
• Saline irrigation and debridement of the wound*
• Skin grafting in first 6 hours
• Intra venous antibiotics
221. Which of the following is a method of provision of parenteral nutrition?
• NGtube
• Jejunostomy
• Gastrostomy
• CVP line*
• Whole food by mouth
• D.
222. What is the most common cause of a transfusion reaction?
• a-Air embolism
• b- Contaminated blood
• e-Unusual circulating antibodies
• d- Human error*
• e-Graft vs Host disease
223. Which of the following clotting factors is the FIRST common factor in both intrinsic and
extrinsic pathways?
• Factor X*
• Factor I
• Factor V
• Factor IX
• Factor XI
224. 45 years old builder brought in to A&E is unable to move after he fell on his back and is
suspected of a spine injury. He has warm perfused extremities, pulse rate of 50/min and Blood
pressure of 70/30 mm Hg. What is the type of shock in this patient?
• Hemorrhagic shock
• Septic shock
• Cardiogenic shock
• Neurogenic shock*
• Restrictive shock
225. Which of the following statements regarding starvation is true?
• After a 12 hour fast plasma insulin levels rise
• Energy expenditure is higher in starvation
• All of the body's glycogen is stored in the liver
• The breakdown of protein during starvation cannot be prevented
• Proteins and fats are broken down when starvation is prolonged*
226. The basal energy expenditure of a 60 kg healthy man would be?
• #- 2500 Kcal/day
• b- 1000 Kcal/day
• c- 1800 kcal/day*
• d- 1200 kcal/day
• e-3000 kcal/day
227. A patient presents with Megaloblastic anemia after gastrectomy, the deficiency of which of
the following is responsible?
• Vitamin D
• Vitamin A
• Vitamin C
• Vitamin B-12*
• Vitamin E
228. D34. After what period of expected starvation should Parenteral Nutritional support? you
consider
• 1 day
• 5-7 days*
• C-2 weeks
• 12 hours
• e 1 month
229. 35. Which of the following tools is used in assessing the Nutritional status of patients?
• a MUST score*
• b- APACHE score
• c-Ranson's score
• Glasgow score
• e-Alvarado score
230. 36. What is the expected normal BMI of a 70 kg male?
• 15-20
• 20-25*
• C- 26-30
• 31-35
• 35-40
231. Which of the following is used as a Clinical measure of Nutritional assessment?
• & Triceps Skin fold thickness*
• b- Blood pressure pattern
• Dryness of tongue
• Walking distance
• Ability to climb flight of stairs
232. An 80 years old frail and cachectic man is diagnosed with Esophageal malignancy after having
dysphagia for 9 months. What would be the best method to improve his nutritional status
prior to surgery?
• #- Keep NPO
• b- Partial Parenteral Nutrition
• c- Total Parenteral Nutrition
• Feeding Jejunostomy*
• IV Fluids
• 45 old male patient has suffered from myocardial infarction one month back and wants
surgery for Inguinal hernia. Which one of the following is true?
• Surgery should never be done on this patient
• Surgery should be performed after 6 months*
• Surgery can be done on the next operation list
• Surgery should be performed after one month
• Surgery should be performed after eighteen months
233. 40.A. middle aged man presents with pain RUQ, fever, anorexia, malaise and cough. He is
toxic, anemic and mildly jaundiced. There is tender hepatomegaly with tenderness over Right
lower intercostal spaces. USG showed a hypoechoic lesion in the right lobe of liver and CT
showed a fluid filled cavity in the right lobe of liver. What is the likely diagnosis?
• Hydatid cyst
• Amoebic abscess*
• Hamartoma
• Adenoma
• Focal Nodular hyperplasia
234. A 19 years old lady presents with intermittent colicky abdominal pain for 6 months. There is
history of alternating constipation and diarrhea. She recently lost her father to a prolonged
respiratory illness. On examination there is fullness in the RIF and a Barium follow-through
showed irregular narrowing of terminal ileum. What is the most likely diagnosis?
• s- Actinomycetoma
• b- Ascariasis
• c Hydatid Disease
• d- Ileocaecal tuberculosis*
• e-Psoas abscess
• D
235. 42, 42year old woman has a history of 3rd degree burn to her right arm in childhood. Now
she has come with an ulcerating lesion over the burn scar for 6 months. She states that this
lesion appeared after she had a fall and wound over the previous burn scan. Biopsy will most
likely reveal
• A Basal cell carcinoma
• B Mariolin ulcer*
• C Keloid
• D Malignant melanoma
• E Kaposi sarcoma
• B
236. Commonest type of brain hematoma after roadside accidents in young adults is
• A: Acute subdural hematoma
• B: Acute extradural hematoma*
• C: Chronic subdural hematoma
• D: Intracerebral hematoma
• E: Pontine haemorrhage
237. What is the optimum time and temperature for sterilization using an autoclave?
• 5 minutes at 100C
• 10 minutes at 190C
• 15 minutes at 121C*
• 20 minutes at 95C
• e-8 minutes at 212C
• C
238. Which of the following does NOT lower Surgical site infection rate?
• Aseptic technique during surgery
• Short hospital stay
• Trimming hair using an electric trimmer just before surgery
• Patient wearing a clean gown
• Having a large team of people during surgery*
239. Which of the following is NOT used as a medium of skin preparation during surgery?
• 2- Ethyl Alcohol*
• b-Isopropyl Alcohol
• c-Chlorhexidine
• d- Povidone Iodine
• e-Methylated spirit
240. The ideal temperature of an Operating theatre should be?
• 10-12C
• 14-16C
• 20-22C*
• 28-30C
• c- 32-34C
241. A 21 years old lady presents with H/O scald by boiling water over her both lower limbs. What
is the percentage of the TESA that is burned?
• a-9%
• b- 18%
• c- 27%
• d- 36%*
• c- 45%
• D
242. How much IV fluids should be given in the first 24 hours to an 80 kg patient with burns of
20% TBSA?
• 2700ml
• 3200ml
• 4200ml
• 5000ml
• 6400ml*
243. Which of the following formulae is used to calculate fluid requirement in a burned patient?
• 5x% of TBSA burned x BMI
• 10x % of TBSA burned x height x weight
• 4x% of TBSA burned x weight*
• 7 x% of TBSA burned x height
• e 2 x % of TBSA burned x BMI x time since burn in minutes
244. 51.30- year old man presents to the Emergency Department following a high speed motor
vehicle accident. He has marked abdominal distension, a pulse rate of 130 and a blood
pressure of 80/50 mmHg. The most appropriate initial investigation would be
• Abdominal angiogram
• Abdominal paracentesis
• CT scan of the abdomen with contrast
• Plain X-ray of the abdomen
• FAST (focused abdominal sonography for trauma) scan*
• E
245. 52. A 30 years old factory worker was extricated from a burning factory by fire fighters. On
arrival in A&E he is tachycardiac and has burns over his back, chest, neck and face. While
resuscitating him with IV Fluids you notice that his nose hairs and moustache are singed.
Which of the following should be the probable course in management of this patient?
• Continue IV fluids
• Start IV antibiotics
• Prepare for escharotomy
• Early elective intubation*
• Perform CXR
• D
246. 53. You are asked to see a 30 years old man complaining of a swelling on his back, OVE it is a
2x3cm swelling over left scapular region, it has a smooth surface, regular margins and is
mobile over the underlying structures, overlying skin is not pinchable, a small black punctum
is visible in the center of the swelling. What is the diagnosis?
• Lipoma
• Fibroma
• Neurofibroma
• Sebaceous cyst*
• Hemangioma
247. 54. While examining a swelling over the left flank of a 50 years old woman you notice that
the swelling is soft, mobile in relation to underlying structures and overlying skin, the surface
feels septated and the swelling tends to slip under your examining finger. What is the
diagnosis?
• Lipoma*
• Fibroma
• Neurofibroma
• Dermoid cyst
• Hemangioma
• A
248. 56. A 6 month old boy is brought to your OPD by his mother who is concerned about a
discolored swelling over his left check, the swelling is reddish brown in color and turns pale
when pressure is applied although the color returns upon lifting your hand. What is the
diagnosis?
• Lipoma
• Fibroma
• Neurofibroma
• Dermoid cyst
• Hemangioma*
• E
249. 57. Regarding blood supply of Thyroid gland which of the following is most appropriate
statement.
• Inferior thyroid artery is branch of external carotid artery
• Inferior thyroid artery is branch of Internal carotid artery
• Superior thyroid artery is branch of external carotid artery*
• Superior thyroid artery is branch of internal carotid artery
• Thyroid Ima artery is a branch of external carotid artery
• Which of the following is NOT useful in treatment of Keloid scars?
• Topical silicone gel
• Intralesional steroid injections
• Radiotherapy
• Intralesional surgical excision
• e-Systemic steroids*
• E
250. Which of the following is a good donor area for taking a split thickness skin graft in a female
patient?
• Abdomen
• Forearm
• Buttock
• Thigh*
• e Neck
251. 61.2-year-old toddler falls while playing, sustaining bruise on his knee. Which cell type is
responsible for cessation of blood flow in his wound healing process?
• Fibroblasts
• lymphocytes
• Macrophage
• Neutrophils
• Platelets*
• E
252. A 35 years old lady undergoes debridement for infected 2nd degree burns of left arm, after
the infection has settled, which of the following is the best to cover the wound?
• Split thickness skin graft*
• Full thickness graft
• Free flap
• Pedicled graft
• e-Myocutaneous flap
• A
253. Hemorrhage can be classified as primary, secondary & reactionary, Which is true regarding
secondary hemorrhage,
• Occurs at the time of surgery
• Occurs within 24 hours of surgery
• Occurs due to slip of ligature
• Occurs between 7-10 post-operative days*
• e.Occurs due to bleeding disorder
• D
254. Which of the following does NOT increase the risk of myocardial ischemia in the
perioperative period?
• Tachycardia
• Hypertension
• Pain
• Anemia
• e-Beta Blockers*
• E
255. 65.24 year old person had a chest injury. Clinical and radiological examination revealed 2-3
Ribs fracture with associated haemo pneumothorax. Which of the following is best initial step
in Emergency Management of this patient?
• Endo-Tracheal Intubation and positive pressure ventilation
• Immediate Thoracotomy
• Placement of chest Tube with under-water seal*
• Simple Analgesics and Strapping of fractured ribs
• Surgical fixation of fracture ribs
• C
256. Which of the following is not a risk factor for thrombosis?
• Pregnancy
257. Which of the following should be kept in mind while operating on diabetic patients?
• & Tight glycemic control is not necessary
• b- Diabetic patients have the same risk of cardiac events as normal patients
• c-HbAle reflects glucose control over preceding 1 month
• d- Diabetic patients should be prioritized on theatre list*
• Poor wound healing is rare in diabetic patients
• D
258. Which of the following should be performed at minimum to screen elective surgical patients
during COVID-19 pandemic?
• COVID-19 IgG and IgM levels
• COVID-19 PCR and CXR*
• HRCT Thorax
• COVID-19 Rapid antigen test I
• History and examination only
259. 69.35 years old car driver had head on collision with truck resulting in fracture of 4 left lower
ribs. Auscultation reveals positive bowel sounds in left chest.
• Cardiac Tamponade
• Hemothera
• Rupture of the diaphragm*
• Sliding hiatus hernia
• Tension Pneumothorax
• C
260. 70. Which of the following is the correct dose of Lignocaine for local infiltration in a 60 kg
man?
• 100 mg
• 180 mg*
• 250 mg
• 300 mg
• 350 mg
261. 71. A patient with severe head injury came to the emergency with GCS 8/15. What is the
most important step in the management of a severe head injury, as in this patient?
• Airway, Breathing & Circulation*
• CT Scan Brain
• I/V Fluids
• Managing skull fracture
• Stop the bleeding
262. 72. 45 years old male presents with head injury after a road traffic accident. His CT-scan
reveals a biconvex lesion. What is the most likely blood vessel injured?
• Maxillary artery
• Middle Cerebral artery
• Middle cerebellar artery
• Middle Meningeal artery*
• Posterior Cerebral artery
• D.
263. 73. A patient with head injury comes to the emergency with Bilateral peri orbital ecchymosis
and CSF rhinorrhea. What is your initial diagnosis?
• Base of Skull Fracture*
• Brain stem bleed
• Intra cerebral bleed
• Occipital Fracture
• Severe Head Injury
• A.
264. 74. Post-operative analgesic requirements vary in different patients. Which of the following
is the best way to assess adequacy of pain relief?
• Measure the degree of tachycardia
• Assess the level of hypertension
• Ask the patient using a visual analogue scale*
• Look for tachypnea
• Assess the level of sedation
• 75. All of the following are complications of liver trauma EXCEPT.
• Bile collection
• Biliary fistula
• Hydatid cyst*
• Intrahepatic hematoma
• Liver abscess
• C
265. 76. 45 years old Diabetic male has presented in E/R with h/o painful swelling in Peri-anal
region for last 1 week. On examination there is red, hot and tender swelling in left buttock
which is fluctuant. What is most likely diagnosis:
• Anal carcinoma
• Anal Fissure
• Hemorrhoids
• Perianal fistula
• Peri-anal Abscess*
• E
266. 77. You are called to see a patient who had Left inguinal hernia repair after a difficult spinal
anesthetic yesterday. The patient complains of intense generalized headache involving both
sides which gets worse on sitting up and improves on lying down, the patient is not able to
mobilize because of this headache and has vomited once. What is the most likely diagnosis?
267. 79.35 years old sheep farmer has presented with right hypochondrial pain and fever for last 1
week. On examination there is tender mass in right hypochondrium. Complete blood count
reflects marked eosinophilia. Which of the following statement is most appropriate regarding
this condition?
• CT is the best investigating tool*
• Lung is the Commonest organ involved
• Medical therapy has got no role in this condition
• Surgery is mandatory treatment in this condition
• X-ray is best investigating tool
268. Parental mutrition is given either by central line or Peripheral line. Which of the following is a
complication of parenteral nutrition on Central line insertion?
• Abdominal cramps*
• Bloating
• Constipation
• Diarrhea
• Pneumothorax
• A
269. 81. Which of the following is true regarding Magnetic resonance imaging?
• It has excellent contrast resolution without any radiation exposure*
• It is widely available and cheap
• MRI is based on the same principle as X-rays
• Patients who are claustrophobic or unable to remain calm can be easily imaged using MRI
• e Metallic implants are better visualized in MRI
• A
270. 82. Which of the following investigation to determine pathology in ductal customs
• A*
• b- CT-Abdomen
• c- Ultrasound abdomen
• X-ray abdomen
• e Complete Blood count
271. 83. While assisting your consultant in the endoscopy suite, you are quizzed about the extent
of visualization of GI tract by Upper Gil endoscopy?
• It is only able to visualize up to the pyloric sphincter
• It can only visualize the upper part of stomach and esophagus
• The 2nd part of the duodenum is the most common extent of visualization*
• d. It is able to visualize the GI tract beyond the ligament of Trietz
• e-Proximal Ileum can be easily visualized
272. 89. Which of the following staging systems is used for Colorectal Carcinoma?
• Dukes staging*
• Nevin staging
• Manchester Classification
• Clarks Staging
• Tokyo classification
• A
273. 91. A 62-year-old man complains of early morning spurious diarrhea with rectal bleeding of 6
weeks duration. Clinical examination, investigations and staging showed an adenocarcinoma 5
cm from the anal verge, which has spread locally to the lateral pelvic wall without any distant
metastases. What is the appropriate course of treatment?
• n- Neoadjuvant chemotherapy followed by surgery and adjuvant therapy*
• Surgery alone
• Surgery and postoperative chemotherapy
• Endoscopic stenting*
• Chemotherapy and radiotherapy
274. 92. Routine cancer screening should be performed for which of the following cancers?
• a-Leukemia
• b- Renal cancer
• c- Sarcoma
• Breast cancer*
• e-Pancreatic cancer
• D
275. 95. A middle aged man presented with c/o pain in legs after walking few steps. On
examination there is superficial thrombophelibitis of left leg. Which of the following is the
most important question to be asked next?
• Family h/o peripheral vascular disease
• H/O smoking*
• Insect bite
• Recent trauma
• Allergic contact dermatitis
• B
276. 96. In a patient with arterial insufficiency & Raynaud's phenomena what is the most
important advice given to the patient?
• Regular exercise
• Wearing TED stockings
• Avoid cold exposure*
• Bed rest
• Hydrotherapy to increase blood flow
• C
277. 97. In a patient with peripheral arterial disease, pain at rest is improved by which of the
following?
• Elevating the limb above the heart level
• Massaging the limb after cold compresses
• Placing the limb at horizontal level with heart
• Application of topical anesthetics
• Lowering the limb so that it is dependent*
• E
278. 101. 50 years old female was operated for knee replacement. On 3d post-operative day she
developed sudden painful swelling of her left lower limb. On examination her limb is swollen
with pain on passive movement of limb. What is most likely diagnosis?
• Acute limb ischemia
• Chronic limb ischemia
• Chronic Lumbago
• Deep venous thrombosis*
• Sciatica
• D
279. 104, 54-year-old fermele returning from Australia develops a tender, warm, swollen right call
after 12hour long flight. She smokes five cigarettes daily and takes the oral contraceptive pill.
She has no chest pain or shortness of breath. Which single investigation is the most
appropriate?
• Abdominal ultrasound
• Ascending venography
• CT angiography
• Doppler ultrasound*
• VQ scan
• D
280. 105. 52 years old male with poorly controlled type 2 diabetes, developed an infected ulcer in
his right foot. The infection spreads to involve the soft tissue of the foot resulting in necapsis
& gangrene. Which of the following is the best type of amputation for this patient?
• a-Ray amputation
• b- Symes amputation
• c- Below knee amputation*
• d- Through knee Amputation
• Above knee amputation
281. 106. During stripping and avulsion of great saphenous vein while performing Trendelenburg's
operation for varicosee veins which nerve is at risk of damage
• Common peroneal nerve
• Deep peroneal nerve
• Saphenous nerve*
• Superficial peroneal nerve
• Sural nerve
• C
282. 107. 55 year old women with h/o tiredness, aching & feeling of heaviness in left leg for past
three months. Symptoms are relieved by leg elevation & massage. On examination there are
superficial varicosities, non-pitting edema & a superficial, slightly painful 2*2 cm ulcer above
& behind the medial mallelus. What is the most appropriate diagnosis?
• Superficial thrombophlebitis
• Venous insufficiency
• Arterial embolism
• 113.
283. 50 years old male presents with pulsatile mass in central abdomen, What is first line
investigation to diagnose the disease?
• Ultrasound abdomen*
• IVU
• X-ray erect abdomen
• X-ray KUB
• Arteriography
284. 114. 35 A 55 years old diabetic male presented in emergency room with complaint of a
painful swelling spreading in left lower leg. On examination of leg it has red, hot, tender and
non-fluctuant but spreading over whole leg. On investigations his Hemoglobin is 10 g/dl, WBC
count is 15000/ul, Random blood sugar is 450 mg/dl. What is the best first line treatment?
• Daily aseptic dressing
• Excision of infected area,
• Incision and drainage
• Intramuscular antibiotic
• Intravenous antibiotics*
• E
285. 115. Which of the following is NOT a part of Primary Survey in the management of Trauma?
• Airway Management
• Breathing and ventilation
• Cervical Spine protection
• Neurological status assessment
• Definitive Fracture management*
• Answer. E
286. 116. What is the Neurological threshold for consideration of a definitive airway in an acutely
injured patient?
• GCS ≤14
• GCS ≤10
• GCS ≤8*
• GCS ≤5
• B- GCS ≥10
• Answer: C
287. 117. You have been asked to assess a trauma patient in the ER, on assessment the patient is
a 20 years old Male who was involved in a motor bike accident. He opens his eyes upon calling
his name; he is confused about his whereabouts and time of the year but can point towards
painful stimuli. What is the GCS score of this patient?
• 10
• 11
• 12*
• 13
• 14
• Answer: C
288. 118. Patency of Airway of the trauma patient can be rapidly assessed by:
• Asking the patient to point to painful area
• Asking the patient to flex his neck
• Asking the patient to extend his neck
• Asking the patient his name
• Asking the patient to take a deep breath
289. While working in the ER you receive a 42 years old man who was the restrained driver in a
head on collision. On arrival in the trauma room he has a heart rate of 142/min, Blood
Pressure is 77/39 mm of Hg. Respiratory rate of 30/min and Temperature is 98 F. You have
passed 2 wide bore IV lines, taken bloods and started IV Fluids. Further examination reveals
bilaterally audible breath sounds; muffled heart sounds and engorged neck veins. What is the
next step in the management of this patient?
• Continue IV fluids and arrange blood for transfusion
• Insert Foley's Catheter to measure Urine Output
• Needle Thoracostomy in 2nd Intercostal Space on Left side
• Perform Pericardiocentesis*
• Insert Endotracheal tube
• Answer: D
290. 121.A 28 years old male stabbing victim has been brought to the ER. On arrival his Pulse is
120/min, Blood Pressure is 83/42, Respiratory Rate is 40/min and Temperature is 98.2 F. Upon
examination there is a stab wound on the Left chest in the 4th intercostal space with tracheal
deviation to the right, engorged neck veins, absent breath sounds and a hyper-resonant
percussion note over the left chest. What is the next best step in the management of this
patient?
• Pericardiocentesis
• Chest tube in 2nd Intercostal space on Right Side
• Insert Endotracheal tube
• Needle thoracostomy in 2nd Intercostal space on Left side*
• Observe the patient
• Answer: D
291. 122.As per NICE guidelines, a CT Brain is indicated after head injury for which of the
following?
• Age >40 years
• Scalp laceration
• Multiple episodes of Vomiting*
• Initial headache
• GCS 15 at presentation
• Answer: C
292. 123.A 30 years old motor vehicle accident victim presents with a GCS EI V1 M4. You perform
primary survey, secure his airway and find a fixed dilated left pupil. An urgent CT Brain shows
a convex bleed over the surface of the left hemisphere. The most probable diagnosis in this
patient is?
• Acute Sub-dural hemorrhage
• Chronic Sub-dural hemorrhage
• Acute Extra-dural hemorrhage*
• Diffuse Axonal Injury
• Subarachnoid hemorrhage
• Answer: C
293. 124. A six-month-old male baby is presented in emergency room after burns with hot liquid.
On examination his head, face, right upper and lower limbs are found burnt. What formula
will give accurate calculation of burns?
• Rule of three
• Rule of seven
• Rule of eleven
• The Lund and Browder chart*
• The parkland formula
• D.
294. 125. Which of the following injuries is notorious for not being seen on initial radiographs?
• Anterior Dislocation of Shoulder
• Hip Fracture
• Scaphoid Fracture*
• Supracondylar fracture of Humerus
• R. Distal Radius fracture
• Answer: C
295. 126.A 50 years old woman presented with a displaced fracture of shaft of Right Humerus, She
was treated with Analgesia, U-slab and Humeral brace. During her follow-up visit after 4
months, she still has pain and mobility at fracture site. X-rays do not show callus formation
and no cortical bridging is noticed. What is the probable diagnosis?
• A-Atrophic non-union*
• Delayed Union
• Hypertrophic non-union
• Mal-Union
• Uniion
• Answer: A
296. 127.A 40 years old wrestler has been brought to the ER after he sustained a blunt chest
injury. He complains of severe left sided chest pain and on examination there is bruise over
the chest and a 6 x 8 cm segment of the Left chest wall which unusually moves inwards on
inspiration paradoxically. What is the probable diagnosis in this case?
• Tension Pneumothorax
• Cardiac Tamponade
• Flail chest*
• Pneumothorax
• Rib fracture
• Answer: C
297. 128. A 57 years old man has been brought to the ER after a blast in local market. On arrival
he is tachypneic and has a right sided open chest wound approximately 3 x 4 cm in size. What
would be the appropriate management of this patient?
• Chest tube insertion on the right side
• Pack the wound using aseptic technique
• Stitch the wound using absorbable sutures for muscles and non absorbable sutures for skin
• 3-sided occlusive dressing along with chest tube insertion*
• Emergency department thoracotomy
• Answer: D
298. 129. A 45 years old male is presented in emergency room with 70% burns after explosion of a
boiler in a factory. On examination, he got burn on face, neck & chest. His blood pressure is
100/60 mm of Hg, pulse is 110/min. He is also feeling difficulty in swallowing and breathing.
Which system requires immediate attention after admission to the hospital?
• Gastrointestinal system.
• Musculoskeletal system
• Respiratory system*
• Renal System
• Skin and integument
• C
299. 130.A 25 years old man is brought to the ER after suffering blunt abdominal trauma in a
motor vehicle accident. He complains of abdominal pain and has a pulse of 105/min, blood
pressure of 93/57 mm of Hg, Respiratory rate of 28/min and temperature of 97.8 F. His Chest
and Pelvic radiographs do not demonstrate any injuries. What is the next best step in his
management?
• Exploratory Laparotomy
• FAST Scan*
• MRI Abdomen
• X-ray Abdomen
• X-rays Femur B/L
• Answer: B
300. 131.A 10 years old girl presents to you after a playground accident. She complains of Pain in
her Right Elbow. On examination there is swelling, tenderness and deformity at the right
elbow and a radiograph shows a Supra-condylar fracture of Humerus. The patient also
complains of numbness over the thenar eminence and has a very weak thumb opposition and
is unable to pincer grip a piece of paper. Which nerve has been injured in this patient?
• Median Nerve*
• Ulnar Nerve
• Radial Nerve
• Musculo-cutaneous nerve
• Axillary nerve
• Answer: A
301. 132.A 47 years old farmer presents to the ER after a history to trauma to his left leg 7 days
back. He sought treatment from a local bone setter who applied an ill-shaped cast. The
patient now complains of swelling of the limb as well as loss of sensation in his left foot. On
examination his foot is dropped and everted, sensations are absent on the dorsum of the foot.
Radiographs show fracture of the upper end of fibula. What is the likely nerve injury in this
patient?
• Common Peroneal nerve*
• Sciatic nerve
• Tibial nerve
• Obturator nerve
• Femoral nerve
• Answer: A
302. A 28 years old man is trapped under a collapsed building for 18 hours. His right arm and leg
were crushed under a beam. When he is finally freed, he is confused, his pulse is faint but
regular and his right arm and leg are cold, pale and pulseless. He does not appear to be able to
feel or move either of them. What is the most probable diagnosis?
• Frost bite
• Crush Syndrome*
• Hypothermia
• Tetanus
• Necrotising Fascitis
• Answer: B
303. 134.A 16 years old girl sustains a left leg fracture caused by a falling tree branch. The fracture
is only slightly displaced, so a simple reduction is performed and the arm is put in a cast. 12
hours later she returns to ER with severe pain in the effected leg, which is not controlled by
analgesics. The distal pulse is absent and so is the sensation in her fingers, but as soon as the
fingers are extended passively, she cries out in pain. What is the probable diagnosis?
• Crush syndrome
• Limb ischemia
• Compartment syndrome
• Gas Gangrene
• Necrotising Fascitis
304. 135.10 days ago, a 60 years old rescue worker injured his foot on a nail. He did not seek
treatment and the wound healed itself. Now he presents with fever, sweating, and drooling.
He is unable to open his mouth properly. Each time there is a loud noise his muscles go into
spasm and he seems to be having difficulty breathing. What is the probable diagnosis?
• Gas Gangrene
• Necrotising Fascitis
• Streptococcal Sepsis
• Guillian-Barre Syndrome
• Tetanus
305. 4. Commonest cause of pancreatitis is:
• A Viral infection
• BERCP
• C Gallstones*
• D. Drugs E Alcohol
306. ERCP is superior to MRCP in providing
• Diagnostic role
• Therapeutic role*
• Excellent Imaging of ductal anatomy Identification of choledocholithiasis
• Detecting malignant strictures
307. TSH receptor antibodies are elevated in:
• Autoimmune thyroiditis*
• Grave disease
• Myxedema
• Cretinism
• E. Thyroid Cancer
308. A patient presents to surgical OPD six days after appendectomy with splicing fever, malaise
and anorexia. Most likely cause:
• A Deep Venous thrombosis
• Urinary tract infection
• Intra abdominal abscess
• Wound infection*
• E. Pneumonia
309. 14. The causative organism of Hydatid disease is
• Entamoeba histolytica
• Clostridium difficile
• Echinococcus granulosis *
• Echinococcus multilocularis
• E. Clostridium perfringens
310. 18 A 50 years old female with known gall stone disease presents with right uppar quadrant
pain, fever with rigors and chills, and jaundice. Cholangitis is suspected. Which statement
concerning cholangitis is correct?
• The most infecting organism is staphylococcus aureus
• The diagnosis is suggested by Charcot's triad*
• The disease occur primarily in young. Immunocompromised patients
• Cholecystectomy is the procedure of choice in affected patients
• Surgery is indicated once the diagnosis of cholangitis is made
311. A 45 year old male presents in the emergency with history of colicky abdominal pain for 1
day with vomiting, abdominal distention and constipation. He gives history of laparotomy for
gun shot injury five years back. The likely cause of his condition is:
• Paralytic ileus
• B Adhesions*
• C Mesenteric ischemia
• D. Volvulus
• E. Intussception
312. A 14 year old boy sustains a closed stable fracture of the upper tibia. After reduction and
application of a cast, he complains of excessive pain. The next step in his management will be:
• X-rays to check for cast fragments
• B elevate the limb by 5-10'
• C Elevate the limb by about 90
• D. Remove the cast *
• E Increase the dose of the analgesic
313. A 60 year old alcoholic presents with severe epigastric pain which radiates to the back for last
three hours and vomiting. On examination, he is tender with guarding in the epigastrium. His
serum amylase is alevated, WBC count is 18x10^9/L blood glucose is 20mmol/L. His Ranson
score is:
• 4
• 3
• 1
• 2
• 8*
314. 19 A 60 year old smoker presents in surgical Out patient with progressive dysphagia for last
three months. He is also complaining of hoarseness of voice and has lost approximately 10 kgs
in last three months. Barium swallow reveals an irregular stricture in the upper third of
esophagus. The lesion is most likely:
• Chronic esophagitis
• Squamous cell carcinoma of esophagus*
• Adenocarcinoma
• Barret’s esophagus
• Achalasia
315. 20 A patient underwent total thyroidectomy for a long standing, huge multinodu ar goiter
with pressure symptoms in morning. In the ward he became breathless with severe amxiety,
sweating and stridor. What should be the immediate Initial step?
• Endotracheal intubation immediately*
• Prop up the patient
• Oxygen inhalation
• Urgent opening of all layers of thyroidectomy incision*
• Opening of the skin wound only
316. 40 years old female has presented to surgical outpatient department with a painless swelling
just below and infront of right ear. Investigations confirm pleomorphic adenoma of right
parotid gland. What is the best treatment for this patient?
• Radiotherapy
• Chemotherapy
• C Wedge excision
• D Radical parotidectomy
• Superficial parotidectomy*
317. A 25 year old woman,20 weeks pregnant, presents with 1.5 cm right thyroid mass Fine
needle aspiration cytology is consistent with papillary neoplasm. The mass is ca on scan and
solid on ultrasound Which of the following is the most appropriate treatment?
• A Right thyroid lobectomy
• Subtotal thyroidectomy
• C Total thyroidectomy*
• Radioactive lodine ablation
• Anti thyroid medication
318. 23, Staghorn calculi are formed due to:
• A Increased intake of calcium in diet
• Increase intake of protein diet
• C Increased ingestion of milk
• D Hyper-oxaluria
• infection with urea splitting organism*
319. 24. Gold standard surgical procedure for cand amat
• Trans vesical prostatectomy
• B Retropubic (Million) prostatectomy
• C TURP
• D Transperineal prostatectomy
• Transrectal prostatectomy
320. 25. Most common pathogen responsible for UTI is:
• E.coli*
• Klebsiella
• Streptococcus
• Proteus
• Staph aureus
321. Following a 2-hour firefighting episode, a 36 year old fins throbbing headache, nausea,
dizziness and visual dit his carboxyhaemoglobin level is found to appropriate next step in his
treatment?
• Begin an immediate exchange transfusion
• B Transfer the patient to a hyperbaric oxygen chamber
• C Begin bicarbonate infusion and give mg acetazolamide intravenously
• Administer 100% oxygen by mask
• E Perform flexible bronchoscopy with further re
322. 34. A fifty years old female has presented to the surgical outpatient department with history
of ache while walking in both legs. On examination there are multiple bluish swellings over
the both legs and thighs. You are suspecting varicose veins. What is the gold standard for
diagnosing varicose veins nowadays?
• Conventional angiography
• Conventional ultrasound
• Duplex scan
• Computerized tomography scan
• Clinical examination
323. 35. A 40 years old male patient has developed a left calf swelling and pain after surgery for a
rectal tumor. You are suspecting deep vein thrombosis. Prior to starting treatment, you order
a duplex scan. What is true regarding this scan?
• It is not very sensitive in patients with symptomatic calf deep vein thrombosis
• A negative duplex scan will rule out deep vein thrombosis
• It cannot differentiate between acute and chronic thrombosis
• Lack of compressibility of veins is the hallmark of chronic thrombosis*
• The results are similar as compared to clinical examination alone.
324. 36. A 50 years old heavy smoker has got intermittent claudication for past three wees in the
right lower limb but now has developed severe rest pain. What is typical feature of rust pain?
• It improves on lying down
• It improves on elevating the legs
• It improves by hanging the foot outside bed
• it is worst at day time
• It is least at night
325. A greenstick fracture?
• Occurs mainly in elderly
• is a fatigue fracture
• is a spiral fracture of a long bone
• is a fracture where part of cortex is intact and part is crumpled
• Is treated always by surgery
326. A 12 years old girl has presented to the surgical outpatient department with history of right
sided neck swelling for four months. She also has low grade fever, evening pyrexiar weight
loss. On examination the swelling is firin and rubbery. ESR is raised. What is the musikely
diagnosis?
• Tuberculous cervical lymphadenitis
• Cystic hygroma
• Metastatic disease
• DJ Lymphoma
• El Squamous cell carcinoma
327. Neurogenic shock is characterized by the presence of:
• Decreased Cardiac Output
• Decreased Blood volume
• Decreased Peripheral Vascular Resistance
• Increased Cardiac output
• Increased Peripheral Vascular Resistance
328. A 25 year old woman presented in Emergency & Accident with history of severe vomiting for
last one day, her pulse is 120/min and B.P. is 60/40. Which of the following fluids is most
appropriate for replacement
• Albumin
• 5% Dextrose water
• Ringer Lactate
• Dextrose Saline
• Blood
329. A 13 year old boy presents in O.P.C. with painful swelling on his left forearm for last three
days. He also complains of high grade fever for last one day. On examination the swelling is
red, 5 x 5 cm in size, hot tender and fluctuant, The most likely cousative micro-organism is
• A Streptococcus pyogenes
• Staphylococcus epidermidis
• Staenophylus influenza
• Staphylococcus aureus
330. Which of the following is the best guide to fluid replacement in a burns patient?
• Haemoglobin and haematocrit
• Urine output*
• Urine osmolality
• Plasma volume
• Specific gravity
331. The most correct statement regarding a 70 kg male who has sustained third degree burns to
all of his right arm and second degree burns to the anterior aspect of his trunk is that
• He has burns to 27% of his body
• He will require 2 to 4 ml of Hartmann's solution per kilogram. per percent be first 8 hours
after admission
• A normal blood gas and chest X-ray on admission will be useful in excluding ational injury
• Prophylactic antibiotics and steroids are of proven benefit if he has suffered inhalational
injury
• Alkalinizing the urine will prevent renal damage if the burns were electrical
332. Regarding scars, which one of the following statements is true?
• A Hypertrophic scar extends beyond the original wound.
• 8 Keloid scar does not extend beyond the original wound
• C Pathophysiology of both keloid and hypertrophic scar is excess deposition of lagen fibers*
• D Steroids have no role in its management
333. 48, A70-year-old cigarette smoker presents with a right inguinal mass that has enlarged and
has caused discomfort in recent months. He complains of recent difficulty with micturition
and nocturia. The swelling, which does not extend to the scrotum reduces when resting. What
is the likely diagnosis?
• Direct inguinal hernia
• Strangulated indirect inguinal hernia
• Hydrocele
• Aneurysm of the femoral artery
• 16) Cyst of the cord
334. A 20 years old man receives multiple blood transfusions, for abdominal gin si.ct wound, he
complains of numbness around his mouth and displaced carpopedal spasm, and a positive
chvostek sign. Treatment requires:
• a intravenous bicarbonate
• intravenous potassium
• cintravenous calcium gluconate
• d intravenous digoxin
• e intravenous parathormone.
335. A 15 years old boy, sustains head injury, while doing one wheeling on his bike he opens his
eyes to painful stimulus, his verbal response is confused, his motor response is lesion to pain,
his Glasgow coma scale would be:
• 3
• 6
• 9*
• 12
• e 15.
336. A 60-year-old male presents with an Inguinal hernia of recent onset. Which of the following
statements is TRUE?
• The hernia is more likely to be direct than indirect*
• Presents through the posterior wall of the inguinal canal, lateral to the deep inguinal ring.
• is covered anteriorly by the transversalis fascia.
• is more likely than a femoral hernia to strangulate.
• The sac is congenital.
337. 02. Fracture femur:
• A Blood loss is may be up to 1-1.5 liter*
• Very common sports injury
• is always open fracture
• is always closed fracture
• Mid shaft fracture is common in old age
338. Q3. The gold standard for diagnosis of varicose veins nowadays is:
• Conventional angiography
• Conventional ultrasound
• Duplex scan*
• DJ CL scan
• Clinical examination
339. Q5. A 40 years old male patient has developed a left calf swelling and pain post-surgery of a
rectal tumor. You are suspecting DVT. Prior to starting treatment of DVT, you order a duplex
scan. What is true about this scan?
• It is not very sensitive in patients with symptomatic calf DVT.
• A negative duplex scan will rule out DVT
• It cannot differentiate between acute and chronic thrombosis
• Lack of compressibility of veins is the hallmark of chronic thrombosis*
• The results are same as compared to clinical examination alone.
340. Q6. In repair of a femoral hernia, the structure most vulnerable to major injury lies:
• Medially
• Laterally*
• Anteriorly
• Posteriorly
• Superficially
341. Q7. An 20 years old female is complaining painless abdominal mass. On examination the
mass is occupying the whole abdomen mid part and is also pulsatile. What is true regarding
this patient condition?
• A The lady is suffering from mesenteric ischemia
• 8) Surgery should only be performed if symptoms develop.
• Surgery should only be performed if increase in size occurs
• Surgery improves 5 year survival rate
• It should be left as such as it is a very common condition
342. 08. A 2-year-old boy presents with a reducible umbilical hernia, under 2-cm diameter. This is
best managed by:
• Immediate surgery and repair with mesh
• Immediate surgery repair without mesh
• Laparoscopic repair with mesh
• Laparoscopic repair without mesh
• Periodic observation and evaluation*
343. 09. A healthy appearing, 8-year-old boy is evaluated for an abdominal mass, felt by his
mother during a bath. What is the most likely diagnosis?
• Lymphoma
• Rhabdomyoscarcoma
• Wilms tumor*
• Neuroblastoma
• Renal cell carcinoma
344. 10. A 24 years old female has suffered a burn of her right upper limb, head and neck and sa
front of chest. What is the total percentage of burn in this lady?
• 50 %
• 45%
• 27%*
• 30%
• 67%
345. Q11. A 42-year-old man presents with cancer of the left testis. To exclude lymphatic
metastasi which is the site that should be initially examined?
• Vertical chain of inguinal glands
• Horizontal chain of inguinal glands
• Retrorectal glands
• Para-aortic glands
• Obturator nodes
346. Q 13. A 60 years old female patient has presented to the OPD with severe epigastric pain for
6 hours. A diagnosis of acute pancreatitis is confirmed. The WBC count is 18000,She also has
raised blood sugar (290 mg/dl). What is the Ranson Score of this patient on admission?
• 2
• 4
• 3
• 1
• 5
347. Q14. A 45 years old male was treated in surgical ward with diagnosis of acute pancreatitis 4
weeks back. Now he complains of increasing abdominal mass with pain. On examination there
is a 6x5 cm mass in the epigastric region. Which of the following statements is true regarding
this patient?
• This patient needs urgent surgery and removal of the mass.
• He has developed a very rare and life threatening complication of acute pancreatitis
• Cannot be confused with carcinoma pancreas
• DCI scan abdomen is the investigation of choice*
• Fine needle aspiration cytology is contraindicated
348. Q15. Fracture Neck of femur.
• A Is a fracture of necessity
• Not an emergency
• Osteonecrosis is not one complication
• Non-union is very common if not fixed*
• Common in children
349. Q16. Which one of the following is true of the scarring associated with burn?
• A: Partial thickness burns do not scar.
• Excision and grafting in a deep burn will minimize starring
• Starring is most in burns that heal quicker
• D: Starring is less in burns that heal slowly
• E There is no role of antibiotics in starring process. *
350. 017, A 50 years old heavy smoker has got intermittent daudication for past 3 weeks in the
right lower tirab but now has developed severe rest pain. What is typical feature of rest pain?
• A Il amprove on lying down
• Is It improves on elevating the leg
• It improves by hanging the foot outside the bed*
• it is worst at daytime
• it is least at night
351. Q 18. A patient known to have gallstone disease has presented with epigastric pain radiating
to the back and vomiting for 6 hours. Serum amylase is 1600.What is the indication of CT scan
in this patient?
• A: To confirm presence of gallstones
• B: If condition does not resolve in 24 hours
• C: If patient develops local complications of acute pancreatitis. *
• D: On patient's demand.
• E: To assess the anatomy of pancreas.
352. 19. Which one of the following is a complication of pancreatic pseudocyst?
• A: Cystic fibrosis
• B: Annular pancreas
• C: ARDS
• D: Pressure symptoms including pain and vomiting*
• E: Chronic pancreatitis
353. Q20. Posterior hip dislocation:
• Is not associated with dashboard injury
• May cause sciatic nerve injury
• May be due to trivial trauma
• Common in old aged people*
• Commonly associated with femoral neck injury
354. Q21. A 45-year-old male presents with a 6-month history of an ulcerative nodular lesion, 1.5
cm in diameter in the region of the right oral comissure. Biopsy reveals basal cell carcinoma.
The preferred treatment is:
• Mohs micrographic surgery and subsequent reconstruction
• Excision with a clinical margin and local flap repair*
• Topical 5-FU
• Local radiation therapy
• Cryotherapy
355. Q22. A 61-year-old man with a long history of heavy smoking shows on computed axial
tomography (CAT) scanning a tumor of upper lobe of right lung and enlarged paratracheal
nodes. The tumor has been diagnosed as malignant by bronchoscopy. Your next move should
be:
• Esophagoscopy to rule out invasion of the esophagus. *
• Proceed with lobectomy and paratracheal node dissection.
• Begin radiation of the tumor and paratracheal area.
• Perform a mediastinoscopy for staging.
• [E] Wait 3 months and repeat CAT scan to evaluate further disease progression
356. Q 23. A young man is shot at the level of the right sternoclavicular joint. His blood pressure is
30/60 mm Hig, pulse 120 bpm, and a chest x-ray shows a right hydropneumothorax. The first
step should be:
• Insert a chest tube and observe for drainage. *
• Perform an immediate right thoracotomy.
• Perform an angiogram to rule out great vessels injury.
• Perform median sternotomy with extension along with right anterior border of the
sternocleidomastoid muscle.
• Perform a CAT scan with contrast, to evaluate extent of injury.
357. Q24. Immediately following a bout of pneumonia, a young woman develops a large pleural
effusion. A chest tube is inserted and 600 ml of thin pus is obtained. A CAT scan shows
incomplete drainage and multiple intrapleural loculations. Management of this empyema
requires: b
• Insertion of multiple chest tubes under CAT guidance to drain either most or all loculations.
• Treat the patient with antibiotics and continue single chest tube drainage. *
• Treat patient with antibiotics and continue single chest tube drainage waiting for a thick
peel to develop and then proceed with open total lung decortication.
• Proceed with thoracoscopy and intrapleural toilette. Break the loculations and place drains
• A thorough open total lung decortication immediately.
358. Q25. A male neonate is born with an omphalocele. This entity can be distinguished from
gastroschisis, because in an omphalocele, the protrusion is:
• Not covered by a sac
• A defect in the abdominal musculature*
• Associated with an umbilicus attached to the abdominal wall musculature
• Associated with partial or complete malrotation of the bowel
• Really contains abdominal viscera
359. Q26. During a car crash a young man suffers bilateral multiple fracture ribs. He is alert and
present with shortness of breath. His blood pressure is 100/60 min Hg and chest is unstable
Treatment for this is:
• (4) Prolonged intubation and ventilator support until rib fractures heal along with aggressive
bronchial
• toilette*
• Once the patient is stable, open rib fracture reduction and stabilization with plates
• Fracture stabilization, with towel clips on ribs and attached to weights (external fixation)
• Avoid intubation, control pain, and perform aggressive bronchial toilette
• Temporary extracorporeal circulation to allow fractures to heal
360. 027. A 75 years old man undergoes a radical prostatectomy for prostate cancer. The
pathology report reveals Gleason score 9/10 and involvement of several pelvic lymph nudes.
Which is the most likely site for prostatic cancer metastasis?
• JA) Liver
• (8) Kidney
• Lung
• Bone*
• Brain
361. 28. A 38-year-old woman presents with shortness of breath and abdominal distention.
Workup reveals presence of ascites and hydrothorax. What is the name of this condition?
• Brenner tumor
• (6) Dysgerminoma
• Wolffian duct remnant
• Krukenberg's tumor
• Meigs's syndrome*
362. Q 29. A 23-Year-Old Woman Has A Cesarean Section In Which A Plannenstiel Incision is Used
In The Plannenstiel Incision, Which Of The Following Is True?
• The recti and fascia are separated transversely,
• These and fascia are separated vertically*
• Fascia lata graft is used.
• A prosthetic graft is used.
• The upper abdomen can readily be explored.
363. 030 A 56-year-old woman is admitted to the emergency department complaining of upper
abdominal pain. An ultrasound of the abdomen reveals a thin-walled gallbladder filled with
Risid and a solid, left renal mass. What should be the next test ordered?
• Hydroxy iminodiacetic acid (HIDA) scan
• (6) Intravenous pyelogram (IVP)
• CT scan of the abdomen and pelvis
• Oral cholecystogram
• Upper gastrointestinal (G) series
364. Q21, During evaluation of the cause of varicocele in a 36-year-old man, attention is directed
to the method of drainage of the left testicular vein, which usually enters which of the
following?
• Left adrenal vein
• (8) Lett renal ven*
• Left inferior mesenteric vein
• Interior vena cava (TVC)
• Left inferior epigastric vein
365. Which of the following statement is true about Greenstick fractures:
• Commonly associated with high velocity injury
• May occur in osteogenesis imperfecta. *
• Incomplete fracture in children
• Complete fracture in adults
• Comminuted fractures of bone in children
366. Q33. Classical triad of symptoms (1.e pain, haematuria and Mass) is present in what
percentage of patient with renal cell carcinoma?
• 10-15%
• 20-30%
• 35-50%
• 50-60%
• E 65-75%
367. Q34. Which is common neurotransmitter in penile blood vessels?
• A Serotonine
• B. Dopamine
• C Nor epinephrine
• D Nitric oxide*
• E Epinephrine
368. Q35. Compartment syndrome:
• Pain out of proportion is a cardinal sign*
• Is not an emergency
• Tissue ischemia may not occur if left alone
• Does not occur in abdomen
• Is not common in leg
369. Q37. Gold standard surgical procedure for enlarged prostate:
• A Transvesical prostatectomy
• B Retropubic (Millon) prostatectomy
• C TURP*
• Transperineal prostatectomy
• E Transtectal prostatectomy
370. 038. What is true of Spigelian hernia?
• A It occurs exclusively in males.
• B. It involves part of the circumference of the bowel wall.
• It is best repaired by the classical Bassini technique of inguinal ligament repair.
• It occurs at the lateral edge of the linea semilunaris. *
• It always contains the vermiform appendix.
371. Q41. Most common renal stone in Pakistan is:
• Uric acid calculus
• Staghorn calculus
• Calcium oxalate calculus*
• Phosphate calculus
• Cystine calculus
372. Q42. The most common abdominal hernia in a male is:
• Inguinal hernia*
• femoral hernia
• päraumblical
• epigastric hernia
• Spigelian hernia
373. Q45. A fifty year old obese female presents in surgical out patient department with history of
severe episodic pain in the Right hypochondrium radiating to the back for the last three
months. She is also complaining of dyspepsia. What is the best investigation for diagnosing
her condition?
• CT-Scan of abdomen*
• Endoscopy
• Endoscopic ultra sound
• D] Abdominal ultra sound
• MRI of abdomen
374. Q 45. A fifty year old diabetic woman presents with fever, anorexia and malaise,
accompanied by right upper quadrant discomfort. Examination reveals tender enlarged liver.
Ultrasonography reveals a multiloculated cystic mass. What is the diagnosis?
• Polycystic liver disease
• Amoebic liver abscess
• Pyogenic liver abscess
• Haemangioma of liver*
• E Hepatoma
375. Q46. Flail chest is defined as:
• Multiple rib fractures with subsequent subcutaneous emphysema
• Chyle in the pleural space
• Excess fluid in pericardium
• Two or more ribs fractured at two points*
• Blood in the pleural space
376. Q49. A 20 years old lady has presented to Out patient department with oral swelling which
increases in size and becomes painful during meals. On examination there is a translucent
swelling at the floor of the mouth with characteristic frog belly appearance. What is the most
likely diagnosis?
• Frey's syndrome
• Pleomorphic adenoma
• Ranula*
• Sjogren syndrome
• Mucus Distula
377. Q50, A 15 years old boy has sustained an injury to the right arm while playing. On
examination there is swelling of the right elbow with painful movements. You order an ray
which shows supracondylar humeral fracture. What is the most threatening immediate
complication of this fracture?
• Malunion
• Bleeding
• Neurovascular compromise*
• DJ Compartment syndrome
• Deformity
378. Q56. A severely traumatized woman was seen in emergency with a Glassyow Coma Scale
(GCS) of 5. Which of the following statements regarding GCS is most appropriate?
• A high score correlates with a high immortality*
• It includes measurement of intracranial pressure
• It includes measurement of verbal response
• It includes measurement of papillary reflexes
• It is an obsolete clinical tool
379. Q57. While operating on middle aged male with gunshot injury to abdomen, the surgeon
noticed that there is generalized ooze from the raw surfaces. What is the most suitable blood
product to be transfused in this particular patient?
• Whole blood*
• Platelets
• Fresh Frozen Plasma
• Packed red cells
• Cryoprecipitate
380. Q5B. A 16 year old male presents to surgical OPD with history of Right Illac fossa pain. On
examination, you find that on pressing the left illac fossa, he experiences pain in right iliac
fossa. This sign is known as?
• Psoas sign
• Pointing sign
• Obturator sign
• Rovsing sign*
• Murphy sign
381. Q60. Following a 2-hour firefighting episode, a 36 year old fireman begins complaining of a
throbbing headache, nausea, dizziness and visual disturbances. He is taken to the ER, where
his carboxyhaemoglobin level is found to be 31%. Which of the following is the most
appropriate next step in his treatment?
• Begin an immediate exchange transfusion
• Transfer the patient to a hyperbaric oxygen chamber
• Begin bicarbonate infusion and give acetazolamide intravenously
• Administer 100% oxygen by mask*
• Perform flexible bronchoscopy with further therapy determined by findings
382. Q61. An 18 year old male was assaulted and sustained significant head and facial trauma.
Which of the following is the most common initial manifestation of increased intra cranial
pressure?
• Change in level of consciousness
• Ipsilateral pupillary dilation
• Contralateral pupillary dilation*
• Hemiparesis
• E Hypertension
383. Q 62. A 14 year old boy sustains a closed stable fracture of the upper tibia. After reduction
and application of a cast, he complains of excessive pain. The next step in his management
will be
• X-rays to check for cast fragments
• elevate the limb by 5-10 degrees*
• Elevate the limb by about 90 degrees
• Remove the cast
• Increase the dose of the analgesic
384. Q 63. A twenty year old male is brought to the emergency after a road traffic accident thirty
minutes back. On examination, he is conscious, tachypneic with a pulse of 120/min, BP of
60/40 mmitg and respiratory rate of 20/min with tender abdomen. He is suffering from which
type of shock:
• Hourogenic shock
• Anaphylactic shock
• Hypovolemic shock*
• Obstructive shock
• Cardiogenic shock
385. Q 64. Which one of the following is a complication of massive transfusion?
• Hyperthermia
• BJ Hyperkalemia*
• Hypercalcemia
• Hyponatremis
• Hypermagnesemia

Paper B
1. A 35 year old male has been diagnosed as suffering from salivary gland tumor. Which of the
following is true regarding salivary gland tumors in general?
• Most of the tumors arising from minor salivary glands are benign
• Most of the tumors arising from minor glands are malignant
• Fine needle aspiration cytology is the first line investigation
• CT scan and MRI are not very helpful investigations
2. A 10 year old child has been diagnosed with thyrotoxicosis. Which of the following modalities
of treatment is best for this age group?
• Radioiodine
• Subtotal thyroidectomy
• Total thyroidectomy
• Antithyroid drugs
3. A 30 year old female has undergone total thyroidectomy in the morning. On evening round you
find that she has difficulty in breathing which is getting severe. Regarding respiratory
obstruction after thyroidectomy, which one of the following is the most important cause?
• Tracheal injury
• Unilateral vocal cord paralysis
• Tension hematoma
• :Laryngeal injury
4. Regarding thyroid malignancies, which of the following statements is true?
• These malignancies have high mortality
• Follicular carcinoma metastasizes usually to lymph nodes
• Papillary carcinoma metastasizes by blood stream
• Papillary carcinoma spreads to cervical lymph nodes
5. A 30 year old male, traffic constable by profession presents in the surgical outpatient
department with history of ache in both lower limbs. On examination, the findings are
suggestive of bilateral varicose veins. His Body Mass Index is 25. What is the most probable
cause of his varicose veins?
• Obesity
• Age factor
• Gender
• Profession
6. Which of the following statements is true about arterial occlusion?
• Acute arterial occlusion is most commonly caused by atherosclerosis
• In occlusion caused by thrombus, there is no time for collaterals to develop
• Vascular occlusion in different parts of the body produces similar symptoms
• Pain, pallor, parasthesia and pulselessnes is the hallmark of acute ischemia
7. Which of the following is true about role of plain X-ray abdomen in acute intestinal
obstruction?
• Step ladder pattern is seen only in obstruction caused by tuberculosis
• Xray supine is helpful to diagnose intestinal perforation
• Xray erect is used to have an idea about level of obstruction
• Xray erect is used to detect free intraperitoneal air.
8. A 42 years old male patient was a diagnosed case of right submandibular gland tumor and was
operated two days back.Now in the ward he complains of absence of sensations in the
submentalregion.What complication has occurred?
• Facial nerve injury
• Lingual nerve injury
• Hypoglossal nerve injury
• Marginal mandibular nerve injury
9. Regarding achalasia:
• It is a disease of the large intestine
• It is associated with H.pylori infection
• It is closely related to hiatal hernia
• It is due to loss of ganglion cells in the mysentric plexus
10. A 30 years old male presents to OPD with history of abdominal pain,constipation and altered
behavior. Investigations revealed increased serum calcium &PTH and renal stones on
ultrasound KUB.The most likely diagnosis is:
• Primary hyperparathyroidism
• Thyrotoxicosis
• Secondary Hyperparathyroidism
• Pheochromocytoma
11. Following is an immediately life threatening chest injury:
• Aortic injury
• Diaphragmatic injury
• Airway obstruction
• Pulmonary contusion
12. Which one of the following is a cause of hypercalcemia?
• Hypothyroidism
• Hyperparathyroidism
• Hypoparathyroidism
• Intestinal tumors
13. Regarding tension pneumothorax,which one of the following statements is true?
• Should be managed conservatively
• Requires C.T scan for diagnosis
• Needs urgent decompression by wide bore cannula
• Urgent thoracotomy should be done
14. A 50 years old hypertensive patient is booked for cholecystectomy in next week.The best
advice for him would be:
• Stop his antihypertensive medications
• Continue the drug but stop it on the night before surgery
• Continue the drug and take it on the morning of surgery with sip of water
• Increase the dose of medication
15. Late dumping syndrome following peptic ulcer surgery is caused by:
• Bile vomiting
• Diarrhea
• Reactive hypoglycemia
• Small stomach
16. A middle aged female presents to surgical OPD with severe right upper abdominal pain for 1
hr radiating to right shoulder.On examination there is tenderness in right
hypochondrium.WBC count is normal.Most likely diagnosis is :
• Acute appendicitis
• Acute pancreatitis
• Biliary colic
• Acute cholecystitis
17. Most common incision used for open cholecystectomy is known as:
• Right upper paramedian
• Kocker’s incision
• Gridiron incision
• Midline incision
18. An elderly male patient presents to surgical OPD with a white patch on the lateral border of
the tongue.The lesion is most likely:
• Carcinoma tongue
• Erythroplakia
• Leukoplakia
• Candidiasis
19. Which of the following is true for gall stones?
• Pigment variety is most common
• Gall stones are best treated by cholesterol lowering drugs
• Pigment stones usually form after bile stasis or infection
• Pigment stones are green in color
20. Most common cause of hematemesis and malena is:
• Mallory Weiss tear
• Peptic ulcer
• Erosions
• Gastric tumor
21. Which one of the following is an important feature of hyperparathyroidism?
• Headache
• Renal stones
• Fever
• Loss of weight
22. Regarding chest trauma,which of the following statements is true?
• Accounts for only 5% of all trauma cases
• Is almost never fatal
• Accounts directly or indirectly for about 50% of all trauma cases
• Is always easy to diagnose
23. 55 years old diabetic female controlled on oral hypoglycemic is to be operated for
Paraumbilicalhernia.Which of the following is true?
• Should be last on operation list
• Must take oral hypoglycemic drug on the morning of surgery
• Should be first on operation list
• Should be kept hypoglycemic during surgery
24. Which one of the following is a risk factor for post-operative thrombosis?
• Short surgery time
• BMI<18
• BMI>30
• Surgery on face
25. 30 years old male known case of rheumatoid arthritis on NSAIDS for 2 years presented to
surgical OPD with history of severe generalized abdominal pain for 1 day.On examination the
patient is tachycardic and hypotensive.There is severe guarding all over the abdomen.X ray
chest erect showed gas under diaphragm.The most likely diagnosis is:
• Acute pancreatitis
• Perforated peptic ulcer
• Acute gastritis
• Intestinal perforation
26. Which one of the following is the best first line investigation for gall stones?
• MRCP
• ERCP
• Ultrasound abdomen
• CT scan abdomen
27. Which one of the following is the first line investigation in chest trauma?
• CT chest
• Xray chest
• Ultrasound chest
• Diagnostic peritoneal lavage
28. Regarding Trousseaus sign of hypoparathyroidism:
• Is due to hypercalcemia
• Also occurs on thyrotoxicosis
• Also known as carpopedal spasm
• Is induced by tapping the facial nerve
29. Following is an important cause of enlarged cervical lymphnodes?
• Prolonged fasting
• Hypothyroidism
• Thyrotoxicosis
• Metastatic disease
30. Regarding refeeding syndrome,which of the following statements is true?
• Is a complication of fasting
• Results in hyperphosphatemia
• Is more common in enteral nutrition
• Occurs due to overfeeding
31. A 12 year old girl presented to OPD with history of right sided neck swelling for 4 months.She
also has low grade fever,evening pyrexia and weight loss.On examination the swelling is firm
and rubbery.ESR is raised.What is the most likely diagnosis?
• Tuberculous cervical lymphadenitis
• Thyroid swelling
• Cystic hygroma
• Lipoma
32. Which of the following is the commonest cause of cervical lymphadenitis in our country?
• Brucellosis
• Tuberculosis
• Malignancy
• HIV
33. A 60 years old lady presented in emergency with multiple vomitings containing food
particles.She also complains of significant weight loss in past 6 months.Herxray abdomen
shows markedly dilated stomach.What is the most likely diagnosis?
• Gastric outlet obstruction secondary to malignancy
• Gastric outlet obstruction secondary to peptic ulceration
• Gastric bands
• Gastric volvulus
34. Regarding body mass index,which one of the following is true?
• Is weight corrected for gender
• A BMI of >30 predicts a good postoperative recovery
• A BMI of 20 is associated with increased hospital mortality
• A BMI of <15 is associated with increased hospital mortality
35. As a resident on call,you are informed about 60 years old male who underwent laparotomy 12
hours back has passed only 20 ml urine in previous hour,The previous output was
satisfactory.What will be your first step?
• Give fluid challenge of 500 ml saline
• Give 20 mg diuretic
• Check central venous pressure line
• Check urinary catheter for blockage
36. A 30 years old male came to surgical OPD with history of intramuscular injection over left
deltoid and now painful swelling over that region.On examination the swelling is red hot
tender and of about 7 by 5 cms in size.Impression of deltoid injection abscess is made.What is
the most appropriate management?
• Antibiotics alone
• Analgesics alone
• Incision and drainage without antibiotic cover
• Incision and drainage with antibiotic cover
37. The treatment of choice for uncomplicated peptic ulcer disease is:
• Truncal vagotomy
• Gastrectomy alone
• Highly selective vagotomy
• Eradication therapy
38. The commonest site of metastasis for gastric cancer is:
• Bones
• Liver
• Brain
• Lungs
39. A 35 years old female presents with severe biliary colic.Gallstones are confirmed on
sonography.The most appropriate management plan is:
• Emergency cholecystectomy
• Advise surgery only when complications occur
• Treat her conservatively,prepare for surgery and operate on next available list
• Treat her conservatively and operate after 4 weeks
40. Which of the following is the commonest complication of jejunostomy feeding?
• Tube blockage
• Wrong placement
• Tube dislodgement
• Diarrhea
41. What is the typical feature of nerve entrapment in carpel tunnel syndrome?
• Symptoms diminish by wrist flexion
• Pain worsens at night
• Erythema is present in cutaneous distribution of median nerve
• Motor symptoms precede sensory symptoms
42. Which one of the following nerves is involved in anterior dislocation of shoulder?
• Radial nerve
• Median nerve
• Ulnar nerve
• Axillary nerve
43. A 65 years old man has an enterocutaneous fistula of jejunum secondary to inflammatory
bowel disease.What is the most appropriate fluid for replacement of his enteric losses?
• 5% dextrose water
• 3% dextrose saline
• Sodium bicarbonate solution
• Ringer lactate solution
44. The normal daily requirement of potassium in a 70 kg man is:
• Less than 5 mmol per day
• 10-25 mmol per day
• 70-80 mmol per day
• 100-200 mmol per day
45. In a 70 kg man,the total body water is about 42 liters.The majority of this is within.
• Extracellular extravascular compartment
• Extracellular intravascular compartment
• Intracellular compartment
• Intracellular compartment
46. A 50 years old smoker develops whitish plaques in several parts of oral mucosa.The most
useful step in diagnosis is:
• Cytology of oral washings
• Punch biopsy of the lesion
• FNAC of submandibular nodes
• FNAC of submental lymph nodes
47. A 60 years smoker has marked dysphagia due to carcinoma of middle 1/3 of esophagus.He
requires nutritional support for atleast two weeks before any surgery or radiation.What is the
most appropriate method of nutrition in this patient?
• Central line
• Oral soft diet which is easily digested
• Nasogastric feeding
• Enteral feed via jejunostomy
48. Regarding flail chest,which of the following is true?
• Is fracture of one rib only
• Always requires ventilator support
• Is a complication of blunt chest trauma
• Involves lower ribs only
49. The commonest malignat tumor of oral cavity is:
• Adenocarcinoma
• Squamous cell carcinoma
• Melanoma
• Stromal tumor
50. A 50 years old patient develops sudden left lower chest pain and epigastric pain after
vomiting. The patient shows diaphoresis, breath sounds are decreased on the left, and there is
abdominal guarding. The most appropriate diagnostic test is:
• Esophagoscopy
• Electrocardiogram
• X-ray Chest
• White blood count
51. The most common indication for operation in Crohn’s disease of the colon is:
• Obstruction
• Chronic debility
• Bleeding
• Perforation
52. Acute appendicitis is most commonly associated with which of the following signs?
• Frequent loose stools.
• Anorexia, abdominal pain, and right lower quardrant tenderness
• White blood cell count greater than 20,000 per cu. Mm.
• Constipation
53. The diagnosis of acute appendicitis is most difficult to establish in:
• Persons aged 60 and older
• Women aged 18 to 35
• Infants younger than 1 year
• Pregnant women
54. When progressive enlargement of a multinodular goiter causes symptomatic tracheal
compression, the preferred management in otherwise good-risk patient is:
• Iodine treatment
• Thyroid hormone treatment
• Surgical resection of the abnormal thyroid
• Radioactive iodine treatment
55. A 45 years old woman has a solitary, nonfunctioning thyroid nodule and fine needle cytology
is nondiagnostic. Which of the following is the initial surgical procedure of choice?
• Total thyroidectomy
• Subtotal thyroid lobectomy and resection of the isthmus
• Total thyroid lobectomy, resection of the isthmus, and modified unilateral neck dissection
• Thyroid lobectomy and resection of the isthmus
56. A 40 years old male undergoes an apparently uneventful total thyroidectomy for follicular
carcinoma of the thyroid. 48 hours later he develops circumoral numbness, followed by
laryngospasm, and then has a generalized seizure. Of the following, which is the first priority?
• Proceed to OR for exploration of the operative site
• Administer 25 ml of 10% calcium gluconate intravenously
• Obtain a serum magnesium measurement and administer intravenous magnesium chloride
STAT
• Obtain a CT scan of the head to evaluate the possibility of brain metastases
57. What is the most appropriate method of feeding for high output fistula following small bowel
resection?
• Oral
• Nasogastric
• Total Parenteral nutrition
• Jejunal
58. ERCP is superior to MRCP in providing
• Diagnostic role
• Therapeutic role
• Excellent imaging of ductal anatomy
• Detecting malignant strictures
59. Calot’s triangle:
• Is bounded medially by right hepatic duct
• Is bounded laterally by cystic duct
• Contains the left hepatic duct
• Contains hepatic artery
60. Which of the following conditions is more likely to result after spinal anaesthesia?
• CSF leak
• Temporal lobe herniation
• Hypotension
• Infection
61. A 42 year old woman involved in a traffic accident presents to the emergency room
complaining of flank pain and gross hematuria, she is hemodynamically stable. The next step
in management is?
• Exploratory laparotomy
• Open Lavage and, if positive, immediate laparotomy
• Immobilization of the pelvis
• Computed axial tomography (CAT) scan with the use of intravenous Contrast
62. During evaluation of the cause of varicocele in a 36- year- old man, attention is directed to the
method of drainage of the left testicular vein, which usually enters which of the following?
• Left adrenal vein
• Left renal vein
• Left inferior mesenteric vein
• Inferior vena cava (IVC)
63. A young female patient presents with a discharging ulcer at middle of the neck on left side.
The edges of the ulcer are undermined. What is the most probable diagnosis?
• Rodent ulcer
• Nonspecific ulcer
• Tuberculosis
• Epithelioma
64. Water is lost from body through a number of routes.Which of the following is INCORRECT?
• Urine
• Faeces
• Respiratory tract
• Saliva
65. Extracellular fluid losses may be extensive in following clinical settings EXCEPT?
• Acute intestinal obstruction
• Major burns
• Hepatic coma
• Acute peritonitis
66. Which of the following conditions CANNOT lead to water depletion in a patient?
• Diabetes inspidis
• Profuse sweating
• Head injury
• Peripheral oedema
67. The body responds to surgical trauma by alterations in water and electrolyte balance. Which
of the following is NOT part of this phenomenon?
• Retention of water
• Retention of sodium
• Sweating caused by fever
• Release of steroid hormones
68. In a normal adult person of 70 kg:
• Insensible water loss is about 200 ml/day
• Sodium requirement is 70 mEq/day
• Daily urine output is 1000-2000 ml
• Protein requirement is 17 gm/24 hours
69. What is normal serum sodium level in adults?
• 3.5 – 5.5 mEq/L
• 5.0 – 10.0 mEq/L
• 35 – 55 mEq/L
• 135 – 145 mEq/L
70. In a normal person of average built:
• Extracellular fluid volume is about 8 liters
• Plasma volume measures about 5 liters
• Total body water measures approximately 40 liters
• Sodium requirement is 150 mEq
71. The blood collection bags contain anticoagulant solution that contains following substances
EXCEPT:
• Phosphate
• Sodium
• Dextrose
• Calcium
72. Which of the following complications is NOT associated with blood transfusion?
• Serum sickness
• Hyperkalemia
• Hypocalcemia
• Incompatibility reactions
73. What is the most suitable temperature for storing blood in the blood bank?
• 20oC
• -4oC to 0oC
• + 2oC to + 6oC
• + 10oC to + 14oC
74. This is TRUE about haemorrhage:
• Bright red blood spurting with pulse is venous in origin
• Bleeding during surgery is primary haemorrhage
• Bleeding caused by blunt injury of abdomen is ‘revealed’ haemorrhage
• Secondary haemorrhage occurs few hours after surgery
75. Shock is common in clinical practice. In all forms of shock there is:
• Diminished circulatory volume
• A hypotensive state with peripheral vasoconstriction
• An impairment of cellular oxygenation and perfusion
• Inadequate cardiac output
76. Which of the following is NOT a feature of hypovolemic shock?
• Peripheral vasoconstriction
• Air hunger
• Increased blood viscosity
• Bradycardia
77. Septic shock:
• Can only be caused by Gram negative bacteria
• Can only be caused by Gram positive bacteria
• Carries a favourable prognosis
• Can be most effectively treated with antibiotics
78. 5% dextrose saline solution is often used in surgical patients. Which of the following
statements is CORRECT about it?
• It is isotonic
• It is hypotonic
• It is hypertonic
• It has good nutritive value
79. A patient has developed shock after trauma due to an accident. Which of the following
solutions has NO ROLE in treating hypovolemic shock?
• Blood transfusion
• Ringer’s lactate solution
• Fresh frozen plasma (FFP)
• Increased amount of oral fluids
80. One gram of nitrogen is equivalent to how much protein?
• 6.25 gm
• 9.25 gm
• 13 gm
• 16 gm
81. Total parentral nutrition (TPN):
• Should deliver about 2500 calories/day to an adult of average built
• Should deliver at least 30 gm nitrogen (180 gm protein)/day
• Is free of complications in today’s settings
• Provides better nutritional support than the enteral route
82. Which of the following statements is CORRECT about Enteral feeding?
• It results in constipation
• It should be considered as second best after intravenous nutrition
• It is not an effective method for long term nutrition
• It often results in uremia and dehydration
83. Mark the INCORRECT statement about oesophagus:
• It is 25 cm long
• Its upper part has striated muscles
• It starts at about 15 cm from the incisors
• It extends from cricopharyngeal sphincter to cruri of diaphragm
84. A middle aged patient complains of dyspepsia of recent origin that is not responding to
medical treatment. Which of the following investigations will be your FIRST CHOICE for further
investigation?
• Stool examination for occult blood
• Barium swallow
• Barium meal follow through examination
• Upper GI endoscopy
85. Following are some statements about carcinoma of oesophagus. Which one of these is
INCORRECT?
• Squamous cell carcinoma is the most frequent type
• Lower third of the oesophagus is most commonly involved
• It usually presents with progressive dysphagia
• It can be effectively diagnosed on history and upper GI endoscopy
86. The length of instrument used for upper GI endoscopy is:
• 30 cm
• 40 cm
• 100 cm
• 120 cm
87. The ‘Tea-pot’ stomach deformity is caused by scarring as a result of long standing gastric ulcer
located at:
• Gastric fundus
• Gastro-oesophageal junction
• Lesser curvature
• Greater curvature
88. Gastrodoudenal ulcers developing as a result of acute physical stress are best termed as:
• Cushing’s ulcer
• Curling’s ulcer
• Kissing ulcer
• Rodent ulcer
89. The pain in gastro-oesophageal reflux disease (GERD) is very closely linked to:
• Food intake
• Change of posture (stooping forward)
• Induction of vomiting
• Feeling hungry
90. The best investigation for bleeding peptic ulcer in ER management is:
• Blood complete picture
• Upper GI endoscopy
• Barium meal examination
• Serum electrolytes
91. A patient presenting to ER with severe abdominal pain and vomiting is labeled as an ‘acute
abdomen’.Which of the following investigations is the FIRST to be advised?
• Blood complete picture
• Plain x-ray abdomen
• Barium meal
• MRI
92. The level of intestinal obstruction can be determined by following steps EXCEPT:
• Appropriate history of the events over last few days
• Thorough clinical examination
• Repeated measurement of abdominal girth
• Radiological examination by contrast studies
93. Number of segments in adult liver is?
• 2
• 4
• 6
• 8
94. Which of the following types of gall stones are most common?
• Calcium oxalate stone
• Phosphate stones
• Cholesterol stone
• Pigment stones
95. Obstructive jaundice may be associated with following clinical findings EXCEPT:
• Generalized pruritis
• Enlarged liver
• Anemia
• Clay coloured stools and dark coloured urine
96. A 50 years old man developed obstructive jaundice that is progressive in nature and painless.
He has a smooth globular mass in RHC. Which of the following statements INCORRECT about
obstructive jaundice in this patient?
• It is due obstruction somewhere in the biliary tract.
• Liver enzymes (ALT, AST) are expected to be markedly raised.
• It is associated with clay coloured stools.
• Treatment of choice is surgery to relieve obstruction.
97. 25 years old male footballer has presented with painful swelling of left knee. MRI report of left
knee reveals ligamentous injury. Most common ligament torn in football players is;
• Anterior cruciate ligament
• Lateral collateral Ligament
• Medial Collateral Ligament
• Patellar ligament
• Posterior Cruciate ligament
• True. A
98. An old lady had a fall on ground with outstretched hand (Rt). The x rays showed the bone
fracture and the distal fragment collapse into extension, dorsal displacement, radial tilt and
shortening. What is your diagnose
• Colle’s fracture
• Dorsal Barton fracture
• Fractured radial Styloid
• Smith fracture
• Volar Barton fracture
• True A
99. During fight, patient’s right arm was extended, abducted and externally rotated. He came to
emergency department while holding his right arm with his left one. Pain was so severe that
he did not give permission for examinations. He is suffering from,
• Fracture proximal humerus head
• Fracture lower 1/3 humerus shaft
• Posterior dislocation Shoulder Joint
• Inferior dislocation Shoulder Joint
• Anterior dislocation Shoulder Joint
• True E
100. One of the premalignant condition of esophagus is Barrett’s esophagus which involves lower
end of esophagus. It is a predisposing factor for:
• Adeno-carcinoma
• Adenoid cystic carcinoma
• Basal cell carcinoma
• Squamous cell carcinoma.
• Malignant melanoma.
• True A
101. 35 years old male has presented with pain epigastrium on/off for last few months. Which of
the following conditions is most likely associated with person having chronic non-specific
gastritis?
• Chronic malabsorption
• Crohn disease
• Helicobacter pylori infection
• Pernicious anemia
• Sprue
• True C
102. Carcinoma of stomach is now the fifth commonest malignancy. Commonest location of
gastric carcinoma within the stomach is:-
• Antrum
• Body
• Cardia
• Gastro-oesophageal junction.
• Fundus
• True A
103. Crohns disease is the chronic granulomatous condition of bowel. Which one of the following
is a specific feature of this disease?
• Continuous colonic involvement
• Inflammation limited to mucosa
• Pseudopolyp
• Toxic megacolon
• Transmural inflammation
• True E
104. Ulcerative colitis is a chronic disease of GIT which present with remission & relapses.
Characteristic of UC is presence of:
• Granulomas
• Fistulae
• Pseudo polyps
• Sinuses
• Skip lesions
• True E
105. A patient has had years of intermittent diarrhea and abdominal pain, but has never
consulted a physician. Eventually he begins to pass fecal material in his urine and he seeks
medical attention. Which one of the following diseases is most likely to cause this
complication?
• Celiac disease
• Crohn's disease
• Diverticulosis
• Ulcerative colitis
• Whipple disease
• True B
106. Which one of the following colonic conditions has the highest risk of developing colorectal
carcinoma?
• Familial polyposis coli
• Hyperplastic polyp
• Inflammatory polyp
• Juvenile rectal polyp
• Peutz-Jeghers polyp
• True A
107. A 72-year-old male has presented with history of progressive dysphagia. He has lost 10 kg
weight in last 4 months. On examination he has supra-clavicular lymph node. What is next
step in his management?
• Barium meal
• Barium swallow
• CT-Chest
• CT-Chest with contrast
• Upper GI-Endoscopy
• True E
108. In patients with inflammatory bowel disease which, one of the following extra
gastrointestinal conditions is more commonly associated with ulcerative colitis than with
Crohn’s disease?
• Gallstones
• Renal stones
• Uveitis
• Venous thrombosis
• Erythema nodosum
• True E
109. An elderly male with history of cardiac bypass presents in emergency with gross abdominal
distention and constipation for past few days and is in great distress. His vitals are not very
promising and his plain X ray abdomen shows large intestinal obstruction. You plan a
laparotomy and find a mass in sigmoid colon. What is the best treatment option in this
patient?
• Abdomino-perenial resection
• Pass a rectal tube and decompress the colon
• Resection of mass with colorectal anastomosis
• Take biopsy and close the patient
• Take biopsy and perform anterior resection
• True C
110. A 45 years old male patient presents with repeated episodes of severe pain in epigastrium,
retrosternal burning and profuse diarrhea for over a year. His upper GI endoscopy shows
several ulcerations in stomach and duodenum. His gastric pH is very low but serum gastrin
levels are high and he is not responding to any acid lowering drug. Most likely diagnosis would
be
• Chronic atrophic gastritis
• Gastric outlet stenosis
• GORD
• MENs II syndrome
• Zollinger Ellison Syndrome
• True E
111. A middle aged male with long history of dysphagia to liquids and solids and retrosternal
region. Pain is usually relieved with a loud belch. His X-ray chest shows air fluid level behind
heart and a gas bubble shadow. Most likely diagnosis would be.
• Achalasia cardia
• Barrett's esophagus
• CA esophagus
• Mallory-Weiss tear
• Para esophageal hiatus hernia
• True E
112. A 45 years old female with prolong history of dyspepsia and pain right hypochondrium
presents in emergency with Right hypochondrial pain, for last 2 days. O/E he febrile with
temperature 102 degree Fahrenheit. she is clinically tender and has got mass RHC. Most
probable diagnosis would be:
• Acute pancreatitis
• Cholangitis
• Chronic Cholecystitis
• Empyema Gall Bladder
• Mucocele Gall Bladder
• True C
113. A 70 years old female with long history of diarrhea, bloating, and facial flushing presents in
ER with pain, tenderness and mass right iliac fossa. Considering this history and examination
most suitable diagnosis would be
• Acute appendicitis
• Carcinoid tumor
• Crohn's colitis
• Diverticulitis
• Ulcerative colitis
• True B
114. A middle aged lady previously on contraceptives for a long time presents with acute severe
central abdominal pain, vomiting, blood mixed stools, severe hypovolemia, leucopenia and
shock. X ray plain abdomen shows thickened gasless small intestinal loops with air bubbles in
a large abdominal vein. She is most probably suffering from
• Acute mesenteric ischemia
• Diverticulitis
• Intestinal amebiasis
• Typhoid
• Ulcerative colitisA
• True A
115. 50 years old male with history of chronic diarrhea, mucous discharge, hematochezia,
polyarthritis and mild to moderate pain in abdomen is diagnosed with cholangiocarcinoma.
Which other disease is associated with these symptoms.
• Ca gallbladder
• Crohn's disease
• Peutz Jeghers syndrome
• Sclerosing cholangitis
• Ulcerative colitis
• True E
116. 50 years old male with h/o bleeding P/R has presented with h/o pneumaturia. The test with
the highest diagnostic yield for detecting a colo-vesical fistula is:
• Barium enema.
• Colonoscopy.
• Computed tomography (CT).
• Cystoscopy
• Ultrasonography.
• True C
117. 25-year-old man presented with bleeding P/R for last 2 weeks. He is giving h/o painful
defecation and the pain persists for 1 to hours after defecation. There is no h/o weight loss,
mucus discharge. What is the most likely diagnosis:
• Anal fissure.
• Carcinoma anal canal.
• Carcinoma rectum.
• Hemorrhoids
• Perianal fistula
• True A
118. A 5 month old baby brought in emergency by Parents. According to them, baby is vomiting
and is in agony for the last one day. On abdominal examination, there is palpable lump in the
right iliac fossa. What is the most likely diagnosis?
• Caelic disease
• Hirschsprung’s disease
• Intussception
• Necrotizing enterocolitis
• Volvulus
• True C
119. A 02 years old male child was brought by his mother with c/o left inguinal swelling, which
appears whenever the child strains. On examination a lump was felt in the left scrotum. What
is the best treatment option in this case
• Bassini’s repair
• Darnining’s repair
• Herniotomy
• Lord’s plication.
• Mesh repair.
• True C
120. 08 months old male child was brought in surgical OPD with c/o empty right scrotum. O/E
testis was palpable at the level of pubic tubercle which could not be brought at the base of
scrotum. What is the best treatment option for this child?
• Orchidopexy at 01 year of age
• Ochidopexy at 03 years of age
• Orchidopexy at 05 years of age
• Orchidopexy at 07 years of age
• Orchidopexy at 09 years of age
• True A
121. A non Muslim uncircumcised child of 08 years of age was brought in emergency with c/o
painful swelling of the glans penis for 02 hours. O/E the glans was swollen and tender with a
retracted foreskin over the glans.
• What is the most likely diagnosis?
• Balanitis xerotica obliterans
• Gonorrheal UTI
• Paraphimosis
• Phimosis
• Syphilis
• True D
122. 1 week old full term male child was diagnosed as a case of umbilical hernia. At what
appropriate age the repair of umbilical hernia should be done if fails to close spontaneously.
• 02 year
• 04 year
• 06 year
• 08 year
• 10 year
• True B
123. 74 years old male has left testicular swelling. On examination its hard. Regarding testicular
seminoma.
• It is common testicular tumor of young children
• Inguinal lymph nodes are usually involved by the tumor
• It is the commonest cause of male infertility
• Orchidectomy via inguinal approach is the standard surgical management
• Ochidectomy via scrotal approach is the standard surgical management
• True D
124. Micturating cysto-urethrogram is the investigation of choice to diagnosis
• BPH
• Vesicoureteric reflux
• Renal stones
• Vesical stones
• Ureteric stones
• True B
125. A 60 year female is seen in emergency with high temperature, rigors, pain with swelling right
loin. Blood pressure recorded was 160/70mmHg and pulse 100/min. There was extreme
tenderness in right loin. Most probably, this patient is suffering from
• Bladder stones
• Pyelonephritis
• Hydronephrosis
• Renal stone
• Ureteric stone
• True C
126. A 52 years old male presented in OPD with progressively enlarging lump right scrotum. On
examination the swelling is tense with negative cough reflex. The testis is not separately
palpable from this swelling and trans-llumination test is positive. What is th most likely
diagnosis?
• Hernia
• Hematocele
• Hydrocele
• Testicular torsion
• Testicular tumor
• True C
127. 68 years old male was presented in emergency with retention of urine and palpable bladder.
On examination prostate was enlarged. What would be first line of management in the
emergency room?
• Foley’s catheterization
• Suprapubic catheterization
• Suprapubic decompression by needle insertion
• Transuretheral resection of prostate
• Trans-vesical prostatectomy
• True A
128. 28 years old male presented with right loin pain. His plain x-ray KUB revealed big stag horn
calculus in the RT renal pelvis. What should be the appropriate treatment option?
• ESWL
• Nephrolithotomy
• Pyelolithotomy
• Nephrectomy
• Nephrostomy
• True C
129. 70 years old male has presented in in OPD with h/o shortness of breath, easy fatigability and
marked weight loss in last few months. On GPE he is anemic and abdominal examination
reveals mass in right iliac fossa. Most likely diagnosis is
• Amebiasis
• Appendicular mass
• Carcinoma caecum
• Diverticulitis
• Intussuception.
• True C

130. 15 years old female diagnosed as a case of thyroglossal cyst. A plan for Sistrunk operation
was made. Which one of the following investigation should be done before surgery?
• Bone scan
• CT scan of chest
• CT scan of neck
• MRI neck
• Thyroid scan
• True E
131. 50 years old male underwent surgery for parotid tumor and presents in OPD with C/O
erythema and sweating at the scar site whenever he tasted food. What is the most likely
diagnosis?
• Facial nerve weakness
• Frey’s syndrome
• Horner’s syndrome
• Occulomotor nerve weakness
• Trigeminal nerve weakness
• True E
132. 34 years old female presented with H/O, fatigue, myalgia, weight loss with increased
appetite. On examination there is a diffuse swelling in front of neck. Investigations revealed
no evidence of infection. What is the most likely diagnosis?
• Grave’s disease
• De-Quervain’s thyroiditis
• Hashimoto’s thyroiditis
• Toxic Adenoma
• Thyroid Malignancy
• True A
133. 25 years old female presented with solitary nodule right lobe of the thyroid. Thyroid Function
Tests report was normal. What should be next investigation of choice?
• Anti TSH antibodies
• Anti thyroglobulin antibodies
• C.T. scan
• FNAC
• Thyroid scan
• True D
134. 35 years female underwent subtotal thyroidectomy for simple multi nodular goiter and
suction drain was placed in the wound cavity. 03 hours after the surgery the patient became
dyspneic. On examination the dressing was soaked with blood, drain contained 200 ml of
blood. What should be your immediate step in the management?
• Change of dressing
• Open the wound urgently.
• Prop up the patient
• Shift the patient in operation theater
• Wash the drain
• True B
135. A 40 years old multiparous, non-lactating women presents with a 2x2 cm tender lump
underneath areola with surrounding inflammation. On examination there is greenish black
discharge from the nipple. Most common cause leading to this condition would be;
• Breast abscess
• Bacterial mastitis
• Duct ectasia.
• Duct papilloma
• Intraductal carcinoma.
• True C
136. A 30 years old female notices a firm 3x4 cm lump in one breast which slips while palpation
and is sometimes tender. FNAC report revealed C2 cells. What would be the appropriate
treatment option for this patient?
• Excision biopsy of the lump
• Wide local excision
• Simple mastectomy
• Modified radical mastectomy
• Quardrantectomy.
• True A
137. A 45 years old female was diagnosed to have Ca Breast. Her tumor was 3.5 cm in greatest
dimensions. She had fixed hard nodes on same side axilla. There was no clinical evidence of
metastasis. In which stage the patient is according to TNM classification.
• Satge IIa
• Stage IIb
• Stage IIIa
• Stage IIIb
• Stage IIIc
• True D
138. In young patients, best treatment option for a right sided Ca breast patient with tumor size of
2.5 cm in greatest dimension with mobile ipsilateral palpable axillary lymph node, without
clinical evidence of distant metastasis is.
• Lumpectomy with axillary sampling
• Lumpectomy with axillary clearance
• Wide local excision with axillary sampling
• Wide local excision with axillary clearance
• Simple mastectomy with axillary clearance
• True C
• I
139. A 30 years old female notices a firm 2 cm painless lump in one breast which is firm in
consistency. It is neither attached to the skin nor to the underlying structures What would be
the next logical step in management.
• Complete triple assessment
• CT scan of Breast
• MRI of breast
• Excision biopsy
• Modified Radical Mastectomy
• True A
140. A 36 year old gentleman presented to the ER with head injury and when the doctor checked
the patient, he was not able to open his eyes because of the swelling around the eyes (black
eye). What is this called?
• Exophthalmos
• Raccoon Sign
• Battles sign
• Panda sign
• Black eye
• True D
141. A 45years old gentleman presented to the OPD with 20 days history of severe headache and
vomiting (multiple episodes) with history of fits and weakness of the right side. What is your
diagnosis?
• Left cerebral Brain Tumor
• Right cerebral brain tumor
• Hematoma
• Encephalitis
• Meningitis
• True A
142. A 35 years old male C/O severe backache after heavy weight lifting. On examination he was
unable to straighten his back and weakness of big toe extension with loss of sensation, lateral
aspect of the calf. What is the most likely diagnosis?
• Pagets disease
• Ankylosing spondylitis
• Verteberal disc polapse
• Rheumatoid arthritis
• Osteoarthritis
• True C
143. 75 years old female developed profound foot drop after fibular fracture. Which one of the
following nerve is most likely injured
• Tibial
• Common peroneal
• Superior gluteal
• Sciatic
• Femoral
• True B
144. 22 years old boy presented in OPD with C/O loss of sensation over a patch of right arm lateral
aspect, following anterior dislocation of the shoulder. Which one of the following nerve is
most likely injured.
• Axillary
• Suprascapular
• Subscapular
• Long thoracic nerve
• Musculocutaneous nerve
• True A
145. A 65 years old male hypertensive presented in the OPD for routine checkup. During
examination an expansile and pulsatile mass was palpable at the umbilical region. Which one
the following should be the first investigation of choice?
• X-ray abdomen supine
• Ultrasound abdomen
• C.T. scan abdomen
• M.R.A.
• C.T. angiography
• True B
146. 60 years old male was found to 40 % of the internal carotid artery stenosis. What is
appropriate management of choice in this patient?
• Carotid endartrectomy.
• Anti-platelet agents
• Watch and wait
• Oral anticoagulants
• I.V. anticoagulants
• True B
147. Which investigation is of highest diagnostic value in the management of benign prostatic
hyperplasia.
• Blood complete
• Cytological examination of urine.
• Renal function tests.
• Prostate specific antigen
• Transrectal ultrasound.
• True E
148. The normal value of urine peak flow from full bladder into the flow meter is?
• >5ml/s
• >10ml/s
• .>15ml/s
• >20 ml/s
• >25ml/s.
• True B
149. Following delivery of her second baby, a statistician develops a tender fluctuating swelling in
her right breast. She had pyrexia of 102 C. She should be treated by
• Antibiotics for gram positive organisms.
• Antibiotics and she should express milk from the infected breast.
• Incision and drainage.
• Mastectomy as underlying malignancy can be a cause.
• Needle aspiration and antibiotics.
• True C
150. 35 years old male presents with h/o of fever and diarrhea 3 weeks after appendectomy, what
is the most likely diagnosis. Ondigital rectal examination rectum is boggy.
• Acute Gastro-entritis
• Amebic dysentery
• diverticulitis
• Pelvic abscess
• Post operative wound infection
• True D
151. A young female patient presents with hard swelling in the neck. There is numbness and pain
with the wasting of hand muscles. Most probably, patient is suffering from
• Raynand’s disease.
• Patient has thromboangtitis obliterans.
• The condition is Acrocynosis.
• Patient has simply sagging of shoulder Girdle
• She is suffering from cervical rib.
• True E
152. 38 years old female patient presented with headache and flushing. O/E she has got
tachycardia and is anxious at the time of presentationShe has a family history of his relative
having died of a thyroid tumor. Most appropriate initial investigation for this patient is
• Chest x-ray
• Measurement of 5-HIAA
• c- Measurement of catecholamine
• d- Thyroid function test
• e- FNAC of thyroid
• True C
153. 42 years old office administrator undergoes left wide local excision and axillary clearance for
a 4 cm invasive ductal carcinoma. After surgery she has developed numbness of her left arm.
• Intercostobrachial nerve.
• Latissimus dorsi muscle.
• Long thoracic nerve.
• Musculocutaneous nerve.
• Serratus anterior muscle.
• True A
154. After undergoing modified radical mastectomy for invasive carcinoma, 50 years old female
aware of winging scapula of the same side. It is due to damage of following structure during
the surgery.
• Intercostobrachial nerve.
• Latissimus dorsi muscle.
• Long thoracic nerve.
• Musculocutaneous nerve.
• Serratus anterior muscle.
• True C
155. A premenopausal female complains of pain and lumps in breast. Examination reveals firm
breast with fine nodularity but no discrete mass. There is cyclical pain and local tenderness.
Most probably, this patient is suffering from
• Fibroadenosis
• Carcinoma breast
• Duct ectazia
• Giant fibroadenoma
• Phyllodes tumour
• True A
156. 56 years old smoker diagnosed as a case of Ca lung by pulmonologist. Anesthetist declare
that patient is fit for lobectomy if he has FEV1 of more than
• 100 cc
• 200 cc
• 500 cc
• 1000 cc
• 1500 cc
• True E
157. Professor of Physiology told the first year class that the normal quantity of Gastric juice is 2
to 2.5. what is the total amount of GI secretions in 24 hour
• 1 liters
• 4 liters
• 8 liters
• 10 liters
• 12 liters
• True C
158. Most widely used system to stage cancer all around the world is TNM staging system.
Regarding TNM staging of Ca Breast T 1 is less than
• 1 cm
• 2 cm
• 3 cm
• 4 cm
• 5 cm
• True B
159. Ranson criteria is used widely in surgical ICU to predict the outcome of patients of acute
pancreatitis. Which of the following is not a parameter to predict the severity of an attack of
acute pancreatitis on admission in Ranson score?
• White cell count
• Serum amylase
• Age
• Blood glucose
• LDH and AST
• True B
160. A patient 6 weeks after an attack of acute pancreatitis develops a swelling in upper abdomen
which is increasing in size. An ultrasound scan reports pancreatic pseudocyst of more than
8cm. Which of the following treatment is most appropriate in this patient?
• Excision of cyst
• Cystogastrostomy
• Pancreatectomy
• Whipple operation
• Watch and see
• True B
161. 1. A Sportsman is complaining of pain around is left elbow joint. On examination the elbow is
tender over lateral epicondyle. This type of injury usually occurs in:
• a. Badminton player
• b. Basket ball player
• c. Cricket player
• d. Hockey player
• e. Tennis player*
162. 4. 50 years old smoker has presented with progressive dysphagia. A gastroenterologist is
performing upper GI endoscopy. Which of the following is true regarding findings on upper Gl
endoscopy?
• a. Distance between incisors and lower esophageal sphincter is 25 cm
• b. Endoscopy is performed routinely under General Anesthesia
• c. Lining of esophagus is ciliated columnar
• d. Second constriction appreciated during endoscopy is aortic and bronchial constriction*
• e. Total length of esophagus is 40 cms
163. 15. 45 years old female presents in OPD with ho painful defecation for last 5 days. On further
inquiry she reveals that pain settles I hour after passing stools and there is also bleeding per
rectum during defecation. There is no history of discharge or weight loss along with it. Most
likely diagnosis is
• a. Anal fissure*
• b. Anal stenosis
• c. Perianal Abscess
• d. Peri-anal fistula
• e. 3 degree hemorrhoids
164. 18. A 45 years old diabetic female with prolonged history of dyspepsia presents in emergency
with right hypochondrial pain, for last 2 days. O/E she has toxic look and is febrile with
temperature 102 degree Fahrenheit. Abdominal examination reveals tender mass in right
hypochondrium. Most probable diagnosis would be:
• a Acute pancreatitis
• b. Cholangitis
• c. Chronic Cholecystitis
• 3. Empyema Gall Bladder*
• e. Mucocele Gall Bladder
165. 22. 55 years old male presents with ho bleeding per rectum and altered bowel habits for last
6 months. He says he has lost about 15 kilograms of weight in last 3 months. On examination
there is no mass in abdomen nor is there any positive finding on digital rectal
examination/proctoscopy. Next logical step in management is:
• & Colonoscopy*
• b. CT abdomen with contrast
• c. Serum Carcino-embrionic antigen
• d. Ultrasound abdomen.
• e. X-ray erect abdomen
166. 30. 8 month old infant is brought to OPD with undescended testes. Best time for orchidopexi
is:
• a. 6 to 12 months*
• b. 13 to 18 months
• c. 18 to 24 months
• d. 24 to 30 months
• e. 30 to 36 months

IMDC Surgery MCQs

Paper A
Surgical Anatomy: (06)
1. Regarding blood supply of Thyroid gland which of the following is true.
• Inferior thyroid artery is branch of external carotid artery
• Inferior thyroid artery is branch of Internal carotid artery
• Superior thyroid artery is branch of external carotid artery
• Superior thyroid artery
• is branch of internal carotid artery
• Thyroid Ima artery is a branch of external carotid artery
2. Which of following statement is true regarding anatomy of Breast.
• Breast tissue extends from 2nd to 4rth rib
• Lobules are more dense in old age
• Sentinel node is last node to drain involved Breast tissue
• 50% Lymphatic drainage of breast is through Internal mammary lymph nodes.
• 85% Lymphatic drainage of breast is through Axillary lymph node.
3. Which of the following is true regarding calots triangle
• Common bile duct forms its medial boundary
• Common hepatic duct forms its medial boundary
• Cystic artery forms its medial boundary
• Portal vein forms its medial boundary
• Right Hepatic artery forms its medial boundary
4. Common site of obstruction of an abdominal hernia is
• Body of hernia
• Fundus of hernia
• Inside abdomen
• Neck of hernia
• Wall of hernia
5. Which of the following cells found in the stomach secrete pepsinogen?
• APUD Cells
• Chief Cells
• Goblet cells
• Parietal cells
• Stomach Cell
6. Five years old boy was brought to the pediatrician for routine checkup at the time of
admission in the school. On examination, pediatrician was not able to palpate the right testis
in the scrotum. Which of the following statement is true regarding undescended testes.
• Chances of malignancy are 50 times more than normal person.
• His right testis is functionless organ
• Orchidopexy should be attempted before the age of 1 years
• Surgical treatment does not reduce the risk of malignancy.
• Undescended testes are always functionless.

Wounds, Tissue repair, scars , accident & Emergency, warfare injuries: (14)
1. A 27 years old lady was admitted in female surgical ward with para-umbilical hernia. Her
hernioplasty is planned. What are the chances of wound infection in this operation?
• 01-02 %
• 04-05 %
• 06-10 %
• 10-20 %
• 20-40 %
2. 25 years old male presented in A & E department after RTA with painful swelling of left leg.
On examination his left leg is swollen, tense and he feels excruciating pain on dorsiflexion.
Distal pulses are feeble.
• What is the next step in management:
• Application of crape bandage
• Fasciotomy
• Opioid analgesic
• NSAIDs
• Repair of vessels
3. A young man is driving a car under influence of alcohol. He is brought to ER with degloving
injuries of upper limbs. On examination there is contamination of his upper limb injuries with
dirt and pebbles. What type of wound is it?
• Clean Wound
• Crushed Wound
• Puncture wound
• Tidy wound
• Untidy Wound
4. 10. 55 years old male presents in emergency after a bomb blast in a mosque. In a case of
explosive blast injury, the structure most commonly injured by blast wind is.
• Brain
• Extremities
• Heart
• Middle ear
• Spinal cord
5. 11. A teenager gets a cut to palmar surface of his hand extending to forearm while punching
mirror in a fit of rage. She is unable to move her thumb and index finger properly. Which
Nerve is injured in this case.
• Anterior Interosseous Nerve
• Median Nerve
• Radial Nerve
• Superficial palmer nerve
• Ulnar nerve
6. 12. A 40 years old man presents to E.R after road traffic accident. On examination his Pulse is
120/min and Blood pressure is 90/70. His resuscitation is started during secondary survey.
What is the fluid of choice for resuscitation?
• 5%Dextrose water
• 10% Dextrose water
• Pead’s Saline
• Dextrose Saline
• Ringer’s Lactate
7. 13. 35 years old female underwent appendicectomy. Most commonly injured nerve during
appendectomy
• Genital branch of genitofemoral nerve
• Ilioinguinal nerve
• Ilio-hypogastric nerve
• Obturator nerve
• Sciatic Nerve
8. 14. A 2-year-old toddler falls while playing sustaining bruise on his knee. Which cell type is
responsible for cessation of blood flow in his wound healing process?
• Fibroblasts
• Macrophage
• Neutrophils
• Platelets
• lymphocytes
9. 15. A young man is driving a car under influence of alcohol. He is brought to ER with fracture
of multiple ribs in right hemithorax. He is complaining of severe chest pain. On examination
he has severe respiratory distress with absent breath sounds over the whole of right lung
field along with engorged neck veins.
• What is the first step to do in this particular case?
• Chest Tube insertion in triangle of safety in Right lung
• Emergency tracheostomy
• Emergency endotracheal Intubation
• Maintaining IV line and starting blood transfusion
• Needle thoracotomy with wide bore Canula in right mid clavicular line in 2nd intercostal
space
10. A hypertrophic scar has been formed over the site of burn of a 12-year-old boy’s arm. What
type of collagen is more predominant in a scar?
• Type 1
• Type 2
• Type 3
• Type 4
• Type 5
11. A poly trauma patient is brought to emergency department. His x-ray pelvis shows iliac bone
fracture with disruption of pubic symphysis. Which of the following statement is true?
• Pelvic fractures can never be fatal
• Rupture of iliac arteries can cause hypovolemic shock
• This patient requires urgent fixation by surgery
• These fractures usually are not associated with any other injury
• These fractures usually cause trivial injuries
12. Hemorrhage can be classified as primary, secondary & reactionary. Which is true regarding
secondary hemorrhage.
• Occurs at the time of surgery
• Occurs within 24 hours of surgery
• Occurs due to slip of ligature
• Occurs between 7-10 post-operative days
• Occurs due to bleeding disorder
13. 20 years old motorcyclist presents in Emergency department with severe chest pain and
difficulty in breathing. O/E his pulse is 134/min, B.P is 80/50 mm of HG with absent breath
sounds in right chest, percussion note is hyper-resonant. Chest x-ray reveals rib fractures with
mediastinal shift and Next step in management is:
• CT chest with contrast
• Elective thoracotomy
• Emergency room thoracotomy
• Inserting wide bore cannula in second intercostal space
• Start mechanical ventilation
14. 24-year old person had a chest injury. Clinical and radiological examination revealed 2-3 Ribs
fracture with associated haemo-pneumothorax. Which of the following is best initial step in
Emergency Management of this patient?
• Endo-Tracheal Intubation and positive pressure ventilation
• Immediate Thoracotomy
• Placement of chest Tube with under-water seal
• Simple Analgesics and Strapping of fractured ribs
• Surgical fixation of fractured ribs

Trauma & Life support (10)


1. 45 years old man presents in Emergency department after a street fight in state of confusion
with distended neck veins. He is assaulted by an icepick in left chest. O/E his pulse is 124/min
B.P is 80/50 mm of HG. Auscultation of chest reveals bilateral breath sounds in both
hemithorax with muffled heart sounds.
• Most likely diagnosis is
• Aortic Rupture
• Cardiac Tamponade
• Hemothorax
• Pneumothorax
• Tension pneumothorax
2. 35 years old car driver had head on collision with truck resulting in fracture of 4 left lower ribs.
Auscultation reveals positive bowel sounds in left chest.
• Cardiac Tamponade
• Hemothorax
• Rupture of the diaphragm
• Sliding hiatus hernia
• Tension Pneumothorax
3. 34years old male presented in emergency after motorbike accident. He complained of severe
pain in left chest. O/E he was vitally stable with tenderness over anterior chest & on
auscultation he has decreased breath sounds in left chest with dull percussion note. X-Ray
chest reveals fractures of left 6th and 7th ribs. Most probable diagnosis is:
• Cardiac Tamponade
• Flail chest
• Hemothorax
• Pneumothorax
• Tension Pneumothorax
4. 28years old shopkeeper was brought in emergency department by police after he was beaten
by a thief with the stick. The patient has complaint of severe pain in right chest and dyspnea.
A chest tube was inserted after making a diagnosis of right hemothorax which drained1500 ml
of blood instantly. What would be the next step in the management of the patient?
• Conservative management
• Exploratory laparotomy
• Monitor chest drain output for next 24 hours & transfuse blood
• Planned Emergency thoracotomy
• Ventilator support
5. A patient with severe head injury came to the emergency with GCS 8/15. What is the most
important step in the management of a severe head injury as in this patient?
• Airway, Breathing & Circulation
• CT Scan Brain
• I/V Fluids
• Managing skull fracture
• Stop the bleeding
6. A patient presents to the ER with RTA in unconscious state and after resuscitation becomes
conscious without any deficit. After 4 hours, patient suddenly becomes unconscious with one
sided constricted pupil. What is this condition called?
• Amnesia
• Chronic interval
• Dementia
• Lucid Interval
• Parkinson’s
7. What is the most common source of Extradural hematoma?
• Middle Cerebral artery
• Middle cerebellar artery
• Posterior Cerebral artery
• Middle Meningeal artery
• Maxillary artery
8. A patient with head injury comes to the emergency with B/L peri orbital ecchymosis and CSF
rhinorrhea. What is your initial diagnosis?
• Severe Head Injury
• Occipital Fracture
• Base of Skull Fracture
• Intra cerebral bleed
• Brain stem bleed
9. 50 years old soldier has presented in Accident and Emergency department with swollen left
leg. There is blackish discoloration of skin with brownish serous fluid having sweet smelling
odor. On palpation there is crepitus. Organism involved in this condition is:
• Bacteroides Species
• Clostridium perfringes
• E. Coli
• Staphylococcus
• Streptococcus
10. The initial assessment of a severely injured patient includes the following except?
• Airway obstruction
• Breathing
• Circulation
• Intracranial haematoma
• Routine imaging
11. All patients involved in road traffic accidents are treated according to ATLS protocol.
• Which of the following is not a part of ATLS protocol?
• Definitive treatment
• Primary survey
• Investigations as first priority
• Resuscitation
• Secondary survey
12. A 35 year male sustains a blunt trauma to his abdomen when falling from a building. There is
pain and tenderness all over the abdomen. Pulse 120/min and BP 90/60mmHg. Resuscitation
is in progress according to ATLS protocol. A DPL( Diagnostic peritoneal lavage) is planned for
internal bleeding.
• Which statement is not trueregarding DPL?
• Aspirate of more than 10ml of blood is deemed as positive
• Presence of > 100,000 red cells/µL is deemed positive
• FAST( Focused abdominal sonar for trauma) has replaced DPL
• Presence of blood on DPL always requires surgery
• A NG tube is passed to empty the stomach before DPL
13. A man fell of a bridge while driving a motor cycle at high speed. He is in severe pain and
tender in left lower chest(9th to 11th ribs) and in abdomen. On examination the pulse is
120/min, BP 90/70. Abdomen is rigid and still. DRE reveals local tendness and boggy feel
anteriorly. What is the diagnosis?
• Pneumothorax
• Rupture of spleen
• Liver trauma
• Colonic rupture
• Rupture of diaphragm

Wound infection, special infections AIDS, Sterilization: (16)


1. 35 years old man presents with pain right hypochondrium and fever with rigors and chills for
last 3 days. On further inquiry he revealed that he is suffering from diarrhea for last 3 weeks.
There is tender mass in right hypochondrium extending down to right lumber region. Most
likely diagnosis is:
• Acute appendicitis
• Chronic Cholecystitis
• Empyema Gall Bladder
• Hydatid cyst of liver
• Liver Abscess
2. All of the following are complication of liver trauma except
• Intrahepatic hematoma
• Liver abscess
• Bile collection
• Biliary fistula
• Hydatid cyst
3. 35 years old man is undergoing hernioplasty. In a case of inguinal hernia repair, prophylactic
antibiotic should be given
• At the time of incision
• At the time of induction of anesthesia
• One hours before operation
• Two hours before operation
• Three hours before operation
4. 50 years old gardener has presented with tonic clonic convulsions for last one day. There is
also h/o difficulty in swallowing and spasms of different body muscles. Which of the following
is true statement regarding this condition
• Organism involved are gram positive aerobic cocci
• Organism involved are gram positive an-aerobic cocci
• Organism involved are gram positive aerobic rods
• Organism involved are gram negative aerobic cocci
• Organism involved are gram positive an-aerobic rods
5. 45 years old Diabetic male has presented in E/R with h/o painful swelling in Peri-anal region
for last 1 week. On examination there is red, hot and tender swelling in left buttock which is
fluctuant. The classical bug involved in this condition is:
• Bacteroides
• E. coli
• Klebciella
• Proteus
• Staphylococcus
6. A 22 years old female who had been on broad spectrum antibiotic for last 1 week was
discharged on oral antibiotics and painkillers. After 3 days she is presented with fever and
bloody diarrhea about 7 episodes per day. Which medicine will be the drug of choice in this
condition?
• Amoxicillin
• Ampicillin
• Ciprofloxacin
• Erythromycin
• Vancomycin
7. Gastroenterologist has to sterilize endoscope after every intervention. In an endoscopy
service commonest method employed all over the world for sterilizing delicate endoscopes is.
• Autoclaving at 121o
• Ethylene oxide
• Gamma irradiation
• Gluteraldehyde
• Hot air oven
8. . A 5-year-old toddler spills over hot tea over himself while playing. What is immediate first
aid to be given at site of accident?
• Butter
• Ice
• Honey
• Tap water
• Tooth paste
9. A 2-year-old toddler falls while playing sustaining bruise to his knee. Which cell type is
responsible for cessation of blood flow in his wound healing process?
• Fibroblasts
• lymphocytes
• Platelets
• Macrophage
• Neutrophils
10. 44. 50 years old male has presented with h/o of pain RHC for last 10 hours. He is also running
fever for last 3 day along with bloody diarrhea. He is laborer by occupation and eats in
restaurant mostly. His sigmoidoscopy reveals flask shaped ulcers. Most likely causative
organism is:
• E coli
• Entamoeba Histolytica
• Staph aureus
• Neisseria meningitis
• Pseudomonas aerigenosa
11. 35 years old IV drug addict develops multiple abscess in body. He is diagnosed to have HIV
infection. Which of the following blood cell type is most affected in this HIV-positive
individuals.
• CD-4 Lymphocytes
• CD-8 Lymphocytes
• Platelets
• Neutrophils
• Red blood cells
12. 25 years old male presents on 5th post -operative day after appendicectomy with a pussy
discharge from his wound and pain in wound site. On general physical examination he has a
pulse rate of 110/mins and temperature 100 F. Local examination reveals swollen and tense
wound pouring puss on pressure.
• What is the next step in management.
• Apply dressing and discharge the patient
• Open stiches drain puss and leave the wound open
• Open stitches drain puss and restitch wound
• Start analgesic
• Start anti-pyretics
13. 35-year-old IV drug addict is screened for HIV and his ELIZA turns out to be positive. He has
developed chest infection which is not responding to antibiotics. What can be the most likely
cause?
• COVID- Pneumonia
• H-influenza Pneumonia
• Pneumococcus pneumonia
• Pneumocystis carinii pneumonia
• SARS-Infection
14. 35 years old sheep farmer has presented with right hypochondrial pain and fever last 1 week.
On examination there is tender mass in right hypochondrium. Complete blood count reflects
marked eosinophilia. Which of the following statement is true regarding this condition?
• CT is the best investigating tool
• Lung is the Commonest organ involved
• Medical therapy has got no role in this condition
• Surgery is mandatory treatment in this condition
• Ultrasound is best investigating tool
15. 35 years old male presents with intestinal obstruction. He has a family history of tuberculosis
and consultant is suspecting intestinal tuberculosis as a cause of obstruction. Which of the
following is true regarding intestinal tuberculosis?
• Common site of stricture is ileocecal junction area
• Hyperplastic form is more aggressive than ulcerative form
• Ischemic necrosis is pathognomonic feature
• Surgery is contraindicated in Intestinal tuberculosis
• Ultrasound abdomen is gold standard to confirm diagnosis
16. 50 years old male truck driver has presented in emergency with h/o abdominal pain,
constipation & multiple episodes of vomiting for last 3 days. He is also running fever for last 1
week for which he was taking medication from GP. There is no past history of epigastric pain
or dyspepsia although he frequently eats food from different Dhaaba hotels as he is a truck
driver. On examination his abdomen is tense and he is tender with board like rigidity. What is
the most likely diagnosis.
• Diverticulitis
• Intestinal tuberculosis
• Perforated duodenal ulcer
• Mesenteric lymphadenitis
• Typhoid perforation

Nutrition: (06)
1. 51. All of the following are fat soluble vitamins except
• Vit A
• Vit C
• Vit D
• Vit E
• Vit K
2. 52. Which of the following is the site of absorption of Vit B-12
• Stomach
• Duodenum
• Ileum
• Transverse Colon
• Descending Colon
3. 53.60 years old is male is admitted in ICU with h/o stroke. A prolonged recovery was
expected. Which one of the following technique would be appropriate to fulfill the nutritional
requirement of this patient?
• Feeding jejunostomy
• Open gastrostomy
• Nasogastric feeding
• Total parenteral nutrition
• Percutaneous endoscopic gastrostomy
4. Hartmann solution is the main solution for resuscitation. Which of following ion is not
component of Hartman solution.
• Bicarbonate
• Chloride
• Floride
• Potassium
• c. Sodium
5. Potassium is the main ion of intracellular compartment. Average daily requirement of
potassium in 50 kg woman is approximately
• 0.5 mili mole/day
• 5 millimole
• 50 millimole/day
• 500 millimole/day
• 5000 millimole/day
6. 56. Parental nutrition is given either by central line or Peripheral line. Which of the following
is a complication of parenteral nutrition on Central line insertion?
• Abdominal cramps
• Bloating
• Constipation
• Diarrhea
• d. Pneumothorax

Fluid & electrolyte & Acid-Base Balance, Blood transfusion, shock & Metabolic response
to injury. (18)
1. 57. Reactionary hemorrhage is the one which occurs:
• As a result of slip of ligature
• As a result of violent coughing on recovery from anesthesia
• Due to a blood transfusion line being disconnected
• Within 6 hours of operation
• 7-14 days after operation
2. Commonest cause of death in surgical patient is:
• Breast cancer
• Colo-rectal carcinoma
• Lung cancer
• Myocardial infarction
• Shock
3. A rescue team recovered a patient from a collapsed building after an earthquake. The patient
was suspected to have crush syndrome. Which of the following is best step in emergency
management of crush syndrome?
• Blood transfusion
• Colloids
• Normal Saline
• Plasma transfusion
• 5% Dextrose water
4. 65 years old male presented in emergency after RTA. He had a fracture shaft of femur after a
vehicle ran over his thigh and crushed it. Which of the following complication is specific for
this type of injury.
• Anaphylactic shock
• Hypovolemic shock
• Hemorrhagic shock
• Myoglobinuria
• Septic shock
5. In acute emergency if blood has to be given immediately without full laboratory cross-
matching it is best to give blood which is :
• Group-O RhD -ve
• Group-O RhD +ve
• Group AB RhD -ve
• Group AB RhD +ve
• Group A RhD -ve
6. A 60-year-old female operated upon for perforated appendix. She was kept nil per oral. On 3rd
post operative day patient developed weakness and fatigue associated with vomiting and
absent bowel sounds. Her ECG revealed S-T depression and appearance of U wave. The most
likely diagnosis is :
• Hypercalcemia
• Hypocalcemia
• Hyperkalemia
• Hypokalemia
• Hyponatremia
7. Blood transfusion is one of the commonest procedures done in public sector hospitals. Main
hazard of blood transfusion in Pakistan is
• Transfer of bacterial infections
• Transfer of Hepatitis C
• Transfer of malarial parasite
• Transfer of syphilis
• Transfer of HIV
8. During massive transfusion therapy, which of the following electrolyte abnormalities may lead
to disaster if not addressed immediately?
• Hypocalcemia.
• HCO3 deficiency.
• Hypochloremia.
• Hypomagnesaemia
• Hyperglycemia
9. A 35-year-old motor cyclist involved in road traffic accident. He has open fracture of his right
femur. On arrival he is tachypneic and confused with cold clammy skin. Which of the following
physiological changes is most likely present?
• Decreased reabsorption of water from tubules
• Metabolic Alkalosis
• Respiratory acidosis
• Respiratory alkalosis
• Increased synthesis of glycogen in liver
10. 35 years old female presents with colicky abdominal pain & distention for last one day. She is
constipated and has had multiple episodes of projectile vomiting. Her Arterial blood gases are
ordered. What abnormality do you anticipate in this patient?
• Metabolic acidosis
• Metabolic Alkalosis
• Respiratory Acidosis
• Respiratory alkalosis
• All of above
11. 32 years old motorcyclist had RTA 30 minutes ago in a confusional state. On arrival in A&E, his
pulse is 120/ minute, BP is 90/60 and he is looking very pale. He has massively swollen right
thigh, which is tender immobile and shortened. His state of shock is
• Anaphylactic shock
• Hypovolemic shock
• Neurogenic shock
• Septic shock
• Vasovagal shock
12. Hypovolemic shock can be classified into 3 categories. In mild hypovolumic shock, there is
only
• Increase pulse rate
• Lactic acidosis
• Marked decrease in urine out put
• Marked increase in breathing rate
• Pain
13. 15 years old girl presents in E/R with difficulty in breathing state of confusion after a bee
sting. Her pulse is 102/min and her blood pressure is 90/70. Condition of this young girl is due
to
• Anaphylactic shock
• Cardiogenic shock
• Hypovolemic shock
• Neurogenic shock
• Septic shock
14. 44 years old male has presented in ER with history of abdominal pain and multiple episodes of
vomiting for last 2 days. X-ray erect abdomen reveals Gas under right dome of diaphragm. His
pulse is 120/min, blood pressure is 90/60 but he has warm extremities. Which of the following
statement is true regarding septic shock.
• Allergic reaction is the root cause
• Endotoxin are the cause of septic shock
• Hypovolemia is the root cause
• Myocardial dysfunction is the root cause
• Reduction in preload is due to mechanical obstruction
15. While performing cholecystectomy iatrogenic injury to right hepatic artery results in profuse
bleeding. This type of hemorrhage is defined as
• Concealed bleeding
• Massive hemorrhage
• Primary hemorrhage
• Reactionary hemorrhage
• Secondary hemorrhage
16. Which of the following is true regarding hemorrhagic shock?
• Blood volume loss than 20% of total blood is class 1 shock
• Blood volume loss between 15-30 of total blood volume is class 2 shock
• Blood volume loss between 15-30 of total blood volume is class 3 shock
• Blood volume loss between 15-30 of total blood volume is class 4 shock
• Blood volume loss greater than 30% of total blood volume is class 4 shock
17. 35 years old male was transfused 1 pint of blood after thyroidectomy. Which of the following
is not a complication of single blood transfusion?
• Allergic reaction
• Febrile reaction
• HCV transmission
• Hemolytic reaction
• Hypothermia
18. Which of the following statement is true regarding resuscitation in shock.
• Colloids are better than crystalloids for resuscitation
• Blood loss must always be replaced by blood transfusion
• Long & wide bore cannulas are best for fluid resuscitation
• Resuscitation should not be started before determining cause of shock
• Short & wide bore cannulas are best for fluid resuscitation
19. Which of the following therapies will improve End-Organ perfusion in the patient with severe
Hemorrhagic Shock?
• A litre of Isotonic crystalloid
• Epinephrine injection
• Sodium bicarbonate infusion
• Trendelenburg position
• O-negative Blood without cross match
20. An 80 years old female is admitted with Hb of 5.7g/dl. She is transfused 4 units of blood over
the period of 6 hours. Four hours later she is found breathless with bilateral basal crepts. X ray
shows Fluffy haziness in lower lobes. What is the diagnosis?
• DIC
• Fluid overload
• Hypocalcemia
• Infection
• Transfusion reaction

Principle of Radiology/Endoscopic procedures (12)


1. 65 years old male presents with Intestinal perforation on X-ray it can be seen as
• Double bubble appearance
• Air under diaphragm
• Fluid under diaphragm
• Honey comb appearance
• None of the above
2. 60 years old male is diagnosed to have well-differentiated adeno-carcinoma of sigmoid colon.
What is the next best investigation required in this patient.
• Colonoscopy
• CT abdomen with oral and IV contrast
• Radionuclide studies
• Ultrasound abdomen
• Ultrasound abdomen & Pelvis
3. 25 years old male presents with head injury in accident and emergency department after road
traffic accident. His GCS is 10/15 which is best investigation to determine the extend of injury.
• Ct angiography
• CT-Scan
• MRI
• USG
• X-ray skull
4. 25 years old motorcyclist presents with head injury after RTA. He is suspected to have extra-
dural hematoma. What is the finding on CT Brain of a patient with Extra Dural (Epidural)
Hematoma
• Bilateral Intracranial Bleed
• Concavo-convex shaped Bleed
• Bi-convex shaped bleed
• Intra-cerebral bleed
• Depressed skull fracture
5. 55 years old female presents with pain epigastrium radiating towards back. She is suspected
to have Gall stone pancreatitis. Investigation of choice to confirm diagnosis is:
• CT Abdomen
• CT- Abdomen with contrast
• Serum Amylase
• Serum Lipase
• Ultrasound abdomen
6. 50 years old female was operated for Pelvic malignancy under General anesthesia. On 3 rd
post-operative day she developed sudden pain in chest with shortness of breath. ECG rules
out myocardial infarction. Her ABGs reflect hypoxic changes with decreased CO2
concentration in blood. Which investigation is Gold standard to confirm diagnosis
• CT Angiogram
• Doppler ultrasound
• Echocardiogram
• MRI chest
• X-ray chest PA view
7. Ultrasound is the first line investigation to detect pathologies in:
• Liver
• Biliary tree
• Kidney
• uterus
• All of above
8. 50 years old obese lady presented in ER with swelling of both legs with dull pain. She is taking
OCPs. On examination her legs are swollen and tense what is the most accurate way to
determine diagnosis?
• CT of both legs
• CT Pelvis
• Dopplers ultrasound of legs
• MRI of legs
• Ultrasound abdomen
9. 35 years old female presents with red, hot and tender swelling of left breast. She is a lactating
mother. What investigation will you suggest to confirm diagnosis.
• CT- Chest
• Complete blood count
• MRI chest
• Ultrasound Breast
• X-ray Chest
10. 70 years old patient with atrial fibrillation presents with severe pain in right lower limb. On
examination his distal pulses are absent. What initial investigation will you order in this
patient.
• Angiography
• CT
• Doppler ultrasound
• MRI
• X-ray
11. 65 years old lady presented with pain epigastrium after taking fatty meal. She has a long
history of heartburn for which she is taking antacid. What is the best investigation to confirm
the diagnosis.
• Barium meal
• CT abdomen
• MRI abdomen
• Upper GI endoscopy
• X-Ray erect abdomen
12. 35 years old lady presented with pain epigastrium. She is advised upper GI endoscopy.
Gastroenterologist is counselling her about the complications of upper GI endoscopy. Which
of the following is not the complication of upper GI Endoscopy.
• Perforation of common bile duct
• Perforation of duodenum
• Perforation of esophagus
• Perforation of Jejunum
• Perforation of stomach
13. 55 years old male presented with multinodular goiter. Clinically it’s a simple goiter but there is
a discrete swelling which is tense and fluctuant. What is next step to reach diagnosis.
• CT neck
• FNAC of swelling
• MRI of neck
• Thyroid scan
• Ultrasound neck
Principle of Anesthesia & pain Management 08
1. 48 years old male is undergoing open heart surgery. Anesthetist wants to assess the
respiratory tract before operation. Best investigation to assess lung function is
• Blood gases
• Chest x-ray
• Forced expiratory volume in 1st second
• Forced vital capacity
• Peak expiratory flow rate
2. 45 years old man presents with a big swelling in right arm. Surgeon has planned to perform
excision under local anesthesia. The effective maximum dose of lignocaine is
• 1mg/kg body weight
• 2mg/kg body weight
• 3mg/kg body weight
• 4mg/kg body weight
• 5mg/kg body weight
3. Surgeon plans to excise sebaceous cyst under local anesthesia. He has planned to use
lidocaine which is short acting local anesthetic. The suggested mechanism of action of
Lignocaine at the cellular level is
• It blocks chloride channels
• It blocks potassium channel
• It blocks the peripheral receptors
• It blocks sodium channel
• It decreases the threshold of receptors
4. Young anesthetist is having problem while intubating a short-necked female. Which is the
most reliable way to ascertain correct placement of an endotracheal tube?
• Detection of breath sounds on auscultation
• Detection of pressure in inflated balloon
• Measurement of end-tidal CO2 concentration
• Movement of chest wall on manual inflation
• Trying to visualize trachea
5. 25 years old male presents with numbness of skin around right inguinal region and medial
aspect of right thigh after right sided hernioplasty. Nerve injury resulting in this condition is:
• Femoral branch of genitofemoral nerve
• Genital branch of Genitofemoral nerve
• Illio-inguinal Nerve
• Illio-hypogastric Nerve
• Obturator Nerve
6. Bier’s block is useful technique to provide anesthesia for upper limb surgery. Which is the best
anesthetic agent used for Bier block?
• Amethocaine
• Bupivacaine
• Lignocaine
• Levobupivacaine
• Prilocaine
7. 55 years old male has presented with h/o severe excruciating epigastric pain radiating
towards back with multiple episodes of vomiting. His TLC Count is 20,000/ IU and Serum
amylase is 800 I/U. You are ordered by consultant in charge to relieve his pain. Which
amongst the following would be best analgesic for this patient.
• Intramuscular Diclofenac
• Intra venous Hyoscine
• Intra venous Ketorolac
• Intra venous Nalbuphine
• Intravenous Paracetamol
8. 2 years old boy is booked for herniotomy on elective list. He is resisting staff nurse while she is
trying to pass an intravenous cannula. Staff nurse requests the anesthetist to sedate the
patient. Best inhalational agent for him would be
• Enflurane
• Ether
• Halothane
• Isoflurane
• Sevoflurane
9. Most appropriate statement regarding the choice of antibiotics for prophylaxis is
• Antibiotic prophylaxis is harmful in excessive blood loss
• Continuation therapy is contraindicated in unexpected contamination
• Prosthetic insertion requires antibiotics on completion of surgery
• IV antibiotics half an hour before induction of anesthesia is recommended
• Urethral instrumentation for stricture needs no prophylaxis
10. 45 years old female has developed very high-grade fever with severe muscle pains in early
postoperative period. She is diagnosed to have Malignant Hyperthermia. Best treatment
option for this patient would be
• Atropine
• Dantrolene
• Morphine
• Neostigmine
• Valium

Arterial, Venous & Lymphatic disorders (08)


1. A multigravida female presented in OPD with history of varicose veins for 8 years and recently
has developed a non-healing wound near medial malleolus for 6 months. What is definitive
indication for surgical management of varicose veins
• Dilated veins
• Prominent Veins
• Reticular veins
• Tortuous veins
• Ulcer formation
2. 65 years old man presents with intermittent claudication. He is unable to walk for more than
10 minutes. He is being evaluated for severity of disease. The normal ankle brachial pressure
index is
• 0.1-0.2
• 0.3-0.5
• 0.5-0.7
• 0.9-1.3
• 1.4-1.6
3. 50 years old female was operated for knee replacement. On 3rd post-operative day she
developed sudden painful swelling of her left lower limb. On examination her limb is swollen
with pain on passive movement of limb. What is most likely diagnosis?
• Acute limb Ischemia
• Chronic limb ischemia
• Chronic Lumbago
• Deep venous thrombosis
• Sciatica
4. 65 years old female presents with severe pain in her left leg for last 2 hours. She is a patient of
ischemic heart disease. Her doctor is suspecting embolus blocking her left femoral artery.
Golden window in which patients’ limb can be saved in acute limb ischemia is.
• Less than 6 hours
• b Less than 8 hours
• Less than 10 hours
• Less than 12 hours
• Less than 14 hours
5. 54-year-old female returning from Australia develops a tender, warm, swollen right calf after
12hour long flight. She smokes five cigarettes daily and takes the oral contraceptive pill. She
has no chest pain or shortness of breath. Which single investigation is the most appropriate?
• Ascending venography
• CT pulmonary angiography
• abdominal ultrasound
• Duplex ultrasound scan
• VQ scan
6. 60-year-old female develops a tender, warm, swollen right calf after hip surgery. She smokes
five cigarettes daily and takes the oral contraceptive pill. She has no chest pain or shortness of
breath. Which blood investigation is more specific for this scenario?
• Complete Blood count
• D-dimer
• Liver function test
• Urea creatinine
• Serum electrolytes
7. During stripping and avulsion of great saphenous vein while performing trendelenbergs
operation for varicosee veins which nerve is at risk of damage
• Sural nerve
• Deep peroneal nerve
• Saphenous nerve
• Superficial peroneal nerve
• Common peroneal nerve
8. 45 years old female presents after RTA in ER with right femur fracture for which surgery was
done, after 48 hours, he developed dyspnea with petechiae and body rash. What is the
probable diagnosis?
• Aspiration pneumonia
• Atelectasis
• Drug reaction
• Fat embolism
• Pulmonary embolism
9. 65 years old female taking NSAIDS for lower backache presented in surgical O.P.D. with
complain of pain in right calf area after taking few steps. She had to take rest for sometimes
to walk again. What is the most likely diagnosis?
• Acute limb ischemia
• Burger’s disease
• Chronic limb ischemia
• Deep vein thrombosis
• Raynaud’s phenomena
10. 60 years old lady presents with h/o sudden severe pain and paresthesia of left leg for last 1
hour. She is a diagnosed case of ischemic heart disease with recent history of myocardial
infarction. On examination her left leg is pale, cold and pulseless. Which of the following
would be the best emergency surgical management for this patient?
• Above knee amputation
• Below knee amputation
• Emergency embolectomy
• Intravenous heparin infusion
• Oral warfarin

Tumors, cysts; ulcers & sinuses, Burns skin lesions, skin grafts & flaps 16
1. A 50 years old diabetic was diagnosed to have peri-anal abscess. Incision and drainage was
done and wound was left open to heal. Wound wash and daily dressing was advised. Such
type of wound healing is classified as:
• Healing by primary intention
• Healing by primary stitching
• Healing by quaternary intention
• Healing by secondary intention
• Healing by tertiary intention
2. 45 years old female presents with burn scar to cosmetic surgeon. Which of the following
statement is most appropriate regarding scars?
• Hypertrophic scar is red, itchy and flat
• Keloid scars can improve spontaneously
• Keloid scar extends beyond its boundaries
• Monofilament suture excites ugly scarring
• Scars of abdominal wounds gains full strength in first 3 months
3. A 50 years old diabetic has presented in OPD with complaint of bleeding from a lesion near
anal verge off/on for last 6 month. On further inquiry he says that lesion stops pouring and
again starts discharging. Discharge is mostly blood but sometimes its frank puss. On
examination 1X1 cm swelling is present at 3 o clock position which pours puss when pressed.
What is the most likely diagnosis?
• Anal fissure
• Hemorrhoids
• Peri-Abscess
• Peri-anal fistula
• Prei-anal hematoma
4. A 55 years old diabetic male presented in emergency room with complaint of a painful
swelling spreading in left lower leg. On examination of leg it has red, hot, tender and non-
fluctuant but spreading over whole leg. On investigations his Haemoglobin is 10 g/dl, WBC
count is 15000/ul, Random blood sugar is 450 mg/dl. What is the best first line treatment?
• Daily aseptic dressing
• Excision of infected area.
• Incision and drainage
• Intramuscular antibiotic
• Intravenous antibiotics
5. A 65 years old male farmer is presented in OPD with a 3 into 3 mm non-healing ulcer on the
outer end of left eye. On examination ulcer has burrowed deep up to bone. Lymph nodes are
not palpable. What is the most likely diagnosis?
• Basal cell carcinoma
• Malignant Melanoma
• Squamous cell carcinoma
• Traumatic ulcer
• Tuberculous ulcer
6. A teenager has a fall from bike on the road and gets a de gloving injury of upper limb with
mud and sand in it. How do you classify this wound?
• Clean
• Clean Contaminated
• Contaminated
• Dirty
• Infected
7. A surgeon gives incision on a prepped and draped patient’s abdomen for hernia. How do you
classify this wound?
• Clean
• Clean Contaminated
• Contaminated
• dirty
• Infected
8. A 21 years old female is presented in emergency room with history of flame burns 1 hour ago.
On examination she has full thickness burn involving thorax and abdomen on front and back.
Her body weight is 70kg. She has given antibiotics and painkillers in the emergency. Now she
is advised intravenous fluid replacement. Calculate the fluid requirement for the first 8 hours?
• 6080
• 8080
• 10080
• 11080
• 12080
9. A six-month-old male baby is presented in emergency room after burns with hot liquid. On
examination his head, face, right upper and lower limbs are found burnt. What formula will
give accurate calculation of burns?
• Rule of three
• Rule of seven
• Rule of eleven
• The Lund and Browder chart
• The parkland formula
10. A 45 years old male is presented in emergency room with 70% burns after explosion of a
boiler in a factory. On examination, he got burn on face, neck & chest. His blood pressure is
100/60 mm of Hg, pulse is 110/min. He is also feeling difficulty in swallowing and breathing.
Which system requires immediate attention after admission to the hospital?
• Gastrointestinal system.
• Musculoskeletal system
• Respiratory system
• Renal System
• Skin and integument
11. An 18 years old female is admitted in surgical ward after history of burns. She had 5X5 cm
wound on left thigh. At the moment, wound is clean, healthy granulation tissue is seen.
Wound culture and sensitivity test shows no growth of bacteria. What is the best
management of the wound?
• Full thickness skin graft
• Myocutaneous flaps
• Partial thickness skin graft
• Skin closure with prolene
• Wound dressing
12. 35 years old lady has presented in OPD for follow up after skin grafting. He has patches of
graft failure. Which of the following bacteria may cause total loss of skin graft?
• β-haemolytic streptococci
• Esch. Coli
• Klebsiella
• Pseudomonas aeruginosa
• Staph. Albus
13. A 30 years old male presents in OPD with complaint of itching and ulceration in the naevus on
his fore-arm. He is also worried about the size of the naevus that has increased during the last
few weeks. What is the most likely diagnosis?
• Basal Cell Carcinoma
• Malignant Melanoma
• Squamous Cell Carcinoma
• Traumatic ulcer
• Tuberculous ulcer
14. 25 years old female presents with 2 into 2 cm sized lump in right breast which is smooth, well
circumscribed and firm in consistency. What is most likely diagnosis
• Carcinoma
• Cyst
• Fibro-adenoma
• Phylloides tumor
• Inflammatory lymph node
15. 45 years old female has presented with soft small painless swelling on her scalp. On
examination this swelling can not be separated from the skin of scalp and there is a small
opening on top it. What is the most likely diagnosis
• Abscess
• Angular dermoid
• Cystic hygroma
• Retention cyst
• Sebaceous cyst
16. 50 years old male presents with swelling in front of neck which move on deglutition but does
not move on protrusion of tongue. On examination this swelling is hard with irregular nodules
and fixed to the underlying structures. What is most likely diagnosis
• Simple multinodular goiter
• Graves’ disease
• Malignant goiter
• Toxic adenoma
• Thyroid cyst
17. Which is the gold standard test for lymphedema diagnosis
• Lymphoflouroscopy
• .MRI
• CT scan
• Lymphoscintigraphy
• ultrasound
18. A 45 years old male who was bed ridden for last 10 years after spinal cord injury develops
bluish , oedematous swollen left lower limb. On duplex ultrasound, there is massive deep vein
thrombosis with sparing of collateral channels. What is the condition anticipated?
• Acute limb ischemia
• Cellulitis
• Phlegmasia alba dolmens
• Phlegmasia cerulea dolens
• Lymphatic obstruction
19. A 65-year old person presents with a large ,hard ,Exophytic growth on the lateral border of the
tongue, with palpable bilateral cervical Lymph Nodes. Biopsy shows Squamous cell Carcinoma.
What is the best treatment option.
• Chemotherapy
• Chemo-Radiotherapy
• Surgery with supra-hyoid Neck Dissection is better choice
• Surgery and Radiation therapy for cervical Nodes
• Treatment with Radiotherapy is equally effective as surgery
20. Years old male presents with swelling on left arm after an intra-muscular injection. Swelling is
red, hot and fluctuant. Which of the following statement is false regarding this swelling ?
• Antibiotics are treatment of choice for an abscess
• The abscess wall is composed of granulation tissue
• Causative organism most likely is E. Coli
• Incision & drainage is the only treatment of Abscess
• Surgical drainage of abscess requires additional antibiotic support
21. Which of the following statement is true regarding scars?
• A hypertrophic scar extends beyond its boundaries
• A keloid scar is red, itchy and flat
• Suture marks can be reduced by using monofilament suture
• Scar gains full strength in first 3 months
• Keloid scars can improve spontaneously

Radio therapy and chemotherapy


1. TNM classification of a malignant tumor was designed as
• A clinical staging
• An histological staging
• Staging carried out at operation
• A staging dependent upon radio scanning and skeletal survey
• A staging based on sentinel node biopsy
2. 45 years old female has presented to breast clinic with a breast lump. She is diagnosed to have
stage 3 infiltrating ductal carcinoma. Best management approach for her is
• Chemotherapy only
• Radiotherapy only
• Surgery only
• Chemotherapy followed by surgery*
• Radiotherapy followed by Surgery
3. 56 years old patient with hepatocellular carcinoma had liver transplant. Post operatively he is
on one of the necessary medicine with IV antibiotics since operation. He is running continues
fever.Most dreadful complication of this necessary medicine after organ transplant is
• Bacterial infection.
• Opportunistic infection*.
• Post-transplant lymphoproliferative disorder.
• Reactivation of latent virus.
• Squamous cell Ca
4. A patient presents with a swelling in front of ear. which clinical feature is Not suggestive of
malignant parotid gland
• Fixity to skin
• Fixity to underlying structures
• Facial nerve involvement
• Purulent discharge from its duct*
• Cervical Lymphadenopathy
5. Regarding Histopathology; which one Is the most common malignancy encountered in oral
cavity
• Adeno-carcinoma of minor salivary glands
• Adenoid cystic carcinoma
• Hodgkin Lymphoma
• Non-Hodgkin Lymphoma
• Squamous cell carcinoma*

PAPER A
1. A Pott’s Puffy Tumour is:
• Infected sebaceous cyst of scalp
• Subperiosteal abscess with osteomyelitis of skull*
• TB brain
• Sebaceous horn
• Lipoma
2. In a burn patient, what is the best indicator for volume replacement:
• Normal blood pressure
• Low blood creatinine level
• Normal pulse rate
• Mental alertness
• Urinary output*
3. Hidradenitis suppurativa is an infection of:
• Apocrine glands*
• Adenoids
• Hair follicles
• Middle ear
• Tonsils
4. Onychogryphosis is:
• An absent nail
• A broken nail
• An infected nail
• An ingrowing nail
• An overgrown curved nail *
5. Lucid interval in a patient after a head injury typically is present in:
• Dural sinus haematoma
• Extra-dural haematoma*
• Intra-cerebral haematoma
• Scalp haematoma
• Sub-dural haematoma
6. A dilated pupil immediately after head injury is due to:
• Anoxia
• Damage to optic nerves
• Damage to iris
• Fear
• Intra cranial hypertension*
7. Parotid gland duct is also called:
• Charcot’s duct
• Santorini’s duct
• Stensen’s duct*
• Wharton’s duct
• Wirsung’s duct
8. Rodent ulcer is another name for:
• Aphtous ulcer
• Basal cell carcinoma*
• Melanoma
• Peptic ulcer
• Squamous cell carcinoma
9. In an hypotensive trauma victim initially following amount of crystalloid bolus is given to see
the response:
• 1/2 liter
• 1 liter
• 2 liter*
• 3 liter
• 4 liter
10. Stage I shock results after loss of following percentage of blood:
• 5%
• 10%
• 15%*
• 20%
• 25%
11. Main stay of treatment in lymphoedema is:
• Chemo-radiation
• Conservative*
• Radiotherapy
• Steroids
• Surgery
12. Cold abscess is caused by:
• Actinomycosis
• Common cold
• Staph aureus
• Strep pyogenes
• Tuberculosis*
13. Student’s elbow is also known is:
• Biceps bursitis
• Golfer’s elbow
• Olecranon bursitis *
• Tennis elbow
• Triceps bursitis
14. Commonest complication after spinal anaesthesia is:
• Backache
• Bleeding
• Headache*
• Meningitis
• Urinary retention
15. Hirshprung’s disease mostly affects:
• Colon
• Duodenum
• Small intestine
• Stomach
• Rectosigmoid*
16. Which is the most common intra-abdominal organ injured with blunt trauma?
• Colon
• Liver
• Small gut
• Spleen*
• Stomach
17. For which of the following organ injuries diagnostic peritoneal lavage (DPL) is least likely to be
helpful:
• Kidney*
• Liver
• Sigmoid colon
• Spleen
• Stomach
18. The initial maneuver to establish a clear airway in a trauma victim is:
• Jaw thrust
• Oropharyngeal airway
• Nasopharyngeal airway
• Removal of foreign body from mouth*
• Endotracheal intubation
19. In burn patients, which one of the following features is indicative of inhalational injury?
• Carbonaceous sputum*
• Perineal burns
• History of outdoor burning environment
• Explosion injury with burns involving lower limbs
• Electric burns involving back
20. A young man with automobile accident is having a tension pneumothorax. Immediate
treatment is:
• Chest inubation
• Intercostal block and strapping
• Needle thoracostomy*
• Endotracheal intubation and pleural aspiration
• Immediate thoracotomy
21. Select the most appropriate initial step in the management of a patient with multiple injuries:
• Splinting of fractures
• Control of external hemorrhage
• Relief of tension pneumothorax
• Blood transfusion
• Maintenance of airway*
22. During the process of wound healing, collagen is laid down by:
• Eosinophils
• Basophils
• Fibroblasts*
• Neutrophils
• Platelets
23. Which of the following studies is most helpful in evaluating a patient’s pulmonary status for a
Thoracic surgical procedure?
• ECG
• Chest X-ray
• History alone
• Lung functions test*
• ETT
24. Regarding wound healing which one of the following statement is true:
• It is characterized by decreased in vascular permeability
• Type I collagen is formed first
• It is retarted by vitamin A excess
• Macrophage plays a central role*
• It is poor in children
25. Malignant hyperthermia is a complication of:
• Atropine
• Pentothal
• Halothane
• Atracurium
• Succinylcholine*
26. Best resuscitation in hemorrhagic shock is done with:
• Ringer’s lactate
• Sodium chloride
• 5% Glucose
• Blood*
• Mannitol
27. During anaesthesia, Succinylcholine injection is used for:
• Induction
• Maintenance
• Reversal
• Relaxation*
• Analgesia
28. ABI stands for:
• Ankle Brachial Index*
• Arm Body Index
• Airway Breathing Investigation
• Ankle Body Index
• Airway Blood Interface
29. Lignocain with Adrenalin is contraindicated in:
• Finger*
• Arm
• Scalp
• Palm
• Sole
30. Thomas splint is used for fracture of:
• Lower limb*
• Cervical vertebra
• Upper limb
• Lumbar vertebra
• Rib
31. External fixator is used for fracture of:
• Atlas vertebra
• Axis vertebra
• Limb bones*
• Rib
• Skull
32. Axillary lymph nodes receive following percentage of lymphatics from breast:
• 15%
• 25%
• 50%
• 75%*
• 100%
33. Parathyroid glands derive their blood supply from:
• Common carotid arteries
• Inferior thyroid arteries*
• Parathyroid arteries
• Superior thyroid arteries
• Tracheal arteries
34. ERCP is investigation of choice in a patient having:
• Pre-hepatic jaundice
• Hepatic jaundice
• Post-hepatic jaundice*
• Hemolytic jaundice
• Septicemic jaundice
35. An abdominal trauma patient should not be sent for CT scan abdomen if he is having:
• Hypertension
• Hypotension*
• Pain abdomen
• Bradycardia
• Fever
36. Intravenous Urography is not done if a patient is having:
• Tachycardia
• Jaundice
• Renal failure*
• Haematuria
• Cough
37. In laparoscopy surgery, gas used to create pneumo-peritoneum is:
• Carbon dioxide*
• Helium
• Hydrogen
• Nitrous oxide
• Oxygen
38. Cuffed endo-tracheal tube is not used in children as it leads to:
• Allergic reaction
• Bleeding
• Cough
• Cyanosis
• Tracheal stenosis*
39. Fluid used for resuscitation in a burn victim is:
• Ringer’s lactate *
• Sodium chloride
• 5% Glucose
• Hypertonic saline
• Blood
40. Massive blood transfusion is defined as when an adult patient receives following units of RCC
in 24 hours:
• 2
• 4
• 6
• 8
• 10*
41. Following airway can be used in a conscious patient:
• Endotracheal tube
• Oropharyngeal airway
• Laryngeal mask airway
• Nasopharyngeal airway*
• Tracheostomy tube
42. Following suture is a monofilament:
• Catgut
• Cotton
• Prolene*
• Silk
• Vicryl
43. Standard temperature for autoclave is Celsius:
• 101
• 111
• 121 *
• 131
• 141
44. Samples for routine histopathology are immediately placed in following fixative, to preserve
morphology:
• 10% Alcohol
• 10% Formaldehyde*
• 10% Glucose
• 10% Mannitol
• 10% Saline
45. SEMS stand for:
• Surgical Emergency Medical Service
• Single Electrode Medical Stent
• Self Expanding Metallic Stent*
• Surgical Emergency Mobile Service
• Self-Expanding Malleable Stent
46. 5-Fluorouracil acts by:
• Interfering with mitosis
• Interfering with DNA synthesis*
• Direct damage to DNA
• Inhibiting Tyrosine kinase
• Damaging the cell membrane
47. Which one of the following is most radiosensitive tumour?
• Neuroblastoma*
• Renal cell carcinoma
• Gastric carcinoma
• Germ cell tumour
• Leiomyosarcoma
48. Earliest source of glycogen during starvation is:
• Blood
• Bone
• Liver*
• Muscle
• Spleen
49. In prolonged starvation, main source of energy is:
• Liver glycogen
• Lipid stores*
• Muscle protein
• Muscle glycogen
• Inhaled oxygen only
50. The total energy requirement of a stable patient with a normal or moderately increased need
is approximately:
• 10–20 kcal/kg per day
• 20–30 kcal/kg per day*
• 30–40 kcal/kg per day
• 40–50 kcal/kg per day
• 50–60 kcal/kg per day
51. Commonest presentation of arterial disease is:
• Acute limb ischemia
• Critical limb ischemia
• Gangrene
• Intermittent claudication*
• Venous thrombosis
52. Commonest cause of peripheral vascular disease is:
• Atherosclerosis*
• Arteritis
• Congenital
• Embolism
• Trauma
53. Commonest type of lymphedema is:
• Primary
• Secondary*
• Tertiary
• Idiopathic
• Congenital
54. Which of the following is a retroperitoneal structure:
• Liver
• Spleen
• Small gut
• Stomach
• Pancreas*
55. Commonest site of neuropathic ulcer is:
• Sole of foot*
• Heel of foot
• Dorsum of foot
• Fore foot
• Tip of toes
56. Lipodermatosclerosis is a feature of:
• Arterial disease
• Cardiac disease
• Hyperlipidemia
• Lymphatic disease
• Venous disease*
57. Most common cause of leg ulcer is:
• Arterial
• Fungal
• Malignant
• Traumatic
• Venous*
58. Slipping sign is a feature of:
• Ganglion
• Lipoma*
• Melanoma
• Varicocele
• Neurofibroma
59. An infected war wound was left open to heal. This type of healing is called:
• Healing by primary intention
• Healing by secondary intention*
• Healing by tertiary intention
• Healing by quaternary intention
• Healing by delayed primary intention
60. Compartment syndromes typically occurs in:
• Closed upper limb injuries
• Open upper limb injuries
• Closed lower limb injuries*
• Open lower limb injuries
• Head injuries
61. After a proven myocardial infarction, to reduce the risk of perioperative re-infarction, elective
surgery should be postponed for:
• 1-2 months
• 2-3 months
• 3-6 months*
• 6-12 months
• 12-24 months
62. Patients are advised not to take solids before anaesthesia to avoid the risk of acid aspiration
syndrome. They should be nil per oral for:
• 2 hours
• 4 hours
• 6 hours*
• 8 hours
o hours
63. 64. PAIR procedure is used for treatment of:
• Bone cyst
• Choledochal cyst
• Hydatid cyst*
• Ovarian cyst
• Sebaceous cyst
64. 65. Bier’s block is usually used for surgery of:
• Chest
• Digit
• Scalp
• Lower limb
• Upper limb*

Paper B
Musculo-skeletal (16)
1. 25 years old male footballer has presented with painful swelling of left knee. MRI report of
left knee reveals ligamentous injury. Most common ligament torn in football players is;
• Anterior cruciate ligament
• Lateral collateral Ligament
• Medial Collateral Ligament
• Patellar ligament
• Posterior Cruciate ligament
2. An old lady had a fall on ground with outstretched hand (Rt). The x rays showed the bone
fracture and the distal fragment collapse into extension, dorsal displacement, radial tilt and
shortening. What is your diagnose
• Colle’s fracture
• Dorsal Barton fracture
• Fractured radial Styloid
• Smith fracture
• Volar Barton fracture
3. During fight, patient’s right arm was extended, abducted and externally rotated. He came to
emergency department while holding his right arm with his left one. Pain was so severe that
he did not give permission for examinations. He is suffering from,
• Fracture proximal humerus head
• Fracture lower 1/3 humerus shaft
• Posterior dislocation Shoulder Joint
• Inferior dislocation Shoulder Joint
• Anterior dislocation Shoulder Joint
4. 22 years old boy presented in OPD with C/O loss of sensation over a patch of right arm lateral
aspect, following anterior dislocation of the shoulder. Which one of the following nerve is
most likely injured.
• Axillary
• Suprascapular
• Subscapular
• Long thoracic nerve
• Musculocutaneous nerve
5. An old lady has presented in OPD with numbness in little and ring finger of right hand. She has
also developed wasting of medial half of right hand. The cause of this problem is:
• Entrapment of anterior interosseous nerve
• Entrapment of posterior interosseous nerve
• Entrapment of Median nerve
• Entrapment of Radial nerve
• Entrapment of ulnar nerve
6. The predominant blood supply to the head of femur in adults is from
• Deep femoral artery
• Lateral Circumflex Femoral Artery
• Medial Circumflex Femoral Artery
• Peroneal artery
• None of above
7. 70 years old female presents with pain in both hips. She is diagnosed to have Avascular
necrosis(AVN). The AVN capitus femoris is:
• Osteonecrosis of the head of the femur due to loss of blood supply
• Osteonecrosis of the neck of the femur
• Osteonecrosis of the acetabulum
• Osteonecrosis of shaft of femur
• None of the above
8. AVN is most common in;

• Basi-cervical fractures
• Inter trochanteric fracture
• In sub-capitus fracture
• Shaft of femur fracture
• Tran- cervical fracture
9. The blood loss in the fracture shaft femur, which may cause class two hemorrhagic shock is;
• Up to 500 ml
• Up to 750 ml
• Up to 1500 ml
• Up to 2000 ml
• Up to 2500 ml
10. The pelvic trauma may lead to serious life-threatening complication due to;
• Bladder injury
• Blood loss
• Gut perforation
• Rectal trauma
• Uterine injury
11. 40 years old male presents in a state of shock after RTA. He has an open book pelvic fracture.
Simple immediate measure in ER to control pelvic hemorrhage is by;
• Bilateral Skin traction
• External fixation
• Open reduction & internal fixation
• Pelvic binder
• Skeletal traction
12. The management of fracture neck of femur in young adult is;
• Hip Spica
• Immediate fixation
• Immobilizing the patient
• Skin traction
• Skeletal traction
13. The fracture neck of femur in elderly is managed by;
• Hemi arthroplasty if the patient is physiologically active & community ambulant
• Hip spika if patient is physically active
• Total hip replacement if the patient is wheel chair bound
• Total hip replacement if the patient pre morbid status is community ambulant, fit for
surgery & have active life.
• Skeletal traction if the patient is physically active and has to live active life.
14. Femoral shaft fracture definitive management in adults is;
• Hip Spica
• Intra medullary nailing
• Long leg cast
• Skin traction
• Skeletal traction
15. Compartment syndrome in leg is managed by;
• Crepe bandage application
• Drainage
• Fasciotomy
• Icing
• Leg elevation
16. A Sportsman is complaining of inability to extend his right wrist after a fall while playing. His
radiograph shows fracture shaft of right humerus. Most likely cause is injury to:
• Anterior interosseous Nerve
• Median Nerve
• Musculocutaneous nerve
• Radial Nerve
• Ulnar nerve

GIT (52)
1. One of the premalignant conditions of esophagus is Barrett’s esophagus which involves lower
end of esophagus. It is a predisposing factor for:
• Adeno-carcinoma
• Adenoid cystic carcinoma
• Basal cell carcinoma
• Squamous cell carcinoma.
• Malignant melanoma.
2. 35 years old male has presented with pain epigastrium on/off for last few months. Which of
the following conditions is most likely associated with person having chronic non-specific
gastritis?
• Chronic mal-absorption
• Crohn disease
• Helicobacter pylori infection
• Pernicious anemia
• Sprue
3. Carcinoma of stomach is now the fifth commonest malignancy. Commonest location of gastric
carcinoma within the stomach is:-
• Antrum
• Body
• Cardia
• Gastro-esophageal junction.
• Fundus
4. Cohn’s disease is the chronic granulomatous condition of bowel. Which one of the following is
a specific feature of this disease?
• Continuous colonic involvement
• Inflammation limited to mucosa
• Pseudo polyp
• Toxic megacolon
• Transmural inflammation
5. 45 years old male has presented with difficulty in swallowing. A Barium swallow shows a
dilated esophageal body with a smooth tapering at the lower esophageal end. What is the
most likely diagnosis?
• Achalasia Cardia
• Diffuse esophageal spasm
• Incompetent lower esophageal sphincter
• Oropharyngeal dysphagia
• Scleroderma
6. Which one of the following colonic conditions has the highest risk of developing colorectal
carcinoma?
• Familial polyposis coli
• Hyperplastic polyp
• Inflammatory polyp
• Juvenile rectal polyp
• Peutz-Jeghers polyp
7. A 72-year-old male has presented with history of progressive dysphagia. He has lost 10 kg
weight in last 4 months. On examination he has supra-clavicular lymph node. What is next
step in his management?
• Barium meal
• Barium swallow
• CT-Chest
• CT-Chest with contrast
• Upper GI-Endoscopy
8. In patients with inflammatory bowel disease which, one of the following extra gastrointestinal
conditions is more commonly associated with ulcerative colitis than with Crohn’s disease?

• Gallstones
• Renal stones
• Uveitis
• Venous thrombosis
• Erythema nodosum
9. An elderly male with history of cardiac bypass presents in emergency with gross abdominal
distention and constipation for past few days and is in great distress. His vitals are not very
promising and his plain X ray abdomen shows large intestinal obstruction. You plan a
laparotomy and find a mass in sigmoid colon. What is the best treatment option in this
patient?
• Abdomino-perenial resection
• Pass a rectal tube and decompress the colon
• Resection of mass with colorectal anastomosis
• Take biopsy and close the patient
• Take biopsy and perform anterior resection
10. A middle aged male with long history of dysphagia to liquids and solids and pain in
retrosternal region. Pain is usually relieved with a loud belch. His X-ray chest shows air fluid
level behind heart and a gas bubble shadow. Most likely diagnosis would be.
• Achalasia cardia
• Barrett's esophagus
• CA esophagus
• Mallory-Weiss tear
• Para esophageal hiatus hernia
11. A 45 years old female with prolonged history of dyspepsia and pain right hypochondrium
presents in emergency with Right hypochondrial pain, for last 2 days. O/E she is febrile with
temperature 102 degree Fahrenheit. she is clinically tender and has got mass RHC. Most
probable diagnosis would be:
• Acute pancreatitis
• Cholangitis
• Chronic Cholecystitis
• Empyema Gall Bladder
• Mucocele Gall Bladder
12. A 70 years old female with long history of diarrhea, bloating, and facial flushing presents in ER
with pain, tenderness and mass right iliac fossa. Considering this history and examination
most suitable diagnosis would be
• Acute appendicitis
• Carcinoid tumor
• Crohn's colitis
• Diverticulitis
• Ulcerative colitis
13. 70 years old male has presented in in OPD with h/o shortness of breath, easy fatigability and
marked weight loss in last few months. On GPE he is markedly anemic and abdominal
examination reveals mass in right iliac fossa. Most likely diagnosis is
• Amebiasis
• Appendicular mass
• Carcinoma caecum
• Diverticulitis
• Intussusception
14. Ranson criteria is used widely in surgical ICU to predict the outcome of patients of acute
pancreatitis. Which of the following is not a parameter to predict the severity of an attack of
acute pancreatitis on admission in Ranson score?
• Age
• Blood glucose
• LDH and AST
• Serum amylase
• White cell count
15. 60 years old male presents in Emergency with pain epigastrium for last 3 days. There is h/o
multiple episodes of vomiting bilious in consistency. He is diagnosed case of Gall stone
disease. O/E his pulse is 124/mins his BP is 90/60 mm/hg & he has distended abdomen with
tender mass in epigastrium. Which is the best investigation to confirm your diagnosis.
• Amylase
• CRP
• CT-abdomen with contrast
• MRCP
• Serum Lipase
16. A patient 6 weeks after an attack of acute pancreatitis develops a swelling in upper abdomen
which is increasing in size. An ultrasound scan reports pancreatic pseudocyst of more than
8cm. Which of the following treatment is most appropriate in this patient?
• Excision of cyst
• Cystogastrostomy
• Pancreatectomy
• Whipple operation
• Watch and see
17. Professor of Physiology told the first year class that the normal quantity of Gastric juice is 2 to
2.5. what is the total amount of GI secretions in 70 kg adult in 24 hours
• 1 liters
• 4 liters
• 8 liters
• 10 liters
• liters
18. 35 years old male presents with h/o of fever and diarrhea 3 weeks after appendectomy, what
is the most likely diagnosis. On digital rectal examination rectum is boggy.
• Acute Gastro-entritis
• Amebic dysentery
• diverticulitis
• Pelvic abscess
• Post-operative wound infection
19. 60 years old male presents in Emergency with pain epigastrium for last 1 day. There is h/o 1
episode of vomiting. He has previously had similar episodes and relates them to fatty meal or
dinner party. O/E his pulse is 110/mins his BP is 100/70 mm/hg. He is tender in Right
hypochondrium. What is most likely diagnosis
• Acute Cholecystitis
• Chronic Cholecystitis
• Hepatitis
• Pancreatitis
• Peptic ulcer
20. 60 years old male presents in Emergency with pain epigastrium for last 3 days. There is h/o
multiple episodes of bilious vomiting. He is diagnosed case of Gall stone disease. O/E his pulse
is 124/mins his BP is 90/60 mm/hg & he has distended abdomen with tender mass in
epigastrium.
• What is most likely diagnosis
• Acute Cholecystitis
• Chronic Cholecystitis
• Gastritis
• Pancreatitis
• Peptic ulcer
21. 25 years old female presents with pain abdomen which started in periumbilical region and
now has shifted to Right iliac fossa on examination she is tender in right lower quadrant.
Ultrasound shows a blind ending loop with a peristalsis of gut in right lower quadrant and no
other significant finding. Most likely diagnosis is
• Acute appendicitis
• Intestinal obstruction
• Pelvic inflammatory disease
• Mesenteric lymphadenitis
• Ureteric colic
22. 35 years old female is booked for appendicectomy. Surgeon has planned an open
appendicectomy with grid iron incision. What is true about McBurney point
• Gall bladder fundus is found at this point
• It indicates the position of tip of appendix
• It presents at joining point of medial one third and lateral two third in line joining umbilicus
and anterior superior iliac spine
• Lanz skin incision is given over this point
• Base of appendix is present at this point.
23. All of the following are true about paralytic ileus except
• Clinically patient present with abdominal distension
• It can occur in post-operative patient
• It can occur with peritonitis
• Prokinetic drugs always relieve this condition
• There can be associated hypokalemia
24. 50 years old male presents in emergency with intestinal obstruction. Which of the following is
not a classical feature of intestinal obstruction?
• Abdominal pain
• Absolute constipation
• Abdominal Distention
• Tenesmes
• vomiting
25. Pringle maneuver is carried to
• Control hemorrhage from liver
• Deliver spleen during laparotomy
• To mobilize duodenum
• To mobilize the right side of colon
• Stop bleeding g from spleen
26. Most common site of metastasis in colorectal CA is
• Brain
• Bone
• Liver
• Lungs
• Muscles
27. A patient presents in trauma room with history of RTA. Initially he was maintaining his vitals
and plain CT scan was carried out which showed grade 2 liver injury. Suddenly he became
hypotensive and developed tachycardia despite of adequate fluid resuscitation. What’s next
appropriate step in management
• Blood transfusion
• MRI
• Endoscopy
• Laparotomy
• Transfusion of FFPs
28. Which of the following investigations of hepatobiliary channels is therapeutic as well as
diagnostic
• Endoscopic retrograde cholangio pancreatogram(ERCP)
• Endoscopic ultrasound
• HIDA Scan
• Magnetic Resonant cholangio pancreatography(MRCP)
• Percutaneous cholangiogram
29. All of the following are the local complication of pancreatitis except
• ARDS
• Pancreatic abscess
• Peripancreatic sterile collection
• Pseudocyst
• Splenic vein thrombosis
30. Valvulae conniventes is feature of
• Duodenum
• Esophagus
• Jejunum
• Stomach
• Transverse colon
31. Regarding colorectal Carcinoma which of the following statement is true
• Fecal occult blood and colonoscopy can be used for screening purpose
• Left sided tumors usually present with anemia
• Most common site is transverse colon
• Most common site of metastasis is lung
• Right sided tumors usually present with obstruction
32. Which of the following is not the intraluminal cause of intestinal obstruction?
• Fecal impaction
• Gallstones
• Bezoars
• Foreign bodies
• Volvulus
33. Most common cause of intestinal obstruction is
• Adhesions
• Carcinoma
• Fecal impaction
• Inflammation
• Obstructed hernia
34. Most common organism found in pyogenic liver abscess is
• E coli
• Staph aureus
• Neisseria meningitis
• Pseudomonas aeruginosa
• H Influenza
35. Which test is not part of routinely advised liver function test
• Alanine transferase
• AST
• Bilirubin
• Creatinine
• PT/APTT

36. In case of liver trauma liver packing should be removed after


• 24hrs
• 48hrs
• 82hrs
• 96hrs
• 102hrs
37. 65 Years old male presents with progressive dysphagia. Biopsy after upper GI endoscopy
reveals carcinoma esophagus. Which one of the following statement is true regarding
carcinoma esophagus.
• Best investigation to diagnose the disease is CT scan thorax
• Dysphagia usually presents as very early feature
• Incidence of adenocarcinoma is decreasing
• Squamous cell carcinoma usually affects upper two third
• Recurrent laryngeal nerve is never involved
38. 50 years old male presents with abdominal pain, absolute constipation and multiple episodes
of vomiting. On abdominal examination there is marked distention and multiple scars of
previous surgery. Next logical step in management is:
• Giving laxative
• Emergency Laparotomy
• Diagnostic laparoscopy
• Prescribing antiemetic
• X-ray erect abdomen
39. A 55 years old man presents with bleeding P/R. His colonoscopy revealed a malignant lesion.
The commonest site for colorectal carcinoma is:
• Ascending colon
• Descending colon
• Rectum
• Sigmoid colon
• Transverse colon
40. 68 years old male presents with multiple episodes of vomiting. His vomitus consists of food
particles which he has taken. He is diagnosed to have Gastric outlet obstruction. The
electrolyte disorder associated with this condition is:
• Hyperkalemia
• Hypernatremia
• Hypochloremic alkalosis
• Hyperchloremic alkalosis
• Respiratory acidosis
41. A 72-year-old male has presented with history of progressive dysphagia. He has lost 10 kg
weight in last 4 months. On examination he has supra-clavicular lymph node. What is next
step in his management?
• Barium meal
• Barium swallow
• Ultrasound abdomen
• CT-Chest and abdomen with contrast
• Upper GI-Endoscopy
42. A pregnant young female presented in surgical OPD with an ultrasound report in her hand
after routine check-up. She was concerned about Gall stones which were incidentally
diagnosed during routine ultrasound performed for her pregnancy. On further inquiry it was
found that she never had any symptoms of gall stone disease What is the correct statement in
this regard:
• CT scan should be performed to confirm diagnosis
• Laparoscopic cholecystectomy must immediately be performed
• Medical treatment should be initiated to dissolve gall stones
• Open cholecystectomy must immediately be performed
• 80 to 90% of gall stones are asymptomatic so she should be kept on follow up
43. A 65 years old male with prolonged history of dyspepsia presents in OPD with jaundice. There
is no history of fever but he says he has lost 12 kg of weight in last 6 months. O/E he has a
non-tender mass in right hypochondrium. Most probable diagnosis would be:
• Acute pancreatitis
• Carcinoma head of Pancreas
• Cholangitis
• Empyema Gall Bladder
• Mucocele Gall Bladder
44. A middle-aged lady presents with severe epigastric pain and multiple episodes of vomiting
after iftar dinner. Detailed history revealed she had arthritis for which she was using NSAIDs
for years. On examination her abdomen is tense and tender.
45. Most important relevant investigation is:
• Complete Blood picture
• Diagnostic peritoneal lavage
• Serum Amylase
• Upper GI endoscopy
• X-ray erect abdomen
46. On 4rth post-operative day, following pelvic surgery due to CA Rectum, a 54-years-old woman
becomes dyspnoeic, her peripheral arterial 02 saturation falls from 94% to 81 % and her
measured Pa02 is 52 on 100% non-rebreathing mask. This is associated with which of the
following conditions?
• Atelectasis
• Congestive cardiac failure
• Myocardial infarct
• Pneumothorax
• Pulmonary thrombo-embolism
47. A 02 years old male child was brought by his mother with c/o left inguinal swelling, which
appears whenever the child strains & some times become painful. On examination a lump was
felt in the left scrotum. What is the worst complication which can occur in this scenario
• Cyst formation
• Infection.
• Irreducibility
• Obstruction
• Strangulation

UROGENITAL (14)
1. A patient has had years of intermittent diarrhea and abdominal pain, but has never consulted
a physician. Eventually he begins to pass fecal material in his urine and he seeks medical
attention. Which one of the following diseases is most likely to cause this complication?
• Celiac disease
• Crohn's disease
• Meckel’s Diverticulosis
• Ulcerative colitis
• Whipple disease
2. 50 years old male with h/o bleeding P/R has presented with h/o pneumaturia. The test with
the highest diagnostic yield for detecting a colo-vesical fistula is:
• Barium enema.
• Colonoscopy.
• Computed tomography (CT).
• Cystoscopy
• Ultrasonography.
3. 74 years old male has left testicular swelling. On examination its hard. Regarding testicular
seminoma.
• It is common testicular tumor of young children
• Inguinal lymph nodes are usually involved by the tumor
• It is the commonest cause of male infertility
• Orchidectomy via inguinal approach is the standard surgical management
• Orchidectomy via scrotal approach is the standard surgical management
4. Micturating cysto-urethrogram is the investigation of choice to diagnosis
• BPH
• Vesicoureteric reflux
• Renal stones
• Vesical stones
• Ureteric stones
5. A 60 year old female is seen in emergency with high temperature, rigors, pain with swelling
right loin. Blood pressure recorded was 160/70mmHg and pulse 100/min. There was extreme
tenderness in right loin. Most probably, this patient is suffering from
• Bladder stones
• Pyelonephritis
• Hydronephrosis
• Renal stone
• Ureteric stone
6. A 52 years old male presented in OPD with progressively enlarging lump right scrotum. On
examination the swelling is tense with negative cough reflex. The testis is not separately
palpable from this swelling and trans-illumination test is positive. What is most likely
diagnosis?
• Hernia
• Hematocele
• Hydrocele
• Testicular torsion
• Testicular tumor
7. 68 years old male was presented in emergency with retention of urine and palpable bladder.
On examination prostate was enlarged. What would be first line of management in the
emergency room?
• Foley’s catheterization
• Suprapubic catheterization
• Suprapubic decompression by needle insertion
• Transurethral resection of prostate
• Trans-vesical prostatectomy
8. 28 years old male presented with right iliac fossa pain radiating from loin. His plain x-ray KUB
revealed big stag horn calculus in the RT renal pelvis. What should be the appropriate
treatment option?
• ESWL
• Nephrolithotomy
• Pyelolithotomy
• Nephrectomy
• Nephrostomy
9. Which investigation is of highest diagnostic value in the management of benign prostatic
hyperplasia.
• Blood complete
• Cytological examination of urine.
• Renal function tests.
• Prostate specific antigen
• Transrectal ultrasound.
10. 75 years old male was presented in emergency with retention of urine and palpable bladder.
On digital rectal examination prostate is enlarged, hard in consisitency and fixed with rectum.
What’s is true regarding prostatic cancer
• It is most common malignancy in men over 65 years
• There is no correlation of family history and prostatic malignancy
• Prostatic specific antigen is always raised in Prostatic malignancy
• Bones involved most frequently with prostatic cancers are ribs
• Hematogenous spread is not present in prostate cancers
11. 58 years old male presents with hematuria & right loin pain. On examination he has got a
palpable mass in right lumber region. He is suspected to have renal cell carcinoma. What is
true regarding renal cell carcinoma?
• Clear cell carcinoma is rare subtype
• CT with contrast is best investigation to stage the disease
• It is more common in females
• Non-surgical options are mainstay of treatment these days
• Radicle nephrectomy is the only surgical option
12. 55 years old female has presented in accident and emergency department after road traffic
accident with severe pain in hypogastrium, abdominal distention and syncope. She does not
have urge to pass urine. X-ray pelvis revealed fracture of pubic bones bilaterally. What is the
most likely diagnosis?
• Extra peritoneal rupture of bladder
• Intra-peritoneal rupture of bladder
• Ovarian rupture
• Rectal injury
• Urethral injury
13. 70 years old male presents with pain less scrotal swelling. There is no history of trauma in
recent past. On examination swelling is hard in consistency and limited to scrotum. True
statement regarding testicular tumors is:
• Hematogenous spread is more common in Seminomas
• Incidence of testicular tumors is decreasing in last few decades
• Inguinal lymph nodes are enlarged in testicular tumors
• Non Germ cell tumors are more common as compared to germ cell tumors
• Teratomas are more common in younger age group as compared to Seminoma
14. 48 years old lady presents with pain in right lumber region. She is diagnosed case of gout.The
type of renal stones associated with it is:
• Enzyme disorders
• Hypercalcemic disorders
• Idiopathic calcium urolithiasis
• Renal tubular syndrome
• Uric acid lithiasis*

HEAD & NECK (11)


1. 15 years old female diagnosed as a case of thyroglossal cyst. A plan for Sistrunk operation was
made. Which one of the following investigations should be done before surgery?
• Bone scan
• CT scan of chest
• CT scan of neck
• MRI neck
• Thyroid scan
2. 50 years old male underwent surgery for parotid tumor and presents in OPD with C/O
erythema and sweating at the scar site whenever he tasted food. What is the most likely
diagnosis?
• Facial nerve weakness
• Frey’s syndrome
• Horner’s syndrome
• Oculomotor nerve weakness
• Trigeminal nerve weakness
3. 34 years old female presented with H/O, fatigue, myalgia, weight loss with increased appetite.
On examination there is a diffuse swelling in front of neck. Investigations revealed no
evidence of infection. What is the most likely diagnosis?
• Grave’s disease
• De-Quervain’s thyroiditis
• Hashimoto’s thyroiditis
• Toxic Adenoma
• Thyroid Malignancy
4. 25 years old female presented with solitary nodule right lobe of the thyroid. Thyroid Function
Tests report was normal. What should be next investigation of choice?
• Anti TSH antibodies
• Anti thyroglobulin antibodies
• C.T. scan
• FNAC
• Thyroid scan
5. 35 years female underwent subtotal thyroidectomy for simple multi nodular goiter and
suction drain was placed in the wound cavity. 03 hours after the surgery the patient became
dyspneic. On examination the dressing was soaked with blood, drain contained 200 ml of
blood. What should be your immediate step in the management?
• Change of dressing
• Open the wound urgently.
• Prop up the patient
• Shift the patient in operation theater
• Wash the drain
6. A 20 year male presented with a small swelling in front of neck in midline, swelling moves
upward on protrusion of tongue. What is diagnosis?
• Branchial cyst
• Cystic hygroma
• Sub-mental lymph node
• Thyroglossal cyst
• Thyroid nodule
7. A patient presenting with a slowly enlarging Painless- lump in front of the ear; which is
movable and not fixed to skin and deeper structures. WQhat is most likely clinical diagnosis
• An enlarged cervical lymph node.
• Lipoma of skin
• Dermoid cyst
• pleomorphic adenoma of Deep parotid lobe
• pleomorphic adenoma of superficial parotid lobe
8. Regarding -Pleomorphic Adenoma ,what is most appropriate treatment:
• Conservative medical therapy
• Enucleation of the gland
• Superficial parotidectomy
• Total parotidectomy
• Radiotherapy
9. Which of the following antibodies is diagnostic of patients with Grave’s disease?
• Anti-thyroglobulin antibodies (Anti Tg)
• Anti-thyroid peroxidase antibodies (Anti TPO)
• Thyroid stimulating antibodies (Anti TSH)
• Anti-cardiolipin antibodies
• Anti-microsomal antibodies
10. 35 years old female presents with swelling in front of neck which moves on deglutition. She
complains of palpitation and heat intolerance. Which is the most reliable test to determine
her thyroid status.
• Free serum T4 level
• Free serum T3 level
• Serum TSH level
• Thyroid scan
• Ultrasound neck
11. Which of the following imaging studies is best for the localization of a parathyroid adenoma?
• Ultrasound neck
• Ct scan head and neck with contrast
• Sestamibi scan
• MRI
• PET scan
12. A 53 year old female, resident of Sawat, with a long standing Multinodular goiter presents in
OPD with heat intolerance, palpitations and tremors for the last 5 months. TSH levels were
undetectable. What is the next course of action:
• Start thyroxine 100 microgram OD
• Start propranolol 40mg TDS
• Make the patient Euthyroid and go for near thyroidectomy
• Start Carbimazole 10mg TDS to cure it
• Start dietary supplementation of iodine

BREAST (10)
1. A 40 years old multiparous, non-lactating women presents with a 2x2 cm tender lump
underneath areola with surrounding inflammation. On examination there is greenish black
discharge from the nipple. Most common cause leading to this condition would be;
• Breast abscess
• Bacterial mastitis
• Duct ectasia.
• Duct papilloma
• Intraductal carcinoma.
2. A 45 years old female was diagnosed to have Ca Breast. Her tumor was 3.5 cm in greatest
dimensions. She had fixed hard nodes on same side of axilla. There was no clinical evidence of
metastasis. In which stage the patient is according to TNM classification.
• Stage I
• Stage IIa
• Stage IIb
• Stage III
• Stage IV
3. In young patients, best treatment option for a right sided Ca breast patient with tumor size of
2.5 cm in greatest dimension with mobile ipsilateral palpable axillary lymph node, without
clinical evidence of distant metastasis is.
• Lumpectomy with axillary sampling
• Lumpectomy with axillary clearance
• Wide local excision with axillary sampling
• Wide local excision with axillary clearance
• Simple mastectomy with axillary clearance
4. Following delivery of her second baby, a statistician develops a tender fluctuating swelling in
her right breast. She had pyrexia of 102 C. She should be treated by
• Antibiotics for gram positive organisms.
• Antibiotics and she should express milk from the infected breast.
• Incision and drainage.
• Mastectomy as underlying malignancy can be a cause.
• Needle aspiration and antibiotics.
5. Most widely used system to stage cancer all around the world is TNM staging system.
Regarding TNM staging of Ca Breast T 1 is less than
• 1 cm
• 2 cm
• 3 cm
• 4 cm
• 5 cm
6. 42 years old office administrator undergoes left wide local excision and axillary clearance for a
4 cm invasive ductal carcinoma. After surgery she has developed numbness of her left arm.
• Intercostobrachial nerve.
• Latissimus dorsi muscle.
• Long thoracic nerve.
• Musculocutaneous nerve.
• Serratus anterior muscle.
7. After undergoing modified radical mastectomy for invasive carcinoma, 50 years old female
aware of winging scapula of the same side. It is due to damage of following structure during
the surgery.
• Inter-costobrachial nerve.
• Latissimus dorsi muscle.
• Long thoracic nerve.
• Musculocutaneous nerve.
• Serratus anterior muscle.
8. A premenopausal female complains of pain and lumps in breast. Examination reveals firm
breast with fine nodularity but no discrete mass. There is cyclical pain and local tenderness.
Most probably, this patient is suffering from
• Fibroadenosis
• Carcinoma breast
• Duct ectasia
• Giant fibroadenoma
• Phyllodes tumor

NERVOUS SYSTEM (08)


1. A 36-year-old gentleman presented to the ER with head injury and when the doctor checked
the patient, he was not able to open his eyes because of the swelling around the eyes (black
eye). What is this called?
• Exophthalmos
• Battles sign
• Black eye
• Panda sign
• Raccoon Sign
2. A 45years old gentleman presented to the OPD with 20 days history of severe headache and
vomiting (multiple episodes) with history of fits and weakness of the right side. What is your
diagnosis?
• Encephalitis
• Hematoma
• Left cerebral Brain Tumor
• Meningitis
• Right cerebral brain tumor
3. A 35 years old male C/O severe backache after heavy weight lifting. On examination he was
unable to straighten his back and weakness of big toe extension with loss of sensation, lateral
aspect of the calf. What is the most likely diagnosis?
• Ankylosing spondylitis
• Osteoarthritis
• Paget’s disease
• Rheumatoid arthritis
• Verteberal disc polapse
4. 75 years old female developed profound foot drop after fibular fracture. Which one of the
following nerve is most likely injured
• Tibial
• Common peroneal
• Superior gluteal
• Sciatic
• Femoral
5. 1 year old child presents with progressive increasing in head size and delayed milestone. On
examination head circumference is twice normal for age and gender. CT scan brain showed
dilated ventricles. What is the most likely diagnosis?
• Arnold chiari malformation
• Dandy walker malformation
• Hydrocephalus
• Meningitis
• Subdural hematoma
6. 2 months old infant presents in ER with skin rashes for last 10 days followed by dizziness and
fits for last 1 day. On neurological examination he showed signs of meningeal irritation , what
is the most likely diagnosis ?
• Cerebral infarction
• Encephalitis
• Intracerebral bleed
• Meningitis
• Metabolic/toxic encephalopathy
7. A 65 years old male hypertensive presented in the OPD for routine checkup. During
examination an expansible and pulsatile mass was palpable at the umbilical region. Which one
the following should be the first investigation of choice?
• X-ray abdomen supine
• Ultrasound abdomen
• C.T. scan abdomen
• M.R.A.
• C.T. angiography
8. 60 years old male was found to 40 % of the internal carotid artery stenosis. What is
appropriate management of choice in this patient?
• Carotid endarterectomy.
• Anti-platelet agents
• Watch and wait
• Oral anticoagulants
• I.V. anticoagulants
9. A young female patient presents with hard swelling in the neck. There is numbness and pain
with the wasting of hand muscles. Most probably, patient is suffering from
• Raynaud’s disease.
• Patient has thromboangtitis obliterans.
• The condition is Acrocyanosis.
• Patient has simply sagging of shoulder Girdle
• She is suffering from cervical rib.
10. A 19-year-old man is brought to emergency department after being stabbed in the left side of
the chest. His initial vital signs after 2 L of fluid are HR 107 beats per minute and blood
pressure 80/50 mm Hg. O/E you are not able to hear heart sounds and breathe sounds are
distant on the left. What should be your next step?
• Chest x-ray while preparing a thoracostomy set
• Emergency Pericardiocentesis
• Intubation
• Large volume resuscitation
• Chest Intubation

ORODENTAL (04)
1. . Regarding Le Fort facial fractures, which is the only one that involves the inferior orbital rim
• Le Fort -I fracture
• Le Fort-II fracture
• Le Fort III fracture
• Compound Mandibular fracture
• Condylar fracture
2. A 65-years old smoker presents with an ulcerating mass on his Tongue. Biopsy establishes that
this is cancer. Which of the following is most common site for cancer of tongue?
• Lateral and ventral surface of the anterior 2/3rd tongue.
• Posterior 1/3 rd. Tongue
• Tip of tongue
• Dorsum of tongue
• Ventral side of tongue
3. The parotid gland is divided into superficial and deep portions by
• Ramus of mandible
• Facial nerve branches
• Internal maxillary artery
• Tragal cartilage
• Deep fascia
4. Sialo lithiasis is most commonly found in
• Submandibular gland
• Accessory salivary gland
• Sublingual gland
• Parotid gland
• Inter-Dental spaces

PEADIATRIC SURGERY (08)


1. A 5 month old baby brought in emergency by Parents. According to them, baby is vomiting
and is in agony for the last one day. On abdominal examination, there is palpable lump in the
right iliac fossa. What is the most likely diagnosis?
• Caelic disease
• Hirschsprung’s disease
• Intussception
• Necrotizing enterocolitis
• Volvulus
2. A 02 years old male child was brought by his mother with c/o left inguinal swelling, which
appears whenever the child strains. On examination a lump was felt in the left scrotum. What
is the best treatment option in this case
• Bassini’s repair
• Darnining’s repair
• Herniotomy
• Lord’s plication.
• Mesh repair.
3. 08 months old male child was brought in surgical OPD with c/o empty right scrotum. O/E
testis was palpable at the level of pubic tubercle which could not be brought at the base of
scrotum. What is the best treatment option for this child?
• Orchidopexy at 01 year of age
• Ochidopexy at 03 years of age
• Orchidopexy at 05 years of age
• Orchidopexy at 07 years of age
• Orchidopexy at 09 years of age
4. A non Muslim uncircumcised child of 08 years of age was brought in emergency with c/o
painful swelling of the glans penis for 02 hours. O/E the glans was swollen and tender with a
retracted foreskin over the glans. What is the most likely diagnosis?
• Balanitis xerotica obliterans
• Gonorrheal UTI
• Paraphimosis
• Phimosis
• Syphilis
5. 1 week old full term male child was diagnosed as a case of umbilical hernia. At what
appropriate age the repair of umbilical hernia should be done if it fails to close spontaneously.
• 02 year
• 04 year
• 06 year
• 08 year
• 10 year
6. 5 years old boy is booked for herniotomy. He is shifted to operation theatre which of the
following problem can easily occur in this age group.
• Hypocalcemia
• Hypokalemia
• Hyponatremia
• Hypothermia
• Hypomagnesemia
7. 1-month old child presents with projectile vomiting. O/E What is the operation for congenital
hypertrophic pyloric stenosis called?
• Hartmann procedure
• Whipple operation
• Heller myotomy
• Ramstad procedure
• Ivor-Lewis surgery

THORAX (08)
1. 56 years old smoker diagnosed as a case of Ca lung by pulmonologist. Anesthetist declare that
patient is fit for lobectomy if he has FEV1 of more than
• 100 cc
• 200 cc
• 500 cc
• 1000 cc
• 1500
2. After fall of heavy building material on patient’s chest, a 35-year-old man has been sent to the
A&E department with severe pain in his chest and marked bruising of the chest wall on the
left. O/E his blood pressure is 80/50 mmHg, the left chest is dull on percussion and there is no
air entry
• Tension pneumothorax
• Cardiac tamponade
• Flail chest
• Massive hemothorax
• Open pneumothorax
3. 50 years old smoker has presented with progressive dysphagia. A gastroenterologist is
performing upper GI endoscopy after which he is complaining of severe pain in thorax and
episodes of vomiting. examination there is subcutaneous emphysema in thorax and neck
region. Most likely cause is
• Acute Myocardial infarction
• Angina pectoris
• Iatrogenic esophageal perforation
• Mucosal injury to esophagus
• Variceal bleeding
4. 40 years old boy presents in E/R with RTA. Which of the following is not immediate life
threatening condition in chest trauma.
• Rupture of diaphragm
• Pericardial tamponade
• Tension pneumothorax
• Massive hemothorax
• Flail chest with contused lungs
5. Normal amount of pleural fluid produced in 24 hours in an adult is:
• 1-2 liters
• 2-3 liters
• 3-4 liters
• 4-5 liters
• 5-6 liters
6. Patient presents with sudden shortness of breath in emergency. X-ray chest reveals right sided
pneumothorax. Which of the following statement is not true regarding pneumothorax?
• Chest tube is placed in triangle of safety for pneumothorax
• Intubation is always required for pneumothorax
• Tension pneumothorax is dire emergency
• There is mediastinal shift in tension pneumothorax
• There is tracheal deviation in tension pneumothorax
Heart and great vessels (04)
1. A patient while driving collided with a high-speed vehicle. He sustained a steering wheel injury
over the sternum. On examination the neck veins are distended, systolic blood pressure is
80mmHg, pulse 130/min, heart sound are muffled. What is the diagnosis?
• Tension pneumothorax
• Cardiac tamponade
• Flail chest
• Open pneumothorax
• Massive haemothorax
2. A motor cyclist, driving at high speed, collides with a trawler. He is hypotensive, with
distended neck veins. The cardiac dullness is increased. Cardiac tamponade is suspected.
• Which statement is untrue regarding cardiac tamponade?
• Cardiac echo will show fluid in the pericardial cavity
• The increased pulse pressures
• Cardioparcetesis with removal of small amount of blood is life saving
• Enlarged cardiac shadow on CXR
• The correct immediate treatment of tamponade is intervention
3. Which of the following is least suited for use in central venous access for feeding?
• Basilic vein
• External jugular
• Femoral
• Jugular
• Subclavian
4. Methods used to investigate the arterial disease include the following except?
• Angiography
• Angioplasty
• Auscultation
• CT angiography
• Duplex scan

Surgery A
1. 1: A mother complained that her 04 weeks old male child vomits after every breast feeding.
The vomiting is non billous and contains milk. On abdominal Inspection visible peristalsis was
seen and mass was palpable in epigastrium. What kind of electrolyte abnormality you expect
in this child
• Hyperchloremia
• Hyperkalemia
• Hypernatremia
• Hypokalemia*
• Hyponatremia
2. 2: 50 years old female is booked for cholecystectomy. Which one of the following technique is
used best in maintaining the airway and preventing complications for a patient undergoing.
• Cricothyrodotomy
• (b.) Endotracheal intubation*
• Guedal airway
• Jaw thrust
• Tracheostomy
3. 3: While doing TRIAGE of mass casualty, immediate care should be given to which of the
following group.
• Dead patients for their proper burial
• Patients with compound limb fractures
• Patients with maxillofacial injury*
• Patients with simple limb fractures
• Patients with trivial injury
4. 4:36 years old car driver had head on collision with truck and had chest injury. In a case of
trauma to the chest, following condition needs definitive surgical repair.
• Cardiac temponade
• b.Hemothorax
• Rupture of the diaphragm*
• Sliding hiatus hernia
• e.Tension Pneumothorax
5. 5: Bier's block is useful technique to provide anaesthesia for upper limb surgery. Which is the
best local anaesthetic agent in use?
• Amethcaine
• b) Bupivacaine*
• Levobupivacaine
• Lidocaine
• Prilocaine
6. 11: A 45 year old female underwent subtotal thyroidectomy for simple multinodular goiter.
Eight hours after the surgery patient had shortness of breath and neck pain. She was
diagnosed to have post operative hematoma due to post operative hemorrhage. Such
hemorrhage is classified as:
• Primary hemorrhage
• Reactionary hemorrhage*
• Secondary hemorrhage
• Tertiary hemorrhage
• Quaternary hemorrhage
7. 13: A 50 year old patient of chronic renal failure underwent hemodialysis. In post dialysis
phase, patient developed weakness, nausea, vomiting and colicky abdominal pain. His ECG
revealed tall tented T waves. The most likely diagnosis is:
• Hypercalcemia
• Hypocalcemia
• C. Hyperklemia*
• Hypokalemia
• Hyponatremia
8. 9 14: A 40 year old female underwent total thyroidectomy for papillary carcinoma of thyroid.
On the first post operative day she developed circum-oral numbness and later on tetany, her
ECG revealed prolonged Q-T interval. What is the most likely diagnosis?
• Hypercalcemia
• Hypocalcemia*
• Hyperkalemia
• Hypokalemia
• Hyponatremia
9. 20 years old male motorist had an accident and brought in the emergency department. On
examination he was bleeding profusely from the wound right thigh. The most appropriate
fluid for resuscitation of this patient should be
• Dextrose / saline
• Dextrose Saline
• 5% dextrose water
• 10% dextrose water
• e Ringers Lactate*
10. Blood transfusion is one of the commonest procedures done in public sector hospitals. Main
hazard of blood transfusion in Pakistan is
• Transfer of bacterial infections
• Transfer of Hepatitis C*
• Transfer of HIV
• Transfer of malarial parasite
• Transfer of syphilis
11. 22: A 45 years old male farmer is presented in OPD with complaint of itching and ulceration in
the naevus on his forearm. He is also worried about the size of the naevus that has increased
during the last few weeks. His excision biopsy has turned out to be a malignancy. What is the
most likely diagnosis?
• Basal Cell Carcinoma
• Malignant Melanoma*
• Squamous Cell Carcinoma
• Kaposi Sarcoma
• Tuberculous ulcer.
12. 23: A 22 years old female was discharged on oral antibiotics and painkillers one week ago.
Now she is presented with fever and diarrhea about 7 episodes per day. She is taking
metronidazole for the last 2 days but frequency of loose stools has increased to 10 episodes
per day. Which medicine will be the drug of choice in this condition?
• Amoxicillin
• Ampicillin
• Ciprofloxacin
• Erythromycin
• Vancomycin*
13. , 24: A 21 years old female is presented in emergency room with history of flame burns 1 hour
ago. On examination she has 50% total body surface burns. Her body weight is 70kg. She was
given antibiotics and painkillers in the emergency. Now she is advised intravenous fluid
replacement. Calculate the fluid requirement for the first 8 hours?
• 3000
• 5000
• 7000*
• 9000
• 14000
14. 25: A six month old male baby is presented in emergency room after burns with hot liquid. On
examination his head, face, right upper and lower limbs are found burnt. What formula will
give accurate calculation of burns?
• Rule of three
• Rule of seven
• Rule of eleven
• The parkland formula
• The Lund and Browder chart*
15. 33: 48 year old male is undergoing open heart surgery. Anesthetist wants to assess the
respiratory tract before noperation. Best investigation to assess lung function is
• Blood gases
• Chest x-ray
• Forced expiratory volume in 1st second*
• Forced vital capacity
• Peak expiratory flow rate
16. 34: Preoperative blood transfusion is indicated in patients with hemoglobin level of:
• <8 gm /dl*
• <9 gm /dl
• <10 gm /dl
• <11 gm/dl
• <12 gm/dl
17. 35. The effective maximum dose of lignocaine is ?
• 2mg/kg body weight*
• 3mg/kg body weight
• 4mg/kg body weight
• 5mg/kg body weight
• 6mg/kg body weight
18. 36. The suggested mechanism of action of Lignocaine at the cellular level is
• It blocks chloride channels
• It blocks peripheral receptors
• It blocks potassium channel*
• It blocks sodium channel
• It decreases the threshold of receptors
19. 37. Which is the most reliable way to ascertain correct placement of an endotracheal tube?
• Detection of breath sounds on auscultation
• Detection of pressure in inflated balloon
• C. Direct visualization
• Movement of chest wall on manual inflation
• e Measurement of end-tidal CO₂ concentration*
20. 38. Commonest cause of bleeding during Cholecystectomy is from injury to Cystic artery which
is a branch of 22
• Caelic artery
• Common hepatic artery
• Left hepatic artery
• Right hepatic artery*
• Superior mesenteric artery
21. 239. 60 years old female who had recent history of myocardial infarction presented to
emergency department with hlo sudden severe pain and parasthesia left leg for last 1 hour.
O/E the left leg was pale, cold and pulseless. Which of the following would be the best
emergency surgical management for this patient?
• Above knee amputation
• Below knee amputation
• Emergency embolectomy*
• Intravenous heparin infusion
• Oral warfarin
22. 46. Five years old boy was brought to the pediatrician for routine checkup at the time of
admission in the school. On examination, pediatrician was not able to palpate the right testis
in the scrotum and right hemiscrotum was not developed. Following is the most important
point regarding diagnosis in this boy.
• Chances of malignancy are 20 times more than normal person.
• His right testis is functionless organ
• Orchidopexy should be attempted before the age of 2 years *
• With time, testis can descend in scrotum.
• e Surgical treatment reduces the risk of malignancy.
23. 247. 22 years old motorcyclist had RTA 30 minutes ago. On arrival in A&E, his pulse is 120/
minute, BP is 90/60 and he is looking very pale. He is suffering from
• Hypovolemic shock*
• Neurogenic shock
• Septic shock
• Traumatic shock
• Vasovagal shock
24. 48. Hypovolumic shock can be classified into 3 categories. In mild hypovolumic shock, there is
only
• Pain
• Decrease urine out put
• Increase breathing rate
• Increase pulse rate*
• Lactic acidosis
25. 250. 24-year old person had a chest injury .Clinical and Radiological Examination revealed 2-3
Ribs Fracture with associated Haemo-Pneumothorax. This is the following best step in
Management.
• Endo-Tracheal Intubation and positive pressure ventilation
• Immediate Thoracotomy
• Placement of chest Tube with under-water seal *
• Simple Analgesics and Strapping of fractured ribs
• Surgical fixation of fractured ribs
26. 2451. 35-year old man sustained a stab wound to the middle of the chest, left side. On arrival
to the ER, he was found hypotensive with distended neck veins .What is the most likely
diagnosis.
• Aortic Tear
• Cardiac Temponade*
• Mediastinal Haematoma
• Pulmonary contusion
• Tracheal Tear
27. 359. 26 years old female has developed very high grade fever with severe muscle pains in
early postoperative period. She is diagnosed to have Malignant Hyperthermia. Best treatment
option for this patient would be
• Atropine
• Dantrolene*
• Morphine
• Neostigmine
• Valium
28. 360..55 years old male has presented with h/o severe excruciating epigastric pain radiating
towards back with multiple episodes of vomiting. His TLC Count is 20,000/ IU and Serum
amylase is 800 VU. You are ordered by consultant in charge to relieve his pain. Which amongst
the following would be best analgesic for this patient.
• Diclofenac infusion
• Intramuscular Diclofenac
• Intra venous Hyosine
• Intra venous Ketoralac
• e Intra venous Tramadol*
29. 61. 25 years old male presented in A & E with Head injury after RTA. On Examination he is
confused, his eyes are open and he can localize pain. His GCS would be
• 8
• 9
• C. 11
• d 13*
• 15
30. 363. In a case of explosive blast injury, the structure most commonly injured by blast wind is.
• Brain
• Extremities
• Heart
• Middle ear*
• Spinal cord
31. 364. As compare to chest X-ray PA view, X-ray lumbosacral spine AP & Lateral view, quantity
of radiation received is more than
• Five times
• Fifteen times
• Ten times
• Twenty five times*
• Twenty times
32. 65. Which of the following is the most appropriate diagnosis?
• Intestinal perforation
• Haemothorax
• (C) Pneumothorax*
• Diaphragmatic Rupture
• Flail chest
33. Perforation of esophagus is a serious condition with high mortality rate. Investigation of
choice for diagnosis and early treatment of esophageal perforation is
• Barium swallow
• Flexible endoscopy*
• c. Gastrograffin swallow
• Rigid endoscopy
• X-Ray chest
34. 56 year's old male patient of hepatocellular carcinoma had liver transplant. Post operatively
he is on one of the necessary medicine with IV antibiotics since operation. He is running
continues fever. Most dreadful complication of this necessary medicine (cyclosporine for
immunosuppressive therapy) after organ transplant is
• Bacterial infection.
• (b) Opportunistic infection. *
• lymphoproleferative disorder.
• Reactivation of latent virus.
• Squamous cell Ca.
35. Different tumor markers are available for diagnosis of certain malignant as well as non
malignant conditions. CA19-9 is one of the tumor marker widely used in surgery. Raised CA19-
9 is helpful in the diagnosis of
• Biliary obstruction
• Cirrhosis
• Esophageal cancer
• Pancreatic cancer
• Pancreatitis
36. 21 years old motorcyclist had RTA and fracture of right tibia with 5 cm laceration over fracture
site. He should be treated by
• Wound debridement and daily antiseptic dressing.
• Wound debridement with antibiotics.
• Wound debridement with antibiotics and internal fixation. *
• Wound debridement with antibiotics and external fixation.
• Wound debridement with antibiotics and POP.
37. 36 years old female had chronic discharging sinus on right radius. Radiologist marked a lesion
in the under lying bane. Following is true for this patient.
• 8. Acute osteomyelitis usually become chronic if treated Improperly.
• Malignant transformation is common.
• Proper antibiotics can cure all similar cases.
• Systemic signs are very common in such cases.
• 8. Tubercle Bacillus could be a causative organism. *
38. A poly trauma patient was brought to emergency department. X-rays showed right iliac bone
fracture with disruption of pubic symphisis. This patient needs
• First priority is to treat shock.
• Require urgent stabilization of fracture.
• Pelvic fraclures are never fatal alone.
• Usually not associated with other injuries.
• Rupture of iliac artery can cause hypovolumic shock. *
39. 30 % burn patient was treated by partial thickness skin grafting in a plastic surgery
department. His donor skin consists of.
• Contains epidermis only.
• Contains epidermis and a part of dermis. *
• Contains epidermis and whole of dermis.
• Donor site heals usually with contracture.
• Transfer to recipient with its blood supply,
40. Hemorrhage can be classified as primary, reactionary and secondary. While discussing the
secondary hemorrhage, it is true that
• Follows the primary hemorrhage.
• Occur after 7 days.
• Occur within 6-8 weeks. *
• Occurs in first 24 hours.
• Causes are restlessness, coughing and vomiting.
41. 30 years old butcher had accidental stab on right wrist. He was brought to trauma center with
fresh bleeding. Treatment of hemorrhage in such cases includes all of the following except,
• Antibiotics. *
• b. Blood transfusion.
• IV fluids.
• Ligation/repair.
• Pressure dressing.
42. 30 years old laborer was brought to neurosurgeon with history of fall and unconsciousness.
Neurosurgeon diagnosed him as a case of extra dural hematoma and operated immediately.
Blood vessel commonly involved in this case is
• Common carotid artery.
• Circle of villus.
• Middle cerebral artery.
• Middle meningeal artery. *
• Vertebral artery.
43. Following delivery of her second baby, a statistician develops a tender fluctuating swelling in
her right breast. She should be treated by
• Antibictics for gram positive organisms.
• Antibiotics and she should express milk from the infected breast. *
• Incision and drainage.
• Mastectomy as underlying malignancy can be a cause.
• Needle aspiration and antibiotics.
44. 12.29 years old lactating mother developed high grade fever and painful swelling on her right
breast since last 6 days. O/E there is a swelling of about 4 x 3 cm behind the nipple; it is
severely tender with overlying skin reddened and warm. Which of the following investigation
will be most helpful in arriving at appropriate diagnosis?
• CT Scan
• Mammogram
• C. MRI
• Ultrasound*
• X-Ray chest
45. 3. 45 year-old female came OPD with FNAC report showing mitotic lesion. On physical
examination there is a lump size is 5 cm. There are two mobile lymph nodes in the axilla. No
evidence of any other lesion in the whole body. According to clinical examination the
appropriate stage of this patient is:
• Stage l
• Stage Il
• Stage III
• Stage IV
• Inflammatory carcinoma
46. 14. 65 years old male presents in OPD with complaint of pain in his right lower limb. On
examination his Blood pressure is 140/90 mm of Hg, right lower limb is pale, posterior tibial
dorsalis pedis arteries are not palpable. ECG shows atrial fibrillation. What is the most
probable diagnosis?
• Cellulitis.
• Lymphoedema.
• Deep venous thrombosis.
• d.) d. Acute limb ischemia. *
• Compartment syndrome.
47. 15. 30 years old male has presented in E/R after a car crash. His right leg got stuck for 2 hours
and he was evacuated from car after cutting the seat. O/E his leg is grossly swollen and painful
with fracture of tibia. After starting resuscitation next step in management is
• a Elevating the affected limb
• b. External fixation of affected limb.
• Open reduction and internal fixation of affected limb.
• Performing Fasciotomy of affected limb.
• Starting DVT prophylaxis.
48. 16.70 years old patient with atrial fibrillation presented in A&E. O/E he has no distal pulses &
severe pain in right lower limb. What is the initial investigation of choice you like to do in such
a patient?
• Angiography.
• CT scan.
• Doppler USG*
• ECG.
• MRI.
49. 17. Years old office administrator undergoes left wide local excision and axillary clearance for
a 4 cm invasive ductal carcinoma. She has recently become engaged and is anxious about her
decision to go through with her marriage. She should be informed that the factor that will
most likely provide the greatest impact on her future life will be
• Hormone receptor status of primary tumor.
• Presence of BRCA! gene.
• The size of primary tumor
• The histological type of carcinoma.
• e. The number of positive axillary nodes.
50. 18.23 years old female was brought to the emergency with 50 % burn while she was cooking.
Her weight was 60 kg. Calculate her hourly fluid requirement for first 8 hours.
• 2.5 liters
• 3 liters.
• 3.5 liters.
• 1 liter.
• 6 liters. *
51. 20 years old clenk who works at a boutique present with a well circumscribed 2 cm mass in
hor right breast. The mass is pain less and has a rubbery consistency and appears to move
freely through the breast tissue. The likeliest discrete borders. It diagnosis is:
• Carcinoma
• Cyst
• Fibroadenoma*
• Cystosarcoma phylloides
• Intramammary lymph node
52. 21.45 years old male has presented in E/R with severe left sided chest pain and shortness of
breath after road traffic accident. O/E his neck veins are engorged and there is tracheal shift
towards right side. Most likely diagnosis is.
• a Cardiac Tamponade
• Hemotherax
• Pneumothorax*
• Rupture of aorta
• Tension Pneumothorax
53. 22.25 years old male has presented in E/R with massively bleeding wound from his right thigh
after road traffic accident. He is anxious, his pulse is 120 beats per minute and B.P is 90/50.
First step in Emergency management of this patient is,
• Blood transfusion
• Crystalloids transfusion
• Giving analgesic
• Giving anxiolytic
• Maintaining airway*
54. 23.45 years old male has presented in E/R with road traffic accident. O/E he has got a wound
on left thigh and multiple wounds on head and neck. There is tenderness in region of back of
neck and thorax. Which one of the following is true for this scenario
• Anaphylactic shock
• Hypovolemic shock*
• C. Obstructive shock
• Neurogenic shock
• Spinal shock
55. 24.45 years old male has presented in E/R with multiple rib fractures on left and shortness of
breath after road traffic accident. After performing tube thoracostomy 500 ml of blood came
in chest tube. Indication of open thoracotomy is,
• 1100 ml of blood in drain
• 800 ml of blood in drain
• 750 ml of blood in drain
• 500 ml of blood in drain
• 200 ml of blood per hour for more than 4 hours*
56. 25.35 years old male presented in E/R with fracture shaft of right femur after road traffic
accident. He is brought in E/R department within 2 hours of incident & O/E his pulse rate is
120/min & B.P is 120/75. Which statement is true considering his condition,
• Fracture shaft of femur is not the cause of this condition
• He cannot be in state of shock as his BP is normal
• He may fall in category of class 1 and class 2 shock *
• His condition is explained by neurogenic shock
• His condition is explained by septic shock
57. 26.55 years old male presented with painful swelling in lower leg. He is known diabetic on
drugs treatment. On Examination erythema and tenderness is present with enlargement of
Inguinal - Lymph nodes. The lesion is seen to spread proximally in superficial tissues. What is
the diagnosis?
• Abscess
• Carbuncle
• Cellulitis. *
• Gangrene
• Impetigo
58. 27. In patient present three weeks after a thoracaic surgery, the wound appears firm, raised
and hyperaemic and has extended beyond its normal limits. The scar is called?
• Hyperpigmented scar
• Hypertrophic Scar*
• Keloid scar
• Normal scar
• Puckered scar
59. 28. Which of the following statement is true regarding wound healing?
• Primary repair of all structures are attempted in untidy wound
• Delayed primary repair involves immediate closure of wound
• A crush wound is suitable for primary healing
• Healing by primary intention results in best scar*
• Healing by secondary intention leaves minimal scar and deformity
60. 29.65 Years old male presented in OPD with h/o 3 into 3cm non-healing wound in sacral
region. Which of the following statement is true regarding chronic wounds?
• Bed bound patients are turned every 6 hours
• The most common chronic leg ulcer is Marjolin's ulcer
• C) A chronic non healing ulcer must be biopsied to exclude malignancy
• Most leg ulcers are managed by surgery*
• Recurrence rate is low in venous ulcers
61. 30. Which of the following statement is true regarding necrotizing soft tissue necrosis?
• Onset is gradual and run a chronic course
• B) Clostridial infection cause toxic shock syndrome
• This is usually monomicrobial Infection
• Treatment is essentially medical
• Commonly superficial tissues are involved*
62. 32. A variety of injuries may occur in road traffic accident. Consistent patterns on injuries are
observed. Which of the following is not a typical combination?
• a Abdominal runover and diaphragmatic injury*
• Head injury & Clavicle injury
• Pelvic fracture and urinary tract injury
• Ribs fracture and liver or splenic injury
• Sternal injury and dorsal spine injury
63. 34. 45 year old male presented A & E department with severe chest pain and shortness
Emergency consultant diagnosed him case tension pneumothorax. which of the following
statement is untrue regarding tension pneumothorax
• Chest intubation best Emergency management
• Distended neck vein are classical feature
• Endotracheal intubation is first step in treatment *
• Needle aspiration performed before chest intubation
• Shift of mediastinum towards opposite side
64. 35. A 35 year male sustains blunt trauma to abdomen when falling from building. There is pain
and tenderness over abdomen. Pulse 120/min and 90/60mmHg. Resuscitation is in progress
according ATLS protocol. A DPL (Diagnostic peritoneal lavage) is planned for internal bleeding.
Which statement does regarding DPL
• Aspirate of more 10ml of blood is deemed as positive
• Presence of >100,000 red cells/uL is deemed as positive
• Presence of blood DPL always requires surgery *
• NG tube is passed to empty stomach before DPL
• Urinary catheter should be passed before performing DPL
65. 36. Which of the following is not a complication of massive blood transfusion?
• Coagulopathy
• Hypercalcemia*
• Hyperkalemia
• Hypokalemia
• Hypothermia
66. 37. An 80 year old female is admitted with Hb of 5.7g/dL. She is transfused 4 units of blood
over 6 hour. Four hours later she is found breathless with bilateral basal crepts. X ray shows
Fluffy haziness in lower lobes. What is the diagnosis?
• DIC
• Fluid overload*
• Hypocalcaemia
• Infection
• Transfusion reaction
67. 38. If hepatitis B virus infection is caused by blood transfusion the ELISA test in recipient is
positive?
• One week later
• One year later
• Six months later
• Two weeks later
• Two years later
68. 39. 18 Years old female is brought to a nutritionist for her nutritional assessment. She was
labelled as mal-nourished because her BMI was below.
• 18
• 20
• C. 23
• 25
• 35
69. Which of the following is least suited for use in central venous access for feeding?
• Basilic
• Cephalicx
• Femoral*
• Jugular
• Subclavian
70. 41. Which of the following statement is true regarding Clostridium tetani?
• Clostridium tetani is an aerobic terminal spore bearing gram positive organism
• It releases exotoxins tetanospasmin affecting motor neurons *
• Toxoid is a vaccine derive from horse serum
• Toxoid vaccine is given after dermal sensitivity check
• Tracheostomy is contraindicated in respiratory cyanotic attacks
71. 42. Which of following is regarding surgical wound infections?
• Local antibiotic preparation effective oral
• films (opsite) be used in infected
• Subcuticular decrease the risk of wound infection*
• Wounds best delayed primary secondary
• Wounds with jagged edges are less likely to be infected compared to incised wounds.
72. 43. Which of the following is useful in reducing the surgical wound infection?
• Compulsory Chlorhexidine antiseptic baths prior to surgery Hypothermia in perioperative
period
• Infrequent of hand between operations
• Hypothermia in post-operative
• Shaving operation site the night before surgery.
• e Short hospital stay*
73. 44. Which of the statement is about risk of infection in the types of wounds?
• Antibiotic prophylaxis in dirty wounds is mandatory
• Prophylactic antibiotics are ineffective in clean-contaminated wounds
• The infection rate clean wounds between 1-2%*
• The dirty wounds have an infection rate of < 10%
• Wound after biliary surgery is classified as contaminated
74. 45. the statement is correct regarding positive findings on histological picture concerning
tuberculosis?
• Askanazi Cells
• C-cells
• Langhan's giant cell*
• Orphan eye cells
• e Reed Sternberg giant cells
75. 46. Which statement is true regarding hyderadenitis suppurativa?
• Groin is common site
• Is chronic cicatrizing suppuration of apocrine glands
• Is tuberculous in nature
• More common in males
• Occurs in elderly
76. 47. Which of the skin lesion is not premalignant/ malignant condition?
• Aktinic Keratosis
• Bowens disease
• Marjolin ulcer
• d.) Salmon patch*
• Radiodermatitis
77. 48.70 Years old smoker presented in OPD with h/o pain in lower limbs. Which of the following
statement is true regarding intermittent claudication?
• Claudication distance is variable on day to day basis
• is commonly felt in the calf*
• Is due to compression of nerve compression in leg x
• It may be present at rest
• Relieved by getting out of bed X
78. 49. Which of the following statement is true regarding critical ischaemia in lower limb?
• Absent foot pulses means critical ischaemia
• Paralysis of leg may suggest acute limb ischaemia
• Rest pain is commonly found in calf*
• Rest pain is relieved by absolute rest
• Hyperaesthesia is feature of pregangrene
79. 50. A 22 year male involved in motorcycle accident presents to casualty department with
fracture femur. The fracture is fixed with inramedullary nail. He is noted to have become
disorientated with several areas of patechial haemorrhages in skin. What is the diagnosis?
• Air embolism
• b) Fat embolism. *
• Foreign body embolism
• Mycotic embolism
• Stroke
80. 51.A 75 year female present with sudden acute pain in her right leg. On examination she has
irregular tachycardia of 120 beats /min and a white, paraesthetic right lower leg. No palpable
pulses below groin on right side but pulses in left limb are normal. What is the diagnosis?
• Air embolism
• Acute aortic thrombosis
• Cholesterol embolism*
• Fat embolism
• Popliteal artery embolism
81. 52. Which of the following statement is true regarding the surgical anatomy of inguinal herria?
• The length of inguinal canal in adults is 10cm
• The inferior epigastric artery is lateral to deep ring
• In infants and adults internal and external rings are superimposed
• The internal ring is U shaped opening in external oblique apponeurosis
• The internal ring is 1.25cm above the mid inguinal point
82. 53. Which of the following statements is true for thyroid gland.
• C-cells are calcium secreting cells in thyroid gland
• Superior thyroid artery is branch of Internal carotid artery*
• T3 is most sensitive part of Thyroid function test
• Thyroid gland is present in front of lower tracheal rings
• Thyrocervical trunk is branch of subclavian artery
83. 54.35 years old male presented with Right sided inguinal hernia. Which of the following
statements is t regarding anatomy of inguinal region
• Conjoint tendon is strongest part of anterior wall of inguinal canal
• Deep inguinal ring is triangular in shape
• External oblique form the floor of inguinal canal
• Femoral artery passes above the roof of inguinal canal*
• First layer incised for opening inguinal canal is internal oblique muscle
84. 55 65 years old female suddenly complaints of chest pain & shortness of breath for last 2. her
first post operative day after hysterectomy for matapaney OE her puisis n and BP is 100/70-
Her ultrasound shows no free fluid in cuide sac Most likely diagnosis is,
• Cardiomyopathy
• Congestive cardiac failure
• Hypovolemic shock
• Pulmonary Embolism*
• 8. Septic shock
85. 56.65 years old male has presented with grade 1 varicose veins which are asymptomatic. He is
not cosmetically concerned Best management of this patient is
• a Injection sclerotherapy
• Laser ablation
• c Polidocanol therapy
• A Flush ligation and multiple stab avulsions
• TED stockings and reassurance*
86. 57 A 65 year male present with pain in his right leg after walking He is hypertensive for last 2
decades on examination his pulse is 55 /min and BP is 16090 Best indicator to assess the
seventy of disease clinically is
• Ankle brachial pressure index*
• b Capillary refill time
• c Perthes test
• d Sensory loss determination
• e Temperature measurement of affected limb
87. 58 Which of the following statement is true regarding amputations?
• a Above knee amputation is better choice than below knee
• b Below knee amputation is end bearing amputation
• Gritti stokes amputation is performed at the ankle*
• Toe amputation usually heais well in diabetes
• Transmetatarsal amputation can relieve rest pain
88. 59 65 years old male has presented with dilated tortutous veins of both lower limbs Best
initial investigation to diagnose the disease is
• a CT angiography of lower limbs
• Duplex imaging of lower limbs
• MRI of lower limbs
• Ultrasound of pelvis
• Venography*
89. 60. A 15 year old male has had appendicectomy for gangrenous appendix a week ago Ha now
presents with diarrhea, fever and lower abdominal pain White cell count is raised What is the
diagnosis?
• (ra) a Surgical wound infection*
• 6 Pelvic abscess
• Necret sing fasciitis
• d Pseudomembranous colts
• e Diverticular abscess
90. 61. A road traffic accident victim presents with a multi-fragment and displaced fracture of
pelvis. which now needs reconstruction. He is clinically stable. Which investigation will give
maximum information and be helpful?
• a CT Scan*
• D. Bone scan
• Fluoroscopy
• MRI
• Ultrasound
91. 62. 25 years old healthy male is booked for hernioplasty. Which of following statement is true
regarding pre-operative preparation of this patient
• a.NPO for 10 hours before
• b. Hepatitis profile is all he requires from labs
• Xray chest is mandatory
• Shaving is best done 2 hours prior to the surgery*
• Hernioplasty is commonly done under G/A
92. 63.35 years old male has developed tachypnea & high grade fever soon after Laparoscopic
cholecystectomy. He is also complaining of severe musyle pains along with stiffness involving
all muscles of body. Labs reflect metabolic and respiratory acaidosis with hyperkalemia. Best
treatment for this patient is.
• Cold sponging
• (b) Dantrolene*
• Paracetamol
• Valium orally
• Valium intravenously
93. 64.40 years old female was operated for perforated duodenal ulcer. She developed dryness of
mouth in immediate post-operative period. She was reassured by duty doctor who told her
that its known side effect of anesthesia drug. Which was the one told by doctor
• a Atropine*
• Enflorane
• Neostigmine
• Pentothal
• Suxamethonium
94. 65.50 years old obese female was operated for Breast cancer. Her surgery underwent for 3
hours. Surgeon is concerned about this patient because she may develop
• Deep venous thrombosis*
• b. Post-operative fever
• c. Post-operative vomiting
• Postoperative hepatitis
• Malignant Hyperthemia
95. Secondary hemorrhage occurs:
• As a result of slip of ligature
• As a result of violent coughing on recovery from anaesthesia
• Due to a blood transfusion line being disconnected
• Within 6 hours of operation
• 7-14 days after operation*
96. Commonest cause of death in surgical patient is:
• Breast cancer
• Colo-rectal carcinoma
• Lung cancer*
• Myocardial infarction
• Shock
97. 35 years old motorcyclist presents in Emergency department with severe chest pain and
difficulty in breathing O/E his pulse is 134/min, B.P is 80/50 mm of HG and dilated neck veins.
Chest examination reveals rib fractures with mediastinal shift and absent breath sounds in
right chest, percussion note is hyper resonant. Next step in management is;
• CT chest with contrast
• Elective thoracotomy
• Emergency room thoracotomy
• Inserting wide bore cannula in second intercostal space*
• Start mechanical ventilation
98. 35 years old man presents in Emergency department after a street fight in state of confusion
with distended neck veins. He is assaulted by an icepick in left chest. O/E his pulse is 124/min
B.P is 80/50 mm of HG. Auscultation of chest reveals bilateral breath sounds in both
hemithorax with muffled heart sound. Most likely diagnosis is
• Aortic Rupture
• Cardiac Tamponade*
• Haemothorax
• Pneumothorax
• Tension pneumothorax
99. A rescue team recovered a patient from a collapsed building after an earthquake. The patient
was suspected to have crush syndrome. Which of the following is best step in emergency
management of crush syndrome:
• a Blood transfusion
• b. Colloids
• C. Normal Saline*
• Plasma transfusion
• e 5% Dextrose water
100. 45 years old male presented in emergency after motorbike accident. He complained of
severe pain left chest. O/E he was vitally stable with tenderness over anterior chest on
auscultation he has decreased breath sounds in left chest with dull percussion note. X-Ray
chest revealed fractures of left 6th and 7th ribs. Most probable diagnosis is:
• Cardiac Tamponade
• Flail chest
• Haemothorax*
• Pneumothorax
• Tension Pneumothorax
101. 28 years old shopkeeper was brought in emergency department by police after he was
beaten by a thief with the stick. The patient C/O severe pain right chest and dyspnea. A chest
tube was inserted after making a diagnosis of right haemothorax which drained 1500 ml of
blood instantly. What would be the next step in the management of the patient?
• Conservative management
• Planned Emergency thoracotomy*
• Ventilator support
• Exploratory laparotomy
• Monitor chest drain output for next 24 hours & transfuse blood
102. A 50 years old diabetic male has presented in OPD with complaint of bleeding from a
lesion near anal verge off/on for last 6 month. On further inquiry he says that lesion stops
pouring and again starts discharging. Discharge is mostly blood but sometimes its frank puss.
On examination swelling is at 3 o clock position and is 1X1 cm in size. What is the most likely
diagnosis?
• Anal fissure
• Hemorrhoids
• Peri-Abscess*
• Peri-anal fistula
• Prei-anal hematoma
103. A 21 years old female is presented in emergency room with history of flame burns 1
hour ago. On examination she has full thickness burn involving thorax and abdomen on front
and back. Her body weight is 70kg. She has given antibiotics and painkillers in the emergency.
Now she is advised intravenous fluid replacement. Calculate the fluid requirement for the first
8 hours?
• 6080
• 8080
• C. 10080*
• 11080
• 12080
104. 25 years old male presents in A & E department after RTA with painful swelling of left
leg. On examination his left leg is swollen and he feels excruciating pain on dorsiflexion. Distal
puises are not palpable. Next step in management is:
• Application of crape bandage
• Fasciotomy*
• C Opioid analgesic
• d NSAIDs
• Repair of vessels
105. 35 years old man is undergoing hemioplasty. In a case of inguinal hernia repair,
prophylactic antibiotic should be given
• At the time of induction of anesthesia*
• At the time of incision
• C. One hours before operation ani
• Two hours before operation
• Three hours before operation
106. 40. A road traffic accident victim came to tertiary care hospital after 03 days of
accident. He had been managed by a quack for open fracture of the tibia and fibula. O.E. there
was a tight splintage applied to the leg. On removal of the splintage, there was foul smelling
pus oozing out from the leg wound with blister formation and black discolouration of skin up
to the level of the knee joint. Crepitus was also detected around the wound. What would be
the best management for this patient?
• Above knee amputation*
• Below knee amputation
• Debridement of the wound and back slab
• Debridement of the wound and POP cast
• IV. antibiotics and aseptic dressing of the wound
107. 65 years old obese lady underwent pelvic surgery for malignancy. On 3rd post
operative day she complained of pain and swelling left calf. Examination revealed tense,
tender red calf. What would be the investigation of choice in this patient to confirm your
diagnosis?
• C.T. scan of the leg
• Contrast MRI
• Duplex scan*
• Photoplathysmography
• e Ultrasound of the leg
108. 44. 60 years old male patient with IHD presented in O.P.D with h/o severe rest pain of
the leg His work up revealed complete aortoillac occlusive disease. What would be the
appropriato treatment option for this patient?
• Aortoiliac bypass
• Apriobifemoral bypass*
• Aortoiliac angioplasty and stent placement
• Axillobifemoral bypass
• Axilloilac bypass
109. 45. 45 years old female has developed very high grade fever with severe muscle pains
in early postoperative period. She is diagnosed to have Malignant Hyperthermia. Best
treatment option for this patient would be
• Atropine
• Dantrolene*
• Morphine
• Neostigmine
• Valium
110. 51. Hypovolumic shock can be classified into 3 categories. In mild hypovolumic shock,
there is only
• Increase pulse rate*
• Lactic acidosis
• Marked decrease in urine out put
• Marked increase in breathing rate
• Pain
111. 53 24-year old person had a chest injury Clinical and Radiological Examination
revealed 2-3 Ribs Fracture with associated Haemo-Pneumothorax. Which is the following best
step in Management.
• Endo-Tracheal Intubation and positive pressure ventilation
• Immediate Thoracotomy
• Placement of chest Tube with under-water seal*
• Simple Analgesics and Strapping of fractured ribs
• Surgical fixation of fractured ribs
112. 35, 56 years old male patient of hepatocellular carcinoma had liver transplant. Post
operatively he is on one of the necessary medicine with IV antibiotics since operation. He is
running continues fever. Most dreadful complication of this necessary medicine after organ
transplant is
• Bacterial infection.
• Opportunistic infection. *
• Post-transplant lymphoproleferative disorder.
• Reactivation of latent virus.
• Squamous cell Ca
113. 56. Prophylactic antibiotics are characterized by
• Should given two hours before operation
• b Should be given at the time of induction*
• Should be given IM
• Should be continued for three days
• Should be given in the presence of infection
114. 57. 40 years old gardner presnts in emergency department with tonic clonic fits. He is
complaining of generalized body aches and pain with difficulty in swallowing. He has a
drossing on his right index finger and on enquiry reveals that he has a small puncture wound
due to thorn prick under the dressing. Most likely cause of this condition is:
• a Enterobactercia
• b. Gram positive aerobic
• o. Gram aerobi negative rods
• Gram anaerobi negative rods
• e Viral infection
115. 58. Most appropriate statement regarding the choice of antibiotics for prophylaxis is
• IV antibiotics at induction of anesthesia is necessary*
• Continuation therapy is contraindicated in unexpected contamination
• Urethral instrumentation for stricture needs no prophylaxis
• Antibiotic prophylaxis is harmful in excessive blood loss
• Prosthetic insertion requires antibiotics on completion of surgery
116. 61. 25 years old male presented in A & E with Head injury after RTA. On Examination
he is confused, his eyes are open and he can localize pain. His GCS has fallen due to
intracranial bleed. Most likely cause of this bleed is
• Anterior cerebral artery
• Basilar artery
• Internal carotid artery
• Middle meningeal artery*
• Posterior cerebral artery
117. 63. A 35 year old motor cyclist involved in road traffic accident. He has open fracture
of his right femur. On arrival he is tachepnic and confused with cold clammy skin. Which of the
following physiological changes is most likely present.
• decreased reabsorption of water from tubules
• decreased serum bicarbonate
• increased pH of blood*
• increased secretion of sodium in Urine
• e increased synthsis of glycogen in liver
118. 64. During massiva transfusion therapy, which of the following electrolyte
abnormalities may lead to disaster if not addressed immediately?
• Hypocalcaemia. *
• Mild hypokalemia.
• Hypochloraemia.
• Hypomagnesaemia
• Hyperglycemia
119. 65. 50 years old female was operated for Pelvic malignancy under General anesthesia.
On 3rd post-operative day she developed sudden painful swelling of her left lower limb. On
examination her limb is swollen with painful movement of limb. Which one amongst the
following would be the best investigation to diagnose the condition?
• a CT abdomen & pelvis
• b. Dopplers ultrasound of limb*
• MRI abdomen & pelvis
• Ultrasound abdomen & pelvis
• X-ray Lumbosacral spine.
120. 65 years old man has presented with abdominal pain and more than a dezen episodes
of vomiting for last 1 day Further inquiry reveals that he has not passed stool or flatus for last
3 days. On examination his abdomen is distended. What kind of electrolyte/metabolic
abnormality do you expect in this man
• Hyperchloremia
• Hyperkalemia
• Hypernatremia
• Hypokalemia*
• Metabolic acidosis
121. Commonest cause of death in surgical patient is:
• Breast cancer
• Colo-rectal carcinoma
• Lung cancer
• Myocardial infarction
• Shock*
122. 25 years old motorcyclist presents in Emergency department with severe chest pain
and difficulty in breathing: fals pulse is 124/min, B.P is 80/50 mm of HG and dilated neck
veins. Chest examination reveals rib fractures with mediastinal shift and absent breath sounds
in hight chest, percussion note is hyper resonant. Next step in management is
• a CT chest with contrast
• b. Elective thoracotomy
• c. Emergency room thoracotomy
• Inserting wide bore cannula in second intercostal spacer*
• Start mechanical ventilation
123. A 30 years old lady was diagnosed to have breast abscess. Incision and drainage was
done and wound was left open to heal. Wound wash and daily dressing was advised. Such
type of wound healing is classified as:
• Healing by primary intention
• Healing by primary stitching
• Healing by quatemary intention
• Healing by secondary intention*
• e Healing by tertiary intention
124. A rescue team rècovered a patient from a collapsed building after an earthquake. The
patient was suspected to have crush syndrome. The problems associated specifically with
crush syndrome are due to:
• Hypokalemia
• Hypematremia
• Hypernatremia
• Metabolic alkalosis
• Myoglobinuria*
125. A 45 years old man underwent Laparotomy for mult-trauma. Massive blood
transfusion was done due to solid organ injury. 6 days taler he presented in a state of shock
and ultrasound revealed free fluid which was confirmed to be a blood on diagnostic peritoneal
lavage. Such hemorrhage is classified as:
• Hemorrhage due to clotting disorder
• Primary hemorrhage
• Reactionary hemorrhage
• Secondary hemorrhage*
• Tertiary hemorrhage
126. A 40 year old female underwent total thyroidectomy for papillary carcinoma of
thyroid. On the first post operative day she developed circum oral numbness and later on
telany, her ECG revealed prolonged Q-T interval. What is the most likely diagnosis
• Hypercalcemia
• Hypocalcemia
• Hyperkalemia
• Hypokalemia*
• Hyponatremi
127. 15. During massive transfusion therapy, which of the following electrolyte
abnormalities will require active intervention?
• a Hypocalcaemia*
• b. Mild hypokalemia,
• C. Hypochloraemia.
• Hypomagnesaemia
• Hyperglycemia
128. 17. A 27 years old female was admitted in female surgical ward with the diagnosis of
appendicular abscess, Exploratory laparotomy was performed perforated gangrenous
appendix was found. What are the chances of wound infection?
• 1-2%
• 02-05%
• C.. 05-10%
• 10-20%
• 20-40%*
129. 18. A 30 years old mais is presented in OPD with complaint of pahless swelling on
scalp. On examination the size of swelling is 1x1 cm which is mobile on the undering statture
but foxed with skin What is the most likely diagres?
• a Abscess
• b Breast lump
• c Dermoid cyst
• Lipoma
• Sebaceous cyst*
130. 20. A 65 years old male farmer is presented in OPD with a recurrent 5X9 mm ulcer on
the left leg. On examination ulcer margins are everted, surrounding skin is inflamed and
indurated. Inguinal lymph nodes are palpable. What is the most likely diagnosis?
• Basal cell carcinoma
• Parotid carcinoma
• Squamous cell carcinoma*
• Traumatic ulcer
• Tuberculous ulcer
131. 21. A 22 years old female was discharged on oral antibiotics and painkillers one week
ago. Now she is presented with fever and bloody dianhea about 7 episodes per day. Which
medicine will be the drug of choice in this condition?
• Amoxicillin
• Ampicillin
• Ciprofloxacin
• Erythromycin*
• Vancomycin
132. 26.35 years old male presents with urinary tract infection. He is advised Ceftriaxone.
Third generation cephalosporin are effective predominantly against
• Anaerobes
• Both gram positive and negative bacteria
• Clostridium tetani
• Gram positive bacteria
• e Gram negative bacteria*
133. 29. 35 years old man is undergoing herioplasty. In a case of inguinal hernia repair,
prophylactic antibiotic should be given
• At the time of induction of anesthesia*
• At the time of incision
• C. One hours before operation
• Two hours before operation
• e Three hours before operation
134. While preparing a 65 years old female patient for elective cholecystectomy under
general anesthesia, it is mandatory to perform following investigation as a part of routine
investigations On history, she is chronic smoker.
• a C.T, scan chest
• b Lung function test
• CMRI chest
• Ultrasound chest
• e X-ray chest*
135. 34, 38 years old man presents with a big swelling in right lower limb. Surgeon has
planned to perform that excision under local anesthesia. The effective maximum dose of
lignocaine is
• 2mg/kg body weight*
• 3mg/kg body weight
• 4mg/kg body weight
• d 5mg/kg body weight
• 6mg/kg body weight
136. 35. The suggested mechanism of action of Lignocaine at the cellular level is
• It blocks sodium channel*
• It blocks potassium channel
• It blocks the peripheral receptors
• It decreases the threshold of receptors
• It blocks chloride channels
• A
137. 36. Which is the most reliable way to ascertain correct placement of an endotracheal
tube?
• Detection of breath sounds on auscultation
• Movement of chest wall on manual inflation
• C. Measurement of end-tidal CO2 concentration*
• Direct visualization
• Detection of pressure in inflated balloon
138. 37. Nerve passing through spermatic cord is:
• Femoral branch of genitofemoral nerve
• Genital branch of Gentofemoral nerve*
• Ilo-inguinal Nerve
• Ulohypogastric Nerve
• Cbturator Nerve
139. 39. 60 years old female who had recent history of myocardial infarction presented to
emergency department with h/o sudden severe pain and parasthesia left leg. O/E the left leg
was pale, cold and pulseless. Which of the following would be the best emergency surgical
management for this patient?
• Above knee amputation
• Below knee amputation
• Emergency embolectomy*
• Intravenous heparin infusion
• Oral warfarin
• с
140. 43.65 years old female underwent total knee replacement. On 3d post operative day
she complained of pain and swelling left call. Examination revealed tense, tender red call.
What would be the investigation of choice in this patient to confirm your diagnosis?
• C.T. scan of the leg
• Contrast MRI
• Duplex scan*
• Photoplathysmography
• Ultrasound of the leg
141. 44. 50 years old gardener has presented with tonic clonic convulsions for last one day.
There is also hilo difficulty in swallowing and spasms of different body muscles. Which the
following is true statement regarding this condition
• Endotoxin plays a major sole producing symptoms*
• b Exploxin affects on motor neurons of anterior hom cells to produce symptoms
• Organism involve grows best in aerobic media
• The condition is due to gram negative spore forming organism
• e Third generation cephalosporin is treatment of choice
142. A 45. 60 years old male patient with HD presented in OPD with No severe rest pain of
the leg. His work up revealed complete aortoillac occlusive disease. What would be the
appropriate treatment option for this patient?
• à Aortolic bypass
• b. Aprobifemoral bypass*
• cAortolac angloptasty and stent placement
• d Axiobifemoral bypas
• e Axilloillac bypass
143. 48. A 37 years old male farmer is presented in OPD with complaint of itching and
ulceration in the naevus on his nose. He is also worried about the size of the naevus that has
increased during the last few weeks. What is the most likely diagnosis?
• Basal Cell Carcinoma
• Malignant Melanoma*
• C. Squamous Cell Carcinoma
• Traumatic ulcer
• Tuberculous ulcer
144. 49. In an endoscopy service commonest method employed all over the world of
sterilizing delicate endoscopes is.
• Autoclaving at 121°
• Ethylene oxide
• Gamma irradiation
• Gluteraldehyde*
• Hot air oven
145. 52. Hypovolumic shock can be classified into 3 categories. In mild hypovolumic shock,
there is only
• Pain
• Lactic acidosis
• Normal to decrease urine out put
• Normal to increase breathing rate
• e Increase pulse rate*
146. 56. 56 years old male patient of hepatocellular carcinoma had liver transplant. Post
operatively he is on one of the necessary medicine with IV antibiotics since operation. He is
running continues fever. Most dreadful complication of this necessary medicine after organ
transplant is
• Bacterial infection
• b Opportunistic infection. *
• Post-transplant lymphoproleferative disorder.
• Reactivation of latent virus.
• Squamous cell Ca.
147. 57. Prophylactic antibiotics are characterized by
• Should given two hours before operation
• Should be given the time of induction*
• Should be given IM
• for three days
• Should be given the presence infection
148. 58. Professor of Surgery told the medical students about skin infections that it is
unlikely in cellulitis
• To get ß-haemolytic Streptococcus from infection site.
• It is an spreading infection of skin and subcutaneous tissue
• Diabetics are least unaffected*
• To get positive blood cultures.
• To detect systemic signs.
149. 59. Most appropriate statement regarding the choice of antibiotics for prophylaxis is
• IV antibiotics at induction of anesthesia is necessary*
• Continuation therapy is contraindicated in unexpected contamination
• Urethral instrumentation for stricture needs no prophylaxis
• Antibiotic prophylaxis is harmful in excessive blood loss
• Prosthetic insertion requires antibiotics on completion of surgery
150. 65.A 35 year old motor cyclist involved in road traffic accident. He has open fracture of
his right femur. On arrival he is tachepnic and confused with cold clammy skin. Which of the
following physiological changes is most likely present.
• a decreased reabsorption of water from tubules
• b. decreased serum bicarbonate*
• increased pH of blood
• increased secretion of sodium in urine
• e increased synthsis of glycogen in liver
151. 29. The best treatment for a localized basal cell carcinoma of the inner canthus is:
• External radiation*
• Radium needle implantation
• Surgery with chemotherapy
• Chemotherapy
• Combination of radiotherapy and chemotherapy
152. 30. A patient presents with fever, swelling, pain in lower leg. He is a known diabetic
on drugs treatment Exam show redness, erythema and tenderness around ankle. The Inguinal
LNs are palpable and also tender. T seen to spread proximally. What is the diagnosis?
• Cellulitis. *
• Gangrene
• C. Carbuncle
• Impetigo
• Abscess
153. 31. Which of the following bacteria may cause total loss of skin graft?
• Klebsiella
• B-haemolytic streptococci*
• Esch, Coli
• Pseudomonas aeruginosa
• Staph. Albus
154. While preparing a 65 years old female patient for elective cholecystectomy under
general anaesthes to perform following investigation as a part of routine investigations. On
history, she is a chronic smo
• X-ray chest*
• C.T. scan chest
• Lung function test
• MRI.chest
• Ultrasound chest
155. 33, 48 year old male is undergoing open heart surgery. Anesthetist wants to assess the
respiratory in Best investigation to assess lung function is
• Peak expiratory flow rate
• Forced expiratory volume in 1 second*
• Forced vital capacity
• Blood gases
• Chest x-ray
156. 4. Preoperative blood transfusion is indicated in patients with hemoglobin level of:
• <8 gm /dl*
• <9 gm /dl
• <10 gm /dl
• <11 gm /dl
• <12 gm/dl

Surgery B
1. 9 months old infant is brought to OPD with undescended testes. True statement regarding
undescended testes is
• about 10% of boys are born with this condition
• Condition is more common on left side
• C. Condition is 40 percent bilateral
• 6 to 12 months is the best time for orchidopexy*
• Risk of malignancy is less in these cases
2. 70 years old male was presented in emergency with retention of urine and palpable bladder
On examination postate was enlarged. Urologist counseled him for Trans urethral resection of
prostate(TURP) Risk of re-operation after TURP is
• 2% after 8-10 years
• 8% after 8-10 years
• c 15 % after 8-10 years*
• 20% after 8-10 years
• 25% after 8-10 years
3. 11. An 80 years old lady has presented after fall from bed & is diagnosed to have fracture neck
of femur. Most likely complication related to this fracture is:
• Avascular necrosis*
• Compartment syndrome
• Delayed union
• Massive Bleeding
• Nerve entrapment syndrome
4. During fight, patient's right arm was extended, abducted and externally rotated. He came to
emergency department while holding his right arm with his left one. Pain was so severe that
he did not give permission for examinations He is a suffering from,
• Fracture proximal humerus head
• Fracture lower 1/3 humerus shaft
• Posterior dislocation Shoulder Joint
• inferior dislocation Shoulder Joint
• Anterior dislocation Shoulder Joint*
5. 52 years old smoker has presented with progressive dysphagia. A gastroenterologist is
performing upper Gl endoscopy after which he is complaining of severe pain in thorax and
episodes of vomiting examination there is subcutaneous emphysema. Most likely cause is
• Acute Myocardial infarction
• Angina pectoris
• latrogenic esophageal perforation *
• Mucosal injury to esophagus
• Variceal bleeding
6. 35 years old female presents with difficulty in swallowing. Her barium swallow revealed bird
beak appearance. Most likely diagnosis is:
• Achalasia Cardia*
• Barrett's Esophagus
• C Carcinoma esophagus
• Corrosive stricture
• Nut cracker esophagus
7. 15 Best diagnostic tool for esophageal motility disorders is:
• a Barium swallow
• CT-scan of thorax
• c Endoscopic ultrasound
• d Oesophageal manometry*
• Plain X-ray chest
8. 16 55 years old male presents with severe weight loss after gastric outlet obstruction. He has
got long standing history of acid peptic disease. Upper Gl endoscopy reveals a malignant ulcer.
Commonest location of gastric carcinoma within the stomach is:
• Antrum*
• Body
• Cardia
• Gastro-oesophageal junction.
• Fundus
9. 65 yrs. old man presents with vomiting, abdominal pain and inability to pass flatus and stool
of intestinal obstruction. You make a diagnosis of intestinal obstruction. The most common
cause is:
• Adhesions*
• Carcinoma of the colon
• intestinal tuberculosis
• gestructed hernia
• Worms
10. year old male complains of a painful nodule near his anus for the past 3 years it occasionally is
and on but after a few days it resolves with discharge of pus. Your examination reveals a nod
3omo verge with a palpable track. Your diagnosis is:
• Carcinoma of the anus
• Chronic anal fissure
• External hemorrhoids
• Fistula in ano*
• Sebaceous cyst
11. Previously healthy 30 years old male presents in pain right iliac fossa for last 1 week and
multiple episodes of vomiting for last 2 days. On GPE she is toxic with pulse rate 104/min,
temp 101 F. Abdominal examination reveals a fim, mobile tender mass in right iliac fossa
extending towards central abdomen. Most likely diagnosis is
• a Acute appendicitis
• b. Appendicular mass*
• c Colonic malignancy
• d Crohns disease
• Ulcerative colitis
12. A middle aged lady presents with severe epigastric pain and multiple episodes of vomiting
after iftar dinner. Detailed history revealed she had arthritis for which she was using NSAIDs
for years important relevant investigation is:
• Complete Blood picture
• Diagnostic peritoneal lavage
• Serum Amylase
• Upper Gl endoscopy *
• X-ray erect abdomen
13. 58 year old male presents with increasing shortness of breath on exertion and weight loss. On
further questioning admits to passing blood in his stools. Your examination reveals an anemic
patient with no other positive findings colonoscopy tums up a growth in the ascending colon
which is limited to wall of onion. What would be the appropriate surgical procedure for this
patient?
• Left hemicolectomy*
• Loop colostomy
• Right hemicolectomy
• Total colectomy
• d Total proctocolectomy
14. On 4rth post-operative day, following pelvic surgery due to CA Rectum, a 54-years-old woman
becomes dispnoeic, her peripheral arterial 02 saturation falls from 94% to 81 % and her
measured Pa02 is 52 on 100% non rebreathing mask. This is associated with which of the
following conditions?
• a Atelactasis
• b. Congestive cardiac failure
• Myocardial infarct
• Pneumothorax
• Pulmonary thrombo-embolism*
15. 25-year-old LADY presented with bleeding P/R for last 2 weeks. She is giving h/o painful
defecation and the pain persists for 1 to hours after defecation. There is no h/o weight loss,
mucus discharge. What is the most likely diagnosis:
• a Anal fissure. *
• b. Carcinoma anal canal.
• cCarcinoma rectum.
• d Hemorrhoids
• e Perianal fistula
16. if year old presents with weight loss, tremors and heat intolerance. On examination the has
diffuse swelling in t of neck. You suspect hyperthyroidism. What pertinent investigation is
most appropriate?
• Complete Blood Count
• FNA
• Thyroid hormonal profile *
• Thyroid Scan
• d US neck
17. 34 years old female presented with H/O, increased sweating, fatigue, myalgia, weight loss
with increased appetite. On examination there is swelling in front of neck with multiple
nodules. What is the best treatment option?
• a Antithyroid drugs for 6 months
• b. Antithyroid drugs for 3 months followed by surgery*
• Radio-iodine for 6 months*
• Reassure patient and send home
• Thyroxine for 6 months
18. 3 years old female presented with solitary nodule right lobe of the thyroid. Thyroid Function
Tests report was normal. What should be next step in management?
• a Anti TSH antibodies
• b Anti Thyroglobulin antibodies
• C.T. scan
• d FNAC*
• Thyroid scan
19. You are asked to assess a 34 year old mother of two for a lump in her left breast. You think it
likely that the lump may be malignant. What is the next diagnostic step to confirm diagnosis?
• a Ultrasound of the breast
• Mammography*
• Ultrasound of the breast and FNA
• FNAC
• Exosion Blopsy
20. 25 years old female notices a firm 2x2 cm lump in one breast which slips while palpation and
is sometime tender There are no clinically palpable lymph nodes in axill. at would be next step
in management?
• Excision biopsy of the lump
• a Mammogram
• Quardrantectomy
• Ultrasound of breast *
• Wide local excision
21. A 65 years old male presents with hoarseness of voice for last 3 weeks. On examination there
is small swelling in ont of neck which moves on deglutition. His indirect laryngoscopy is done
which shows adducted vocal cord on are side. what is the next step in management of this
patient?
• CT-Neck
• FNAC of neck swelling *
• MRI-Neck
• Surgery of affected vocal cord
• Thyroid Scan
22. 55 years old diabetic presents in OPD with discharging sinus on his left leg for last 1 year. He
has taken multiple courses of different antibiotic during last 12 months. His radiograph shows
peri-osteal reaction. What is the commonest causative agent in this condition?
• E coli
• Klebsialla
• Mycobacterium
• Staphylococcus Arius *
• Streptococcus Pyogenes
23. 45 years old female notices a firm 1 cm painless lump in one breast which is firm in
consistency. It is neither attached to the skin nor to the underlying structures What would be
the next logical step in management.
• Complete triple assessment*
• CT scan of Breast
• MRI of breast
• Excision biopsy
• Modified Radical Mastectomy
24. 6436 year old gentleman presented to the ER with head injury and when the doctor checked
the patient, he was not the to open his eyes because of the swelling around the eyes (black
eye). What is this called?
• a Battles sign
• b. Black eye
• c. Exophthalmos
• d. Panda sign*
• e Red Eyes
25. 445years old gentleman presented to the OPD with 2 days history of severe headache and
vomiting (multiple episodes) with history of fits after RTA. Best diagnostic tool is?
• a CT brain*
• b. MRI Brain
• X-ray head
• d. Ultrasound abdomen
• € Ultrasound brain
26. A35 years old male C/O severe backache after heavy weight lifting On examination he was
unable to straighten his back and weakness of big toe extension with loss of sensation, lateral
aspect of the calf hat is the most likely diagnosis?
• Pagets disease
• Ankylosing spondylitis
• Verteberal disc prolapse*
• Rheumatoid arthritis
• Osteoarthritis
27. 25 years old footballer developed profound foot drop after an injury while playing foot
ballfibular fracture Which of the following nerve is most likely injured
• Tibial
• Common peroneal *
• Superior gluteal
• Sciatic
• e Femoral
28. A 55 years old male hypertensive presented in the OPD for routine checkup. During
examination an expansile pusable mass was palpable at the umbilical region. Which one the
following should be the first investigation od choice
• X-ray abdomen supine
• Ultrasound abdomen*
• C.C.T. scan abdomen
• d. M.R.A.
• C.T. angiography
29. 60 years old male was found to 80 % of the internal carotid artery stenosis. What is
appropriate management of choice in this patient?
• Carotid endartrectomy.
• Anti-platelet agents*
• Watch and wait
• Oral anticoagulants
• IV. Anticoagulants
30. Which investigation is of highest diagnostic value in the management of benign prostatic
hyperplasia.
• Blood complete
• Cytological examination of urine.
• Renal function tests.
• Prostate specific antigen
• e Transrectal ultrasound. *
31. 85 years old lady presents with dysuria and increased frequency. She is suspected to have UTI
Bacteria most likely involved is?
• Clostridium
• E.coli*
• Streptococcus
• Staphylococcus
• Bacteroides
32. Following delivery of her second baby, a statistician develops a tender fluctuating 8 into 8 cm
swelling in her right pest She had pyrexia of 102 C. She should be treated by
• Antibiotics for gram positive organisms.
• Antibiotics and she should express milk from the infected breast
• incision and drainage. *
• Mastectomy as underlying malignancy can be a cause.
• Needle aspiration and antibiotics.
33. 35 years old male presents with h/o of fever and diarrhea 3 weeks after appendectomy, on
digital rectal amination rectum is boggy. What is the most likely diagnosis.
• Acute Gastro-entritis
• b. Amebic dysentery
• c diverticulitis
• d Pelvic abscess *
• e Post-operative wound infection
34. 65 years old lady presents with winging scapula of the same side after modified radical
mastectomy for invasive carcinoma breast. It is due to damage of following structure during
the surgery
• Intercostobrachial nerve.
• Latissimus dorsi muscle.
• Long thoracic nerve. *
• Musculocutaneous nerve.
• Serratus anterior muscle.
35. A Premenopausal female complains of pain and lumps in breast Examination reveals firm
breast with fine modularity but no discrete mass. There is cyclical pain and local tenderness.
Most probably, this patient is suffering from
• Fibroadenosis*
• Carcinoma breast
• Duct ectazia
• Giant fibroadenoma
• Phyllodes tumour
36. 55 years old smoker with 70 kg weight diagnosed as a case of Ca lung by pulmonologist.
Anesthetist declare that e. patent is fit for lobectomy if he has FEV1 of more than
• 100 cc
• 200 cc
• 500 cc
• 1000 cc
• 1500cc*
37. Professor of Physiology told the first year class that the normal quantity of Gastric juice is 2 to
2.5. what is the total e 1500 cc amount of GI secretions in 24 hour in 70 kg person.
• a 1 liters
• 4 liters
• c. 8 liters*
• d. 12 liters
• 16 liters
38. 80 years old lady presents with right sided lumber pain. Ultrasound KUB revals multiple stones
in right kidney. On inquiry she revealed that she has pain in right knee joint and some kind of
arthritis for which she is taking NSAIDs. Stones most commonly related with this scenario are:
• Enzyme disorders
• Hypercalcemic disorders
• Idiopathic calcium urolithiasis
• Oxalate stones
• Uric acid lithiasis*
39. 50 years old chronic smoker has presented with painful swelling in left inguinal region for last
2 day. Detailed inquiry revealed that the swelling was initially reducible but for last 2 days it
has stuck in inguinal region and is not decreasing in size. On examination the swelling is red
hot and tender. What is the most likely diagnosis:
• Abscess in inguinal region
• Obstructed left inguinal hernia
• Irreducible inguinal hernia
• Strangulated left inguinal hernia*
• Reducible left inguinal hernia
40. 55 years old male has presented in Emergency with severe pain epigastrium radiating towards
back. He is diagnosed to have acute pancreatitis. True statement about Ransons criteria is:
• a Ransons criteria has diagnostic value
• 5. Ransons criteria has prognostic value *
• Serum amylase is most significant component of Ransons criteria
• Serum Lipase is most significant component of Ransons criteria
• e White cell count highest significance in Ransons
MCQs Paper-B Surgery
1. A young man has presented with pain in right lower quadrant. Pain initially started in peri-
umbilical region. The reason for localization of pain in right lower quadrant is:
• Appendicular colic
• Distension of appendix
• Intestinal colic
• Peritoneal irritation
• Referred pain
2. A 45 years patient was diagnosed to be having Hepatitis C. He is having distended abdomen
with prominent veins around umbilicus. Enlarged peri-umbilical veins in this patient indicate:
• Bile duct obstruction
• Intestinal obstruction
• Inferior vena caval obstruction
• Portal venous obstruction
• Superior vena caval obstruction
3. A cricketer was struck on his head with a ball. He became unconscious for a while, but
resumed the play afterwards. In the lunch break he was found to be deeply comatosed in the
dressing room. Lucid interval in the head injury typically is present in:
• Dural sinus haematoma
• Extra-dural haematoma
• Intra-cerebral haematoma
• Scalp haematoma
• Sub-dural haematoma
4. A motor cyclist was brought to neuro- surgical emergency after an accident. The duty doctor
found weakness of right side of body with dilated left pupil. A dilated pupil immediately after
head injury is due to:
• Anoxia
• Damage to optic nerves
• Damage to iris
• Fear
• Intra cranial hypertension
5. A young lady was having prominent eyes with weight loss and palpitations. She was
diagnosed to be having Grave’s disease, which is:
• Primary thyrotoxicosis
• Secondary thyrotoxicosis
• Tertiary thyrotoxicosis
• Thyrotoxicosis fictitia
• Treated thyrotoxicosis
6. A 55 years old male is having a lump over right angle of mandible for the last 2
years.Commonest surgical disease of parotid is:
• Calculus
• Carcinoma
• lymphadenitis
• Parotitis
• Pleomorphic adenoma
7. A diagnosed patient of pheochromocytoma is scheduled for adrenelectomy. Preoperative
preparation in this patient is started with:
• Alpha agonist
• Alpha antagonist
• Beta agonist
• Beta antagonist
• Diuretics
8. A wrestler was brought to A & E department as he was unable to move left shoulder joint.
Commonest dislocation of shoulder joint is:
• Anterior
• Inferior
• Lateral
• Posterior
• Superior
9. A medical student has noticed a 3x3 cm nodule in thyroid isthmus. FNAC shows adenomatous
goiter She is undergoing surgery. Minimum operation for solitary thyroid nodule is:
• Excision of nodule
• Subtotal thyroidectomy
• Total lobectomy
• Total lobectomy plus isthmectomy
• Total thyroidectomy
10. Rotor’s lymph nodes are important in relation with:
• Carcinoma breast
• Carcinoma gallbladder
• Carcinoma rectum
• Carcinoma stomach
• Carcinoma thyroid
11. Which one of the following is the malignant condition of breast?
• Cystosarcoma Phylloides
• Duct ectasia
• Fibroadenoma
• Mondor’s disease
• Paget’s Disease
12. A cyclist was run over by a tractor on his lower trunk. The commonest injury associated with
fractured pelvis is:
• Colonic injury
• Ruptured urethra
• Sciatic nerve injury
• Small gut tear
• Torn ureter
13. An abscess is a collection of pus. Brodie’s abscess involves:
• Bone
• brain
• Breast
• Liver
• Lung
14. The commonest bone tumor is:
• Chondroma
• Fibroma
• Metastatis
• Osteoma
• Sarcoma
15. A middle aged man reports to orthopedic outdoor with a swelling over posterior aspect of
right elbow joint for the last 3 years. He was diagnosed to be having Olecranon bursitis, which
is also known is:
• Clergyman’s elbow
• Cricket elbow
• Golfer’s elbow
• Student’s elbow
• Tennis elbow
16. A taxi driver has been brought to A & E department after sustaining a head injury. He was
admitted for neuro-observation. Feature which would cause concern during observation of
head injury patient is:
• Bradycardia
• Hypotension
• Photophobia
• Pyrexia
• Tachycardia
17. Frey’s syndrome follows:
• Cataract surgery
• Mandibulectomy
• Parotidectomy
• SMR for DNS surgery
• Thyroidectomy
18. A 52 year old lady has reported with a huge multinodular goiter with respiratory distress.
Tracheal obstruction in her is tested clinically by:
• Berry’s test
• Kocher’s test
• Joffroy’s test
• Mobius test
• Nafziger test
19. A 44 year old lady, mother of three children, reports to breast clinic with six months
• history of blood stained discharge from her left nipple. Clinical examination is
• unremarkable. She is most likely to be having:
• Duct adenoma
• Duct carcinoma
• Duct ectasia
• Duct fibroma
• Duct papilloma
20. A middle aged smoker has undergone upper GI endoscopy for chronic dyspepsia. Endoscopist
has taken multiple biopsies from a growth in the antrum. Gastric carcinoma is associated with
blood group:
• A
• AB
• B
• O
• Both A & O
21. A middle aged female is having a reducible swelling over left groin for the past four
• months. The most common hernia in females is:
• Direct inguinal hernia
• Femoral hernia
• Indirect inguinal hernia
• Obturator hernia
• Umbilical hernia
22. Duodenal diverticulum is commonest in:
• First & second parts
• Second& third parts
• Third& fourth parts
• First& fourth parts
• First& third parts
23. Pain abdomen is one of the commonest reasons for a surgical referral. Non-surgical causes of
acute abdominal pain may include which of the following?
• Diabetic ketoacidosis
• Hypoglycemia
• Hyperthyroidism
• Hypothyroidism
• Paget’s disease
24. Mallory-Weiss syndrome is:
• Congenital cardiac anomalies
• Congenital diaphragmatic hernia
• Esophageal cardiac tear
• Prominent umbilical veins
• Type of nephrotic syndrome
25. Ludwig’s angina involves:
• Face
• Heart
• Nose
• Palate
• Tonsil
26. Pseudo-cyst pancreas is one of the complications of acute pancreatitis. It should be operated
after an interval of:
• 6 hours
• 6 days
• 6 weeks
• 6 months
• 6 years
27. Following is a translucent swelling in the scrotum:
• Hydrocele
• Inguinal hernia in a baby
• Testicular tumour
• Testicular torsion
• Varicocele
28. Following is NOT a complication of acute appendicitis
• Appendicular abscess
• Appendicular mass
• Adenocarcinoma appendix
• Pelvic abscess
• Pylephlebitis
29. A 13 year old boy is diagnosed to be having branchial cyst on right side of neck. Branchial cleft
remnants most often present with which of the following clinical problem?
• Airway obstruction
• Hemorrhage
• Infection
• Malignant degeneration
• Pain
30. A 2 months old baby boy is going to be operated for congenital pyloric stenosis. He would be
presenting with which of the following biochemical abnormality:
• Acidosis
• Respiratory Alkalosis
• Hypernatremia
• Hyporchloraemic alkylosis
• Paradoxical alkalosis
31. The most common complication in an elderly patient, following hemorrhoidectomy is?
• Fecal impaction
• Bleeding
• Urinary retention
• Infection
• Anal stenosis
32. A lady with gall stones is having biliary colic. Her pain would be radiating towards:
• Umbilicus
• Right shoulder
• Hypogastrium
• Right iliac fossa
• Left iliac fossa
33. Patient with urinary bladder stones are at increased risk for which of the following
• bladder cancers?
• Adenocarcinoma
• Transitional cell carcinoma
• Squamous cell carcinoma
• Choriocarcinoma
• Carcinoma in situ
34. A patient has scrotal swelling. It is non tender, trans-illuminant, and testis and epididymis are
not definable. He has:
• Hydrocele
• Haematocele
• Epididymal cyst
• Spermatocele
• Varicocele
35. Commonest dislocation of hip joint is:
• Anterior
• Inferior
• Lateral
• Posterior
• Superior
36. Treatment of renal adenocarcinoma is:
• Chemotherapy
• Simple Nephrectomy
• Partial nephrectomy
• Radical nephrectomy
• Radiation
37. Alpha-fetoprotein is raised in a patient with:
• Prostatic cancer
• Renal cell carcinoma
• Seminoma testis
• Teratoma testis
• Urinary bladder cancer
38. In case of Litter’s hernia, the hernia sac contains:
• Appendix
• Meckel’s diverticulum
• Testis
• Ovary
• Bladder
39. Raised PSA is suggestive of:
• Carcinoma pancreas
• Carcinoma liver
• Carcinoma prostate
• Carcinoma ovary
• Carcinoma rectum
40. What is raised in acute pancreatitis?
• Serum alkaline phosphatase
• Serum calcium
• Serum amylase
• Serum albumin
• Serum ALT
41. Following is NOT a part of Tetralogy of Fallot:
• VSD
• ASD
• Overriding aorta
• Pulmonary stenosis
• Right ventricular hypertrophy
42. Carcinoma of the breast is the commonest tumor in females. Following is not a component of
Triple assessment
• Clinical examination
• Mammogram
• Sonomammogram
• FNAC
• X ray chest
43. Greenish nipple discharge is a feature of:
• Duct carcinoma
• Duct papilloma
• Duct ectasia
• Duct sarcoma
• Duct abscess
44. Monder’s Disease of breast involves:
• Arteries
• Veins
• Nerves
• Ducts
• Cooper’s ligaments
45. The commonest location of osteogenic sarcoma is:
• Around wrist
• Around elbow
• Around hip
• Around knee
• Around ankle
46. Following is NOT a feature of cardiac tamponade:
• Engorged neck veins
• Hypotension
• Tracheal shift
• Muffled heart sound
• Normal breath sounds
47. Following is NOT a life threatening chest injury to be ruled out in primary survey:
• Tension pneumothorax
• Massive haemothorax
• Bronchial obstruction
• Flail chest
• Open pneumothorax
48. The commonest site of perforated duodenal ulcer is:
• First part
• Second part
• Third part
• Fourth part
• Duodeno-jejunal junction
49. A 70 year old male smoker is having progressive dysphagia for which he has under gone
endoscopy, which has revealed a growth in proximal esophagus. Biopsy is likely
• to show:
• Adenocarcinoma
• Sarcoma
• Squamous cell carcinoma
• Transitional cell carcinoma
• Anaplastic carcinoma

Professor Shabbir MCQs


1. What is the most common cause of Hippocratic facies in a surgical patient
• Bell’s palsy
• Burns
• Dehydration
• Diffuse peritonitis
• D
2. An 11 years boy is having sudden onset of severe pain in right testis. Commonest cause of
torsion of testis in children is
• Associated testicular tumor
• Congenital hydrocele
• High investment of tunica virginals
• Mald scent of testis
• C
3. Following is the malignant condition of breast:
• Cyst sarcoma Phylloides
• Duct ectasia
• Fibro adenoma
• Mondor’s disease
• Paget’s Disease
• E
4. A 13 years boy is diagnosed to be having bronchial cyst on right side of neck. Bronchial cleft
remnants most often present with which of the following clinical problem?
• Airway obstruction
• Hemorrhage
• Infection
• Malignant degeneration
• Pain
• C
5. Patient with bladder stones are at increased risk for which of the following bladder cancers?
• Adenocarcinoma
• Transitional cell carcinoma
• Squamous cell carcinoma
• Choriocarcinoma
• Carcinoma in situ
• B
6. A patient has testicular swelling. It is non tender, trans-illuminant, and testis and epididymis
are not definable. He has:
• Hydrocele
• Haematocele
• Epididymal cyst
• Spermatocele
• Varicocele
• A
7. Raised prostate specific antigen (PSA) is suggestive of:
• Carcinoma pancreas
• Carcinoma liver
• Carcinoma prostate
• Carcinoma ovary
• Carcinoma rectum
• C
8. A 70 years male smoker is having progressive dysphagia for which he has under gone
endoscopy, which has revealed a growth in proximal esophagus. Biopsy is likely to show:
• Adenocarcinoma
• Sarcoma
• Squamous cell carcinoma
• Transitional cell carcinoma
• Anaplastic carcinoma
• C
9. Murphy’s sign is present in:
• Appendicitis
• Ac Cholecystitis
• Hepatitis
• Pancreatitis
• Terminal ileitis
• B

Musculo skeleton
1. A 42-yearlady is involved in a road traffic accident. She presents with a dislocated shoulder.
The shoulder is relocated by you in A&T but afterwards you noticed winging of the scapula.
Which nerve is likely to be damaged?
• Axillary nerve
• Median nerve
• Radial nerve
• Long, thoracic nerve of Bell*
• Ulnar nerve
2. A ‘claw hand’ is usually associated with injury to which of the following nerves?
• Axillary nerve
• Musculocutaneous nerve
• Radial nerve
• Median nerve
• Ulnar nerve*
3. 65. Which of the following statements is true of Colles' fracture?
• Is a cause of carpal tunnel syndrome*
• Results in palmar displacement of the distal
• fractured fragments c. Extends into the wrist (radiocarpal) joint
• Is typically associated with compression of the ulnar nerve
• Open reduction is usual treatment
4. Which one of the following organisms is most commonly associated with septicarthritis?
• Gram negative bacilli
• Haemophilus influenza
• Beta haemolytic streptococci
• Streptococcus pneumonia
• Staphylococcus aureus*
5. 56. A patient presents to surgical outdoor clinic complaining of a swelling on his wrist.
Onexamination there is a focal swelling on the dorsal aspect of the wrist. It is smoothand non
tender. The overlying skin is normal and moves freely over the mass, however, it seems to be
fixed to the tendon. What is the likely diagnosis?
• Sebaceous cyst
• Lipoma
• Ganglion*
• Giant cell of the tendon sheath
• Fibroma
6. Commonest dislocation of shoulder joint is:
• Anterior*
• o. Inferior
• Lateral
• Posterior
• Superior

GIT
1. 7. A man undergoes an open inguinal hernia repair. During the procedure the spermatic cord
is visualized, What structures does this contain?
• Dartos muscle
• Femoral branch of the genitofemoral nerve
• Ilioinguinal nerve
• Inferior epigastric artery
• e Pampiniform plexus*
2. Splenectomy patients are at particular risk of helming sepsis from which organisms?
• aerobic bacteria
• Bacteroides fragilis
• Haemophilus influenzae*
• Fungi
• e aureus
3. Which of the following tumour markers is most commonly associated with pancreatic cancer?
• Canvinoembryonic antigen
• Cancer antigen (CA)125
• CA19-9*
• alpha-fetoprotein (AFP)
• Beta-human chorionie gonadotrophin
4. 62. Regarding, hernias in females, the most common is:
• Epigastric
• Lumbar
• Femoral
• Inguinal*
• Incisional
5. 63. A 35-year-old professional weightlifter presents with a red and swollen lump in the left
groin. An inguinal hernia is suspected and, at the time of operation, thelump is found to
contain a loop of necrotic bowel. This type of herma is best described as:
• Irreducible
• Strangilated*
• Obstructed
• Sliding
• Richter's
6. 13. Which statement is true of Meckel's diverticulum?
• Is present in 20 per cent of the population
• Arises from the mesenteric border of the jejunum
• May contain heterotropic pancreas*
• Is present only in males
• Is a diverticulum of the bladder
7. 22. A 58-year-old patient presents with a 6-week history of increasing difficulty in swallowing.
He first noticed problems when eating meat which became stuck behind his heart', but this n
to include other foods. The patient is currently worried because he is now struggling with
thick fluids and has noticed some involuntary weight loss. What is the most appropriate first
investigation?
• Staging computed tomography
• Barium meal*
• Upper gastrointestinal endoscopy
• Barium swallow
• Electrocardiography
8. 14. Human papilloma virus (HPV) can cause which of the following malignancy?
• Carcinoma of breast
• Carcinoma of anal canal*
• Carcinoma of esophagus
• Bladder carcinoma e. Gastric carcinoma
9. 16. During a cholecystectomy, the cystic artery must be located and ligated. This artery most
commonly arises from the:
• Gastroduodenal artery
• Hepatic artery proper
• Right hepatic artery*
• Left hepatic artery
• Superior pancreatico-duodenal artery
10. 27. A 62-year-old patient is admitted with jaundice. His stool is pale and urine is dark. On
examination he has a palpable gallbladder. The most likely cause is:
• Ascending cholangitis
• Impacted stone in the common bile duct (choledocholithiasis)
• Tumour of the head of pancreas*
• Impacted stone in the neck of the gallbladder
• Viral hepatitis
11. 29.A 2-year-old female patient presents with a 6-week history of bloody motions She has
noticed significant wright loss over the preceding & works with increasing lethargy and fatigue
She has previously had constipation and admits to reflex laxative use What is the most likely
diagnose
• Bowel cancer*
• Imitable bowel syndrome
• Diverticular disease
• Inflammatory bowel disease
• e Anal fissure
12. 24, A 45-year-old man presents to the emergency department with a history of coffee ground
vomiting. He also reports that for 2 days his stool has appeared darker than usual. Which of
the following gives the most sensitive guide as to the severity of his gastrointestinal
haemorrhage?
• Haemoglobin
• Systolic blood pressure
• Pulse rate
• Volume of vomitus/melaena
• Lying and standing blood pressure
13. 26. A 29-year-old pattent presents with a short history of right upper quadrant pain. She is
jaundiced with dark urine and pale stool. She 9 °C. Abdominal examination gives a suggestion
of a palpable gallbladder. The diagnosis is:
• Ascending cholangitls
• Acute cholecystitis
• Biliary colic with bile duct obstruction*
• Pancreatitis
• Mirizzi's syndrome
14. 31. An 18-year-old patient presents with a 5-day history of abdominal pain and pyrexia. On
examination bowel sounds are present and the abdomen is soft with no rebound tenderness.
A mass in the right iliac fossa is palpable. Abdominal computed tomography confirms the
diagnosis of an appendicular mass with an associated abscess. The patient is started on
metronidazole and admitted for observation and conservative management. After 2 days the
mass has not reduced in size and the temperature remains raised. The next stage in
management is:
• Continue metromdazole for further 14 days cefuroxime
• Proceed to appendicectomy
• Drainage of Appendicular abscess*
• Colonoscopy
• e Laparoscopy
15. 33. which of the following is NOT an option when treating an anal fissure?
• a Propranolol*
• b. Botulinum toxin A injections
• Glyceryl trattate cream
• Diltiazem cream
• e Laxatives
16. 34. Which of the following is NOT sign associated with acute appendicitis?
• Murphy's sign*
• Cope sign
• Psoas sign
• Rovsing's sign
• e Pain on rectal examination
17. 36. Which of the following is the most common site for colonic carcinoma?
• Rectum
• Sigmoid colon*
• Caecum and ascending colon
• Descending colon
• Transverse colon
18. 37. The most common cause of acute intestinal bowel obstruction is:
• Adhesions *
• Intestinal parasites
• Volvulus
• Hernia
• Neoplasia
19. 38. Which of the following histopathological features is NOTcharacteristic of Crohn's disease?
• Rose thorn abscesses*
• Cobble stoning
• Skip lesions
• d I ead piping
• Serosal involvement
20. 25. A patient presents with a short history of perfuse, projectile vomiting without bile
staining. He has a history of peptic ulceration and chronic dyspepsia and has noticed increased
bloating over the preceding 9 months. On examination there is distension in the epigastric
region and a succession splash. The abdominal radiograph shows a grossly distended stomach
and collapsed bowel. The most likely cause is:
• Carcinoma of the esophagus
• Carcinoma of the head of pancreas
• Fibrotic stricture*
• Compression by malignant nodes
• Chronic pancreatitis
21. 28. A patient was admitted for acute pancreatitis secondary to alcohol misuse. He was treated
conservatively and discharged for outpatient follow-up. He presents to your clinic after 8
weeks complaining of continued abdominal pain in the epigastrum radiating into the back
Serum amylase is 200 A. On examination be is tender over the epigastrium Blood pressure,
pulse rate and temperature are all diagnosis is:
• Pancreatic pseudocyst*
• Cholecystitis
• Chronic pancreatitis
• Recurrent mente pancreatitis
• e Pancreatic abscess
22. 30. Which one of the following does NOT occur as a systemic manifestation of inflammatory
bowel disease
• Ankylosing spondylitis
• b Pyoderma pangrenosum
• e Sclentis
• d Osteoporosis
• e Sclerosing cholangitis *
23. 32, A patient with a previous anal abscess presents with persistent perianal discharge and
discomfort. On examination a simus is identifiable at the 6 o'clock position with the patient in
the lithotomy position. What procedure the surgeon is most likely to perform?
• Seton insertion
• Colostomy*
• Open exploration of tract
• I&D
• Lateral Syphincterotomy
24. 39. A 55-year-old man, with a 2-year history of dyspepsia. is brought to the emergency
department following a sudden onset of severe epigastric pain. The pain is made worse on
movement. On examination, the patient is cold. sweating profusely and taking shallow
breathe. The abdomen is rigid and bowel sounds are absent. A plain film chest radiograph
reveals free air under the diaphragm. The most likely diagnosis is:
• Perforated appendicitis
• Acute cholecystitis
• Acute pancreatitis
• Myocardial infarction
• Perforated peptic ulcer*

Urogenita
1. Fourier's gangrene of the scrotum is caused by all accept
• a Clostridia*
• b. Bacterolds
• C. Colifarms
• d. Peptostreptococci
• e. StepholocOCUS
2. A 70-year-old man in admitted to the emergency department with acute-onset suprapubic
pain and inability to pass urine for 2 days. On examination, the patient is in discomfort,
neurologically intact, and the abdomen is particularly tender in the suprapubic region. A
digital rectal examination reveals a smooth, enlarged prostate. From the list of options below,
select the most appropriate course of action to takein managing the patient.
• Ask the urology registrar to see the patient
• Obtain blood for urea and electrolytes sampling
• e. Request an abdominal plain film radiograph
• Urinary catheterization *
• Request an intravenous urograto
3. A 12-year-old boy is admitted to the emergency department with sudden onset of severe right
testicular and lower abdominal pain during athletic training. Ile has had one episode of
vomiting and constantly feels nauseous. On examination, the patient is sweating and in
unbearable pain. There is marked tenderness and swelling of the right testicle which is
observed to be lying horizontally. What is the most appropriate next step in this patient's
management?
• Order a CT scan of abdomen
• Send the patient immediately for emergency surgical exploration of the scrotum*
• Perform urine dipstick
• Manage the patient with analgesia and observe
• Obtain a second opinien from your senior colleague, who will only be able to see the patient
in an hour
4. A 6-week-old hoy presents with his parents to the specialist paediatric urology outpatients
department. The parents at the time of the boy’s circumcision were told by the doctor that
the urethral meatus was not in the normal position. On examination, the meatus is on the
ventral surface just below the glans penis. What is the most likely diagnosis?
• Hypospadias *
• Epispadias
• Phimosis
• Chordee
• Perineal urethra
5. 55. A 36-year-old carpenter is diagnosed with a seminoma. To which lymph nodes does a
seminoma most commonly spread first?
• Para-aortic lymph nodes*
• Superficial inguinal lymph nodes
• Anterior cervical chain
• Posterior cervical chain
• Deep inguinal lymph nodes
6. 46. A 75-year-old man is admitted to the emergency department with acute onset suprapubic
pain and inability to par urine for 2 days. On examination, the pet in discomfort,
neurologically intact, and the abdomen is particularly tender in the suprapubic region. A
digital rectal examination reveals a smooth, enlarged prostate. What is the most likely
diagnosis?
• Bladder outflow obstruction due to prostate cancer
• Bladder outflow obstruction due to benign prostatic hypertrophy*
• Bladder cutflow obstruction due to a urethral stricture
• Bladder outflow obstruction due to a spinal cord lesion
• Bladder outflow obstruction due to bladder stone
7. A 19-year-old medical student presents with acute-onset colicky left loin pain and describes a
history suggestive of urinary calculus What one of the following would be the initial
investigation of choice to determine the presence of a calculus in the renal tract
• a KUB (kidney ureter bladder radiograph*
• b. Intravenous urogram
• Flexible cystoscopy
• Diethylene triaminepentaacetic acid (DTPA) xcan
• U/S KUB
8. 6. A patient is known to have exposure to beta naphthylamine. What is he/she at increased
risk of developing?
• Small-cell lung carcinoma
• Bladder cancer*
• Breast cancer
• Chemical pneumonitis

Endo and Salivary glands:


1. 10. Thyrotoxicosis is characterized by which of the following?
• Weight gain
• Proximal myopathy*
• Enopthalmos
• Bradycardia e. Slow tendon reflexes
2. 18. The commonest malignancy of thyroid is:
• Anaplastic carcinoma
• Follicular carcinoma.
• Hurthle cell carcinoma
• Medullary carcinoma
• Papillary carcinoma*
3. 61. A 28-year-old secretary presents with a lump in the midline of the neck that hasgrown
progressively larger over the past several months. On examination, there isa palpable lymph
node in the left submandibular region. An aspirate is taken confirming a malignant cancer of
the thyroid gland. The origin of s is mostlikely to be:
• Follicular
• Anaplastic
• Medullary
• Lymphoma
• Papillary*
4. 11. Which of the following is associated with hyperparathyroidism?
• Paravertebral ossification
• Peptic ulcer*
• Retinal haemorrhages
• Aortic stenosis
• Hoarse voice
5. 65-year-old women with a retrosternal goitre discovered on a routine chest X-ray, which of
the following clinical sign can be present?
• Tracheal deviation
• Berry's sign
• Pemberton's sign*
• Stellwag's sign
• Pretibial myxedema
6. 48. During a ward round, you are asked by your surgical registrar about themanagement of a
phaeochromocytoma. Select from the list below the mostappropriate management plan for a
phaeochromocytoma:
• Surgical resection, followed by beta blockade, followed by alpha blockade
• Lifelong bela and alpha blockade
• Surgical resection
• Beta blockade, followed by alpha blockade. followed by surgical resection
• Alpha blockade, followed by beta blockade followed by surgical resection*
7. 60. You are asked to assess a 25-year-old patient who returned from theatre 2 homi following
a thyroidectomy for a large hyperplastic goitre and is now complaining of difficulty in
breathingis short of breath with a respiratory rate of 40 breaths/min, using beraccessory
muscles of respiration and only able to answer your questions in two or three words. In
addition, there appears to be a fluctuant mass in the midline of theneck underlying, the
surgical clips. Immediate management of this patient wouldbe:
• High-flow oxygen via Hudson mask
• Removal of surgical clips at the bedside
• Intravenous access with two large-bore cannulae and fluid resuscitation
• Removal of surgical clips in theatre under general anaesthesia
• Call your senior and wait for him/her to remove the clips
8. 7. 80 percent of all salivary stones occur in:
• a Parotid gland
• Submandibular gland*
• Submaxillary gland
• Minor salivary glands
• Sublingual gland
9. 10.Aphthous ulcer is found in:
• Bladder
• Colon
• Eye
• Mouth*
• Stomach
10. 8. Commonest surgical disease of parotid is:
• Calculus
• Carcinoma
• lymphadenitis
• Parotitis
• Pleomorphic adenoma*
11. 9. Submandibular gland duct is called:
• Charcot's duct
• Santorini's duct
• Stenson's duct
• Wharton's duct*
• Wirsung's duct
12. 12 Preoperative preparation in a patient of pheochromocytoma is
• Alpha agonist
• b. Alpha antagonist*
• C Beta agonist
• d Beta antagonist
• Diuretics
13. 15.Emergency treatment of post-thyroidectomy bleed is:
• Blood transfusion
• Endotracheal intubation
• Opening up of wound and exploration*
• Pressure bandage
• P Transamine injection
14. 18. Minimum operation for solitary thyroid nodule is:
• Excision of nodule
• Subtotal thyroidectomy
• Total lobectomy
• Total lobectomy plus Isthmectomy*
• Total thyroidectomy
15. Grave's disease is:
• Primary thyrotoxicosis*
• Secondary thyrotoxicosis
• Tertiary thyrotoxicosis
• Thyrotoxicosis fictitia
• Treated thyrotoxicosis

Breast:
1. 19.Rotor's lymph nodes are important in relation with:
• Carcinoma breast*
• Carcinoma gallbladder
• Carcinoma rectum
• Carcinoma stomach
• P. Carcinoma thyroid
2. 15. A worried 23-year-old woman presents with a painless lump in the right breast of one
month du.Onexamination a mobile, discrete fired, non-tender, firm 1 cm lump is d. There is no
lymphadenopathy. The most likely diagnosis is
• Breast cyst
• Lipoma
• Fibroadenoma*
• Sebaceous cyst
• Carcinema of the breast
3. 2 A 16-year-old man is diagnosed with invasive ductal carcinoma of the right breast stage
12N0MB. The most appropriate treatment option for this patient is?
• a Cytotoxic chemotherapy
• b. Wide local excision
• c. Mastertomy pustoperative radiotherapy
• Palliative care programe
• Modified radical mastectomy

CNS
1. A 31-year-old man is admitted following an assault outside a cinema. During the fight he was
hit by a blunt object across the side of the hend. On admission his Glasgow Coma Scale score
is initially 12/15 but falls to 8/15 during his evaluation. The decision is taken to perform a
computed tomography head scan, which identifies a biconvex-shaped space-occupying lesion
within the cranial vault. The diagnosis is:
• Extradural haematoma*
• Subdural haematoma
• Subarachnoid haemorrhage
• Cerebral contusion
• e Intracerebral haemorrhage
2. 59. A 21-year-old female student presents to the emergency department following afall from
a ledge approximately 1.5 m high while walking home. She is drowsyfollowing fall. She opens
her eyes when her name is mentioned and is talking about her studiestomorrow. She localizes
pain you press her nail bed. What is her Glasgow Coma Score
• 5
• 7
• C. 9
• 11
• e 13*
3. A 5-ar-old man is locked off his cycle and huts his head on a stone. He is brought into hospital
with a fluctuating score on the Glasgow Coma Scale. He underwent a CL head scan which
shows a biconvex shaped hematoma. What other finding is he most likely to have?
• Midline shift*
• Skull fracture
• Subdural hematoma
• Hydrocephalus
• Subarachnoid haemorrhage
4. 58. An 85-year-old pensioner is brought to the emergency department by his family who are
concerned that he has become increasingly confused and drowsy in the past 3 weeks. He is
confused and unable to recall events clearly but oriented to time and person and complains
only of occasional frontal headache. His family informs you that he may have fallen while
climbing from the bathrubsome weeks previously. He has also started sleeping for long
periods of time, which is not her normal habit. A head CT scan is performed, which shows mild
generalized atrophy and a crescent-shaped collection. This presentation is consistent with.
• Intracerebral haemorrhage
• Subarachnoid haemorrhage
• Exiradural haemorrhage
• Subdural haemorrhage*
• Intra ventricular haemorrhage
5. 14.Commonest site of origin of neuroblastoma is:
• Adrenal cortex
• Adrenal medulla*
• Bladder
• Spinal cord
• Sympathetic chain

PEADIATRIC SURGERY
1. 16.Commonest solid malignant tumour in children is:
• Fibroblastoma
• Hepatoblastoma
• Nephroblastoma*
• Neuroblastoma
• Retinoblastoma
2. Which one of the following investigation you think would help in antenatel diagnosis of
meningo-myelocvele due to spina bifida?
• Blood Complete Picture (CP)
• B Erythrocyte sedimentation rate (ESR)
• C Computersed Tomography (CT) scan
• D Maternal serum alfa feto protein (AFP)
• Seruma amylase
• Answer D
3. Which one of the following statement is true regarding Intracranial Pressure (ICP)?
• Increases in brain tumours.
• Decreases after head injury
• Increases with lumbar puncture
• Decreases in liydrocephalus
• E Decreases in meningitis
• Answer: A
4. 3 A 32 years old male patient presented to emergency with severe head injury with GCS 6/15
and your senior colleague orders you to administer Mannitol infusion. Which of the following
is essential prior to administration?
• Inquire about history of allergy to mannitel
• B Bilaterally dilated pupils
• C Hemoglobin - 100
• D 95% Oxyge saturation
• Urinary catheterization
• Answer E
5. Which of the following is the most important clinical sign of hydrocephalus in a 2 months old
baby?
• A Dilated pupils
• Facial nerve palay
• CHemiplegia
• Sunset sign
• E Normal fontanelle
6. A 30 years old male presented to emergency with sudden onset of severe headache, neck
stiffness, vomiting and fever. He is seen by emergency doctor and advised lumbar puncture
Whet one is the most important precaution you would consider before the procedure?
• A Do complete blood count
• B Do Fundoscopy and CT scan.
• C Catheterize the patient.
• Do keep patient in left lateral position.
• Do X ray skull
• Answer: B
7. The most common presentation of adult primary brain tumour is
• A Focal neurological deficits
• B. Endocrine dysfunction
• C.Visual loss.
• D. Hemiplegia
• Headache and Seizures
• Answer: E
8. Which one of the following statements is correct regarding Glasgow coma score whic used
worldwide to ascertain the level of consciousness?
• Maximum score is 18.
• Minimum score is 0.
• C GCS of 14/15 means the patient is in deep coma.
• GCS range is from 3 to 15.
• E GCS of 3/15 means the patient is well oriented in time, space and person.
9. 9 Glasgow coma score (GCS) is used for following except:
• F Diagnosis of head injury patient G. Prognosis of head injury patient.
• Treatment of head injury patient
• L Treatment of paraplegic patient.
• J. Monitoring of stroke patient
10. 6 A27 years old driver presented to emergency with road traffic accident and severe head with
GCS 7/15 Which one of the following factor can cause serious secondary brain mjury injury
• Base of skull fracture
• Hypertension
• Hypoxia
• D Increased cerebral perfusion
• Polyuria
• Answer: C
11. 10. Which of the following is clinical evidence of middle cranial fossa fracture is a 39 years old
female who presented with history of fall from height of 10 meters?
• A CSF Otomboca.
• B.. CSF Rhinomboca.
• Fits.
• Hypotension
• Panda eyes.
12. 25. Cushing'sreflex is a physiological nervous system response to increased intracranial
pressure (ICP) that results in which one of the followings:
• A Tachycardia and hypotension
• B. Tachycardia and hypertension
• C. Bradycardia and hypotension
• D) Bradycardia and hypertension
• Hypertension and dyspnoea
• Answer D
13. 10: Which of the following is clinical evidence of middle cranial fossa fracture in a 39 years old
female who presented with history of fall from height of 10 meters?
• CSF Otorrhoea
• CSF Rhinorrhoca
• Fits.
• Hypotension
• Panda eyes
14. 11. Which of the following is clinical evidence of anterior cranial fossa fracture in a 39 years
old female who presented with history of fall from height of 10 meters"
• Fits.
• CSF Otorrboca.
• Hypotension.
• Panda eyes
• Dilated pupils
• Answer: D
15. 12: A35 years old male presented with sudden onset of severe backacke after lifting heavy
weight. He was diagnosed as lumbar disc prolapse. What non invasive mvestigation would you
like to request to diagnose thus patient?
• A Computerised Tomography (CT) scan.
• B. Xray Lumbosacral spine
• C. Magnetic resonance imaging (MRI) scan
• D. Ultrasound scan
• E Lumbar myclography
16. 13: A31 years old female who was 3 month pregnant was involved in road traffic accident and
sustained severe head injury Which of the following you would consider while ordering the
brain CT scan?
• A CT scan is contraindicated in this patient.
• B. Body should be covered with lead shields during CT scan.
• C CT scan bram with contrast is required
• D. CT can be postponed till patient regains consciousness.
• E Priority is given to safety of the baby first.
• Answer: B
17. 14: 45 year old male patient received intramuscular injection of diclofenic sodium in gluteal
region for renal colic. He develops immediate foot drop after injection. Which of the following
segment involved?
• A L1
• BL2
• C. LA
• D. LS
• F S1
• Answer: D
18. 15: 43 year old male patient received intramuscular injection of diclofenic sodium in deltoid
region for renal colic. He develops immediate wrist drop after injection. Which of the
following nerve segment is involved?
• CS
• C6
• C C7
• CS
• E TI
• Answer B
19. 167 45 year old female presented with backache and right sciatica. She is diagnosed as lumbar
disc prolapse. On neurological examination she has absent knee jerk on the right side which
means lesson is situated at one of the following spinal level:
• A L12
• B. 12.3
• 13.4
• I.S.SI
• S1,S2
• Answer C
20. 17. lumber puncture in newborn is risky because spinal cord extends at the following level in
newborn
• A T 12
• B. LI.
• C... L.3.
• D. L4
• LS
• Answer C
21. 18. Acute blood appears white on the computed tomography (CT) brain because of
• Calciuns
• Sodium
• Edema
• Globm
• E Potassium
• Answer: D
22. 19: A 23 years old male presents to Accident and Emergency after being assaulted on the way
home from work. He is able to tell you that he was hit on the head once but does not
remember much after that. He is initially alert and able to answer your questions but
progressively becomes drowser. He also vorals a few times CT scan showing a high density,
biconvex collection of blood on the side of his injury What is the most likely diagnosis?
• A Subdural haematoma
• B. Localised brain oedema
• C. Intacerebral hacontorna
• D Extradural haematoma
• Intraventricular bleeding
23. 20 Which one of the following is indicative of upper motor neuron lesion?
• A Deceased tone of muscles
• B. Depressed tendon reflexes
• C. Increased bulk of muscles
• D. Increased tone of muscles
• Down gomg planter reflex
24. 22 Lucid interval is a temporary improvement in a patient's condition after a traumatic brain
injury, after which the condition deteriorates Lucid interval is seen in
• Sub dural heroorrhage
• Sub arachnoid hemorrhage
• C Extradural bemorrhage
• Pontime bemorrhage
• F.Basal ganglia hemorrhage
• Answer C
25. 23 A 70 years old gentleman who is taking warfarin presented to ER with sudden onset of right
hemiparests Which of the following investigations is urgent and most important to obtain?
• A Carotid duplex scan
• B Echocardiogram
• CCT brain
• Nerve conduction studies
• E Electrocardiography
26. 23: A 70 years old gentleruan who is taking warfarin presented to ER with sudden onset of
right hemiparess. Which of the following investigations is urgent and most important to
obtain?
• Carotid duplex.scm
• B Echocardiogram
• C.CT brain
• Nerve conduction studies
• E Electrocardiography
• Answer, C
27. 24:Failure of the neural tube to close causes:
• Hydrocephalus
• Microcephaly
• Meningmyclococele
• Loss of vision
• Cerebral palsy
28. 21. Which one of the following is indicative of lower motor neuron lesion?
• A Increased tone of muscles
• Depressed tendon reflexes
• Increased bulk of muscles
• Increased tone of muscles
• E Up going planter reflex
• Answer B
29. 26: A patient presents following a motor vehicle accident with eye opening to painful
stimulus, incomprehensible sounds and on painful stimulus flexing arms and extending legs.
What is his Glasgow comm score (GCS)?
• A 5/15
• B. 6/15
• C 7/15
• 8/15
• E 915
• Answer, C
30. 27: A 25-year-old man fell 15 feet from a tree. There was a brief loss of consciousness
witnessed by his friends. When the ambulance arrived he was GCS 15. In A&E he was noted to
have a right temporal scalp laceration One hour later his GCS score dropped to 13/15. Thirty
minutes later he was noted to be GCS 8/15 with a fixed and dilated right pupil CT scan showed
extradural hematoma. All of the following statements are correct except:
• It is often caused by a skull fracture with laceration of the middle meningeal artery.
• It commonly occurs at the plenon.
• C There is often a lucid interval.
• Treatment ts with a burr hole and evacuation of haematoma.
• E Prognosis is poor even if operated.
31. 28: A 65 years old roale was hit by a speeding car while crossing the road. CT scan of brain in
emergency department showed extensive subdural hacmatoma. Which of the following
statements regarding acute subdural hacmatoma is false?
• It is a collection of blood between the dura and arachnoid membranes
• It can be caused by laceration of the brain.
• It can be due to disruption of a cortical blood vessel, usually a vein.
• It has a very good prognosis if operated.
• It has a biconvex shape on CT scans
• Answer D
32. 29 Which of the following statements regarding third nerve paisy in head injury is true?
• It causes a fixed, dilated pupil.
• By cause the eye to deviate upwards.
• C It is caused by pressure on the third nerve by a herniated brain stem.
• It usually occurs on the opposite side to the haematoma
• E it is never briateral
33. 31 A 32-year-old man suffers a spine spinal cord injury with a resultant paraparesis in a
motorcycle accident. He presents to the emergency room and ER doctor advises to apply
cervical collar and log roll the patient. Which of the following statements is false regarding log
roll?
• Maneuver used to move trauma patient with suspected spine injury.
• Patient is carefully rolled in the desired direction without twisting or bending the body.
• Basic idea is to not to convert incomplete spinal injury to complete spinal injury.
• D Used to examine the spine in trauma patient.
• E used only for initial few hours in management of trauma patient.
34. 32 A 28 years old female who is getting married in few weeks time comes to you to inquire
about taking folic acid for prevention of neural tube defects. What advice would you like to
give to this lady?
• She should start taking daily supplement containing 400 to 800 mcg of folic acid.
• She should start taking a daily folic acid supplement if she becomes pregnant.
• She should not take any additional vitamin or iron supplements.
• D She should start taking a daily folic acid supplement until she becomes pregnant, and then
stop.
• E.. If her haemoglobin is adequate than she does not need any supplements

UROLOGY
1. A 35 years old male had blunt abdominal trauma and sustained type IV renal injury. It is
characterized by the following features
• Avulsion of real pedicle*
• Sub capsular hematoma
• Laceration of upper pole of kidney
• Laceration of lower pole of Midney
• Shattered kidney
2. Q-2. A female of 60 years present with haematuria with the passage of worm like clots Which
is the first line investigation for diagnosis
• Intravenous urogram
• CT Urogram
• MRI
• Ultrasonography*
• CT (KUB)
3. 30 years old female las got 1 centimeter stone lodged in tower ureter What is the optimum
treatment for this stone?
• Conservative
• Open ureterolithotomy
• Dormia basketing
• Ureters-resscopy*
• Extra shockwave lithotripsy
4. A 60 years ole male who had LUTS, develop retention of urine. Which is the most common
precipitating factor for this?
• Over indulgence in beer
• Excessive intake of coffee
• Postocnement of micturition*
• Confinement to bed
• Prolonged driving
5. A male of 20 years while walking through street suddenly fell on a sharp object and got
perineal trauma. Which part of the involved in this injury?
• Navicular fossa
• Anterior Urethra*
• Membranous Urethra
• Prostatic Urethra
• Penile urethra
6. Q-6 A 50 year old male has got renal cell carcinoma At what age renal cell carcinoma is more
common.
• Children
• Elderly female
• Elderly male*
• infants
• Teenagers
7. Q-7 A person has got renal cell carcinoma. What is the commonest route of spread of person
is.
• Direct
• dissemination
• Lymphatics
• Transmigration
• Vascular*
8. A person is a known case of stricture urethra. Best palliative treatment for this person is.
• Laser ablation
• Observation
• Optical urethrotomy*
• Urethroplasty
• Urethral dilatation
9. A person had road trame accident and sustained posterior urethral injury. On DRE there is
high riding prostate. Immediate treatment for the person is
• 002 Cystoscopy
• (11) Observation
• (11) Optical repair
• Rail road*
• (v) Suprapubic cystostomy
10. Q-10 A lady of 30 years of age has got 1 cm ureteric stone 5 cm away from Pelviureteric
junction. What is the treatment of choice for this lady.
• Conservative
• ESWL*
• Open ureterolithotomy
• (iv) PCNL
• (v) Uretero renoscopy
11. Q-11 Calcium oxalate stones are
• 3 diagnosed by ultrasonography
• Easily broken by ESWL
• (ii) Radiolucent
• Radio opaque*
• Resistant to ESWL
12. Q-12 Pyelolithotomy is the removal of stone from which part of the kidney
• Central parenchyma of kidney
• (6) Kidney parenchyma
• Lower pole of kidney
• Pelvis of kidney*
• (v) Upper pole of kidney
13. Q-13 A 70 year old male has got LUTS. Which of the following is the obstructive
• Urinary symptom
• Frequency of micturition
• Hesitancy*
• Nocturia
• Urgency
• Urge incontinence
14. Q-14 In prolonged immobilization stones are formed due to
• Decreased metabolism
• (H) Decreased ureteric peristalsis
• Decreased urinary output
• (iv) Dehydration
• Skeletal decalcification*
15. Q-15 A female had ureteric stone. Which is the commonest site of lodgement of this stone
• Anywhere in the ureter
• (11) Crossing of iliac vessels
• Crossing of vasi
• (iv) Pelviureteric junction
• Ureterovesical junction*
16. Q-16 A young male had gonococcal urethritis. What is the most common complication of this
disease
• Epididymitis
• (H) Infertility
• Prostatitis
• Stricture urethra*
• Orchitis
17. Q-17 A 35 years old male has got right testicular tumor. Which of the following Lymph nodes
are the first site of spread of this tumor
• External iliac
• Internal iliac
• Inguinal
• Para-aortic*
• Supraclavicular

MCQ Module Exam Surgery B


1. A 46-year-old man is diagnosed with invasive ductal carcinoma of the right breast stage
T2N0M0. The most appropriate treatment option for this patient is?
• Cytotoxic chemotherapy
• Wide local excision
• Mastectomy + postoperative radiotherapy*
• Palliative care programme
• Modified radical mastectomy
2. A 35-year-old man is knocked off his cycle and hits his head on a stone. He is brought into
hospital with a fluctuating score on the Glasgow Coma Scale. He underwent a CT head scan
which shows a biconvexshaped haematoma. What other finding is he most likely to have?
• Midline shift
• Skull fracture*
• Subdural haematoma
• Hydrocephalus
• Subarachnoid haemorrhage
3. A singer complains of not being able to sing high notes following her thyroidectomy. What is
the likely cause?
• Damage to the recurrent laryngeal nerve
• Damage to the external laryngeal nerve*
• Damage to the vagus nerve
• Tracheal stenosis
• Vocal cord hemiparalysis
4. Radiotherapy can be the sole treatment for which of the following cancers?
• Adenocarcinoma of the oesophagus
• Rectal carcinoma
• Anal cancer*
• Gastric carcinoma
• Phylloides breast tumour
5. Gastrointestinal consequences of major burns include which of the following?
• Splanchnic vasodilation
• Acute gastric dilatation
• Curling’s ulcers*
• Terminal ileal hyperplasia
• Mechanical bowel obstruction
6. A 35 year old house wife sustained accidental full thickness burns involving both lower limbs
and half of back. Her percentage of burnt area as per Wallace Rule of Nines is:
• 25 %
• 35 %
• 45 %*
• 55 %
• 65 %
7. A 45-year-old man presents to the emergency department with a history of coffee ground
vomiting. He also reports that for 2 days his stool has appeared darker than usual. Which of
the following gives the most sensitive guide as to the severity of his gastrointestinal
haemorrhage?
• Haemoglobin*
• Systolic blood pressure
• Pulse rate
• Volume of vomitus/melaena
• Lying and standing blood pressure
8. A 29-year-old patient presents with a short history of right upper quadrant pain. She is
jaundiced with dark urine and pale stool. She has a fever of 38.9 °C. Abdominal examination
gives no suggestion of a palpable gallbladder. The diagnosis is:
• Ascending cholangitis
• Acute cholecystitis
• Biliary colic with bile duct obstruction*
• Pancreatitis
• Mirizzi’s syndrome
9. A 22-year-old female patient presents with a 6-week history of bloody motions. She has
noticed significant weight loss over the preceding 6 weeks with increasing lethargy and
fatigue. She has previously had constipation and admits to regular laxative use. What is the
most likely diagnosis?
• Bowel cancer
• Irritable bowel syndrome
• Diverticular disease*
• Inflammatory bowel disease
• Anal fissure
10. Which one of the following does NOT occur as a systemic manifestation of inflammatory
bowel disease.
• Ankylosing spondylitis
• Pyodermagangrenosum
• Scleritis
• Osteoporosis*
• Sclerosing cholangitis
11. A patient with a previous anal abscess presents with persistent perianal discharge and
discomfort. On examination a sinus is identifiable at the 6 o’clock position with the patient in
the lithotomy position. What procedure the surgeon is most likely to perform?
• Seton insertion
• Colostomy
• Open exploration of tract*
• I&D
• Lateral Syphincterotomy
12. Which of the following is NOT an option when treating an anal fissure?
• Propranolol*
• Botulinum toxin A injections
• Glyceryltrinitrate cream
• Diltiazem cream
• Laxatives
13. Which of the following is NOT a sign associated with acute appendicitis?
• Murphy’s sign*
• Cope sign
• Psoas sign
• Rovsing’s sign
• Pain on rectal examination
14. All of the following may cause a right iliac fossa mass that is palpable on abdominal
examination, except:
• Carcinoma caecum
• Ulcerative colitis*
• Tuberculosis
• Appendicitis
• Ovarian cancer
15. Which of the following is the most common site for colonic carcinoma?
• Rectum
• Sigmoid colon*
• Caecum and ascending colon
• Descending colon
• Transverse colon
16. The most common cause of acute intestinal bowel obstruction is:
• Adhesions*
• Intestinal parasites
• Volvulus
• Hernia
• Neoplasia
17. Which of the following histopathological features is NOT characteristic of Crohn’s disease?
• Rose thorn abscesses
• Cobble stoning
• Skip lesions
• Lead piping*
• Serosal involvement
18. A 26-year-old woman arrives at the emergency department with unbearable intense right iliac
fossa pain. Earlier that day, she was experiencing ‘on and off’ moderate pain in the umbilical
area which gradually moved over to the right iliac fossa. Associated symptoms include
anorexia, nausea and vomiting. On examination, the patient is pyrexial and there is rebound
tenderness and guarding over the right iliac fossa. A beta-human chorionic gonadotrophin test
is negative. What should you do next?
• Send the patient to the emergency operating theatre for an Appendicectomy*
• Alert the obstetrics and gynaecology team, suspecting that she may have a ruptured ectopic
pregnancy
• Manage the patient medically in the emergency department
• Order an ultrasound scan of the abdomen
• Send the patient for a plain film radiograph of the abdomen
19. A 45-year-old man is brought in with an acute onset of epigastric pain, nausea and severe
vomiting. The pain is worse on movement and is only relieved slightly by leaning forward. The
patient is an alcoholic and has been admitted to the emergency department on several
occasions for alcohol intoxication. On examination the patient is tachycardic, pyrexial and
dehydrated. The abdomen is diffusely tender and soft, and bowel sounds are normal. The
patient’s serum amylase is raised by six times the upper limit of normal. The most likely
diagnosis is:
• Perforated peptic ulcer
• Small bowel obstruction
• Acute cholecystitis
• Acute pancreatitis*
• Acute appendicitis
20. A 78-year-old man presents to the emergency department with severe pain arising from his
hernia in the left groin. The patient is also experiencing central colicky abdominal pain. On
examination, the abdomen is generally tender, distended and bowel sounds are raised.
Examination of the hernial orifices reveals a left-sided, irreducible, tense and extremely
tender inguinal hernia. The overlying skin of the hernia is warm and erythematous. What is
the most appropriate course of action in managing this patient?
• Alert theatre and send patient for emergency surgery*
• Request a computed tomography scan of the abdomen
• Request an ultrasound
• Attempt to reduce the hernia
• Observation for 24 hours
21. You are asked to see a 48-year-old woman who has been admitted to the emergency
department with sudden onset of right upper quadrant pain. Your registrar tells you, after
having seen this patient, that the patient has ‘Charcot’s triad’. From the information conveyed
to you, what is the most likely diagnosis
• Biliary colic
• Acute cholecystitis
• Cholangitis*
• Gallstone ileus
• Pancreatitis
22. A 28-year-old man with a 10-year history of ulcerative colitis presents to the emergency
department with an acute severe episode of abdominal pain, nausea and vomiting and blood-
stained, watery diarrhoea. On examination the patient has fever, is tachycardic and the
abdomen is markedly distended. An abdominal plain film radiograph shows that the
transverse colon is dilated at approximately 6.5 cm. What is the most likely diagnosis?
• Large bowel obstruction
• Toxic megacolon*
• Perforated diverticulitis
• Crohn’s colitis
• Sigmoid Volvulus
23. As part of the peripheral vascular examination, you are asked to record the ankle–brachial
pressure index of the patient. Which one of the following values reflects a normal ankle–
brachial pressure index?
• Between 0.9 and 0.6
• Greater than 1.3
• Between 0.6 and 0.3
• Greater or equal to 1.0*
• Less or equal to 0.3
24. A 60-year-old man is diagnosed as having claudication of the lower limbs. The patient is a
smoker and has hyperlipidaemia for which he is taking a‘statin’. You are asked to discuss with
the patient the treatment options available to her. From the list below, choose the best
treatment option for this patient.
• Angioplasty
• Amputation
• Thrombectomy
• Increasing exercise and quitting smoking*
• Continue with the cholesterol-lowering medication and follow up in outpatients in 3 months
25. A 65-year-old man presents for the first time to your clinic with a painless wound in his right
leg, which has been present for over 2 months. On examination you notice a 3 cm leg ulcer in
the gaiter area of the right leg, above the medial malleolus. The shallow bed of the ulcer is
covered with granulation tissue, which is surrounded by sloping edges. There is no history of
trauma. From the list below, choose the most likely diagnosis.
• Arterial leg ulcer
• Neuropathic ulcer
• Venous ulcer*
• Traumatic ulcer
• Neoplastic ulcer
26. A 69-year-old diabetic man presents to the acute surgery unit with a 5- day history of mild
dysuria, frequency and feeling generally unwell. On examination, he is found to be pyrexial
and tachycardic. A genital examination reveals both the penis and the scrotum to be swollen,
red and tender to touch, with erythema also extending into the groin bilaterally and there is
palpable crepitus in the perineum. Routine bloods and cultures are taken(which later grow
both aerobic and anaerobic organisms), and fluid resuscitation and broad-spectrum antibiotics
are commenced. Following further discussion with a urologist, he is taken promptly into the
operating theatre for definitive management. The likely diagnosis in this patient is:
• Fournier’s gangrene*
• Epididymo-orchitis
• Testicular tumour
• Testicular torsion
• Prostatitis

YMDC & Other MCQs

Surgery A
1. A young man of 25 years sustained 30% burns on face and trunk. He is being resuscitated.
What is the best indicator for volume replacement:
• Blood pressure
• à Blood urca level
• C Heart rate
• d Mental alertness
• e Urinary output*
2. 2. Sebaceous cyst has a:
• a Depression b Hair
• Punctum*
• d Shipping sign
• Stalk
3. Onychogryphosis is:
• Aa absent nail
• b A broken nail
• An infected ail
• d. An in growing nail
• C An overgrown curved nail*
4. A young motorcyclist had a lucid interval after head injury. What is the most probable
diagnosis?
• Dural sinus haematoma
• b) Extra-dural hacmatoma*
• C Intra-cerebral lactatoma
• d. Scalp hacmatorna
• e Sub-dural hacmatenta
5. A 15 years boy presented with unilateral dilated papil immediately after head injury. The
commonest cause is.
• 3. Anoxia
• b. Damage to optic nerves
• C. Damage to ins
• Fear
• Intracranial hypertension*
6. Grave's disease is:
• Primary thyrotoxicosis*
• b. Secondary thyrotoxicosis
• Tertiary thyrotoxicosis
• d. Thyrotoxicosis factitia
• Treated thyrotoxicosis
7. 7. A 40 years lady C/O pain during mastication, salivary acct calcu are suspected to cause this
symptom. Name the gland involve most commonly.
• Parotid gland
• b Submandibular gland*
• Submaxillary gland
• d Minor salivary glands
• Sublingual gland
8. 8. Communist surgical disease of parotid is:
• Calculus
• Carcinoma
• c. lymphadenitis
• d. Parolitis
• Pleomorphic adenoma*
9. 9. Submandibular gland duct is called:
• Charcot's duct
• Santorini's duct
• E Stensen's duct
• Wharton's duct*
• e Wirsung's duct
10. 10. Aphthous ulcer is found in:
• a Bladder
• b. Colon
• C Eye
• d Mouth*
• Stomach
11. 11. A 50 years man sustained facial burns with inhalatiocal injury. What is the 1.V fluid of
choice for resuscitation of this patient:
• Glucose 5%
• b. Glucose 10%
• Mannitol
• Ringer's lactate*
• Sodium chloride
12. 12. A young man with malignant hypertension is undergoing adrenal surgery. What is the
pharmacological agent of choice for Preoperative preparation of this patient?
• Alpha agonist
• (B) Alpha antagonist*
• Beta agonist
• Beta
• Diuretics
13. 13. A middle age women sustained poly trauma in an RTA Earliest siga of blood loss is:
• Cyanosis
• Dyspnea
• Hypotension
• d. Pallor
• Tachycardia*
14. 14. Commonest site of origin of neuroblastoma is:
• a Adrenal cortex
• Adrenal medulla*
• Bladder
• d. Spinal cord
• e. Sympathetic chain
15. 15.A young girl of 25 years underwent sustatal thyroidectomy. She had post operative
bleeding. The emergency treatment required is:
• Blood transfusion
• b. Endotracheal intubation
• Opening up of wound and exploration*
• d. Pressure bandage
• Transamine injection
16. 16. Commonest solid malignant tumour in children is:
• Fibroblastoma
• Heramblastoma
• Nephroblastoma*
• Neuroblastoma
• Retinoblastoma
17. 17. A player and a pain and deformity of right shoulder joint during stretching exercise.
Commonest dislocation of shoulder joint suspected?
• Anterior*
• b Inferior
• Lateral
• d Posterior
• C Superior
18. 18. Minimum operation for solitary thyroid nodule is:
• Excision of nodule
• Subtotal thyroidectomy
• Total lobectomy*
• Total lobectomy plus isthmectomy
• Total thyroidectomy
19. 18. A 25 years male had bilateral abnormal swelling of lower limbs. Systemic examination was
unremarkable. He is suspected to have lymphoedema. Main stay of treatment in this patient
is:
• Chemo-radiation
• Conservative*
• Steroids Surgery
20. 21. The term plunging ranula refers to which clinical entity?
• A malignant congenital salivary mass arising from the submandibular gland
• A benign salivary mass involving the parotid submandibular glands
• A mucous retention cyst originating from the sublingual glands, limited by the mylohyoid
muscle
• A mucous retention cyst originating from the submandibular and sublingual glands which
perforates the mylohyoid muscle to enter the neck*
• A milline neck mass which moves on tongue protrusion.
21. 2. Regarding hypertrophic scar and keloid, Spot the wrong statement:
• Hypertrophic sear is common on fictor surface
• Keloid is often familial
• Hypertrophic scar outgrows wound area*
• Keloid is common on stemum, shoulder and face
• Keloid is treated by local injection of steroidi
22. 13. A 65 years diabetic male developed Fournier's gangrene of the scrotum. Which of the
following organism is not expected to involve:
• Clostridia*
• Bacteroides
• Coliform
• d Peptostreptococcus
• Staphylococcus
23. 24. Fogarty catheter is used for:
• Intravenous nutrition
• Ureteric catheterization
• Arteriography
• Urinary retention
• Removing arterial blood clots*
24. 25. Leriche's syndrome is:
• Seen in venous disorders
• Caused by aortoiliac occlusion*
• A disorder of peripheral nerves
• manifestation of spinal disease
• Due to femoropopliteal occlusion
25. 26. The Trendelenburg test is used to detect
• Deep vein thrombosis
• b. Integrity of femoral vein
• Incompetence of valves in the spheno femoral junction*
• Venous ulcer
• Varicocele
26. 27. A bed ridden patient suddenly developed chest pain dyspnoca. The best investigation to
rule out Pelosary embolism:
• Chest X-ray
• b. Ultrasound of Chest
• C MRI
• d. Doppler
• CCT angiography*
27. 28. A young child presents with pain in her shoulder. There is no history of trauma. She
screams if the arm is moved. She is febrile and toxic. What is the likely disgamis?
• a Gout
• b. Irritable joint
• Septic arthritis*
• A fracture
• E A dislocation.
28. 29. A 30 years male sustained trauma to his right arm in an RTA. He had fracture humerus and
wrist dropped. What is your diagnosis?
• Fracture proximal third Fracture of middle third
• Spiral fracture of distal third*
• d. Inter condylar fracture
• Fracture of surgical neck of humerus
29. 30. A factory worker had wound on the dorsum of left hand. He is having blisters, foul smell
and discoloration. The
• causative organism is:
• Clostridium welchii (Perfringens) *
• b. Streptococcus
• C Staphylococcus
• Proteus
30. 31. Bacteroides are not part of normal flora in the following:
• Oropharynx
• Urinary tract
• Colon
• Vagina
• Billary passages*
• 32. Claudication is:
• Pain at rest
• Pain relieved by rest
• Constant pain C
• d. Pain not relieved by rest*
• Pain aggravated by rest
31. 33. A middle age man is having leg ulcer with sleping edges. What is your diagnosis?
• Septic ulcer*
• Syphilitic ulcer
• Carcinomatous ulcer
• Tubercular ulcer
• Venous Ulcer
32. 17. What is not true about keloid scar:
• Is an extensive overgrowth of scar tissue
• b. Common is Africans
• C Is a benign conition
• d. May occur on the car lobes
• e is the same as a hypertrophic seat*
33. What is the most common malignancy encountered in the oropharynx?
• A Adenocarcinoma
• Squamous cell carcinoma (SCC) *
• Adenoid cystic carcinoma
• Non hodgkin’s lymphoma
• Saliviry gland umours.
34. 39. Type of hemorrhage during surgical operations
• b Primary haemorrhage*
• Reactionary haemorrhage
• Secondory haemorrhage
• External haemorrhage
• Internal haemorrhage
35. 6. A 65 years mate sustained poly trauma in an accident. Central Venous Catheter (CVC) has
been placed for optimal volume replacement. Normal central venous pressure is:
• 0-5cm saline
• 5-8cm saline*
• 8-12cm saline
• 12-15cm saline
• 15-20cm saline
36. 41. A 45 years male had blunt trauma to the central part of abdomen abdominal sonogram is
performed to rule out injuries other than:
• Cardiac tamponade
• b. Hepatic Injury
• Splenic injury
• Renal injury*
• Mesenteric vascular injury
37. 42. A patient had developed Compartment syndrome after pol trauma. It is most commonly
in:
• Upper arm
• b. Fore arm*
• Thigh
• Foot
• C Leg
38. 46. Which of the following statements is false?
• a The mechanism of injury and the injury produced are the keystones in management
• b. It is as essential to identify overt (obvious) injuries as it is to identify the covert (hidden]
injury.
• Penetrating injuries usually involve the use of weapons
• d. Blum injuries are the outcome of acceleration/deceleration, such as falls or road traffic
accidents (RTA)
• Knife Injury over a limb is casy to Evaluate*
39. 47. During axillary clearance of stage Il carcinoma breast long thoracic nerve was rejected.
Which muscle is going to be affected?
• Deltoid
• Brachioradialis
• E Serratus anterior*
• Pectoris major
• c Pectoris minor
40. 48. Which of the following statements regarding blat ma is not true? trauma is not true?
• The mechanisms are direct or indirect.
• In indirect injury, associated injuries may be present and should be sought.
• Overt injury should lead the clinician to look for covert injury as well.
• Proper exposure is essential so as not to miss other injuries.
• In chest injuries damage to abdominal organs is rare*
41. Chronic parctitis in children is pathognomonic of chat disease?
• HIV*
• Sialolithiasis
• C Bacterial porotitis
• d. Tuberculous parotitis
• e. granulomatosis
42. 50. The primary growth commonly cervical lymph nodes:
• Nasopharyngeal cancer
• not merasizsizing to
• Cancer in sinuses, laryngopharynx and vallecala
• Breast Cancer*
• d. Thyroid cancer
• Tongue cancer
43. The axillary lymph nodes receive what percentage of lymph from breast
• 50%
• 75%*
• 90%
• 100%
44. 2. What is the most common site for a parotid tamour?
• At the anterior harder of the masseter
• Inferior to the angle of the mandible
• As a parapharyngeal mass
• Anterior to the ear *
• Behind the angle of the mandibles
45. 55. Mate breast cancer is due to excess of:
• Androgen
• Estrogen*
• Both Androgen del Estrogen
• d. Not related to hormones
• Progesterone
46. Ceft palate repair is ideal at:
• 6 months
• b 6-18 months*
• 12-14 months
• 2-4 years
• 6-8 years
47. 55. Submandibular gland is not related to following structure
• Inguinal nerve*
• Hypoglossal nerve
• Facial artery
• Facial nerve
48. Characteristics of cystic hygroma does not include:
• Develops from jugular lymphatic
• Biliary translucent
• Typically occupies the middle third of neck*
• d Enlarges when the child cries
• Is multi locular
49. 57. What is the most significant disadvantage of the laryngeal mass airway (LMA) over an
endotracheal tube?
• Failure to provide a competent airway
• di Risk of pulmonary aspiration*
• C Unreliable placement
• Enhanced risk of tube obstruction
• Failure to allow tracheal suction
50. 58 A 70-year-old female falls and sustains a fractured neck of lemur. She is admitted to
hospital at 7.30pm. She is a brown. hypertensive who is on diuretics and lives alone. You find
her blood pressure is 160/100 mm1g and her pulse is 70/min. Stie is fully alert and orientated
and has no other injuries. The best approach should be;
• Set her up for operation as soon as possible that evening
• Wait until the blood results are l and then discuss the situation with the anesthetist planning
to optimize her for surgery on the morning list,
• Leave all planning until the morning as it is now out of house
• 4. Insist that you operate that night, however long you have to wait
• 2 Apply a Cramer wire splint with 5kg weight on sliding traction*
51. 59. Which of the following is not a coplication submandibutor gland excision?
• Frey's syndrome*
• Anestesia of the insilateral tongue
• Weakness of the corner of the mouth
• d Anestesia of sb mental skin
• Paralysis of the insilateral tongue
52. 60. What is the most reliable way to ascertain tom placement of an endotracheal tube?
• Detection of a pressure waveform on inflation
• b. Direct visualization
• Detection of breath sounds on auscultation*
• Mensurement of end-sical carbon dioxide concentration
• c. Movement of the chest wall on manual Inflation
53. 61. Lidocaine can be injected intravenously, but what is the main reason why hupivacaine
should not be injected into anaesthesia?
• a vein during local
• It lasts longer.
• It is often used with adrenaline.
• It can cause methemoglobinemia.
• It may cause convulsions
• re. It is cardiotoxic*
54. 52. A Bier's block is a useful technique to provide anaesthesia for upper limb surgery. Which is
the best local anaesthetic to use?
• Prilocaine*
• b. Lidocaine
• Levobupivacaine
• Bupivacaine
• Amethocrine
55. Patients vary greatly in their requirement postoperative analgesia. What is the best way
adequacy of pain relief?
• Measure the degree of tachycardia
• Ask the patient to measure the pain*
• Assess the level of hypertension.
• Look for tachypnea.
• Examine for wound splinting.
56. The thyroid stimulating antibody is:
• IgG*
• IgM
• IgA
• IgD
• IgE
57. 65. Which of the following statements regarding fine-needle aspiration cytology (FNAC) in
oropharyngeal cancer is true?
• It causes seeding of disease to the skin*.
• it provides reliable results independent of the operator
• It is used for diagnosis of primary disease.
• It is used in assessment of enlarged cervical lymph nodes.
• It requires adequate fixation for interpretation.
58. A sebaceous cyst has a
• a Depression
• b. Hait
• Punctum*
• d.. Slipping sign
• Stalk
59. 2. In a burn patient, what is the best indicator for volume replacement is:
• a Blood pressure
• b Blood urea level
• Heart caté
• d. Mental alermess
• e. Urinary output*
60. 3. Onychogryphosis is:
• D. An absent nail
• b A broken nail e. An infected nail
• d An ingrowing nail
• An overgrown curved nail*
61. 4. Lucid interval in a patient after a head injury typically is present in:
• Dural sinus haematoma
• b Extra-duralhematoma*
• N Intra-cerebral hematoma
• Scalp locitatoma
• e Sub-duralhematona
62. A dilated pupil immediately after head injury is due to:
• Anoxia
• Damage to optic nerves
• b C Damage to iris
• d. Fear
• e. Intra cranial hypertension*
63. 7. Aphthous ulcer is found in:
• Bladder
• h Colon
• Eye Mouth*
• C Stomach
64. 8. For which of the following organ injuries diagnostic peritoneal lavage (DPL) is least likely to
be helpful:
• Small intestine
• b. Sigmoid colon
• C Pancreas
• d. Spleen
• Stomach*
65. 9. Which is the most common intra-abdominal organ injured with penetrating trauma?
• Colon
• b. Liver
• Small gut*
• 4 Spleen
• e. Stomach
66. 10. LV laid of choice for resuscitation of burn patient is:
• Glucose 5%
• Glucose 10%
• Mannitol
• d. Ringer's lactate*
• e. Sodium chloride
67. 11. Earliest sign of blood loss is:
• Cyanosas
• Dyspnica
• Hypotension
• d. Pallor
• c. Tachycardia*
68. 12. Main stay of treatment in lymphoedema is
• Chemo-radiation
• b. Conservative*
• c. Radiotherapy
• d. Steroids
• e. Surgery
69. 13. Brodie's abscess involves
• a Bone*
• b. Brain
• Breast
• d. Liver
• (e. Lung
70. 14. The commonest bone tumour is:
• Chondroma*
• b. Fibroma
• c. Metastatis
• d: Osteoms
• e Sarcoma
71. 15. Olecranon bursitis is also known is:
• a Clergyman's elbow
• b. Cricket elbow
• Goller's elbow
• Student's elbow*
• e. Tennis elbow
72. A young man with automobile accident has; pulse rate of 110/min, respiratory rate 25 breaths
min and blood pressure 120/95 mm Ilg. He has left haemothorax with rib fracture. Immediate
treatment is:
• Bilateral tube thoracostonty
• Intercostal block and strapping*
• Left tube thoracostomy
• Endotracheal intubation and pleural aspiration
• Immediate thoracolomy
73. Commonest complication after spinal anaesthesia is:
• Backache
• Bleeding
• Headache*
• Meningitis
• Urinary retention
74. The initial maneuver to establish an airway with multiple injuries is:
• a Oropharyngeal airway
• b Nasopharyngeal airway
• Suctioning foreign debris and jaw lift*
• Endotracheal tube
• Tracheostomy
75. In burn patients, which one of the following features is indicative of inhalational injury?
• 4. Carbonaceous sputum*
• Perineal burns
• C History of outdoor burning environment
• Explosion injury with burns involving lower limbs
• Electric burns involving back
76. Select the most appropriate initial In the step management of a patient with multiple injuries;
• 4. Splinting of fractures
• b Control of external hemorrhage
• Relief of tension pneumothorax
• d. IV fluid treatment of shock
• e Maintenance of airway*
77. 22. During the process of wound healing, collagen is laid down by:
• Eosinophils
• b Basophils
• Fibroblasts*
• d Neutrophils
• Platelets
78. 23. Which of the following studies is most helpful in status for a evaluating a patient's
Pulmonary status for a Thoracic surgical procedure?
• ECG
• b Chest X-ray
• History alone
• d Lung functions test*
• ETT
79. 24. Regarding wound healing:
• a It is characterized by decreased in vascular permeability
• b Type I collagen is formed first
• C It is retarted by vitamin A excess
• d. Macrophage plays a central role*
• It is poor in children
80. 25. Commonest site of GANGLION is:
• Back
• Foot a.
• Neck
• Scalp
• Wrist*
81. 26. Atropine injection is used during anesthesiain having:
• a Bradycardia*
• b. Cyanosis
• Hypertension
• Hypotension
• Tachycardia
82. 27. Intravenous fluid used in shock is:
• a Ringer's lactate
• b Sodium chloride*
• e 5th Glucose
• d. 10%-Glucose
• c. Mannitol
83. 28. During anesthesia, Pentothal injection is used for:
• a Induction*
• b. Maintenance
• C Reversal
• d. Relaxation
• Analgesia
84. 29. Commonest site for venesection is:
• Ankle*
• b Arm
• Neck
• d. Scalp
• e. Thigh
85. 30. Post tonsillectomy position is:
• a Head up
• b. Lateral*
• C Leg up
• d. Prone
• e. Straight
86. 31. Lignocaine with Adrenalin is contraindicated in low anesthesia of which of the fallowing:
• a Finger*
• b. Leg
• c. Scalp
• d. Back
• e Thigh
87. 32. Test dose is given
• Intra-muscular
• Intra-osseous
• Intra-thecal
• Intra-venous
• Sub-cutaneous*
88. 33. Thomas splint is used for fracture of:
• Lower limb*
• Spine
• Upper limb
• Skull
• Rib
89. 34. forceps is used mostly during:
• Appendectomy
• Cholecystectomy
• C Hernia repair
• Incision and drainage
• Hysterectomy*
90. 35. Colour of nitrous oxide cylinder is:
• a Blue*
• b. Green
• Red
• White
• Yellow
91. 36. Colour of oxygen cylinder is
• Blue
• Green
• K Rest
• White*
• Yellow
92. 37. ERCP is not successful in a patient having:
• Duodenal obstruction*
• Jejunal obstruction
• Seal ohansction
• Colonic obstruction
• Rectal obstruction
93. A trauma patient should not be sent for CT scan if he is having:
• Bradycardia
• Fever
• Hypertension
• Hypotension*
• Tachycardia
94. 40. Intravenous Urography is not done if a patient is has ing
• Tachycardia
• Jaundice
• Renal failure*
• Haematuria
• Cough
95. 41. In laparoscopy surgery, gas used for pneumoperitoneum is
• a Carbon dioxide*
• b. Helium
• c. Hydrogen
• d. Nitrous oxide
• C. Oxygen
96. 42. Atropine injection should not be used if patient is having
• Bleeding
• b Diabetes mellitus
• Ć Cyanosis
• d. Fever*
• Hypertension
97. 43. Non-cuffed endo-tracheal tube is used in
• Adult
• Adolesence a.
• Children*
• Elderly
• e. Pregnant female
98. 44. Position for thyroidectomy is
• a Head-up*
• b. Lateral
• Lithotomy
• d. Prone
• c. Supine
99. Fluid used in burns is
• Ringer's lactate*
• Sodium chloride
• C 5 Glucose
• 10-Glucose
• Mannitol
100. Intravenous fluid for children is &
• Half strength saline with glucose*
• Hypertonic saline
• Hypertonic glucose
• Normal saline
• 5% dextrose
101. 47. Which blood group is the universal recipient
• A
• AB*
• B
• positive
• negative
102. 48. LMA is used by
• Aesthetist *
• Gynaecylogist
• ophthalmologist
• J Surgeon
• Urologist
103. 49. Which of the follaning suture is molilausent:
• a Catgut*
• h Cotton
• Prolene*
• A and C
• Silk
• vicryl
104. 50. Painless delivery is done by which of the following?
• Epidural anaesthesia*
• Ketamine injection
• Local anaesthesia
• Morphine injection
• Spinal anesthess
105. 51. What is the Standard temperature for sterifiration in autoclave in Celsius?
• 100
• 111
• 121*
• 131
• 141
106. 52. Double ‘J’ stent is normally used by
• Cardiologist
• b. Gastroenterologist
• C. Neurosurgeon
• d. Pulmonologist
• e Urologist*
107. 53. 5-Fluorouracil is a chemotherapeutic agent of choice for:
• a Brain tumour
• b. Colorectal carcinoma*
• Lymphoma
• d. Osteogenic sarcoma
• Seminoma
108. 54. Which one of the following is most radiosensitive tumour?
• Seminoma*
• Squansous cell carcinoma
• Gastric carcinoma
• d. Germ cell tumour
• e Leiomyosarcoma
109. 55. Earliest source of glycogen during starvation is:
• B Blood
• b. Bone
• Liver*
• d. Muscle
• e. Spleen
110. 5. Average daily requirement of potassium in an adult is:
• 30 mM
• 60mM*
• 70mM
111. 57. Which one of the following gastrointestinal secretion has the maximum HCO,
content:
• Saliva
• Gastric
• C Bile
• d Pancreatic*
• d Heal
112. 58. Commonest presentation of arterial disease is:
• Acute limb ischemia
• Critical limb ischemia
• Gangrene.
• d Intermittent claudication*
• Venous thrombosis
113. 59. Commonest cause of peripheral vascular disease is:
• Atherosclerosis*
• Arteritis
• Congenital
• d Embolism
• Trauma
114. 60. Commonest type of lymphedema is:
• Primary
• Secondary*
• Tertiary
• Idiopathic
• Congenital
115. 61. Which of the following is a retroperitoneal structure:
• Liver
• Spleen
• Small gut
• Stomach
• Pancreas*
116. 62. Commonest site of venous ulcer is:
• Broca's area
• Critical area
• Dancer's area
• Gaiter's area*
• Waiter's area
117. 63. Lipodermatosclerosis is a feature of:
• Arterial disease
• Cardiac disease
• Hyperlipidemia
• Lymphatic disease
• Venous disease*
118. Most common cause of leg ulcer
• Arterial
• Fungal
• Malignant
• Traumatic
• Venous*
119. Slipping sign is a feature of
• Ganglion
• Lipoma*
• Melanoma
• Sebaceous cyst
• Thyroglossal cyst

Surgery A MCQs POOL


1. While assessing a patient, the physician should observe all of the following. Mark incorrect
statement
• ITEM A Do not make the patient completely naked during clinical examination
• ITEM B Take specific permission before doing a socially sensitive investigation, such as a test
for HIV
• ITEM C If rectal examination is unlikely to contribute to the management, do not do it
• ITEM D If examination of particular area causes pain, do not examine the area
• ITEM E
• Correct Answer: D
2. STEM: The following is true for pre and intra-operative anesthesia management
• ITEM A Pre-anesthetic complete blood counts are essential even for medically fit patients
about to undergo minor surgery
• ITEM B Recent myocardial infarction is not by itself a contraindication for elective anesthesia
• ITEM C Hepatitis does not affect recovery from anesthesia
• ITEM D Bipolar diathermy is safer than monopolar for patients on pacemaker
• ITEM E
• Correct Answer: D
3. STEM: In the peri-operative management of diabetic patients
• ITEM A Insulin control is preferred even in non-insulin dependent diabetics
• ITEM B Sodium ion derangements are commoner than potassium ion changes
• ITEM C Long-acting insulin is better than short acting insulin
• ITEM D Blood sugar is best measured once daily
• ITEM E
• Correct Answer: A
4. STEM: Safe dosage limits for local anesthetic are
• ITEM A Lignocaine: 40 ml of 2%
• ITEM B Bupivacaine: 40 ml of 1%
• ITEM C Ropivacaine: 40 ml of 1%
• ITEM D Prilocaine: 40 ml of 1%
• ITEM E
• Correct Answer: D
5. STEM: The following are all features of Hypokalemia. Select the incorrect statement?
• ITEM A Prolonged OT, depressed ST, flattening of T on the ECG
• ITEM B Hyperkinetic movements or tremors
• ITEM C Cardiac arrhythmias
• ITEM D Abdominal distension
• ITEM E
• Correct Answer: B
• Description Acute Life Support and Critical Care
6. STEM: Acid base changes are often associated with electrolyte disturbances. Consider the
statements:1. Alkalosis lowers free Ca++ in the serum; 2. hypokalemia worsens alkalosis.
Which of the following comments is correct about these two statements?
• ITEM A Statement 1 is true, statement 2 is false
• ITEM B Statement 2 is true, statement 1 is false
• ITEM C Statement 1 and 2 are both true
• ITEM D Statements 1 and 2 are both false
• ITEM E
• Correct Answer: C
• Description Acute Life Support and Critical Care
7. STEM: Three hours after thyroidectomy, a patient begins to bleed from the
• operated site. This form of bleeding is called
• ITEM A Primary hemorrhage
• ITEM B Reactionary hemorrhage
• ITEM C Secondary hemorrhage
• ITEM D Tertiary hemorrhage
• ITEM E
• Correct Answer: B
8. STEM: A clot the size of a clenched fist contains about
• ITEM A 100-200 ml of blood
• ITEM B 250-300 ml of blood
• ITEM C 350-400 ml of blood
• ITEM D 450-500 ml of blood
• ITEM E 350-500 ml of blood
• Correct Answer: D
9. STEM: A bag of blood for transfusion usually contains about
• ITEM A 200 ml of blood
• ITEM B 250 ml of blood
• ITEM C 300 ml of blood
• ITEM D 350 ml of blood
• ITEM E 400 ml of blood
• Correct Answer: D
• Description Acute Life Support and Critical Care
10. STEM: Thirty minutes after starting a blood transfusion a patient develops mild tachycardia
and fever. There are no other complaints. On examination the blood pressure is normal, there
is no rash, and there are no other abnormal findings. The most likely cause of this
• tachycardia and fever is
• ITEM A Mismatched transfusion due to ABO
• ITEM B Allergic reaction
• ITEM C Transmission of malaria
• ITEM D Pyrexial reaction
• ITEM E
• Correct Answer:
• Description Acute Life Support and Critical Care
11. STEM: Following trauma, a patient receives several units of stored blood, and then begins to
bleed again from his wounds, investigations confirm a coagulation defect. Which of the
following has NO role in the management of his condition;
• ITEM A Calcium
• ITEM B Platelets
• ITEM C Heparin
• ITEM D Steroids
• ITEM E
• Correct Answer: D
• Description Nutrition
12. STEM: Which of the following hormones will show a significant rise after a
• fast?
• ITEM A Insulin
• ITEM B Aldosterone
• ITEM C Glucagon
• ITEM D TSH
• ITEM E
• Correct Answer: C
13. STEM: In severe sepsis and injury the hormonal changes create all of the
• following effects. Mark the incorrect statements
• ITEM A Anabolism
• ITEM B Lipid mobilization
• ITEM C Utilization of amino acids for energy
• ITEM D Increased metabolic rate
• ITEM E Decreased metabolic rate
• Correct Answer: A
14. STEM: A 45 kg lady with gallstone pancreatitis is treated conservatively, with bowel rest,
painkillers and antibiotics. After 5 days of intravenous fluids, the treating physician decides to
begin total parenteral nutrition. Which of the following plans will be most suitable as daily
therapy?
• ITEM A 50 G intralipid, 20 G of albunin, 2000 ml of normal saline via a peripheral vein
• ITEM B 50 G intralipid, 20 G of albumin, 100 G dextrose in 2000 ml of normal saline via
peripheral vein
• ITEM C 50 G intralipid, 60 G of amono acid, 100 G dextrose in 2500 ml of normal saline via a
central venous line
• ITEM D 60 G of amino acid, 500 G of dextrose in 2500 ml of fluid via a central venous line
• ITEM E 50 G intralipid, 30 G of albunin, 2000 ml of normal saline via a peripheral vein
• Correct Answer: D
15. STEM: A 65 year old laborer has marked dysphagia due to a carcinoma of the middle third of
the esophagus. He requires radiotherapy, but is nutritionally unfit for treatment.
Remembering that nutritional support will be required for least two weeks, which is the
optional method of improving his nutritional status prior to radiotherapy?
• ITEM A Total parenteral nutrition through a central line
• ITEM B Oral liquid elemental diet that he may be able to swallow and digest easily
• ITEM C Oral elemental diet given via a nasogastric tube passed through the tumor
• ITEM D Enteral feed via a feeding jejunostomy fashioned operatively
• ITEM E
• Correct Answer: D
16. STEM: Which of the following conditions is LEAST likely to require total parenteral nutrition?
• ITEM A Head injury with coma
• ITEM B Pancreatitis
• ITEM C Small residual intestinal length after surgery
• ITEM D High output ileal fistula
• ITEM E
• Correct Answer: A
17. STEM: A patient suffers a deep, 6 cm wide thigh abrasion, which becomes infected. When he
comes to hospital, the surgeon dresses the wound, planning to allow for healing by secondary
intention. The resultant scar is likely to be
• ITEM A Thin, and easily broken
• ITEM B Thick and vascular
• ITEM C Thick vascular, and resistant to trauma
• ITEM D Of normal thickness, but without senstation
• ITEM E
• Correct Answer: A
18. STEM: A 40 year old diabetic sustains an injury near the anus. Examination reveals marked
local edema, tenderrence and crepitus. A plain X-Ray of the area shows gas in the tissues.
Mark the correct statement?
• ITEM A The condition is likely to be caused by a gram negative bacterium that releases a
potent hemolysin
• ITEM B Most of such patients will have minimal systemic features despite marked local
manifestations
• ITEM C Extensive surgical debridement is required, while hyperbaric oxygen is of little use
• ITEM D Fluoroquinolones and aminoglycosides are effective antibacterials for this condition
• Correct Answer: C
19. STEM: In a wound abscess, the commonest bacterium found is
• ITEM A Staph aureus
• ITEM B Strep pyogenes
• ITEM C Pseudomonas aeruginosa
• ITEM D Bacteriodes
• ITEM E
• Correct Answer: A
20. STEM: A patient after surgery develops a swelling in the wound associated with redness,
indurations and marked pain. The most important step in further management is
• ITEM A Increase the dose of antibiotics
• ITEM B Warm water fomentations
• ITEM C Painkillers and reassurance
• ITEM D Drain the pus
• ITEM E
• Correct Answer: D
21. STEM: A technician in a hospital decontaminates equipment as follows Trolleys in an ethylene
oxide chamber Proctosocpes by thorough soap-water washing Flexible endoscopes by
immersion in fresh 2% glutaraldehyde iv.Scissors for general surgery by autocalave Which of
the following correctly describes his techniques?
• ITEM A “A” is a ppropriate but ‘B’ is insufficient
• ITEM B ‘B’ and ‘C’ are both appropriate in terms of their intended usage
• ITEM C ‘C’ is standard practice
• ITEM D Scissors should be sterilized by flaming but not autoclaving
• Correct Answer: C
22. STEM: Which organism requires the highest temperatures for inactivation?
• ITEM A Vegetative forms of E histolytica
• ITEM B M tuberculosis
• ITEM C HIV
• ITEM D Clostridium tetani spores
• ITEM E
• Correct Answer: D

BAHRIA UNIVERSITY ISLAMABAD

MULTIPLE CHOICE QUESTIONS

SURGERY-A MCQs
1. In a case of confirmed carpal tunnel syndrome, introduction of a steroid in the tunnel is
helpful. In the access to the median nerve at the wrist where should the operator infiltrate the
steroid so the median nerve can be targeted?
• On the dorsum of the wrist
• On the palmar aspect of t he wrist
• On the medical side of the Palmaris longus tendon
• Once the lateral side of the Palmaris longus tendon
• Over the radial styloid.
2. During an operative procedure on the thyroid gland, which nerve other than the recurrent
laryngeal can be damaged to produce hoarseness?
• External laryngeal.
• Inferior subdivision of external laryngeal.
• Superior subdivision of external laryngeal
• Sympathetic trunk
• The phrenic nerve
3. In considering the atherosclerotic changes in ischaemic heart disease, which cardiac artery is
of particular significance as far as prognosis is concerned?
• Right coronary artery
• Left coronary artery
• Left anterior descending
• Circumflex branch of left coronary
• Marginal branch of right coronary
4. A burglar, anticipating a fast get away smashes through a glass window and in the process
gashes the anterior aspect of his right wrist. The trauma surgeon in the A & E notes anesthesia
of the radial three and half digits What is the most likely structure involved?
• Trauma to the radial artery
• Trauma to the Palmaris longus tendon
• Trauma to the flexor retinaculum
• Trauma to the ulnar artery
• Trauma to the median nerve
5. A technician working on electrical equipment touches a live wire. After recovery from the
initial shock, he notices some redness and pain on the palmar aspect of his right hand. After
an uncomfortable night he presents to the hospital where the trauma surgeon notes a brown
discoloration in a band like area of the palmar aspect of his right hand. What is the best
course of action?
• Plan a serial wound excision
• Review patient after 48 hours
• Do a superficial wound excision
• Do a wound excision and apply immediate skin graft.
• Do partial wound excision under GA and tourniquet
6. A 24 year old fireman is trapped under falling masonry in a fire incident. Colleague’s quickly
release him but he complains of pain ink his right leg which bore the brunt during the rapping
incident. On external examination there is no obvious dorsiflexion of the foot causes severe
discomfort in the calf. The most likely cause of its symptoms is:
• Compartment syndrome
• Undisplaced fracture of tibia
• Undisplaced fracture of tibia and fibula
• Deep venous thrombosis
• Large artery thrombosis
7. A 40 year old motorist is involved in a road traffic accident. In the A & E he is noted to have
shortness of breath, with pain in the right side of the chest. O/E the trachea is in the midline
with diminished air entry at the base, of the right chest. An x-ray chest confirms a fluid
collection. A I/C tube drains 350 ml of blood. The most likely cause of bleeding in this injury is
from:
• Aorta
• SVC
• IVC
• Right atrium
• Intercostal vessels
8. In a bungled burglary attempt a security guard receives a single gunshot injury to the medial
aspect of his right thigh. In the ER, he is noted to have a large heamatoma in the medial side
of the thigh. He complains of paraesthesia in his foot. The most appropriate initial
management for this patient is
• Angiography of both lower limbs
• Exploration and repair in the operating room
• Fasciotomy of the anterior compartment of the calf
• Observation for resolution of spasm
• Local wound exploration at bed side.
9. In the management of hyponatraemia following inappropriate administration of low sodium
or sodium free solution, which of the following options is an appropriate management
strategy?
• Plasma ultra filtration
• Aggressive diuresis with frusemide
• Restriction of free water
• Haemodialysis
10. Administration of hypertonic sodium solution. Of all the total body water of a 70 kg male,
which value represents the closest approximation under normal physiological conditions?
• 30 liters
• 34 liters
• 36 liters
• 40 liters
• 42 liters.
11. A 35 year old multipara from a poor background has a chronic discharging sinus in the lower
part of her Rt. Anterior triangle of her neck. OE the sinus has an opening about 3 mm x 4 mm
and the adjoining area blush stained. Underlying the sinus is a 2cm x 2.5cm firm mass. The
most likely diagnosis is:
• Brachial sinus
• Thyroglossal sinus
• Infected sebaceous cyst
• Chronic tuberculous sinus
• Implantation epidermoid
12. In the consensus regarding preoperative administration of prophylactic antibiotics, what is an
appropriate time for its administration?
• 9 hrs before surgery
• 4 hrs before surgery
• 2 hrs before surgery
• 1 hr before surgery
• 30 mins. Before surgery
13. A 30 year old female undergoes an elective cholecystectomy for symptomatic gall stones. In
the modern classification of wounds which of the following wound class best describes her
procedure?
• Clean
• Contaminated
• Clean-contaminated
• Dirty
• None of the above
14. A 35 year old female from a poor background presents with a small ulcer discharging mucoid/
cheesy material in her right anterior axillary fold. Underlying the ulcer are large matted
nodules suggestive of lymph nodes. All features suggest tuberculous ulcer EXCEPT
• The ulcer edge has a bluish margin
• The ulcer base shows no sign of healing
• The ulcer edge is undermined
• There is a mucoid discharge
• There are severe systemic symptoms
15. A 30 year old housewife sustains burns to nearly 40% of the lower half of her body. It takes
about 04 hours to get to a hospital. In A & E the attending doctors find her very restless, in
pain, with a pulse of 100/ml,BP of 100/70. Her clothes are soaked in a straw/pin fluid. Patients
Hb is 17.86 and PCV of 54. Laboratory abnormalities can be explained by:
• Full thickness (Deep) burns
• Pre-existing polycythema
• Predominantly superficial burns
• Laboratory error
• Exaggerated inflammatory response
16. In patients exposed to the Type 1 human immunodeficiency virus (HIV) all of the following are
common opportunistic infections EXCEPT
• Pneumocystis carineii pneuymonia
• Tuberculosis
• Cystomegaly virus infection
• Candida infection
• Hepatitis
17. A 35 year old banker is found to have hypertension which his treating physician is finding hard
to control. On investigation, the ostium of his left renal artery is found to be stenosed. What
are the treatment options?
• Dilatation and insertion of stent
• Dilatation of the stenosed segment
• Removal of the left kidney
• Open operation and endarterectomy
• Open operation and insertion of a prosthetic graft
18. A 16 year old female presents with a 1 cm diameter midline selling just below the hyoid bone.
It feels cystic, moves with deglutition and rises on protrusion of the tongue. The most likely
diagnosis is
• Lipoma
• Thyroglossal cyst
• Bronchial cyst
• Sebaceous cyst
• Implantation epidermoid
19. A 35 years old teacher, a known patient of heart disease, fell down in the washroom. After
sometime he noticed that he could move the left lower limb with great difficulty. In the
hospital his examination revealed a pale liking left leg with no sensations or palpable pulses
below the inguinal ligament. Based on these findings the diagnosis in this patient is:
• Acute arterial thrombosis
• Acute venous obstruction
• Acute arterial embolism
• Acute slipped intervertebral disc
• Fractured neck of femur
20. A 65 year old male presents with a 6 months history of a swelling in his right inguinal region.
He has not been well for sometime and on examination a right direct reducible inguinal hernia
is diagnosed. What are our treatment options?
• Put on waiting list for hernia repair.
• Arrange for a comprehensive assessment of all systems.
• Arrange for x-ray chest with ECG prior to repair.
• Do aerodynamic study prior to repair.
• Reassure patient and defer treatment until symptom appears.
21. A 65 year man presents with acute onset of pain, swelling and erythema of the left knee. He
denies previous episodes or trauma to the knee. The differential diagnosis includes septic
arthritis and gout. Which of the following is the best study to differentiate between gout and
septic arthritis?
• Evaluation of synovial fluid aspirate.
• White blood cell count.
• X-ray of the knee.
• Magnetic resonance imaging (MRI) of the knee.
• Bone scan.
22. A 24 year old takes a heavy fall in a biking accident. He sustains an injury to the right thigh. X-
ray of right thigh shows a transverse fracture of his right femoral shaft. What are our
treatment options?
• Apply skin traction to equalize limb length
• Apply skeletal traction to equalize limb
• Insert a plate and screws
• Insert a interlocking intra-medullary nail
• Insert an external fixator.
23. A 28 year old house wife presents with six months history of ill health, loss of weight,
occasional pyrexia and back ache. 0/E the spine of the 2nd Lumbar vertebra is very prominent.
A x-ray reveals some anterior collapse of the body of the L2 and erosion of adjoining discs.
What are the possible causes of the patient's symptoms?
• Multiple myeloma.
• Secondary deposit.
• Traumatic collapse.
• Leukaema.
• TB spine.
24. A 58 year old female who had partial gastrectomy for duodenal ulcer 12 years ago presents to
you with generalized bone pains and bone tenderness. Radiological exam of pelvis shows
pseudo fractures. Repeated serum calcium levels are normal. These symptoms are due to
which of the following?
• Osteomalacia
• Vitamin B 12 deficiency
• Bile salt mal-absorption
• Hyperparathyroidism
• Small stomach syndrome
25. A 50 years old man is admitted with haematemesis. Upper GI endoscopy reveals a bleeding
vessel in an ulcer in the posterior wall of first part of duodenum. The artery which needs to be
under run for controlling bleeding is
• j. Right gastric artery
• k. Right gastro-epiploic artery
• l. Superior pancreatico duodenal artery
• m. Gastroduodenal artery
• n. Superior mesenteric artery
26. Thirty minutes after a traumatic urethral catheterization a 58 year old male develops rigor,
fever & shivering The most likely cause is:
• Bacteramia.
• Pyaemia.
• Myocardial infarction.
• Septicaemia.
• Allergic reaction.
27. Five days after admission to a medical ward with the diagnosis of a right sided pneumonia, a
46 year of develops right sided severe chest pain, shortness of breath and swinging pyrexia.
On examination there is poor air entry, and an X-Ray chest confirms a right sided fluid
collection. In the early management of empyema chest what is the most successful option.
• o. Start parentral antibiotics.
• p. Repeated aspiration of chest and antibiotics.
• q. Insertion of a dependent chest tube, antibiotics and physiotherapy.
• r. Insertion of a dependent chest tube and antibiotics
• s. Insertion of chest tube through the bed of a resected rib
28. A 65 year old smoker presents with facial swelling and cyanosis specially on bending over.
There are la dilated subcutaneous veins on his upper chest. His jugular veins are prominent
even when he is upright. Which of the following conditions is the most likely cause of these
findings?
• Histoplasmosis(sclerosing mediastinitis).
• Bronchogenic carcinoma.
• Substernal goiter.
• Thoracic aortic aneurysms.
• Constrictive pericarditis.
29. A 70 year old smoker is diagnosed to have a small cell (oat cell) carcinoma in his left main
bronchus. What are our treatment options?
• t. Surgery is the mainstay.
• u. Radiotherapy is the preferred option.
• Chemotherapy has a favourable outcome.
• w. Surgery followed by chemotherapy.
• x. Surgery followed by radiotherapy.
30. A pedestrian is involved in a road traffic accident. Radiological investigation show a fracture of
the pelvis and since the patient is unable to pass urine a retrograde urethrogram confirms a
rupture of the membranous urethra. What is the most appropriate step in the management of
the patient?
• y. A urgent bil. Percutaneous nephrostomy.
• z. A gentle effort made to place a foley catheter through the urethra into the
• bladder.
• aa. Attempt immediate reconstruction of damaged urethra.
• bb. Urgent exploration of the pelvic fracture and drainage of the pelvic haematoma.
• cc. Immediate placement of a suprapubic cystostomy tube.
31. Four hours after an uneventful subtotal thyroidectomy for a euthyroid multinodular goiter a
40 year old female develops, stridor and restlessness. An endotracheal tube is inserted with
relief. The next best step of management in this patient is:
• dd. Administer a steroid IV.
• ee. Administer an antihistaminic IM.
• ff. Administer calcium chloride in IV infusion.
• gg. Put on a ventilator.
• hh. Explore the neck wound.
32. During the surgical exposure of a parotid gland in the performance of a superficial
parotidectomy, what is tl relationship of the facial nerve to the parotid gland?
• ii. The facial nerve passes through the substance of the superficial lobe.
• jj. The facial nerve is superficial to the superficial lobe.
• kk. The facial nerve straddles the isthemus of the parotid gland.
• ll. The facial nerve passes through the deep lobe.
• mm. The facial nerve is deep to the deep lobe.
33. For the accurate localization of a parathyroid adenoma, which investigation provides good
preoperative data?
• Ultrasound scan.
• Single photon emission computorised tomography (SPECT).
• CT scan.
• 99 mTc sestamibi (MIBI) scan.
• PET scan.
34. Thyroid gland develop from Endoderm of
• 1st Pharyngeal Pouch
• 1st & 2nd Ph.Pouch
• 3rd Ph. Pouch
• 4th Ph. Pouch
35. Inferior thyroid artery is branch of
• Carotid artery
• Thyrocervical trunk
• Branch of aorta
• Thyroid ima artery
36. Patient in the ICU become restless and breathless his biochemistry are
• ↑ PH, Normal PCo2 and ↑ HCO3. This patient has
• Metabolic Acidosis
• Metabolic Alkalosis
• AIK
• Resp. acidosis
37. Which of the following blood type can be transfuse in emergency for non-match
• recipient
• Blood group-A+
• Blood group-B-ve
• Blood group-O-ve
• Blood group-O+ve
• Blood groupAB+ve
38. Massive blood transfusion can leads to
• Hyperkalemia
• Hyperkalemia & Hypo-natgremia
• Hypokalemia
• Hyperkalemia & Hypercalcima
39. In thyroid surgery which of the following nerve is more common to be damage
• External laryngeal N
• Superficial laryngeal N
• Rec-Larngeal N
• Accessary-N

General Surgery
• Description Wound Infection, Special infection, Aids: Sterile precautions
1. STEM: The following is true cysticerosis. Marks the correct statement?
• ITEM A It does not occur in vegetarians
• ITEM B The comm0nest site in man is the brain
• ITEM C Serology is specific but not sensitive
• ITEM D Muscle cysticerosis usually requires excision
• ITEM E
• Correct Answer: C
2. STEM: Ascariasis my result in all of the following syndromes. Mark the incorrect statements
• ITEM A Biliary colic, jaundice
• ITEM B Intestinal obstruction
• ITEM C Pancreatitis
• ITEM D Retroperitoneal mass
• ITEM E
• Correct Answer: D
3. STEM: A patient presents with marked lymphedema of the leg. Which of the following parasites
is most likely to have caused this?
• ITEM A Loa loa
• ITEM B Wuchereia bancrofti
• ITEM C Onchocerca volvulus
• ITEM D Cnathostoma spinigerum
• ITEM E
• Correct Answer: B
4. STEM: A patient with a thyroglossal cyst develops a sudden increase in size, and experiences
dyspnea. Which is the most likely complication that has occurred?
• ITEM A Infeciton in the cyst
• ITEM B Bleeding into the cyst
• ITEM C Torsion of the cyst
• ITEM D Malignant change in the cyst
• ITEM E
• Correct Answer: B
5. STEM: A 25 year anxious female presents with a punched out ulcer on the forearm. On
examination, the ulcer has a straight edge. The likely diagnosis is
• ITEM A Basal cell carcinoma
• ITEM B Scrofula
• ITEM C Dematitis artefacta
• ITEM D Trophic ulcer following nerve injury
• ITEM E
• Correct Answer: C
6. STEM: A 50 year old male with long standing varicose veins develops a Marjolin’s ulcer on the
leg. The cause of non healing is
• ITEM A Impaired oxygenation due to stagnant blood I n the varicosities
• ITEM B Perisistent infection in the ulcer
• ITEM C Chronic surrounding lymphedema that results in breaking down of skin before it
heals
• ITEM D Onset of cancer in the ulcer
• ITEM E
• Correct Answer: D
7. STEM: A hirsute young male has a sinus just above the natal cleft. Which is the most likely
diagnosis?
• ITEM A Pilonidal sinus
• ITEM B Tubercular sinus associated with chronic osteomyelitis
• ITEM C Hidradenitis suppurativa
• ITEM D Fistula in ano
• ITEM E
8. STEM: Oncogenes have been identified for all of the following cancers,
• ITEM A Breast cancer
• ITEM B Prostae cancer
• ITEM C Ovarian cancer
• ITEM D Thyroid cancer
• ITEM E
• Correct Answer: C
9. STEM: A 40 year old female is to undergo radical mastectomy for a T2 M0 M0 malignant breast
lump. The objective of “radical” surgery here is
• ITEM A Diagnosis
• ITEM B Local control
• ITEM C Local and systemic control of cancer
• ITEM D To make chemotherapy more viable
• ITEM E
• Correct Answer: B
10. STEM: The following is true of the different types of skin grafts
• ITEM A A split thickness graft contains epidemis but no dermis
• ITEM B A Wolfe graft consists of the entire dermis along with some subcutaneous fat
• ITEM C A graft that contains both skin and bone is called a “multilayer grash”
• ITEM D A rhomboid flap is based on a single feeding artery that allows a length several times
as long as the width
• ITEM E
• Correct Answer: B
11. STEM: A patient reaches the casualty 6 hours after he has sustained a burn in the forearm. On
examination the burn is deep with a circumferential eschar. The estimated total extent of
burn is 5%. The most important next step is. Mark the correct statement?
• ITEM A Intravenous fluid therapy according to the Parkland formula
• ITEM B Occlusive dressing with silver sulfadiazine
• ITEM C Non-occlusive dressing with silver sulfadiazine
• ITEM D Escharotomy
• ITEM E
• Correct Answer: D
12. STEM: A patient sustains extensive deep burns over the torso (estimated 30-40%). The eschar
should be managed by
• ITEM A Allowing spontaneous separation
• ITEM B Surgical excision and healing by secondary intention
• ITEM C Surgical excision and cover with a skin graft
• ITEM D Surgical excision and cover with biological dressings like placenta
• ITEM E
• Correct Answer: C
13. STEM: Following a high velocity road accident, a patient is extracted from the wreck, and is
found conscious but incoherent. He has an obvious fracture in the thigh. Transport should be
in the following manner
• ITEM A Supine, with the injured thigh splinted
• ITEM B Supine, on a hard board, with the injured thigh splinted and the neck immobilized
with a hard collar, sandbag and tapes
• ITEM C Supine, with the injured thigh splinted, and the head turned to one side to prevent
aspiration
• ITEM D Supine, with the injured thigh splinted, the neck hyper extended, and the foot end
raised
• ITEM E
• Correct Answer: B
14. STEM: Which of the following statements regarding cervical spine fractures is False.
• ITEM A Most cervical spine fractures are a result of hyperextension
• ITEM B Fractures of the C1 ring are called Jefferson fractures and are caused by axial loading
• ITEM C Plain x-Rays are insufficient, and most fractures required CT scans for proper
Diagnosis
• ITEM D The majority of unstable fractures will require immediate surgery, reduction,
decompression and fusion.
• ITEM E
• Correct Answer: D
15. STEM: Which statement about burst fractures of the spine is False
• ITEM A The injury most often occurs at the junction of T12 and L1
• ITEM B If the posterior elements are intact, the fracture is stable
• ITEM C The cardinal sign of instability is widening of distance between pedicles
• ITEM D Laminectomy is indicated to provide stability
• ITEM E
• Correct Answer: D
16. STEM: The following statement is true about thoracic spine injuries
• ITEM A The commonest type of fracture is the burst fracture
• ITEM B Because of the splinting action of the ribs, even unstable fracture rarely displace
• ITEM C Associated injuries are extremely uncommon
• ITEM D Fractures are less commonly seen in T1-T9 vertebae than in T10-L15 veertebrae
• ITEM E
• Correct Answer: D
17. STEM: In a critically ill patient in the intensive care unit, a resident performs the “cold water”
test to check for brain death. He instills 50 ml of water into the ear. What should he look for?
• ITEM A Any movement in the body
• ITEM B Eye movement
• ITEM C Convulsions
• ITEM D Gasping
• ITEM E
• Correct Answer: B
18. STEM: Which of the following findings is the LEAST likely to indicate that an injured patient
has a fracture of the upper or lower jaw?
• ITEM A There is bruising over the mastoid process
• ITEM B The mouth is persistently held open
• ITEM C The upper and lower teeth do not fit properly
• ITEM D There is a hematoma in the floor of the mouth
• ITEM E
• Correct Answer: A
19. STEM: An injured patient has fractures his mandible, and the bone ends are pulled apart by
the muscles. These fractures are best treated by
• ITEM A A jaw plaster
• ITEM B Intermaxillary fixation
• ITEM C Fixation with plates
• ITEM D Bone graft
• ITEM E
• Correct Answer: C
20. STEM: The parafollicular ‘C’ cells of the thyroid gland arise from the
• ITEM A Ultimobranchial body
• ITEM B Neural crest
• ITEM C Thyroglossal duct
• ITEM D Fourth pharyngeal pouch
• ITEM E
• Correct Answer: B
21. STEM: A patient known to be hypothyroid presents with an altered mental state and
hypothermia. Which of the following is of least value in the management of this condition?
• ITEM A Intravenous T3
• ITEM B Slow rewarming
• ITEM C Propranolol
• ITEM D Hydrocortisone
• ITEM E
• Correct Answer: C
22. STEM: Within hours of surgery for thyrotoxicosis a patient develops respiratory distress.
Which of the following is the most likely cause?
• ITEM A Laryngeal edema following tension hematoma
• ITEM B Unilateral laryngeal nerve palsy
• ITEM C Bilateral laryngeal nerve plasy
• ITEM D
• ITEM E
• Correct Answer: C
23. STEM: As compared to secondary thyrotoxicosis, which of the following features is NOT true
for primary thyrotoxicosis?
• ITEM A Hyperthyroidism is more severe
• ITEM B Cardiac failure is less common
• ITEM C Orbital proptosis is more common
• ITEM D Pretibial myxedema is less common
• ITEM E
• Correct Answer: D
24. STEM: 24 hours after thyroidectomy for diffuse toxic goiter, a patient develops hyperpyrexia,
marked tachycardia, and restlessness. Which of the following measures is LEAST useful in
management
• ITEM A Propanolol
• ITEM B Iodine
• ITEM C Antibiotics
• ITEM D Steroids
• ITEM E
• Correct Answer: C
25. STEM: The following are all features of primary hyperaldosteronis. Mark the wrong statement
• ITEM A It is common in women than in men
• ITEM B Most cases are due to hyperplasia of the zona glomerulosa
• ITEM C There is increased potassium excretion in the urine
• ITEM D Spironolactone is the first line medical treatment
• ITEM E
• Correct Answer: B
26. STEM: The most common site for extra-adrenal catecholamine secreting paragangliomas is
• ITEM A Pelvis
• ITEM B Abdominal para-aortic region
• ITEM C Thorax para-aortic
• ITEM D Urinary bladder
• Correct Answer: D
27. STEM: Congenital adrenal hyperplasia
• ITEM A Is an autosomal dominant disorder
• ITEM B Most commonly occurs due to a deficiency of 21-hydroxylase
• ITEM C Results in an isolated increase in androgenic cortisol
• ITEM D Is associated with low ACTH levels
• ITEM E
• Correct Answer: B
28. STEM: Prevention of organ failure by preventing multiple organ dysfunction syndrome
(MODS), the targets for intervention are
• ITEM A Established microvascular occlusion
• ITEM B Tissue hypoxia
• ITEM C Cellular dysfunction
• ITEM D Abnormal arachidonic acid metabolism
• ITEM E Coagulation and complement activation
• Correct Answer: A
29. STEM: A cystic hygroma
• ITEM A Usually presents in the neonate
• ITEM B Contains sequestered blood
• ITEM C Contains fresh blood
• ITEM D Usually disappears spontaneously
• Correct Answer: A
30. STEM: Within hours of surgery for thyrotoxicosis, a patient develops respiratory distress.
Which of the following is the most likely cause?
• ITEM A Laryngeal edema following tension hematoma
• ITEM B Unilateral laryngeal nerve plasy
• ITEM C Bilateral faryngeal nerve plasy
• ITEM D Laryngeal spasm due to hypoparathyroidism
• ITEM E
• Correct Answer: A
31. STEM: A young man presents with a solitary thyroid nodule, which of the following is of LEAST
value
• ITEM A Thyroid function tests
• ITEM B Ultrasonography
• ITEM C Radioisotope scan
• ITEM D FNAC
• Correct Answer: C
32. STEM: The best dressing is
• ITEM A Swab
• ITEM B Gauze
• ITEM C Skin
• ITEM D Silver foil
• ITEM E
• Correct Answer: C
33. STEM: Debridement means
• ITEM A Excising 1 mm of skin from the edge of a wound
• ITEM B Not excising skin but excising all damaged muscle
• ITEM C Amputation
• ITEM D Laying open all layers of a wound and excision of devitalized tissue
• ITEM E Suturing of a wound
• Correct Answer: D
34. STEM: All are true about puncture wounds. Mark the wrong statement
• ITEM A Caused by standing on a sharp object
• ITEM B Foreign material may be carried deeply into underlying tissues
• ITEM C Radiography is of no use in their management
• ITEM D Bites are puncture wounds
• ITEM E Best treated by open surgical exploration
• Correct Answer:
35. STEM: Spot the wrong statement with regard to ulcers
• ITEM A Are chronic wounds
• ITEM B Are always caused by mechanical injury
• ITEM C Are common in diabetes and rheumatoid arthritis
• ITEM D Are wounds which fail to heal easily
• ITEM E Are not wounds
• Correct Answer: B
36. STEM: The most frequent cause of hyponatraemia in surgical practice is all. Mark the wrong
statement
• ITEM A Small intestinal obstruction
• ITEM B Duodenal, biliary, pancreatic or high intestinal external fistulas
• ITEM C Severe diarrhea
• ITEM D SIADH
• ITEM E Central diabetes inspidus (CDI)
• Correct Answer: E
37. STEM: In respiratory alkalosis all the statements are true. Mark the wrong statement
• ITEM A PCO2 below 31-342 mmHg
• ITEM B Caused by excessive pulmonary ventilation
• ITEM C Hyperventilatin at high altitude
• ITEM D Hyperpyrexia with hyperventilation
• ITEM E Pituitary adenomas
• Correct Answer: E
38. STEM: The following statements regarding respiratory acidosis are correct. Mark the wrong
statement
• ITEM A PCO2 is above normal range
• ITEM B Caused by impaired alveolar ventilation
• ITEM C Inadequate ventilation by anaesthetized patient
• ITEM D When the effects of muscle relaxants are fully reversed at the end of the
anaesthesia
• ITEM E Prolonged cardiac arrest
• Correct Answer: E
39. STEM: Anticoagulant mixture for blood preservation contains. Mark the wrong statement
• ITEM A Calcium
• ITEM B Potassium
• ITEM C Citrate
• ITEM D Dextrose
• ITEM E
• Correct Answer:
40. STEM: Metabolic disorders that can complicate anesthesia include
• ITEM A Porphyria
• ITEM B Malignant hyperthermia
• ITEM C Diabetes mellitus
• ITEM D Corticosteroid intake
• ITEM E Each of the above
• Correct Answer: E
41. STEM: In jaundiced patient general anesthesia carries the risk of
• ITEM A Bleeding
• ITEM B Thrombosis
• ITEM C Renal failure
• ITEM D Shock
• ITEM E
• Correct Answer: C
42. STEM: Nitrous oxide is
• ITEM A Strng analgesic but weak anaesthetic
• ITEM B Weak analgesic but strong anaesthetic
• ITEM C Strong analgesic and anaesthetic
• ITEM D Analgesic and weak anaesthetic
• ITEM E
• Correct Answer: D
43. STEM: The main organism of endotoxin release in multiple organ dysfunction syndrome is
• ITEM A Proteus vulgaris
• ITEM B E.Coli
• ITEM C Pseudomonas
• ITEM D Acenetobacter
• ITEM E
• Correct Answer: B
44. STEM: Spreading infection is typical of all the following organism, Mark the wrong statement
• ITEM A Beta-haemolytic streptococci
• ITEM B Staph aureus
• ITEM C Cl. Perfringens
• ITEM D Pseudomonas
• ITEM E
• Correct Answer: D
45. STEM: Which of the following surgical dressing has maximum advantage for deep wound
dressing
• ITEM A Polymeric film
• ITEM B Hydrogels
• ITEM C Hydrocolloids
• ITEM D Fibrous polymers
• ITEM E
• Correct Answer: B
46. STEM: Which of the following theatre technique does not help to reduce wound infection
• ITEM A Aseptic technique
• ITEM B Drains and wound guard
• ITEM C Instrument sterilization
• ITEM D Clean dissection
• ITEM E
• Correct Answer: B
47. STEM: In a patient of lepromatous leprosy the principal source of infection is
• ITEM A Skin nodule
• ITEM B Nasal secretion
• ITEM C Sexual contact
• ITEM D Fomites
• ITEM E
• Correct Answer: B
48. STEM: In a patient AIDS needs exclusion when presenting with
• ITEM A Anal warts
• ITEM B Perianal sepsis
• ITEM C Anorectal ulceration
• ITEM D Anal neoplasia
• ITEM E Any of the above
• Correct Answer: E
49. STEM: Antibiotic prophylaxis before surgery should be with
• ITEM A Single low dose of broad spectrum antibiotic
• ITEM B Multiple small doses of broad spectrum antibiotic
• ITEM C Single high dose of broad spectrum antibiotic
• ITEM D Multiple high dose of broad spectrum antionbiotic
• ITEM E
• Correct Answer: C
50. STEM: Ideal steam sterilization of 121°C with pressure of 15lb/inch2 should have hold time of
• ITEM A 10 min
• ITEM B 15 min
• ITEM C 30 min
• ITEM D 35 min
• ITEM E 50 min
• Correct Answer: B
51. STEM: Complications of a cyst include
• ITEM A Torsion
• ITEM B Infection
• ITEM C Rupture
• ITEM D Compression of adjacent structure
• ITEM E All of the above
• Correct Answer: E
52. STEM: A punched out edge is a characteristic of which type of ulcer
• ITEM A Tuberculous
• ITEM B Rodent ulcer
• ITEM C Syphilitic
• ITEM D Nonspecific ulcer
• ITEM E
• Correct Answer: C
53. STEM: A sinus or fistula will be persistent in presence of
• ITEM A Inefficient drainage
• ITEM B Foreign body or necrotic tissue
• ITEM C Epithelialisatin of the wall
• ITEM D Malignant change
• ITEM E All of the above
• Correct Answer: E
54. STEM: What is not true of callosity, corn and wart
• ITEM A Wart is virus induced, corn and callosity are not
• ITEM B Corn has base to the surface and apex inwards
• ITEM C Callosity occurs in areas of friction and pressure so also corn
• ITEM D Resistant multiple warts often indicate immune deficiency
• ITEM E All are true
• Correct Answer: E
55. STEM: In a child with burn inuury IC fluid is a must when BSA exceeds
• ITEM A 5 percent
• ITEM B 10 percent
• ITEM C 15 percent
• ITEM D 20 percent
• ITEM E 25 percent
• Correct Answer: B
56. STEM: Leriche’s syndrome is
• ITEM A Seen n venous disorders
• ITEM B Caused by aortoiliac occlusion
• ITEM C A disorder of the peripheral nerves
• ITEM D A manifestation of spinal disease
• ITEM E Due to femoropopliteal occlusion
• Correct Answer: B
57. STEM: Lumbar sympathectomy is of value in the management of
• ITEM A Intermittent claudication
• ITEM B Arteriovenous fistula
• ITEM C Diabetic neuropathy
• ITEM D Backache
• ITEM E Distal ischaemia affecting skin of toes
58. STEM: Diabetic gangrene is due to. Mark the wrong statement
• ITEM A Excess if sugar in the tissues
• ITEM B Peripheral neuritis
• ITEM C Atheromatous changes in arteries
• ITEM D Reduced peripheral sensations
• ITEM E Clostridium welchii
• Correct Answer: E
59. STEM: All the following are venous symptoms. Mark the wrong statement
• ITEM A Aching
• ITEM B Leg cramps
• ITEM C Palpitation
• ITEM D Tiredness
• ITEM E Ankle swelling
• Correct Answer: A
60. STEM: The cause of venous ulceration is
• ITEM A Ambulatory venous hypertension
• ITEM B Ambulatory venous hypotension
• ITEM C Fibrin cuff around capillaries
• ITEM D Arteriolar spasm
• ITEM E
• Correct Answer: A
61. STEM: Common aetiologic agent of acute lymphangitis is
• ITEM A Staph aureus
• ITEM B S.pyogenes
• ITEM C Bacteroids
• ITEM D Mixed organisms
• ITEM E
• Correct Answer: B
62. STEM: The “golden hours of trauma” refers to
• ITEM A Time immediately after injury
• ITEM B Time few hours after injury
• ITEM C Time days to weeks after injury
• ITEM D
• ITEM E
• Correct Answer: B
63. STEM: Missile wound in abdomen. Mark the wrong statement
• ITEM A The patient should be resuscitated and blood transfusion given
• ITEM B A catheter must be passed into the bladder
• ITEM C A midline incision is used
• ITEM D Every penetrating gunshot wound should not be explored by laparotomy
• ITEM E In all wounds of stomach, lesser sac must be opened
• Correct Answer: D
64. STEM: Bladder and urethral injuries are treated by
• ITEM A Suprapubic puncture
• ITEM B Suprapubic cystostomy
• ITEM C Immediate perurethral catheterization
• ITEM D Rail road technique
• ITEM E
• Correct Answer: B
65. STEM: Spot the wrong statement in case of missile injuries
• ITEM A Incise the skin generously
• ITEM B Incise the deep fascia widely
• ITEM C Identify the neurovascular bundle
• ITEM D Excise all dead muscles
• ITEM E Do not leave the wound open at the end of operation
• Correct Answer: E
66. STEM: Spot the wrong statement in case of blast injuries
• ITEM A The most vulnerable organs to the blast wave are ear drums, lungs and GIT
• ITEM B Have multiple wounds
• ITEM C Deafness occurs due to rupture of tympanic membrane
• ITEM D All blast wounds should be left open and delayed primary suture performed
• ITEM E None of the above
• Correct Answer: E
67. STEM: Epiphyseal injuries causing growth disturbances in children are due to
• ITEM A Avascular necrosis of the plate
• ITEM B Crushing or infection of the plate
• ITEM C Hypermia of the plate
• ITEM D Formation of a callus bridge between bony epiphysis and metaphysic
• ITEM E All of the above
• Correct Answer: E
68. STEM: In fracture open reduction is needed in presence of
• ITEM A Failed closed reduction
• ITEM B Intra-articular fracture in children
• ITEM C Displaced intra-articular fracture
• ITEM D Unstable fracture
• ITEM E Each of the above
• Correct Answer: E
69. STEM: Figure of eight wire construction is commonly done in fracture
• ITEM A Clavicle
• ITEM B Medical femoral condyle
• ITEM C Patella
• ITEM D Talus
• ITEM E
• Correct Answer: C
70. STEM: What is true of clavicle fracture
• ITEM A Non-union is rare
• ITEM B Malunion is of no functional significance
• ITEM C Reduction even if achieved is difficult to maintain
• ITEM D All are true
• ITEM E
• Correct Answer: D
71. STEM: Avascular necrosis of femoral head results from
• ITEM A Fracture neck of femur
• ITEM B Alcoholics
• ITEM C Steroid therapy
• ITEM D Idiopathic
• ITEM E All of the above
• Correct Answer: E
72. STEM: Childhood fractures differ from those in adults by. Mark the wrong statements
• ITEM A Unite more quickly
• ITEM B Malunion to some exten can be corrected by growth
• ITEM C Joint stiffness is severe
• ITEM D May interfere with growth
• ITEM E
• Correct Answer: C
73. STEM: Major factors influencing the time of fracture healing. Spot the wrong
• statement
• ITEM A Age
• ITEM B Type of fracture
• ITEM C Immobilization
• ITEM D High protein diet
• ITEM E Apposition of fragments
• Correct Answer: D
74. STEM: Acute osteomyelitis usually begins at
• ITEM A Epiphysis
• ITEM B Metaphysis
• ITEM C Diaphysis
• ITEM D Any of the above
• ITEM E
• Correct Answer: B
75. STEM: Sequestrum is formed due to
• ITEM A Demineralisation due to cytokines
• ITEM B Vessel thrombosis
• ITEM C Improper stabilisaiton
• ITEM D All of the above
• ITEM E
• Correct Answer: B
76. STEM: In Pott’s spine the pus can present at
• ITEM A Greater trochanter
• ITEM B Lesser trochanter
• ITEM C Ischial tuberosity
• ITEM D Any of the above
• ITEM E Non of the above
• Correct Answer: B
77. STEM: In Pott’s spine, the disease starts in the
• ITEM A Intervertebral disk
• ITEM B Anterior vertebral margin
• ITEM C Posterior vertebral margin
• ITEM D Paravertbebral soft tissue
• ITEM E
• Correct Answer: B
78. STEM: Which of the following is not malignant
• ITEM A Multiple myeloma
• ITEM B Adamantinoma
• ITEM C Ewing’s tumour
• ITEM D Giant cell tumour
• ITEM E
• Correct Answer: D
79. STEM: Bone metastasis in male commonly arises from carcinoma of
• ITEM A Lung
• ITEM B Prostate
• ITEM C Kidney
• ITEM D Thyroid
• ITEM E
• Correct Answer: B
80. STEM: Most common site of metastasis from malignant bone tumours is
• ITEM A Brain
• ITEM B Lungs
• ITEM C Liver
• ITEM D Pleura
• Correct Answer: B
81. STEM: Heberden’s nodes of osteoarthritis are seen in
• ITEM A Carpometacarpal joint
• ITEM B Proximal interphalangeal joint
• ITEM C Distal interphalangeal joint
• ITEM D Metacarpo-phalangeal joint
• ITEM E
• Correct Answer: C
82. STEM: Chronic parotitis in children is almost pathognomonic of
• ITEM A Syphilis
• ITEM B Tubrculosis
• ITEM C AIDs
• ITEM D Scrcoidosis
• ITEM E Sjogren’s syndrome
• Correct Answer: C
83. STEM: Warthin’s tumour refers to
• ITEM A Pleomorphic adenoma
• ITEM B Adenolymphioma
• ITEM C Mucoepidermoid tumour
• ITEM D Adenloid cystic carcinoma
• Correct Answer: B
84. STEM: Sjogren’s syndrome accompanies. Mark the wrong statement
• ITEM A Rheumatoid arthritis
• ITEM B SLE
• ITEM C Chronic active hepatitis
• ITEM D Primary biliary cirrhosis
• ITEM E
• Correct Answer: C
85. STEM: Which of the following statements regarding thyroglassal cyst is ture
• ITEM A This may be present in any part of the thyroglassal tract
• ITEM B Such a cyst occupies the midline except in the region of the thyroid cartilage where
the thyroglassal tract is pushed to one side
• ITEM C The selling moves upward on protrusion of the tongue as well as on swallowing
• ITEM D It should be excised because infection is inevitable
• ITEM E All of the above
• Correct Answer: E
86. STEM: Which of the following statements regarding cretinism is untrue
• ITEM A In endemic area goitrous cretinism is common
• ITEM B Is due to maternal and foetal iodine deficiency
• ITEM C The parents and other children may be normal
• ITEM D Hypothyroidism occurs in 1 in 4000 live births
• ITEM E Routine biochemical screening of neonates for hypothyroidism is done
• Correct Answer: E
87. STEM: All of the following are symptoms of adult hypothyroidism. Mark the wrong statement
• ITEM A Tiredness
• ITEM B Cold intolerance
• ITEM C Weight loss
• ITEM D Mental lethargy
• ITEM E Menstrual disturbances
• Correct Answer: C
88. STEM: Simple goiter commonly develops as a result of
• ITEM A Stimulation of thyroid gland by TSH—as a result of inappropriate secretion from a
microadenoma in anterior pituitary
• ITEM B Stimulation of thyroid gland by TSH to chronically low level of circulating thyroid
hormone
• ITEM C The most important factor is dietary deficiency of iodine
• ITEM D Defective hormone synthesis probably accounts for many sporadic goiter
• ITEM E Well known goitrogens like the vegetables of the brassica family
• Correct Answer: C
89. STEM: All of the following statements regarding de-Quervain’s thyroiditis is true. Mark the
wrong statement
• ITEM A This is due to virus infection
• ITEM B In a typical subacute presentation there is pain in the neck, fever, malaise and a firm
irregular enlargement of thyroid
• ITEM C ESR is raided
• ITEM D Thyroid antibodies are absent
• ITEM E 1123 uptake is high
• Correct Answer: E
90. STEM: Which of the following statements regarding para-thormone is untrue
• ITEM A Stimulates osteoclastic activity
• ITEM B Is an 90-amno acid peptide
• ITEM C Increases the reabsorption of calcium by the renal tubule
• ITEM D Augments the absorption of calcium from the gut
• ITEM E Reduces the renal tubular reabsorption of phosphate, thus promoting
• phosphaturia
• Correct Answer: B
91. STEM: All of the statements regarding parthyroid tetany are true. Mark the wrong statements
• ITEM A Parathyroid tetany due to hypocalcaemia is a common complications of total
thyroidectomy
• ITEM B May also occur after surgery to parathyroids
• ITEM C Symptoms usually appear on the 2nd or 3rd post-operative day
• ITEM D Tetany in newborn may occur within the first grew days of life if mother has
undiagnosed hypoparathyroidism
• ITEM E Permanent hypoparathyroidism is uncommon following radical thyroidectomy for
cancer
• Correct Answer: A
92. STEM: The characteristic features of Water House-Friderichsen Syndrome are the following.
Mark the wrong statement
• ITEM A Massive bilateral adrenal cortical haemorrhage occurs due to fulminating
meningococcal septicemia
• ITEM B Most cases occur in infant and young children
• ITEM C The onset is catastrophic with rigors, hyperpyrexia, cyanosis and vomiting
• ITEM D Death occurs within 48 hours of the onset of symptoms unless correct treatment is
given without delay
• ITEM E Pituitary apoplexy may be associated finding
• Correct Answer: E
93. STEM: All are true of neuroblastoma. Mark the wrong statement
• ITEM A Over 50% occur in children below 2 years
• ITEM B 50% arise from adrenal gland
• ITEM C 50% are malignant
• ITEM D 50% show stippled calcification
• ITEM E
• Correct Answer: C
94. STEM: The acronym QUART stands for
• ITEM A Quadrantectomy and radiotherapy
• ITEM B Quadrantectomy, axillary dissection and radiotherapy
• ITEM C Quadrantic resection and chemotherapy
• ITEM D None of above
• ITEM E
• Correct Answer: B
95. STEM: Indications for chemotherapy in breast cancer include
• ITEM A Premenopausal women with positive lymph nodes
• ITEM B Postmenopausal women with poor prognosis disease
• ITEM C Extensive disease irresptive pre or post menopausal
• ITEM D A + C
• ITEM E A + B
• Correct Answer: E
96. STEM: The vertebral region most vulnerable for fracture is
• ITEM A Cervical
• ITEM B Upper thoracic
• ITEM C Lower thoracic
• ITEM D Lumbar
• ITEM E Sacral
• Correct Answer: A
97. STEM: A haemothorax should always be drained because
• ITEM A Empyema may supervene
• ITEM B Fibrothorxa may occur
• ITEM C Relieves compression of contralateral lung
• ITEM D Helps to reduce bleeding from torn vessels
• ITEM E All of the above
• Correct Answer: E
98. STEM: Transection of aorta should be suspected in presence of
• ITEM A Deceleration injury
• ITEM B Radiofemoral delay
• ITEM C Hoarseness of voice
• ITEM D Left sided pleural effusion
• ITEM E All of the above
• Correct Answer: E
99. STEM: Clinical presentation of mediastinal mass lesion is with
• ITEM A SVC obstruction and facial suffusion
• ITEM B Dry cough
• ITEM C Hoarseness of voice
• ITEM D Elevated hemidiaphragm
• ITEM E All of the above
• Correct Answer: E
100. STEM: Sudden right sided chest pain in post-surgical patient with dyspnoea
• and loud P2 indicates
• ITEM A Myocardial infarction
• ITEM B Plumonary embolism
• ITEM C Acute pneumothorax
• ITEM D Congestive cardiac failue
• ITEM E
• Correct Answer: B
101. STEM: Chronic constrictive pericarditis presents with all. Mark the wrong statement
• ITEM A Raised CVP
• ITEM B Tender hepatomegaly
• ITEM C Ascites and oedema feet
• ITEM D Marked cardiomegaly
• ITEM E
• Correct Answer: D
102. STEM: What is not true of Boerhaave’s syndrome
• ITEM A Occurs due to vomiting with full stomach
• ITEM B There is longitudinal tear in lower oesophagus
• ITEM C Tear involves the mucosa and submucosa
• ITEM D Mediatinitis and left pleural effusion can occur
• ITEM E
• Correct Answer: C
103. STEM: In Mallory-Weiss syndrome all are true. Mark the wrong statement
• ITEM A Follows severe vomiting
• ITEM B Vertical tear
• ITEM C Lower oesophageal tear
• ITEM D Mild bleeding
• ITEM E
• Correct Answer: C
104. STEM: In hypertrophic pyloric stenosis of adult the pathology is
• ITEM A Overgrowth of circular muscle
• ITEM B Overgrowth of longitudinal muscle
• ITEM C Fibrosis of myenteric plexus
• ITEM D None of the above
• ITEM E
• Correct Answer: C
105. STEM: Ulcers at which location have greater chance of malignancy
• ITEM A Antrum
• ITEM B Prepyloric region
• ITEM C Fundus
• ITEM D Body
• ITEM E
• Correct Answer: B
106. STEM: Which peptic ulcer complication manifests with vomiting, abdominal
• pain and rigidity
• ITEM A Pyloric obstruction
• ITEM B Perforation
• ITEM C Malignant transformation
• ITEM D Haemorrhage
• Correct Answer: B
107. STEM: Variceal bleed secondary to portal vein thrombosis is best treated by
• ITEM A TIPSS
• ITEM B Sclerotherapy
• ITEM C Gastroesophageral devascularisation
• ITEM D Octreotide
• ITEM E
• Correct Answer: C
108. STEM: Primary sclerosing cholangitis is associated with
• ITEM A Rheumatoid arthritis
• ITEM B Ulcerative colitis
• ITEM C Mixed connective tissue disease
• ITEM D All of the above
• ITEM E
• Correct Answer: B
109. STEM: Intrahepatic biliary lakes with stone characterize
• ITEM A Hepatic polycystic disease
• ITEM B Primatry biliary cirrhosis
• ITEM C Caroli’s disease
• ITEM D Cholangitis
• ITEM E
• Correct Answer: C
110. STEM: The most common complication of hepatic hydrated disease is
• ITEM A Jaundice
• ITEM B Rupture into peritoneal cavity
• ITEM C Suppuration
• ITEM D Rupture into biliary channel
• ITEM E
• Correct Answer: D
111. STEM: Kehr sign in splenic trauma refers to
• ITEM A Pain and hyperaesthesia in leftr shoulder
• ITEM B Pain and hyperaesthesia in right shoulder
• ITEM C Bruising around left 10th and 11th ribs
• ITEM D Hiccupand haemoptysis on leg elevation
• ITEM E
• Correct Answer: A
112. STEM: A young child having anemia, and gallstones should be investigated for
• ITEM A Cystic fibrosis
• ITEM B Congenital spherocytosis
• ITEM C Malaria
• ITEM D Primary sclerosing cholangitis
• ITEM E
• Correct Answer: B
113. STEM: A 2 year old child with anaemia, hepatosplenomegaly and a large head with slanting
eyes is mots likely to have
• ITEM A Cirrhosis
• ITEM B Spherocytosis
• ITEM C Thalassaemia
• ITEM D Gaucher’s disease
• ITEM E
• Correct Answer: C
114. STEM: The primary investigation of choice in gallstone disease is
• ITEM A Plain x-Ray
• ITEM B OCG
• ITEM C Ultrasonography
• ITEM D IV cholangiography
• ITEM E
• Correct Answer: C
115. STEM: Which of the following is a possibility if symptoms persist after
• cholecystectomy
• ITEM A Stone in CBD
• ITEM B Long stump of cystic duct remnant
• ITEM C Damage to CBD from consequent stricture
• ITEM D Each of above
• ITEM E
• Correct Answer: D
116. STEM: Gallbladder pain persisting for more than 12 hours is due to
• ITEM A Stone colic
• ITEM B Cholecystitis
• ITEM C Perforation of gallbladder
• ITEM D Torsion of gallbladder
• ITEM E
• Correct Answer: B
117. STEM: Clinical manifestations of cystic fibrosis include all. Mark the wrong
• statement
• ITEM A Malabsorption
• ITEM B Obstructive lung disease
• ITEM C Salty child
• ITEM D Mental retardation
• Correct Answer: D
118. STEM: The most common feature of acute pancreatitis is
• ITEM A Severe acute epigastric pain radiating to back
• ITEM B Jaundice
• ITEM C Cullen’s sign and Grey Turner’s sign
• ITEM D Abdominal guarding and loss of bowel sounds
• ITEM E
• Correct Answer: A
119. STEM: Abdominal complications of peritonitis include all. Mark the wrong statement
• ITEM A Paralytic ileus
• ITEM B Portal pyemia
• ITEM C Small bowel obstruction
• ITEM D Colonic volvulus
• ITEM E
• Correct Answer: D
120. STEM: Clinical presentation of Hirschsprung’s disease includes all. Mark the wrong statement
• ITEM A Delayed passage of meconium in newborn
• ITEM B Neonatal intestinal obstruction
• ITEM C Severe chronic constipation
• ITEM D Common to female child and no familial tendency
• ITEM E Common to male child and no familial tendency
• Correct Answer: D
121. STEM: The rule of 2 as applicable to Meckel’s diverticulum is all. Mark the wrong statement
• ITEM A 2” long
• ITEM B Present in 2% cases
• ITEM C 2ʹ from ileocaecal valve
• ITEM D Contains only 2 coats of intestinal wall
• ITEM E
• Correct Answer: D
122. STEM: The most frequent symptom of Meckel’s diverticulum is
• ITEM A Diverticulitis
• ITEM B Bleeding
• ITEM C Intussusception
• ITEM D Intestinal obstruction
• ITEM E
• Correct Answer: B
123. STEM: Intestinal amoebiasis can manifest with which of the following
• ITEM A Typhlitis
• ITEM B Caecal mass
• ITEM C Intestinal obstruction
• ITEM D Fibrous stricture
• ITEM E All of the above
• Correct Answer: E
124. STEM: Juvenile polyp belongs to which of the following
• ITEM A Hamartoma
• ITEM B Villlous adenoma
• ITEM C Tubulovillous adenoma
• ITEM D Tubular adenoma
• ITEM E
• Correct Answer: A
125. STEM: When carcinoma is located in pelvic colon the area to be resected is
• ITEM A Entire colon
• ITEM B Transverse colon to rectum
• ITEM C Splenic flexure to rectum
• ITEM D Hepatic flexue to rectum
• ITEM E
• Correct Answer: C
126. STEM: What is not true of microsopic antomay of appendix
• ITEM A Base of the crypts contain Kultschitzsky cells
• ITEM B Crypts are numerous
• ITEM C Submucosa contains “abdominal tonsils”
• ITEM D The longitudinal muscle coat is due to joining of taenia coli at base of
• appendix
• ITEM E
• Correct Answer: B
127. STEM: In elderly appendicitis need be differentiated from
• ITEM A Mesenteric infarction
• ITEM B Colon cancer
• ITEM C Torsion appendix epiploicoe
• ITEM D Diverticulitis
• ITEM E Each of the above
• Correct Answer: E
128. STEM: Rectal lesions mimicking rectal cancer include
• ITEM A Inflammatory stricture
• ITEM B Amoeboma
• ITEM C Endometrioma
• ITEM D Solitary rectal ulcer
• ITEM E All of the above
• Correct Answer: E
129. STEM: Rectal cancer can be confused with
• ITEM A Benign adenoma
• ITEM B Inflammatory stricture
• ITEM C Amoebic granuloma
• ITEM D Endometrioma
• ITEM E Each of the above
• Correct Answer: E
130. STEM: What is true of local recurrence after surgical treatment of rectal cancer
• ITEM A It is a major problem
• ITEM B Mainstay of therapy is radiotherapy
• ITEM C Most common cause is inadequate removal than micrometastasis
• ITEM D CEA estimation can indicate radical surgery
• ITEM E All are true
• Correct Answer: E
131. STEM: Most common cause of anorectal abscess and fistula is
• ITEM A Unhygenic perianal skin
• ITEM B Anal intercourse
• ITEM C Infection of anal glands
• ITEM D Perianal eczema
• ITEM E
• Correct Answer: C
132. STEM: The most common malignant tumour of anus is
• ITEM A Basaloid carcinoma
• ITEM B Mucoepidermoid carcinoma
• ITEM C Melanoma
• ITEM D Sequamous cell carcinoma
• ITEM E
• Correct Answer: D
133. STEM: Which of the following statements is untrue in obstructed hernia
• ITEM A Synonymous with incarcerated hernia
• ITEM B This is an irreducible hernia containing intestine which is obstructed
• ITEM C There is interference to blood supply
• ITEM D The symptoms are less severe than in strangulation
• ITEM E Often no clear distinction can be made between obstruction and
• strangulation
• Correct Answer: C
134. STEM: All of the following are the causes of ventral hernia. Mark the wrong statement
• ITEM A Occurs most often in obese individuals
• ITEM B High incidence following operations for peritonitis
• ITEM C The placing of a drainage tube through the incision
• ITEM D Persistence of post operative cough and post operative abdominal
• distntion
• ITEM E None of the above
• Correct Answer: E
135. STEM: Which of the following statements regarding infantile polycystic disease of kidney is
not true
• ITEM A Rare condition
• ITEM B The kidneys are large and may cause obstruction at birth
• ITEM C Most of the children are stillborn
• ITEM D Most others die of renal failure in early life
• ITEM E The condition is inherited as autosomal dominant
• Correct Answer: E
136. STEM: The following are the causes of pre renal uraemia. Mark the wrong statement
• ITEM A Hypovolemia
• ITEM B Severe blood loss
• ITEM C GM-negative sepsis
• ITEM D Cardiogenic shock
• ITEM E Acute glomerulonephritis
• Correct Answer: E
137. STEM: The following are the causes of post-renal uraemia. Mark the wrong statement
• ITEM A Renal calculus disease
• ITEM B Pelvic malignancy
• ITEM C Pelvic surgery
• ITEM D Crystalluria
• ITEM E Prolonged hypoxia
• Correct Answer: E
138. STEM: What is not true of ectopia vesicae
• ITEM A Occurrence is 1 in 50,000 births
• ITEM B Absence of anterior abdominal wall and anterior wall of bladder
• ITEM C Early rupture of cloacal membrane is responsible
• ITEM D Common to male child
• ITEM E Common to female child
• Correct Answer: C
139. The following statement is TRUE regarding Lymphedema:
• Clinical examination reveals Non-pitting edema
• Filariasis is an unusual cause
• Treatment is primarily surgical intervention
• It is usually associated with DVT
• CT scan is the gold standard investigation
140. The following statement is TRUE regarding biliary tract injuries:
• Chances of bile duct injury is more in open cholecystectomy than laparoscopic
• cholecystectomy
• Internal stenting is currently the procedure of choice for treating bile duct leaks
• About 75% of injuries are recognized intra-operatively
• Ultrasound abdomen is investigation of choice in biliary tract injury
• Wait and see policy is usually successful
141. Commonest cause of biliary tract injury is:
• Blunt abdominal trauma
• Penetrating abdominal trauma
• Iatrogenic injury
• Spontaneous rupture
• Upper GI endoscopy
142. Regarding amebic liver abscess, following statement is TRUE:
• Cyst of E. histolytica is infective form
• Amebic liver abscess is mostly solitary
• Open surgical drainage is the treatment of choice
• High grade fever is a common presentation
• Left lobe of the liver is the commonest site
143. Regarding Pyogenic liver abscess, following statement is TRUE:
• Infection is usually poly-microbial
• Plain CT scan abdomen is investigation of choice
• Open surgical drainage must be done if patient does not improve within a week
• after initiation of antibiotic therapy
• Pyogenic liver abscess is seldom fatal
• Single antibiotic is mostly used
144. Following is a monofilament suture material:
• Catgut
• Cotton
• Prolene
• Silk
• Vicryl
145. Shock can best be defined as:
• Hypotension
• Hypo-perfusion of tissues
• Hypoxemia
• Drowsiness
• Bradycardia
146. The most common site of Hydatid Cyst is:
• Brain
• Lung
• Liver
• Spleen
• Pancreas
147. Which of the following statement about pyogenic abscess of the liver is TRUE?
• Appendicular abscess is the most common underlying cause
• CT scan is rarely helpful
• Open surgical drainage is the treatment of choice
• Staphylococus aureus is the commonest organism
• The right lobe is more commonly involved than the left lobe
148. The evaluation of a comatose patient with a head injury begins with:
• Computed tomography of the brain
• Establishment of an airway
• Glasgow coma scale
• MRI brain
• Pupillary reflexes
149. The Hunterian Chancre is due to:
• Actinomycosis
• Anthrax
• Leprosy
• Syphilis*
• Tuberculosis
150. Cock’s Peculiar tumour of scalp is:
• A lipoma
• An epithelioma
• An infected haematoma
• A suppurating sebaceous cyst
• Chronic boil
151. If a trauma victim is alert, what is the quickest indication for an adequate airway?
• Absence of cyanosis
• Absence of stridor
• Adequate speech
• Tracheal shift
• Chest movement
152. Galeazzi’s fracture involves:
• Calcaneum
• Fibula
• Radius
• Scapula
• Talus
153. A thirty years old footballer sustained twisting injury to his right knee joint duringpractice
match. He has presented in emergency department after two hours of injury with pain and
swelling of right knee joint, which is locked in thirty degree flexion. Examination of right knee
joint revealed marked tenderness over medial aspect in the joint line. The most likely
diagnosis is:
• Anterior cruciate ligament tear
• Medial meniscus tear
• Medial meniscus with anterior cruciate ligament tear
• Posterior cruciate ligament tear
• Traumatic synovitis
154. Following is the tumour marker for Hepato cellular carcinoma:
• Alpha feto protein
• Carcinoembryonic antigen
• CA 19-9*
• CA 125
• Beta HCG
155. Commonest cause of acute pancreatitis in Pakistan is:
• Alcoholism
• Gall stone disease*
• HCV infection
• Snake bite
• Trauma
156. Verres needle is used during:
• Burr hole
• Cystostomy
• Chest intubation
• Laparoscopy*
• Epidural anaesthesia
157. Following gas is used for sterilization:
• Ethylene oxide*
• Ethylene dioxide
• Ethylene trioxide
• Ethylene tetraoxide
• Ethylene pentaoxide
158. For spinal anaesthesia following drug is used:
• Bupivacain*
• Cocain
• Lignocain
• Pentothal
• Neostigmine
159. Commonest dislocation of shoulder joint is:
• Anterior*
• Inferior
• Lateral
• Posterior
• Superior
160. H. Pylori infection is treated with:
• Single antibiotic
• Double antibiotics
• Triple antibiotics
• Proton Pump inhibitor alone
• Chili free diet
161. The commonest site of varicose ulcer is:
• Broaca’s area
• Gaiter’s area
• Tip of toes
• Calf area
• Heels
162. In acute pancreatitis, which of the following serum enzyme is raised first:
• Amylase
• Chymotrypsin
• Elastase
• Lipase
• Trypsin
163. Which of the following is true about upper gastrointestinal bleeding due to peptic ulcer
disease:
• In patients with normal GI anatomy, hematochezia will not occur.
• Bleeding will stop spontaneously in 70% to 80% of patients.
• Most patients will present with isolated melaena.
• The mortality rate of bleeding due to a peptic ulcer is 25%.
• Perforation may be accompanied by hemorrhage in 50% of cases.
164. 55-year-old woman presents with a laceration on her arm after falling while riding her
bicycle. She does not know the last time she had a tetanus booster, but wants to know why
she should have one. Which of the following is true regarding tetanus:
• Mortality for clinically evident tetanus is almost 50%.
• All patients with tetanus have a history of preceding injury.
• Wound cultures are helpful for diagnostic screening.
• Cardiac dysrhythmia is the most common cause of death.
• Tetanus boosters should be updated every year with clean wounds.
165. Most common carpel bone to get fractured is:
• Trapezoid
• Hamate
• Scaphoid
• Lunate
• Triquetrum
166. A 30yr old man had a road traffic accident and sustained fracture of femur. Two days later he
developed sudden breathlessness. Which of the following can be the most probable cause:
• Pneumonia
• Congestive cardiac failure
• Bronchial asthma
• Fat embolism
• Pulmonary embolism
167. Secondary haemorrhage usually occurs after:
• 7 hours
• 48 hours
• 5 days
• 7 days
• 1 month
168. Cystic artery is a branch of:
• Right hepatic artery*
• Left hepatic artery
• Common hepatic artery
• Right gastric artery
• Leftt gastric artery
169. Following is the pre-malignant condition for carcinoma of gall bladder:
• Ascariasis
• H. pylori infection
• Cholecystitis glandularis proliferans
• Porcelain gall bladder
• Typhoid infection of gall bladder
170. Pseudocyst Pancreas should be operated after:
• 6 hours
• 6 days
• 6 weeks*
• 6 months
• 6 years

Surgery A
1. A 25 years old man is brought to emergency ward with burns of face, left upper limb and chest
with full thickness burns of right upper limb. He is agitated, hypotensive and tachycardiac.
• High dose penicillin should be administered prophylactically.
• Tetnus prophylaxis is not necessary if patient has been immunized in last 3 years.
• The estimated area of burns is 60% total body surface area.
• The most sensitive indicator of adequacy of fluid resuscitation is heart rate.
• The patient should undergo immediate intubation for airway protection and oxygen
administration*
2. A 60 years old woman is to have a suspected melanoma excised from her thigh. It appears to be
a thin melanoma, less then 1 mm deep. The smallest margin to plan for the excision is:
• 3mm.
• 5mm.
• 1cm. *
• 3cm.
• 5cm.
3. A 60 years old woman presents with a cauliflower type skin lesion on hand for 10 years. She
gives a history of radiation treatment to that hand for eczema. Incision biopsy confirms the
diagnosis of squamous cell carcinoma. Correct statement concerning this lesion is:
• It is more malignant than basal cell carcinoma*
• It occurs more frequently in coloured people.
• It rarely metastasizes to regional lymph nodes.
• It should be treated by radiotherapy.
• It is rarely associated with chronic sun exposure.
4. A 50 years old man presents with a painless swelling on scalp. Close inspection shows a
punctum on it. What is the best treatment for the lesion?
• Excision.
• Enucleation.
• Incision & avulsion*
• Incision & drainage.
• Marsupialization.
5. A 40 years old man sustains gunshot wound to his right upper thigh just distal to inguinal
crease. Peripheral pulses are palpable in the foot, but the foot is pale, cool and hypo-
aesthetic. Surgical exploration is decided because the presence of palpable distal pulses not
reliably exclude:
• Significant arterial injury. *
• B. Significant injury to adjacent motor nerve trunks.
• C. Significant injury to adjacent long bones.
• D. Significant injury to adjacent veins.
• E. Subsequent development of compartment syndrome and need for fasciotomy.
6. A 25 years old man sustains a gunshot injury to his left thigh. Exploration reveals that a 5 cm
portion of superficial femoral artery is destroyed. Appropriate management includes:
• Debridement and end-to-end anastomosis.
• Debridement and repair with an interposition arterial graft.
• Debridement and repair with an interposition prosthetic graft.
• Debridement and repair with an interposition vein graft*
• Ligation & observation.
7. Changes consistent with peripheral vascular insufficiency are found in a patient presenting with
calf pain on walking 1 km. Distal pulses are absent, and ankle-brachial index is 0.5.
Conservative management is recommended in which of the following patients?
• Ischaemic ulceration.
• Ischaemic neuropathy.
• Claudication*
• Nocturnal foot pain.
• Toe gangrene.
8. A 50 years old has an industrial and presents with severely injured right lower extremity, which
is pale & pulse-less. Angiography reveals a popliteal artery injury with obstruction. At
operation popliteal vein is also found transected. The patient also has fracture of femur and
pelvis with tender abdomen. His blood pressure is 85/60. The surgeon plans ligation of
popliteal vein after arterial repair because:
• In severe popliteal vascular injuries venous ligation leads to decreased amputation rate
following successful arterial reconstruction.
• Venous ligation leads to decreased incidence of chronic venous insufficiency.
• Venous ligation leads to decreased operative time in patients with multiple injuries*
• In the presence of extensive associated soft tissue injuries venous return is already
sufficiently impaired to render repair pointless.
• Even through ligated veins thrombose, they often recanalize.
9. A 65 years old lady underwent right total hip replacement. Which one of the following test
represents her risk of a fetal pulmonary embolism?
• Less than 1%
• 7%
• 1-5%*
• 10%
• 15%
10. A 60 years old woman complains of calf pain and swelling following left hemicolectomy for
colon cancer. On investigation of deep vein thrombosis confirmed. What is the best early
treatment for this patient?
• Bed rest.
• Pneumatic compression stockings.
• Low-dose heparin*
• Full-dose heparin.
• Warfarin.
11. Mr.XYZ went to see twenty twenty world cup in south Africa in August 2007. 7 months later he
develops fever with rigors and chills, tender left grain lymph nodes with non-pitting oedema
of left leg and recurrent infections in that leg. What is the most likely diagnosis?
• Filariasis*
• Malaria.
• Lymph oedema praecox.
• Non-hodgkins lymphoma.
• Lymph oedema congenita.
12. 12) A neonate presents with a soft brilliantly transilluminant swelling in the base of the neck.
On aspiration lymph comes out. What is the most likely diagnosis?
• Lymphadenitis.
• Lymphangitis.
• Lymphangloma*
• Lymphangiosarcoma.
• Lymphoedema.
13. A 10 years old child sustained injury on left foot while working barefooted. Five days later he
developed fever and pain in left inguinal. Red streaks were seen running longitudinally in left
thigh. What is the diagnosis of these streaks?
• Acute lymphadenitis.
• Chronic lymphadenitis.
• Acute lymphangitis*
• Chronic lymphangitis.
• Cellulitis.
14. A patient is receiving mechanical ventilation after surgery. Arterial blood is sent for blood
analysis. The pCO₂ is 40. It tells us that:
• There is paradoxical acidosis.
• Alveolar ventilation is adequate*
• This patient is suffering from hypoxia.
• This patient is suffering from hypercapnoea.
• Rate of respiration should be increased.
15. On first postoperative day after major surgery, a 70 years old man vomits and immediately
goes into severe respiratory distress. The initial treatment should be:
• Tracheal intubation and suctioning. *
• Steroids.
• Intravenous fluid bolus.
• Cricothyroidectomy.
• High positive end expiratory pressure.
16. A 20 years old female has to get general anesthesia for appendicectomy. Which one of the
following is the best induction agent?
• Thiopentone.
• Nitrous oxide.
• Porpofol*
• Halothane.
• Succinyl choline.
17. A 20 year old nulliparous woman presents to you with a lump in the lower outer quadrant of
her right breast. It is 2/2 cm well defined mobile lump. The patient notice the lump three days
ago while taking the shower. There are no other complains and no family history of breast
cancer. The patient has been taking oral contraceptive pills regularly for the past three years.
For her initial work up you will send patient for:
• X-ray chest.
• Mammography and ultrasound right breast.
• Ultrasound right breast.
• Mammography.
• Ultrasound of right breast and fine needle aspiration cytology of right breast*
18. A 30 years old primigravida complains of headache, restlessness sweating and tachycardia.
She is 18 weeks pregnant and her blood pressure is 200/120 mmHg. Appropriate work up
includes.
• Exploratory laparotomy.
• Mesenteric angiography.
• Head CT scan.
• Abdominal CT scan:
• Abdominal ultra sonogram*
19. A patient is taking CAF regime as adjuvant chemotherapy for breast cancer. What is the
primary toxicity of Adriamycin (Doxorubicin)?
• Cardiomyopathy*
• Pulmonary fibrosis.
• Peripheral neuropathy.
• Uric acid nephropathy.
• Hepatic dysfunction.
20. A 45 years old lady presented to you with 1.5 cm infiltrating ductal carcinoma in the lower
outer quadrant of her left breast. You perform a wide local excision of the lump together with
a sentinel lymph node biopsy on the left side. Histopathology report comes back to show that
the lesion has been completely excised and the margins are not involved. The central lymph
nodes are free of any metastasis. The tumor is ER and PR positive. You will now offer the
patient.
• Simple mastectomy.
• Mastectomy with axillary node clearance.
• Tamoxifen.
• Chemotherapy.
• Radiotherapy and hormonal therapy*
21. A 40 years old female presents with a 3 cm lump in upper outer quadrant of her left breast.
FNAC confirms the diagnosis of infiltrating ductal carcinoma. Of the following lymph node
groups which one is likely to be involved first?
• Internal mammary.
• Axillary*
• Supraclavicular.
• Upper deep cervical.
• Abdominal.
22. A 60 years old male present with dysphagia. Flexible endoscopy shows a growth at cardia.
What is the distance of this growth from incisor teeth?
• 25 cm.
• 30 cm.
• 35 cm.
• 40 cm*
• 45 cm.
23. A 50 years old man presents with haematemesis. Endoscopy shows a bleeding peptic ulcer in
posterior wall of first part of duodenum. Which artery is involved?
• Right gastric artery.
• Right gastroepiploic artery.
• Gastroduodenal artery*
• Superior pancreaticoduodenal artery.
• Inferior pancreaticoduodenal artery.
24. A 50 years old man has undergone laparotomy through a midline incision. On 3dr post-
operative day there is serosanguinous discharge from the wound. A few stitches are removed
which reveal 1 cm dehiscence of abdominal fascia. Which of the following actions is most
appropriate?
• Removing all skin stitches and packing the wound with moist gauze.
• Starting intravenous antibiotics.
• Placing an abdominal binder*
• Prompt resuturing of fascia in operation theatre.
• Bed rest.
25. A 60 years old diabetic man undergoes excision of a 3 cm carbuncle on his back. The wound is
left open and packed. The wound grows smaller by day. Which of the following is true
concerning wound contraction?
• it is a process affecting closure of a sutured surgical wound.
• Infection significantly slows the process of contraction.
• It may account for a maximum 60% decrease in the size of wound.
• It is based on specialized fibroblasts that contain actin myofilaments*
• The percentage reduction of wound size is increased with increased adhesion of skin to
underlying tissues.
26. A 14 years old boy presents one hour after fall through a glass door. He has a 5 cm long tidy
wound on right wrist. There sno skin loss, but wound gap is 2 cm. examination reveals ulnar
nerve injury. How would you close his wound?
• Primary suturing*
• Delayed primary suturing.
• Secondary suturing.
• Skin graft.
• Skin flap..
27. A 40 years old man presents with a stab in the chest. A chest tube is placed, and 800 ml of
blood is drained, with subsequent drainage of 50 ml/h. resuscitation is best facilitated by
which of the following?
• Placement of central venous catheter through subclavian vein.
• Placement of central venous catheter through femoral vein.
• Bilateral sephanous vein cut down.
• Placement of large bore intravenous cannula*
• Placement of Swan Ganz catheter.
28. A 25 years old woman arrives in emergency room following an automobile accident. She is
acutely dyspnoeic with respiratory rate of 60/min. Breath sounds are markedly diminished on
right side. The fist step in the management of this patient should be:
• Take a chest X-ray.
• Draw arterial right pleural space.
• Decompress right pleural space*
• Perform pericardiocentesis.
• Administer intravenous fluids.
29. A 16 years old bay sustains head injury while doing one wheeling on his motorbike. He opens
his eyes to painful stimuli. His verbal response is confused. His motor response is flexion to
pain. What is his Glasgow coma scale?
• 3.
• 6.
• 9*
• 12.
• 15.
30. Two days after cholecystectomy, a 40 years old asymptomatic woman is found to have a
serum sodium level of 120 mEq/l. proper management would be:
• Administration of hypertonic saline solution..
• Restriction of free water*
• Plasma ultra filtration.
• Heamodialysis.
• Aggressive diuresis with furosemide.
31. A 70 years old man is suffering from arrhythmias on the night of coronary bypass. Potassium
has been administered. His urine output is 20-30 ml/h. serum potassium level is 6. Which of
the following medicines counteracts the effects of potassium without reducing the serum
potassium level?
• Sodium polystyrene sulfonate.
• Sodium bicarbonate.
• 50% dextrose.
• Calcium gluconate*
• Insulin.
32. A 50 years old woman presents with a 3 day history of fever, abdominal pain, nausea and
anorexia. She has not urinated for 24 hours. She has a history of previous abdominal surgery
for inflammatory bowel disease. Her blood pressure is 85/60, and pulse is 140. Her response
to this physiological state includes which of the following:
• Increase in sodium and water excretion.
• Increase in renal perfusion.
• Decrease in cortisol level.
• Hyperkalaemia*
• Hypoglycaemia.
33. A 65 years old woman with advanced stage oesophageal carcinoma presents to you in out
patient Department. On examination you find that she has prominent tendons which can be
easily palpated as well as prominent bony prominences of scapula. At this point the patient
has lost:
• More than 30% of body protein stores*
• 20-30% of body protein stores.
• 10-20% of body protein stores.
• 05-10% of body protein stores.
• Less than 5% of body protein stores.
34. A 40 years old man presents to you in the A&E department with 3rd degree burns of 60% body
surface area. Before injury his calculated energy expenditure was 1800 Kcal/day. What will be
his daily energy requirements now?
• 1600 k cal/day.
• 2300 kcal/day.
• 2800 kcal/day.
• 3600 k cal/day*
• 4500 kcal/day.
35. A 70 years old hypertensive asthmatic man who is receiving chemotherapy for colonic
carcinoma is diagnosed with chronic cholecystitis. He has pre-operative haemoglobin of
12g/dl and urinalysis show 20-25 pus cells & gram-negative bacteria. He receives intravenous
penicillin before sending to Operation Theatre. His abdomen is shaved in Operation Theatre.
Open cholecystectomy is performed and despite lack of indications the common bile duct is
explored. The wound is closed, and tube drain is placed through a separate stab wound. On
3rd post operative day, patient develops wound infection. Which of the following changes
could have made this wound a less favorable environment for infection?
• Decreasing the operation time and wound contamination by omitting the common bile duct
exploration*
• Pacing a tube drain exiting directly through the lateral corner of the wound.
• Using oral rother than intravenous penicillin perioperatively.
• Leaving a seroma in the wound to prevent desiccation of the wound.
• Reinforcing the wound closure with polypropylene mesh.
36. A patient suffering from blunt abdominal trauma requires a partial hepatectomy. He is rapidly
transfused with eight units of cross matched blood from blood bank. Post-operatively he is
found to be bleeding from intravenous sites. His coagulopathy is likely to be which clotting
factor/s?
• II only.
• II and VII.
• V and VIII*
• IX and X.
• XI and XII.
37. A patient who has to undergo elective cholecystectomy is taking aspirin for osteoarthritis.
Which of the following tests should be done prior to surgery?
• APTT.
• PT.
• Clotting time.
• Bleeding time*
• Reptilase time.
38. A 20 years old man receives multiple blood transfusions for abdominal gunshot wound. He
complains of numbness around his mouth and displays carpopedal spasm and a positive
Chvostek sign. The treatment required is:
• Intravenous bicarbonate.
• Intravenous potassium.
• Intravenous calcium*
• Intravenous digoxin.
• Intravenous parathormone.
39. A 50 years old woman undergoes laparotomy for adhesolysis. A feeding jejunostomy is also
done. When can feeding begin, using elemental formulae?
• 4-6 hours*
• 48 hours.
• 3 days.
• 5 days.
• 7 days.
40. During the endoscopy list on Saturday afternoon, you are using glutaraldehyde 2% (sidex). To
decontaminate flexible endoscopes. Glutaraldehyde 2% is effective against: The most likely
infecting organism is:
• Vegetative bacteria only.
• T.B., vegetative, spores and viruses.
• Vegetative bacteria and viruses only.
• Spores only.
• TB and spores only*
41. A 12 years old boy is brought in emergency with pain and inflammation at the ball of his left
foot and streaks extending up the inner aspect of his leg. He remembered removing wood
splinter from the sole of his foot on the previous day. The appropriate antibiotics to prescribe
while awaiting specific culture & sensitivity report is:
• Penicillin*
• Erythromycin.
• Tetracycline.
• Cloxacillin..
• Ofloxacin.
42. Which of the following statements is correct for pseudomembranous colitis?
• Surgical intervention is frequently required.
• After appropriate antibiotic therapy, the relapse rate is less than 5%.
• Diagnosis is usually based on clinical findings.
• If surgery is performed, a left hemicolectomy is usually adequate.
• Indications for surgical treatment include intractable disease, failure of medical therapy,
toxic megacolon, and colonic perforation*
43. A 60 years old farmer presents with growth on his lip. The biopsy report confirms squamous
cell carcinoma. Squamous cell carcinoma:
• Is a lesion which arises in areas of persistent hyperkeratosis*
• Accounts for More than 90% of cases that occur in the upper lip.
• Is a lesion that constitutes 30% of all cancers of oral cavity.
• Resistant to radiotherapy.
• Initially metastasizes to the ipsilateral posterior cervical lymph nodes.
44. A 70 years old man hypertensive asthmatic man who is receiving chemotherapy for colonic
cholecystitis. He has pre-operative haemoglobin of 12g/dl and urinalysis show 20-25 pus cells
& gram-negative bacteria. He receives intravenous penicillin before sending to Operation
Theatre. His abdomen is shaved in Operation Theatre. Open cholecystectomy is performed
and despite lack of indications the common bile duct is explored. The wound is closed, and
tube drain is placed through a separate stab wound. On 3rd post-operative day, patient
develops wound infection. Which of the following characteristics of the patient wight have
increased the risk of wound infection?
• History of a colon surgery.
• Hypertension.
• Male sex.
• Receipt of chemotherapy*
• Asthma.
45. A 70 years old man hypertensive asthmatic man who is receiving chemotherapy for colonic
carcinoma is diagnosed with chronic cholecystitis. He has pre-operative haemoglobin of
12g/dl and urinalysis show 20-25 pus cells & gram-negative bacteria. He receives intravenous
penicillin before sending to Operation Threatre. His abdomen is shaved in Operation Theatre.
Open cholecystectomy is performed and despite lack of indications the common bile duct is
explored. The wound is closed, and tube drain is placed through a separate stab wound. On
3rd post operative day, patient develops wound infection. Which of the following changes in
the care of this patient could have decreased the chances of post-operative wound infection?
• Increasing the length of pre-operative hospital stay to prophylactically treat the asthma with
steroids.
• Treating the urinary tract infection prior to surgery*
• Shaving the abdomen, the night prior to surgery.
• Continuing the prophylactic antibiotics for three post-operative days.
• Using a suction drainage system brought out through operative incision.
46. A 70 years old man with a 50 years old history of cigarette smoking presents with a non-tender
hard mass in the lateral neck. The simplest way to establish a diagnosis of a neck mass
suspected to be carcinoma is:
• Fine needle aspiration cytology*
• Bone marrow biopsy..
• Excision biopsy.
• CT scan of head and neck.
• Sinus X-rays.
47. A 35 years old woman has developed a 6 cm mass on her anterior thigh over past ten months.
The mass seems to be fixed to underlying muscles. The most appropriate next step in
management is:
• Above knee amputation.
• Excision biopsy.
• Incision biopsy*
• Bone scan.
• Abdominal CT scan.
48. A 40 years old man had a burn injury inside a confined space. He was in respiratory distress
and endotracheal tube was passed. Soot was suctioned from the wound. Which statement is
correct in the patient?
• Smoke poisoning is a thermal rather than a chemical injury.
• Carbon mono-oxide levels are not likely to be elevated unless there is evidence of skin or
oropharyngeal burns.
• Chest X-rays during early post-inhalation period shows a characteristics ground glass.
appearance.
• Damage to upper respiratory tract is common and is usually found on laryngoscopy.
• Patients with elevated carboxyhaemoglobin levels should be hospitalized for a minimum of
24 hours*
49. A 25 years old man is brought to emergency ward with burns of face, left upper limb and chest
with full thickness burns of right upper limb. He is agitated, hypotensive and tachycardiac.
Which statement is correct concerning initial wound management is this patient?
• Topical antibiotics should not be used as they encourage growth of resistant organisms.
• Early excision of facial and hand burns is especially important.
• Escharotomy should only be performed if neurologic impairment is imminent.
• Excision of areas of full thickness burns should be done 3-7 days after injury*
• Split thickness skin grafting should be done immediately for full thickness burn.
50. Unconcious patient brought to hospital after 1 hour of severe haemoraghe his bp is 50 mmhg,
pulse is rapid, cool n calmy skin, which mechanism is most active at this stage
• baroreceptors reflex
• Cerebral ischemic*
• Cushing reaction
• Chemoreceptors.
51. po2 60 mmhg, hgb 15. cause of injury.
• Hypoxic *
• Anemia
• Overutilization
• Histotoxic
52. primary amylodosis of increase immunoglobulin light chain
• Systemic*
• Reactive systemic
• Superior laryngeal nerve.
• Recurrent laryngeal nerve.
• Non-recurrent laryngeal nerve.
53. Superior parathyroid
• At the level of middle of thyroid gland on posterior surface at 1st tracheal ring( its actually
3rd ring) *
• Variable in position
• In front of peritracheal fascia
• Supplied by superior thyroid artery (inferioris the main supply)
• Upper deep cervical.
54. Patient has done clostomy, its now on parenteral diet.
• fatty liver
• hyperglycemia*
• hypophosphatemia
• Supraclavicular.
• Internal mammary.
55. Opiods are agonist, one with antagonist
• Pethidine
• Buprenorphine*
• Pentazocine (also correct bt is prototype)
• Morphine
• Fentanyl
56. Regarding ANTI-EPILEPTIC DRUGS, which one is INCORRECT a dizipam inhibits gaba channels
• Ethusuxsimide act via blocking of K channels*
• Phenobarbitaol...
• Phenytoin prolongs the inactivated stated on Na Channels
• Vigabatrin irreversibly block the
• enzyme
57. Lumbar puncture done through:
• L2 & L3
• L4 & L5*
• S1-S2
• L1 & L3
• 52 & S4
58. Subduaral hemaorhage
• Superior cerebral veins*
• Middle meningeal artery
• Tension pneumothorax.
• Diaphragmatic rupture.
• Blunt cardiac injury.
59. Most dangerous effect of diphtheria exotoxin on which organ?
• Heart*
• Larynx
• Intestine:
• Dextrose with saline.
• Saline with KCI.
• Ringer's.
60. Renin secretion is increased by?
• Decreased atrial strech
• Macual densa?
• Plasma osmolarity*
• Dinc K+ ?
• Neurogenic
61. Most characteristic symptom of esophageal atresia?
• Regurgitation of all of first feed
• Attacks of cough on feed (this one also by Wikipedia)
• Continuous dribbling*
• Increase in renal perfusion.
• Decrease in cortisol level.
62. After adrenalectomy, cause of death within few days is coz of absent of
• Aldosterone*
• Cortisol?
• bicarbonate.
• potassium.
• parathormone.
63. Which of the following is not neoplasia?
• Choriostoma*
• Memgioma
• Lymphoma
64. 40 years old lady with swelling in pain in knee joint, synovial fluid shows basophilic rhomboid
crystals:
• Calcium pyrophosphate*
• Monosodium urate
• Glycine
65. 90% of anthrax pts have lesions in
• Skin*
• Lungs
• Stomach
• Chest
• Navel
66. Glucose transport is directly proportional to its concentration, this phenomenon is known as:
• Active Transport
• Diffusion
• Facilitated Diffusion*
• Give half strength feed and reduce rate of feeding.
• Boil the feed before administration.
67. nutrient which need further digestion to be absorbed
• Sucrose*
• Tripeptides
• 60 high triglyceride content
• Chylomicrons
68. Cholesterol with negative coronary risk factor
• hdl*
• Idl?
• Proteus vulgaris.
• Escherichia coli.
69. Counter current multiplier
• Requires hypertonic medullary interstitium *
• Produce by vasa recta (maintained by it
• increased diffusion coefficient
• increased solubility
• more affinity
70. Preganlonic nerve fibres contain
• B fibres*
• congestion of liver
• pregnancy
• obesity
• alcohol
71. Muscle tension is decreased to prevent muscle rupture by
• Golgi tendons?? *
• Muscle spindle??
72. Drug to irreversibly impair platelet function
• Aspirin*
• Ibuprofen
• Achondroplasia
• Alpha1 antitrypsin defiency
• Neurofibramatosis
73. A patient comes to a doctor with a complain of pain down the medial side of right forearm &
hand, & wasting of small muscles of hand. On examination, circulation of right upper limb was
also impaired. The anatomical structure most likely to be involved is?
• Head of right humarus
• Left cervical rib
• Right 1st rib
• Right 2nd rib.
• Right cervical rib*
74. Pt presented with hemoptysis & hematuria. It progressed to renal failure. On biopsy there
were focal depositions with linear pattern
• Focal glomerulonephritis
• Rapid Progressive Glomerulonephritis
• Good pasture's*
• renin increases
• efferent arteriolar constriction
75. Scenario male 30 years splenomagaly fever malaise cardiac murmer and splinter hemorrhages
and hematuria. Most appropriate test
• Blood serology
• Blood culture*
• Stool culture
76. Scenario some weeks of disease, fever, lethargic, creatinine was 4, pedal edema n anemia hb,6
rbc 3.6, cause
• Erythropoiten deficiency*
• Acute renal failure (acute does not cause anemia)
77. sceanrio: pt of acromegaly. lips n bone enlarged
• Adenoma of adenohypophsis
• Adeoma of acidophilic cells*
• Adenoma of chromphobic cells
78. Most likely about mammary gland
• Extend vertically from 2 to 6 rib (its right also)
• Contains 15 to 20 lactiferous ducts *
• In pregnancy, contain lactiferous ducts and secretory alveoli
• Sebaceous glands in aereola
79. Scenario of post op. abdominal surgery, thromboembloism cause?
• Stasis n hypercoagubility*
• Stasis
• dothelial injury and
• Endothelial injury and hypercoagulability
80. Biceps reflex absent, knee n ankle reflex hyper, ventilation rate 16/ min, cause spinal cord
section
• L1 L2 L3
• C8 T1
• T8 T9
• C5 C6*
• C3 C4
81. Ventrical cortico spinal fibres ends on
• Lumbar
• Mid thoracic*
• Thoracic
• End of spinal cord
82. Standing from supine position
• Increase heart rate*
• Decrease cardiac output
83. Not of cardiogenic shock
• Hemaorragheb MI*
• CHF
• Dehydration
84. Water enter interstitium
• Filtration*
• Diffusion
• Cotransport with Na
• Decrease hydrostatic pressure
85. Most evident clinical feature of sjogrens
• Enlargement of salivary gland*
• Dry mouth n dry eyes
• Dry mouth only
86. Leprosy
• Intracellular acid fast*
• Intraceelualar non acid fast
• Extracelluar acid fast
87. Scenarios: child with skin eruptions along facial hair line, conjunctivitis, bluish white specs
with red margins at buccal mucosa (measles)
• Kopliks spot*
88. Not or fecal
• Triculais spirailis*
• Triculais tachuli
89. Hepatits B
• By sexual contact*
• b or fecal
90. Not a 2nd line drug in Rheumatoid arthritis
• Antibiotics*
• Anti malarials
• Golds
• Sulphazine
91. From under cook meat
• Tape worm*
• Liver fluke
92. Smooth muscles
• Arrange in longitudinal and circular layer*
• Metoprolol
• Striated muscle
93. Post traumatic patient, memory loss, area
• Hippocampus*
• a hypoxic
• anemia.
• overutilization
• histotoxic
94. Heart muscle show no tetnus an increase absolute refractory period88. Pancreatic fistula,
somatostatin:
• Inhibit other enzymes
• Dec secretions*
• Metoprolol
95. Somatosensory different from primary motor in
• Granular layer*
• At lateral side
• Has lager homunculus
96. 97) A patient in an operation requiring general anaesthesia has high BP which drugs should be
given
• Glycerine nitride?
• Halothane
• Hydralizine*
• Metoprolol
97. 98) Regarding medulla, correct one
• Forth ventricle in upper medulla*
• Fossa Ovalis
• Superior part of Sulcus Terminable
• Lower part of Crista Terminalis
98. 99) A 40 year Male working as a taxi driver in sharjah presented with C/O fever,weight loss,
anorexia. Diarrehea, spleenomegaly,he is emaciated, has jaundice and generalized
lymphadenopathy.whats the diagnosis.
• AIDS*
• Leishmaniasis
• Yellow fever
99. 100) Turner syndrome
• A 44X0*
• B. 44XXX
• 44XXXX
100. 101) Not present in AIDS
• sqaumous cell ca*
• primary brain lyhpoma
• kaposis's
101. 102) Scenario: nodule on dorsum of hand,same side axillary lymph nodes are enlarged,
happened due to? (melanoma case)
• Ultra violet radiations*
• Old job related asbestos
• Latex glove allergy
102. 103) A pt came to gingival hypertrophy, bleeding from gums, petachies, painful sore at lips.
You advice investigation to. Rule out
• Lymphoma?? *
• Leukemia??
• Bechet disease
103. 104) Source of creatinine
• Muscle*
• Heart
• Knee
104. 105) Pt with diabetic nephropathy, investigation of choice
• Albumin*
• Serum creatinine levels
• Creatinine clearance
105. 106) Scenario, pt having swelling on right neck, discharging, routine culture shows nothing.
now first initial investigation for diagnosis
• Biopsy*
• Zehil stain
• Ovulation
106. 107) Question regarding ovary's embryology
• All primary oocytes go in prophase of first meotic in fetal life*
• Most of oocyte undergo degeneration from birth to puberty
• Oocyte completes secondary miosis before ovulation something like that
107. 108) Which structure seperate maternal and fetal blood at last stages of pregnancy
• Cytotrophoblast
• Syncishiotrophoblast
• Cyto and synchio
• Synschiotrophoblast and fetal endothelium*
108. 109) 2nd division of trigeminal nerve NOT supply to
• One air sinus *
• Nasal cavity
• Nasal pharynx?
• Temporal bone
109. 110) Female having HIV but no AIDS CD count 800 now having middle ear infection SOM
(supartive otits media), cause
• Pneumocytes carinii (below 500 typical infections of aids get start) *
• Streptococci pneumonia
• M avium
• Sedomanas aueriginosa (cause ext otits media)
• Staph aureus
110. 111) Men having abcess at left thigh after injury from thorn, cause
• Staph aureus *
• Strept pyogens
111. 112) 19-year-old boy sitting in a well-ventilated room of 26'C. severely sweating, His pulse
120 beats/min, Bp 150/90. What's the diagnosis?
• Heat Stroke (surrounding/ambient temp should be raised in this)
• Mental Stress
• Exercise (Ans) *
• Sympatholytic drugs
112. 113) Most potent mechanism of preventing heat loss
• Shivering (potent mechanism for heat production)
• Vasoconstriction *
113. 114) Old lady, shortened limb, rotated laterally, cause
• Fracture neck of femur*
• Dislocation of hip (Anot dislocation, hip fracture can be true)
• Super gluteal
• Inf gluteal
114. 115) Asthma pt, most decrease volume is
• fev1*
• fev1/fvc ratio
• fvc
115. 116) Cynosis's cause
• Carbon monoxide poisoning (cherry red coloration)
• Deoxyhenoglobin less than 5mg/dl*
• Beta blockers
• Secretin
• Thiazides
116. 117) Regarding Aminoacids, which one of the following amino acids is not synthesized by the
human body and should be included in the diet.
• Alanine
• Tyrosine
• Phenylalanine*
• tryptophan
• Merkels disk
117. 118) Hering-Breuer reflex due to
• Stretch receptors in bronchioles*
• Irritants
• Receptors
118. 119) Protein bound fraction of the drug is:
• Active*
• Inactive
• More lipid soluble
119. 120) Scenario, mother rh negative, baby delivered at 34 weeks with anemia pal our, died
after 6 hrs after transfusion, most likely cause
• Basal ganglia staining*
• Cardiac malformations.
• Merkels disk
• Golgi tendons
• Free nerve
• Muscle spindle
120. 121) Standard deviation for
• Variability*
• Achondroplasia
• Marfans syn
• Alpha1 antitrypsin defiecncy
• Neurofibramatosis
121. 122) You are allowed to breech patient's confidentiality
• insurar claim
• If parents ask
• If pt authorizes you *
• After pt's death
122. 123) Cerebellum is connected to Midbrain by
• Superior Cerebellar Peduncle*
• Middle Cerebellar Peduncle
• Inferior Cerebellar Penduncle
123. 124) A 45 years female came with abnormal vaginal bleeding. biospy shows simple
hyperplasia due to?
• Increase stimulation with estrogen *
• Decrease stimulation with estrogen
• Increase stimulation with LH
124. 124) A patient was lying in spine position ABCESS will enter in lung
• Right upper lobe *
• Left posterior lobe
• Right posterior lobe
• Right apical superior
125. 125) Arterioles diameter
• 0.03 mm with naked eye?*
• ½ to 1mm
• 1mm
• 3mm
• 5 mm
126. 126. Which of the following have the greatest amount of blood?
• Capillaries
• Resistance vessels.
• Large veins*
• Arteries
127. 127) developed a tender Haphazard bundle of nerves (nodule) with pain at the site of
amputation. The nodule most likely is:
• Neuroma*
• Gangliomal
• Ganglionueroma
• Hamartoma
128. 128) adh acts primarily on
• PCT
• Cortical Ct*
• Medullary CT
• loop of henle
• DCT
129. 129) Which cells are not part of CNS
• Oligodendrocytes
• Padular astrocytes
• C..... astrocytes
• Schawn cells*
130. 130) Pus forming cavities in sinuses type or a patient came to ER with fever n multiple
abscess in neck region draining outside, which is the likely organism?
• Pseudomonas
• staph aureus
• Treponema pallidum
• Actinomyces israelli*
131. 131) Scenario: elderly, diplopia, granuloma, raised b.p
• Giant cell arteritis *
• Takayasu disease
• Wegeners Granulomatosis
132. 132) Scientist want to study about telomerase activity so which cells have rapid turnover
• Endothelial
• Erythrocyets
• Leukocytes
• germ cell*
133. 133) Apoptosis, most initial
• Activation of caspases*
• Phagocytosis
• Activates bcL2(its deactivated 1st)
• Spinocerebellar
134. 134) Sight of lodging of inhaled foreign body
• Right lower lobe *
• Giant cell arteritis
• Takayasu disease
• Wegeners Granulomatosis
135. 135) Adiation causes carcinoma in how many years?
• Les than 1 yr
• less than 2 yrs.
• 2-5 yrs
• 10 yrs
• 7-10 yrs*

Surgery B
1. A medical student has noticed a 3x3 cm nodule in thyroid isthmus. Fine needle aspiration
cytology (FNAC) shows follicular neoplasm. She is undergoing surgery. Minimum operation for
solitary thyroid nodule is:
• Excision of nodule
• Subtotal thyroidectomy
• Total lobectomy
• Total lobectomy plus isthmectomy
• Total thyroidectomy *
2. Following is the malignant condition of breast:
• Cyst sarcoma Phylloides
• Duct ectasia
• Fibro adenoma
• Mondor’s disease
• Paget’s Disease *
3. A cyclist was run over by a tractor on his lower trunk. The commonest injury associated with
fractured pelvis is:
• Colonic injury
• Ruptured urethra *
• Sciatic nerve injury
• Small gut tear
• Torn ureter
4. The commonest bone tumor is:
• Chondroma (in adults)*
• Fibroma
• Metastatic
• Osteoma
• Sarcoma (In children) *
5. A middle aged man reports to orthopedic outdoor with a swelling over posterior aspect of
right elbow joint for the last 3 years. He was diagnosed to be
• having Olecranon bursitis, which is also known is:
• Clergyman’s elbow
• Cricket elbow
• Golfer’s elbow
• Student’s elbow *
• Tennis elbow
6. A taxi driver has been brought to A & E department after sustaining a head injury. He was
admitted for neuro-observation. Feature which would cause concern during observation of
head injury patient is:
• Brady cardia *
• Hypotension
• Photophobia
• Pyrexia
• Tachycardia
7. Frey’s syndrome follows:
• Cataract surgery
• Mandibulectomy
• Parotidectomy *
• SMR
• Thyroidectomy
8. A 52 years lady has reported with a huge multinodular goiter with respiratory distress.
Tracheal obstruction in her is tested clinically by:
• Berry’s test
• Kocher’s test *
• Joffroy’s test
• Mobius test
• Nafziger test
9. A 44 years lady, mother of three children, reports to breast clinic with six months history of
blood stained discharge from her left nipple. Clinical examination is unremarkable. She is most
likely to be having:
• Duct adenoma
• Duct carcinoma
• Duct ectasia
• Duct fibroma
• Duct papilloma *
10. A middle aged smoker has undergone upper GI endoscopy for chronic dyspepsia. Endoscopist
has taken multiple biopsies from a growth in the ant rum. Gastric carcinoma is associated with
blood group:
• A*
• AB
• B
• O
• Both A & O
11. A middle aged female is having a reducible swelling over left groin for the past four months.
The most common hernia in females is:
• Direct inguinal hernia
• Femoral hernia
• Indirect inguinal hernia *
• Obturator hernia
• Umbilical hernia
12. Duodenal diverticulum is commonest in:
• First & second parts
• Second& third parts *
• Third& fourth parts
• First& fourth parts
• First& third parts
13. Pain abdomen is one of the commonest reasons for a surgical referral. Nonsurgical causes of
acute abdominal pain may include which of the following?
• Diabetic ketoacidosis *
• Hypoglycemia
• Hyperthyroidism
• Hypothyroidism
• Paget’s disease
14. Mallory-Weiss syndrome is:
• Congenital cardiac anomalies
• Congenital diaphragmatic hernia
• Esophageal cardiac tear *
• Prominent umbilical veins
• Type of nephrotic syndrome
15. Pseudo-cyst pancreas is one of the complications of acute pancreatitis. It should be operated
after an interval of:
• 6 hours
• 6 days
• 6 weeks *
• 6 months
• 6 years
16. Following is a translucent swelling in the scrotum:
• Haematocele
• Inguinal hernia in a baby *
• Testicular tumour
• Testicular torsion
• Varicocele
17. Following is not a complication of acute appendicitis:
• Appendicular abscess
• Appendicular mass
• Aden carcinoma appendix *
• Pelvic abscess
• Pyle phlebitis
18. A 13 years boy is diagnosed to be having bronchial cyst on right side of neck. Bronchial cleft
remnants most often present with which of the following clinical problem?
• Airway obstruction
• Hemorrhage
• Infection *
• Malignant degeneration
• Pain
19. A 2 months baby boy is going to be operated for congenital pyloric stenosis. He would be
presenting with which of the following biochemical abnormality?
• Acidosis
• Alkalosis *
• Hypernatremia
• Hyper chloraemia
• Paradoxical alkalosis
20. Murphy’s sign is present in:
• Appendicitis
• Chole cystitis*
• Hepatitis
• Pancreatitis
• Terminal ileitis
21. The most common complication in an elderly patient with benign prostatic hyperplasia (BPH)
following haemorrhoidectomy is?
• Fecal impaction
• Bleeding
• Urinary retention *
• Infection
• Anal stenosis
22. A lady with gall stones is having biliary colic. Her pain would be radiating towards:
• Umbilicus
• Right shoulder *
• Hypogastrium
• Right iliac fossa
• Left iliac fossa
23. Patient with bladder stones are at increased risk for which of the following bladder cancers?
• Adenocarcinoma
• Transitional cell carcinoma
• Squamous cell carcinoma *
• Choriocarcinoma
• Carcinoma in situ
24. A patient has testicular swelling. It is non tender, trans-illuminant, and testis and epididymis
are not definable. He has:
• Hydrocele *
• Haematocele
• Epididymal cyst
• Spermatocele
• Varicocele
25. Commonest dislocation of hip joint is:
• Anterior
• Inferior
• Lateral
• Posterior *
• Superior
26. Treatment of renal adenocarcinoma is:
• Chemotherapy
• Simple Nephrectomy
• Partial Nephrectomy
• Radical Nephrectomy *
• Radiation
27. In case of Litter’s hernia, the hernia sac contains:
• Appendix
• Meckler’s diverticulum *
• Testis
• Ovary
• Bladder
28. Raised prostate specific antigen (PSA) is suggestive of:
• Carcinoma pancreas
• Carcinoma liver
• Carcinoma prostate *
• Carcinoma ovary
• Carcinoma rectum
29. What is raised in acute pancreatitis?
• Serum alkaline phosphates
• Serum calcium
• Serum amylase *
• Serum albumin
• Serum alenine transaminase (ALT)
30. Following is not a part of Tetra logy of Fallout:
• Ventricular septal defect VSD
• Atrial septal defect ASD *
• Overriding aorta
• Pulmonary stenosis
• Right ventricular hypertrophy
31. Carcinoma of the breast is the commonest tumor in females. Following is not a component of
Triple assessment:
• Clinical examination
• Mammogram
• Son mammogram
• Magnetic resonance imaging (MRI) *
• Fine needle aspiration cytology (FNAC)
32. Greenish nipple discharge is a feature of:
• Duct carcinoma
• Duct papilloma
• Duct ectasia *
• Duct sarcoma
• Duct abscess
33. Monder’s Disease of breast involves:
• Arteries
• Veins *
• Nerves
• Ducts
• Cooper’s ligaments
34. The commonest location of osteogenic sarcoma is:
• Around wrist
• Around elbow
• Around hip
• Around knee *
• Around ankle
35. Following is not a feature of cardiac tamponade:
• Engorged neck veins
• Hypotension
• Tracheal shift *
• Muffled heart sound
• None of the above
36. Following is not a life threatening chest injury to be ruled out in primary survey:
• Tension pneumothorax
• Massive haemothorax
• Bronchial obstruction *
• Flail chest
• Open pneumothorax
37. The commonest site of perforated duodenal ulcer is:
• First part *
• Second part
• Third part
• Fourth part
• Duodeno-jejunal junction
38. A 70 years male smoker is having progressive dysphagia for which he has under gone
endoscopy, which has revealed a growth in proximal esophagus. Biopsy is likely to show:
• Adenocarcinoma
• Sarcoma
• Squamous cell carcinoma *
• Transitional cell carcinoma
• Anaplastic carcinoma
39. Raised PSA is suggestive of:
• Carcinoma pancreas
• Carcinoma liver
• Carcinoma prostate*
• 4 Carcinoma ovary Carcinoma rectum
40. 10. The commonest site of anemysm is
• Ascending aorta
• Descending aorta
• Abdominal aorta*
• Common carotid
• Renal artery
41. 12. Carcinoma of the breast is the commonest tumour in females. Following is not a
component of Triple assessment
• Clinical examination
• Mammogram
• Sonomammogram
• FNAC
• e X ray chest*
42. 16. Which one of the following is the malignant condition of breast?
• Cystosarcomalhylloides
• Duct ectasia
• Mondor's disease
• Paget's Disease*
43. 22. Frey's a y's syndrome follows:
• Cataract surgery
• Mandibulectomy
• Parotidectomy*
• SMR for DNS surgery
• C. Thyroidectomy
44. 24. A 52 years lady has reported with a huge multi nodular goiter with respiratory distress.
Tracheal obstruction in her is tested clinically by:
• Berry's test
• Kocher's test
• Joffroy's test*
• Mobius test
• Nafziger test
45. 28. Treatment t of renal adenocarcinoma is:
• 4 Chemotherapy a
• b. Simple Nephrectomy
• Partial nephrectomy
• Radical nephrectomy *
• Radiation
46. 30. Pre-tibialMyxoedema occurs in:
• a Chronic venous insufficiency
• b. Hypothyroidism
• Lymph-edema
• 4. Nephrotic syndrome
• Thyrotoxicosis*
47. 34. A 13 years boy is diagnosed to be having branchial cyst on right side of neck. Branchial
cleft remnants most often present with which of the following clinical problem?
• Airway obstruction
• Hemorrhage
• Infection*
• Malignant degeneration
• Pain
48. 37. A 2 months baby boy is going to be operated for congenital pyloric stenosis. He would be
presenting with which of the following biochemical abnormality:
• Acidosis
• b. Respiratory Alkalosis
• Hyporchloraemicalikylosis*
• Hypernatremia
• Alvarado al alkalosis
49. 40. Following is NOT feature of cardiac tamponade:
• Engorged neck veins
• Hypotension
• Tracheal shift
• Muffled heart sound
• Normal breath sounds*
50. 42. five years boy is having high grade fever and swelling of leg for the last two days. Acute
osteomyelitis in right leg children usually begins at:
• Diaphysis
• Epiphysis
• Metaphysis*
• Periosteum
• Synovium
51. 61. Monder's Disease of breast
• Arteries
• Veins*
• C. Nerves
• Ducts
• Cooper's ligaments

Prof 2016
1. Regarding hernias in females, the most common is:
• Epigastric
• paraumbilical
• Femoral
• Inguinal*
• Hiatal hernia
2. A 35-year-old professional weightlifter presents with a red and swollen lump in the left groin.
An inguinal hernia suspected and, at the time of operation, the lump is found to contain a
loop of necrotic small intestine. This type c hernia is best described as:
• Irreducible
• Strangulated*
• Obstructed
• Sliding
• Richter's
3. Which of the following tumour markers is most commonly associated with pancreatic cancer?
• Carcinoembryonic antigen
• Cancer antigen (CA)125
• CA19-9*
• alpha-fetoprotein (AFP)
• Beta-human chorionic gonadotrophin
4. A young man has presented with pain in right lawer quadrant. Pain initially started in peri-
umbilical region. The reason for localization of pain in right lower quadrant is:
• Appendicular colic
• Distension of appendix
• C. Intestinal colic
• Peritoneal irritation*
• Referred pain
5. Ranson criteria is used widely in surgical IC to predict the to predict the severity of an attack of
acute pancreatitis on outcome of patients of acute pancreatitis. Which of the following is not
a parameter admission in Ranson score?
• White cell count
• Serum amylase*
• Age
• Blood glucose
• 3. LDH and AST
6. A 55 years old male is having a lump over right of mandible for the last 2 years.Commonest
surgical disease of parotid is:
• Calculus
• Carcinoma
• c lymphadenitis
• Parotitis Pleomorphic adenoma*
7. A 42-year-old lady is involved in a road traffic accident. She presents with a dislocated
shoulder. The shoulder is relocated by you in A&E but afterwards you noticed winging of the
scapula. Which nerve is likely to be damaged?
• Axillary nerve
• Median nerve
• Radial nerve
• Long thoracic nerve of Bell*
• Ulnar nerve
8. A 46-year-old woman is diagnosed with Invasive ductal carcinoma of the right breast stage
TINOMO. The mos appropriate treatment option for this patient is?
• Chemotherapy
• Brest conservation surgery*
• Mastectomy postoperative radiotherapy
• Palliative care programme
• Radical mastectomy
9. A 35-year-old man is knocked off his cycle and hits his head on a stone. He is brought Into
hospital with He underwent a CT scan of skull which shows a biconvex-shaped haematoma a
fluctuating on the Glasgow Coma Scale. What is his most likely diagnosis?
• Extradural hematoma*
• 6. Intra cerebral hematoma
• Subdural haematoma
• Cerebral contusion
• Subarachnoid haemorrhage
10. Radiotherapy is the treatment for which of the following cancers?
• Adenocarcinoma of the oesophagus
• Rectal carcinoma
• Squamous cell Anal cancer*
• Gastric carcinoma
• Phylloides breast tumour A
11. patient is known to have exposure to beta-naphthylamine. What is he/she at increased risk of
developing?
• Small-cell lung carcinoma
• Bladder cancer*
• Breast cancer
• Chemical pneumonitis
• Lymphoma
12. A man undergoes an open inguinal hernia repair. During the procedure the spermatic cord is
visualized. structures does this contain?
• Dartos muscle
• Femoral branch of the genitofemoral nerve
• lio inguinal nerve
• Inferior epigastric
• artery Pampiniform plexus*
13. Splenectomy patients are at particular risk of verwhelming sepsis from which organisms?
• Anaerobic bacteria
• Bacteroidesfragilis
• Haemophilus influenzae*
• Fungi
• Staphylococcus aureus
14. Gastrointestinal consequences of major burns include which of the following?
• Splanchnic vasodilation
• Acute gastric dilatation
• Cushing's ulcers*
• Terminal ileal hyperplasia e. Mechanical bowel obstruction
15. Thyrotoxicosis is characterized by which of the following?
• Weight gain
• Proximal myopathy*
• Enopthalmos
• Bradycardia
• Slow tendon reflexes
16. Which of the following is associated with hyperparathyroidism?
• Paravertebral ossification
• Peptic ulcer*
• Q.22
• Retinal haemorrhages
• Aortic stenosis
• Hoarse voice
17. In a 65-year-old women presents with a retrosternal goitre discovered on a routine chest X-
ray, which of following clinical sien can be present?
• Tracheal deviation
• Berry's sign
• Pemberton's sign*
• Stellwag's sign
• Pretibial mykedema
18. Which statement if true of Meciel's diverticulam?
• is present in 20 per cent of the population
• Arises from the mesenteric border of the
• May contain heterotropic pancreas*
• Is present only in mates
19. Is a diverticulum of the bladder cause which of the following malignancy?
• Human papilloma virus (HPV) can
• Carcinoma of breast
• Carcinoma of anal canal*
o Carcinoma of esophagus
• Bladder carcinoma
20. A worried 23-year-old girl presents with a painless sinele lump in the right breast of one
month duration. examination, a mobile, discrete, welldained, ron-tender, firm 1 cm lump is
found. There is no axill lymphadenopathy. The most likely diagnosis is
• Gastric carcinoma
• Breast cyst
• Lipoma
• Fibroadenoma*
• Fibrocystic disease
• Carcinoma of the breast
21. During a cholecystectomy, the cystic artery must be located and lizated. This artery most
commonly arises from the
• Gastroduodenal artery
• Hepatic artery proper
• Right hepatic artery*
• left hepatic artery
• Superior pancreatico-duodenal artery
22. The commonest malienancy of thyroid is:
• Anaplastic carcinoma
• Follicular carcinoma
• Hunhle cell carcinoma
• Medullary carcinoma
• Papillary carcinoma*
23. A 31-year-old man is admitted following an assault outside a cinema. During the fight he was
hit by a blunt obje across the side of the head. On admission his Glasgow Coma Scale score is
Initially 12/15 but falls to 8/15 during evaluation. The decision is taken to perform a computed
tomography head scan, which identifies a biconvex-shape cranial vault. The diagnosis is:
• space-occupying lesion within the
• Extradural haematoma*
• Subdural haematoma
• Subarachnold haemorrhage
• Cerebral contusion
• Intracerebral haemorrhage
24. A 58-year-old patient presents with a 6-week history of Increasing difficulty in swallowing. He
first noticed problems wh eating meat which became stuck behind his heart, but this gradually
began to Include other foods. The patien currently worried because he is now struggling with
thick fluids and has noticed some involuntary weight loss. What is most anocondate first
Investigation?
• Staging computed tomography
• b. Barium meal
• -Upper gastrointestinal endoscopy and biopsy*
• Barium swallow
• MRI
25. claw hand is usually associated with injury to which of the following nerves?
• Axillary nerve
• Musculocutaneous nerve
• Radial nerve
• Median nerve
• Ulnar nerve*
26. A 45-year-old man presents to the emergency department with a history of coffee ground
vomiting. He also repor that for 2 days his stool has appeared darker than usual. Which of the
following gives the most sensitive ruide as 1 the severity of his gastrointestinal haemorrhage?
• Haemoglobin*
• b. Fecal occult blood test
• Pulse rate
• Volume of vomitus/melaena
• Lying and standing blood pressure)
27. A patient presents with a short history of perfuse, projectile vomiting without bile staining. He
has a history c peptic ulceration and chronic dyspepsia and has noticed increased bloating
over the preceding 9 months. O examination there is distension in the epigastric region and a
succession splash. The abdominal radiograph shows grossly distended stomach and collapsed
bowel. The most likely causos:
• Carcinoma of the esophagus
• Carcinoma of the head of pancreas Gastric outlet obstruction*
• Compression by malignant nodes.
• Chronic pancreatitis
28. A 29-year-old patient presents with a short history of right upper quadrant pain. She is
jaundiced with dark urine and pale stool. She has a fever of 38.9 °C. Abdominal examination
does not reveal any mass. The diagnosis is:
• Ascending cholangitis
• Acute cholecystitis
• Biliary colic with bile duct obstruction*
• Pancreatitis
• Mirizzi's syndrome
29. A patient was admitted for acute pancreatitis secondary to alenhol misuse. He was treated
conservatively and discharged for outpatient follow-up. He presents to your clinic after 8
weeks complaining of continued abdominal pain in the epigastrium radiating Into the back.
Serum amylase is 200 IU/L. On examination he is tender over epigastrium. Blood pressure,
pulse rate and temperature are all unremarkable. The diagnosis Is: the
• Pancreatic pseudocyst*
• b. Cholecystitis
• Chronic pancreatitis
• Recurrent acute pancreatitis
• Pancreatic abscess
30. A 22-year-old female patient presents with a 5-week history of bloody motions. She has
noticed some pain and significant weight loss over the preceding 6 weeks with increasing
lethargy and fatigue. She has previously had constipation and admits to regular laxative use.
What is the most likely diagnosis?
• Bowel cancer
• Irritable bowel syndrome
• Diverticular disease*
• Inflammatory bowel disease
• Anal fissure
31. Which one of the following does NOTorcur as a systemic manifestation
• 3 Ankylosing spondylitis
• b. Pyoderma gangrenosum
• c. Scleritis
• Osteoporosis*
• Sclerosing cholangitis
32. An 15-year-old patient presents with a 5-day history of abdominal pain and pyrexia. On
examination A mass in t right illac fossa is palpable. Bowel sounds are present. an
appendicular mass with an associated abscess. The patient is started on metronidazole and
admitted for t. Abdominal computed tomography confirms the diagnosis of observation and
conservative management. After 2 days remains raised. The next stage in management Is: the
mass has not reduced in size and the temperature
• Continue metronidazolé for further 14 days t cefuraxime
• Proceed to appendicectomy
• Drainage of Appendicular abscess*
• Colonoscopy
• Laparoscopy
33. A patient with a previous anal abscess presents with persistent perianal discharge and
discomfort. On examination a sinus is identifiable at the 6 o'clock position with the patient in
the lithotomy position.. What procedure the surgeon is most likely to perform?
• 3. Seton insertion
• b. Colostomy
• Open exploration of tract*
• 1&D
• Lateral Syphincterotomy
34. Which of the following is NOT an option when treating an anal fissure?
• Propranolol*
• Botulinum toxin A injections
• c. Glyceryltrinitrate cream
• Diltiatem cream
• Laxatives
35. Which of the following is most Important sign associated with acute appendicitis?
• 3. Rebound tenderness*
• b. WBC of 8000/ cmm
• Psoas sign
• Rovsing sign
• Murphy's sign
36. All of the following may cause a right illac fosse mass that is palgable on abdominal
examination, except:
• Carcinoma caecum
• Ulcerative colitis*
• Tuberculosis
• Appendicitis
• Ovarian cancer
37. of the following is the most common site for colonic carcinoma?
• Rectum
• Sigmoid colon*
• Caecum and ascending colon
• Splenic flexure
• Transverse colon
38. The most common cause acute intestinal bowel obstruction is: Postop.
• Adhesions*
• b. Intestinal parasites
• c. Volvulus
• Hernia
• Neoplasia
39. your man, with a 2-year history of dyspy urought to the emergency department following ad
onset of severe epigastric pain. The pain is made worse on movement. On examination, the
abdomen is rigid, moving with respiration and bowel sounds are absent. A plain film chest
radiograph reveals free ale under diaphragm. The most likely diagnosis is:
• Perforated appendicitis
• Acute cholecystitis
• Acute pancreatitis
• Myocardial infarction
• Perforated peptic ulcer*
40. A 26-year-old woman arrives at the emergency department with unbearable Intense right iliac
fossa pain. Ear that day, she was experiencing on and off moderate pain in the umbilical area
which gradually moved over to right liliac fossa. Associated symptoms include anorexia,
nausea and vomiting. On examination, the patien pyrexial and there is rebound tenderness
and guarding over the right illiac fossa. A beta-human choric gonadotrophin test is negative.
What should you do next?
• Send the patient to the emergency operating theatre for an Appendicectomy*
• b Alert the obstetrics and gynaecology team. suspecting that she may have a ruptured
ectopic pregnancy
• C. Manage the patient medically in the emergency department
• Order an CT scan of the abdomen
• Send the patient for a plain film radiograph of the abdomen
41. A 12-year-old boy is admitted to the emergency department with sudden onset of severe right
testicular and lov abdominal pain during athletic training. He had one episode of vomiting and
constantly feels nauseous.. examination, the patient is sweating and in unhearable pain.
There is marked tenderness and swelling of the ri testicle which is observed to be lying
horizontally. What is the most appropriate next step in this patien management?
• Order a CT scan of abdomen
• Send the patient immediately for emergency surgical exploration of the scrotum*
• Perform urine dipstick
• Manage the patient with analgesia and observe
42. Obtain a second opinion from your senior colleague, who will only be able to see the patient
in an hour A 45-year-old man is brought in with an acute met of cnicastric pain, nausea and
severe vomiting. The pain worse on movement and is only relieved slightly by leaning
faoward. The patient is an alcoholic, On examination th patient is pale, tachycardic, pyrexial
and dehydrated. The abdomen is diffusely tender and soft, and bowel soun are normal. The
most likely clinical diagnosis is:
• Perforated peptic ulcer
• Small bowel obstruction
• Acute cholecystitis
• Acute pancreatitis*
• Acute appendicitis
43. A 78-year-old man presents to the emergency department with severe pain arising from The
patient is also experiencing central colicky abdominal pain. On examination, the abdomen is
generally tende his hernia in the left groli distended and bowel sounds are raised.
Examination of the hernial orifices reveals a left-sided, Irreducible, tens and extremely tender
inguinal hergja. The overlying skin of the hernia Is warm and erythematous. What is the mo:
appropriate course of action in managing this patient?
• Alert theatre and send patient for emergency surgery*
• b. Request a computed tomography scan of the abdomen
• c. Request an ultrasound
• Attempt to reduce the herla
• Observation for 24 hours
44. You are asked to see a 48-year-old woman who has been admitted to the emergency
department with sudden onse of high grade fever with rigors and is Jaundiced. What is the
most likely diagnosis that you should be thinking of?
• Biliary colic
• Acute cholecystitis
• -Cholangitis*
• Gallstone ileus
• Pancreatitis
45. A 59-year-old diabetic man presents to the acute surgery unit with a 5-day history both the
penis and the scretum to be swallen, red and tender to touch, with also extending into th
feeling generally unwelt. On examination, he is found to be pyrexial and tachycardic. A genital
CAR both aerobic and anaerobic organisms), and fluid resuscitation and broad-spectrum
antibiotics are commence bilaterally and there is palpable crepitus in the perineum Houtine
bloods and cultures are taken (which fater a urologist, he is taken promptly into the operating
theatre for definiti Following further discussion with manarement. The likely diagnosis in this
patient is:
• Fournier's gangrene*
• b. Epididymo orchitis
• Testicular tumour
• Testicular torsion
46. A 75-year-old man is admitted to the emergency department with acute-onset suprapubic
pain and inability to pass urine for 12 hours. On examination, the patient is in discomfort,
neurologically intact, and the abdomen Is particularly tender in the suprapubic region. A
digital rectal examination reveals a smooth, enlarged prostate. What is the most likely
diagnosis?
• Prostatitis
• Bladder outflow obstruction due to prostate cancer*
• Bladder outflow obstruction due to benign prostatic hypertrophy
• Bladder outflow obstruction due to a urethral stricture
• Bladder outflow obstruction due to a spinal cord lesion
• Bladder outflow obstruction due to bladder stone
47. A 70-year-old man is admitted to the emergency department with acute-onset suprapubic
pain and inability to pass urine for 2 days. On examination, the patient is in discomfort,
neurologically intact, and the abdomen is particularly tender in the suprapubic region. A
digital rectal examination reveals a smooth, enlarged prostate. From the list of take in
managing the patient. options below, select the most appropriate course of action to
• Suprapubic catheterization
• Obtain bloed for urea and electrolytes sampling
• Request an abdominal plain film radiograph Urinary catheterization*
• Request an intravenous urogram
48. As part of the peripheral vascular examination, you are asked to record the ankle-brachial
pressure index of the patient. Which one of the following values reflects a normal ankle-
brachial pressure Index?
• Between 09 and 0.6
• Greater than 1.3
• C. Between 0.6 and 0.3
• Greater or equal to 1.0*
• Less or equal to 0.3
49. A 65-year-old man presents for the first time to your clinic with a painless wound in his right
leg, which has beer present for over 2 months. On examination you notice a 3 cm ler uicer in
the raiter area of the right lez.above the medial malleolus. The shallow bed of the ulcer is
covered with granulation tissue, which is surrounded by slopin edges. There is no history of
trauma. From the list below, choose the most likely diagnosis.
• ischemic leg ulcer
• ulcer Venous ulcer*
• Traumatic ulcer
• marjolin ulcer
50. A 19-year-old medical student presents with acute-onset colicky left loin gain and describes a
history suggestive urinary calcull. Which one of the following would be the initial investigation
of choice to determine the presence of calculus In the renal tract?
• KUB (kidney ureter bladder) radiograph*
• Intravenous urogram
• Flexible cystoscopy
• Diethylenetrlaminepentaacetic acid (OTPA) scan
• .e. U/S KUB
51. ARALLY esents with his parents to the time of the boy's circumcision were told by the doctor
that the urethral meatus was not in the nor ion. On gnosis? pacciatric ursiogy outpatients
department. The par the most i
• Hypospadias*
• Epispadias
• Phimosis
• Chordee
• Perineal urethra
52. A 35-year-old carpenter is diagnosed with a seminoma. To which lymph nodes does a
seminoma spread first?
• Para aortic lymph nodes*
• b. Superficial inguinal lymph nodes
• c. Anterior cervical chain
• Posterior cervical chain
• Deep inguinal lymph nodes
53. A patlent presents to surgical outdoor clinic complaining of a swelling on his wrist. On
examination, there is a fo swelling on the dorsal aspect of the wrist. It is smooth and non-
tender. The overlying skin is normal and mo freely over the mass, however, it seems to be
fixed to the tendon. What is the likely diagnosis?
• Sebaceous cyst
• Lipoma
• Ganglion*
• Glant cell tumour of the tendon sheath
• e Fibroma
54. Which one of the following organisms is most commonly associated with septic arthritis?
• Gram negative bacill
• Haemophilus influenta
• Seta haemolytic streptococci
• Streptococcus pneumonia
• Staphylococcus aureus*
55. An 85-year-old pensioner is brought to the emergency department by his family who are
concerned that he hi become increasingly confused and drowsy in the past 3 weeks. He is
confused and unable to recall events clear but oriented to time and person and complains
only of occasional frontal headache. His family informs you that t may have fallen while
climbing from the bathtub some weeks previously. He has also started sleeping for lor periods
of time, which is not her normal habit. A head CT scan is performed, which shows mild
generalized atrophy and a crescent-shaped collection. This presentation is consistent with:
• Intracerebralhaemorrhage
• Subarachnoid haemorrhage
• Extradural haemorrhage
• Chronic Subdural haemotoma*
• Intra ventricular haemorrhage
56. You are asked to assess a 25-year-old patient who returned from theatre 2 hours ago
following a thyroidectomy fo a lante, hyperplastic goitre and is now complaining of difficulty
in breathing. On examination you noted that she very short of breath with a respiratory rate
of 30 breaths/min, using her accessory muscles of respiration and onl able to answer your
questions in two or three words. In addition, there appears to be a fluctuant mass in the
midlin of the neck underlying the surgical clips. Immediate management of this patient would
be:
• High-flow oxygen via Hudson mask
• Removal of surgical clips at the bedside and open the incision*
• Intravenous access with two large-bore cannulae and fluid resuscitation
• Removal of surgical clips in theatre under general anaesthesia
• Call your senior and wait for him/her to remove the clips
57. A 28-year-old secretary presents with a lump in the midline of the neck that has grown
progressively larger over the past several months. On examination, there is a palpable lymph
node in the left submandibular region. FHAC has confirmed a malignant cancer of the thyroid
gland. The origin of this is most likely to be:
• a Follicular
• Anaplastic
• Medullary
• Lymphoma
• Papillary*

MCQs-SURGERY
1. After intramuscular injection over Lt. Triceps muscles a young girl presented with difficulty in
Dorsiflexion of her wrist. What is most likely cause of this problem?
• Extenser muscle diseases
• Radial nerve injury
• Medial nerve injury
• Ulnar nerve damage
• Axillary nerve damage
2. A young boy presented with a discharging sinus from his Rt. Tibia. He had fracture due to road
traffic accident four months back. What is your diagnosis?
• Mal-union
• Non-Union
• Osteomylitis
• Cellulitis
• Malignancy
3. A lady of five kids presented with soft swelling over periumbilical region. It is nonreducible
• and non tender. What is your diagnosis?
• Lipoma badomen
• Paraumbilical hernia
• Any tumor
• Incisional hernia
• Ca stomach
4. A boy of 20years presented with pain fever and soft cystic mass mover his Rt. Iliac fossa
Ultrasonography confirms cystic nature of mass. What is your diagnosis?
• Appndicular Mass
• Ceacal carcinoma
• Appendicular abcess
• Carcinoid tumor
• Illeoceacal Uhron’s disease
5. An incisional hernia is best treated by
• Repair with catgut
• Repair with prolene suture
• Repair with onlay prolene mesh
• Repair with steel wires
• Conservative treatment
6. Vericose veins of the legs after DVT are best managed by
• Injection selerotherapy
• Multiple vernous ligations
• Great sphanous vein stripping
• No surgical excision of superficial vericosities
• Laser ablation
7. A 70 years male who had recent heart attack developed severe pain left leg. Foot appears pale
and pulses are absent. What is the cause of this vascular emergency?
• Abdominal aortic aneurysm
• Rapture of aorta
• Thromboembolism
• Atherosclerosis
• Dissected aortic aneurysm
8. IV fluids needed for resuscitation if an adult has
• More than 15% burns
• Less than 10% burns
• 40% burs
• Leas than 80%
• Less than 5%
9. Pain control in benign disease is managed by following. Please mark incorrect option?
• Pre-emptive analgesia in order to avoid phantom pain
• Local anesthetic or steroid infiltration around nerve roots to reduce spasm
• Transcutaneous nerve stimulators
• Neuralgias respond to decompression of nerves
• On demand analgesia
10. In a case of failed attempted Endotracheal intubation. The best choice is
• Pass air way
• Do tracheotomy
• Laryngeal mask airway
• Face mask
• Naso tracheal intubation
11. Which of the following nerve is considered to be more vulnerable to injury during
• parotid surgery?
• Hypoglossal nerve
• Glossopharyngeal nerve
• Fascial nerve
• Vagus nerve
• Lingual nerve
12. Which of the following is common cause of bacterial sialadenitis
• Streptococcal pyogenes
• Atypical mycobacteria
• Staph aureus
• Pseudomonas aeruginosa
• Klebsiella
13. A young boy developed painful bilateral parotid swelling. Which of the following
• complication can occur in this patient?
• Salpingitis
• Orehitis
• Typhlitis
• Proctitis
• Oophoritis
14. A 44 year old lady was operated for simple MNG and subtotal thyroidectomy was done. On
3rd post operative day she felt that pitch of her voice is decreased. Which complication has
occurred?
• Damage to external Laryngeal nerve
• Damage to recurrent laryngeal nerve
• Damage to main vagal trunk
• Damage to hypoglossal nerve
• Hypocalcemia
15. A 02 year old boy was brought to OPD for a mid line cystic neck swelling lying just below
thyroid bone that moves with swallowing & tongue protrusion. What is likely diagnosis?
• Branchial cyst
• Cervical cyst
• Thyroglossal cyst
• Cervical cyst
• Dermoid cyst
16. Which of the following is commonest thyroid malignancy?
• Papillary thyroid carcinoma
• Medulllary thyroid carcinoma
• Anaplastic carcinoma
• Thyroid lymphoma
• Metastatic carcinoma
17. A 15 year old female presented in OPD having pain in Umbilicus that radiated to RIF. She is also
having nausea and anorexia. TLC was 11500. On examination when surgeon presses LIF patient
experiences pain in RIF and also tenderness and rebound tenderness in RIF. What is likely
diagnosis?
• Acute appendicitis
• Right ureteric colic
• Ruptured ectopic pregnancy
• Mittleschemerz
• Perforated peptic ulcer
18. Which of the following is type of adynamic intestinal obstruction?
• Adhesions
• Paralytic ileus
• Worms
• Stones
• Tumor
19. A 58 year old female was diagnosed as case of acute pancdreatitis. Which of the following
criteria is most effective in determining severity index?
• Glasgow
• Ranson
• APACHE
• POSSUM
• Parsonet
20. Most common type of gall stones is
• Pigment stones
• Cholesterol stones
• Mixed stones
• Brown
• Black
21. A 40 year old multiparous fatty female, presented in surgical OPD with history of pain in RHC
for few hours associated with few episodes of non bilious vomiting. Examination showed
tachycardia and tenderness in RHC. Which investigation will help in diagnosis?
• CXR
• X-Ray abdomen
• Ultra sound abdomen
• TLC
• Barium study
22. A 40 year old multiparous faty female, presented in surgical OPD with history of recurrent
attacks of pain in RHC especially after fatty meals and flatulent dyspepsia. What is likely
cause?
• Acute cholecystitis
• Acute pancreaitis
• Acute peritonitis
• Biliary colic
• Perforated duodenal ulcer
23. Charcot’s triad refers to
• Fever, Deranged LFTs, Anemia
• Fever, pain right hypochondrium jaundice
• Anemia, hepatomegaly, Jaundice
• Anemia, pain right hypochondrium, jaundice
• Anemia, fever, jaundice
24. A 35 year old female patient complains of dysphagia for last 6 months more for liquids than
solids. Which investigation will confirm diagnosis?
• Barium Sallow
• Barium meal
• CT Scan
• Chest X-Ray
• Ultra sound chest
25. A 50 year old lady presented to surgical OPD complaining of progressive dysphagia, weight
loss and anemia. On examination patient is pale and emaciated. What is likely diagnosis?
• GORD
• Achalsia
• Ca Esophagus
• Ca stomach
• PUD
26. A patient presented in surgical OPD having severe cough especially at bed time and
regurgitation of undigested food material. He is also embarrassed by bad breath. What is
likely diagnosis?
• Ca Esophagus
• Pharyngeal pouch
• Peptic stricture
• Barrett’s Esophagus
• GORD
27. Which of the following is considered to be pre-malignant lesion
• Esophageal candidiasis
• Barrett’s esophagus
• Schatzki rings
• Moniliasis
• Mallory Weiss Tear
28. A young boy sustained cricket ball injury to his Rt. Temporal region. After 4 hours his level of
consciousness deteriorated on examination Rt. Pupil was dilated. What is your clinical
diagnosis?
• Diffuse axonal injury
• Sub-dural haematoma
• Brain Laceration
• Brain contusion
• Extradural haematoma
29. CSF Rhinorechea after head injury is sign of . Mark the right optin?
• Fracture of Post cranial fossa
• Fracture of Middlecranial fossa
• Fracture of Frontal bone
• Fracture of Zygomaic Arch
• Fracture ant. Cranial fossa
30. An Afghani of 50 years presented with pain Rt. Hypochondria. Ultrasonography reveals cystic
swelling well encapsulated in right lobe of liver. The most likely diagnosis would be
• Hepatocellular Carcinoma
• Hydatied Cyst
• Liver Cirrhosis
• Liver Abcess
• Metastatic Liver Disease
31. Lower respiratory tract is protected by following mechnasims. Mark the worng
• statement?
• Closure of the laryngeal inlet
• Closure of the false cords
• Closure of the Glotis
• Cessation of respiration
• No cough reflex
32. After head injury to asses level of consciousness. What is he best clinical
• monitoring scale in the ward?
• Apachi scoring scale/system
• Glasgow comma scale
• Ranson’s criteria
• Alvarado score
• Kings college criteria
33. Cushing, Phenomenon is comprised of different features along with deterioration in level of
consciousness it’s comprised of. Select the right statement?
• Bradycardia and Hypertension
• Taechycardia and Hypertension
• Arrthymias and Hypotension
• Normal pulse rate and normaloension
• Bradycardia and Hypotension
34. A baby with unilateral Clift lip presented in OPD. The best time of surgical repair is?
• 5-6 months
• 4-5 months
• 6 months
• 2-3 months
• First month of life
35. After blunt injury to chest wall an elderly patients develop tension Pneumothorax. Select the
best management option?
• Thoracic chest tube insertion
• Tracheotomy
• Endotracheal intubation
• Urgent surgery
• O2 with mask only
36. Puncture wound over left Ant. Chest wall, resulted in cardiac tamponade in a young man after
a street fight. What is best management option?
• Pericardiaocenteris/Needles aspiration
• I/V fluids only
• Blood transfusions
• Thoracotomy
• Laparotomy
37. A non healing ulcer over nasolabial found in a farmer is most likely. Mark to the
• right option?
• Basal Cell Carcinoma
• Squamus Cell Carcinoma
• Malanoma
• Healing Ulcer
• TB Ulcer
38. Normally metabolism in the body results in production of nitric oxide and carbon dioxide. If a
patient remains in state of shock for delayed time period. What will be
• the outcome?
• Lactic acidosis and hyperkalemia
• Hyper carbia
• Hyponatrimia
• Hypocalcemia
• Hypo albumenemia
39. Most patients in hypovolaemic shock will have cool, pale peripheries, with prolong capillary
refill time in distributive shock as in septic shock this time is. Mark
• right option?
• Normal
• Delayed
• Decreased or brisk
• Increased
• Non appreciable
40. Physiological exhaustion is the end result of shock in any patient with on going traumatic
haemorrhage. What will be the best management plan?
• Avoid haemorrhage
• IV analgesics
• Cold IV infusates
• Inj. Vit-K
• Tranamenic acid intravenously
41. Platelets are very important haemostatic particles. What is its life span? Mark the right
option?
• 1-2 days
• 2-3 days
• 4-5 days
• 7-10 days
• 10-15 days
42. The most common fatal infectious complication of blood transfusion is
• Malaria
• Cytomagalovirus
• Aids virus
• Viral hepatitis
• Typhoid fever
43. 43 What percentage of total insensible loss of water from skin
• 25%
• 45%
• 60%
• 75%
• 80%
44. 44. A patient is on dialysis for last four months his renal function are impaired. Physician
noticed tall T waves on ECG paper. What is the reason for this abnormal T wave?
• Hyperkalemia
• Hypomagnicemia
• Hypercholoremia
• Hyponatriemia
• Hypokalemia
45. 45. A patient was managed by daily dressing and Debridement for one week, now wound
looks pink and granulation tissues are sprouting. You are planning to close it now to minimize
scar and contracture. This wound will heal by
• Secondary intension healing
• Tertiary intension healing
• Reactionary healing
• Delayed healing
• Accelerated healing
46. 46. A bed bound paraplegic patient was brought to hospital with bed sores over his scarum.
Below are different stages of decubitus ulcer. Mark the least true stage?
• Non blenhable skin erythema
• Partial thickness skin loss involving the epidermis
• Full thickness skin loss
• Full thickness tissue loss involving muscles, bones, tendons and joints
• Healthy healing wound edges
47. 47. Venous ulcers in a patient with long standing varicose veins are most commonly
• due to
• Skin infections
• Neuropathy
• Trauma
• Venous hypertension
• Dermatitis
48. 48. Synergistic spreading gangrene is caused by
• Staphylococcal infections
• Streptococcal infections
• Clostidial infections
• Poly microbial
• Aerobics only
49. 49. The best management of 5 cm liver abscess is
• Antibiotic alone
• Aspiration for culture and sensitivity
• Per-cutaneous drainage and antibiotic therapy
• Laparotomy and drainage of abscess
• IV normal saline
50. 50. Systemic inflammatory response syndrome and multiple organ dysfunction
• syndrome is as a result of
• Prostaglandins
• Leucocytes
• Red blood cells
• Cytokines
• Anti inflammatory mediators

Sir Aslam Shah


1. Water is lost from body through a number of routes. Which of the following is INCORRECT?
• Urine
• Faeces
• Skin
• Respiratory tract
• Saliva
2. Extracellular fluid losses may be extensive in following clinical settings EXCEPT:
• Acute intestinal obstruction
• Major burns
• Hepatic coma
• Acute peritonitis
• Acute pancreatitis
3. Which of the following conditions CANNOT lead to water depletion in a patient?
• Diabetes insipidis
• Profuse sweating
• Head injury
• Peripheral oedema*
• Reduced intake
4. The body responds to surgical trauma by alterations in water and electrolyte balance. Which
of the following is NOT part of this phenomenon?
• Retention of water
• Retention of sodium
• Sweating caused by fever*
• Release of steroid hormones
• Release of catecholamines
5. In a normal adult person of 70 kg:
• Insensible water loss is about 200 ml/day
• Sodium requirement is 70 mEq/day
• Daily urine output is 1000-2000 ml*
• Protein requirement is 17 gm/24 hours
• Tidal volume is 300 ml
6. What is normal serum sodium level in adults?
• 3.5-5.5 mEq/L
• 5.0-10.0 mEq/L
• 35-55 mEq/L
• 135-145 mEq/L*
• 150-160 mEq/L
7. Potassium deficiency should be suspected:
• In paralytic ileus*
• When patient's reflexes are exaggerated
• If there is 'tenting' of T waves in patient's ECG
• In renal failure
• In a case of hepatic encephalopathy
8. In a normal person of average built:
• Extracellular fluid volume is about 8 liters
• Plasma volume measures about 5 liters
• Total body water measures approximately 40 liters*
• Sodium requirement is 150 mEq
• Resting energy expenditure decreases after major surgery
9. The pH of extracellular fluid:
• Is maintained in health between 7.4 and 7.8
• Is increased in hypovolemic shock
• Increases abruptly after cardiac arrest
• Can be higher than normal in cases of acute peritonitis
• Should be monitored in a critically ill patient*
10. Metabolic acidosis can occur due to following reasons EXCEPT:
• Episode of diarrhoea
• Repeated vomiting*
• High output fistula
• Renal failure
• Shock
11. In a blood transfusion reaction, following statements are correct EXCEPT:
• It may be due to an incompatibility of the recipient serum and donor's cells
• Is manifested by thrombophlebitis of the infusion site*
• It is most commonly due to clerical mistakes and labeling errors
• It occurs within first 30 minutes of transfusion
• Must be suspected when the patient having blood transfusion complains of loin pain
12. Massive blood transfusion may be complicated by:
• Leukopenia (deficiency of WBCs)
• Coagulopathy*
• Hypokalemia
• Hypercalcemia
• Febrile reactions
13. The blood collection bags contain anticoagulant solution that contains following substances
EXCEPT:
• Phosphate
• Sodium
• Dextrose
• Calcium*
• Citrate
14. Which of the following complications is NOT associated with blood transfusion?
• Serum sickness
• Acidosis
• Hyperkalemia
• Hypocalcemia
• Incompatibility reactions
15. What is the most suitable temperature for storing blood in the blood bank?
• -20°C
• -4°C to 0°C
• C. -30°C
• +2°C to +6°C*
• +10°C to + 14°C
16. This is TRUE about haemorrhage:
• Bright red blood spurting with pulse is venous in origin
• b. Bleeding during surgery is primary haemorrhage
• c. Bleeding caused by blunt injury of abdomen is 'revealed' haemorrhage
• Secondary haemorrhage occurs few hours after surgery
• It can lead to haemorrhagic shock
17. Shock is common in clinical practice. In all forms of shock there is:
• Diminished circulatory volume
• A hypotensive state with peripheral vasoconstriction
• An impairment of cellular oxygenation and perfusion*
• Tissue alkalosis
• Inadequate cardiac output
18. Which of the following is NOT a feature of hypovolemic shock?
• Peripheral vasoconstriction
• Air hunger
• Increased blood viscosity
• Bradycardia*
• Cold sweating
19. In hypovolemic shock:
• The extremities are warm and sweating
• The central venous pressure (CVP) is low
• There is always a source of bleeding
• The neck veins may be engorged
• Urine output is not affected
20. Septic shock:
• Can only be caused by Gram negative bacteria
• Can only be caused by Gram positive bacteria
• Carries a favourable prognosis
• Is particularly associated with infective complications of gastrointestinal and genitourinary
systems
• Can be most effectively treated with antibiotics
21. What is INCORRECT about vasovagal shock?
• Blood pressure is low.
• Pulse is bounding in character
• Patient has tachycardia*
• Pallor is prominent
• It is due to expansion of vascular space
22. 5% dextrose saline solution is often used in surgical patients. Which of the following
statements is CORRECT about it?
• It is isotonic
• It is hypotonic
• It is hypertonic*
• It has good nutritive value
• It is the best solution for use in shock
23. A patient has developed shock after trauma due to an accident. Which of the following
solutions has NO ROLE in treating hypovolemic shock?
• Blood transfusion
• Ringer's lactate solution
• Fresh frozen plasma (FFP)*
• Increased amount of oral fluids
• Colloids solution
• Total parentral nutrition (TPN):
24. Should deliver about 2500 calories/day to an adult of average built
• Should deliver at least 30 gm nitrogen (180 gm protein)/day
• Is free of complications in today's settings
• Provides better nutritional support than the enteral route
• Can be effectively administered through peripheral lines
25. Which of the following statements is CORRECT about Enteral feeding?
• It results in constipation
• It should be considered as second best after intravenous nutrition
• It may result in diarrhea
• It is not an effective method for long term nutrition
• It often results in uremia and dehydration

Abdomen & GIT


1. Mark the INCORRECT statement about oesophagus:
• It is 25 cm long
• Its upper part has striated muscles
• It starts at about 15 cm from the incisors
• It extends from cricopharyngeal sphincter to cruri of diaphragm*
• It is lined by squamous epithelium
2. A middle aged patient complains of dyspepsia of recent origin that is not responding to
medical treatment. Which of the following investigations will be your FIRST CHOICE for further
investigation?
• Stool examination for occult blood
• Barium swallow
• Barium meal follow through examination
• Upper Gl endoscopy*
• CT scan abdomen
3. Following are some statements about carcinoma of oesophagus. Which one of these is
INCORRECT?
• Squamous cell carcinoma is the most frequent type
• Lower third of the oesophagus is most commonly involved*
• It usually presents with progressive dysphagia
• It can be effectively diagnosed on history and upper Gl endoscopy
• There is thickening, rigidity and narrowing of the lumen
4. The length of instrument used for upper Gl endoscopy is:
• 30 cm
• 40 cm
• 80 cm
• 100 cm
• 120 cm*
5. The 'Tea-pot' stomach deformity is caused by scarring as a result of long standing gastric ulcer
located at:
• Gastric fundus
• Gastro-oesophageal junction
• Lesser curvature*
• Greater curvature
• Pyeloric region
6. Gastrodoudenal ulcers developing as a result of acute physical stress are best termed as:
• Cushing's ulcer
• b.) Curling's ulcer*
• Kissing ulcer
• Rodent ulcer
• Marjolin's ulcer
7. The pain in gastro-oesophageal reflux disease (GERD) is very closely linked to:
• Food intake
• Change of posture (stooping forward) *
• Induction of vomiting
• Feeling hungry
• Emotional stress
8. The best investigation for bleeding peptic ulcer in ER management is:
• Blood complete picture
• (b.) Upper Gl endoscopy*
• Barium meal examination
• Serum electrolytes
• Selective angiography
9. A patient presenting to ER with severe abdominal pain and vomiting is labeled as an 'acute
abdomen'. Which of the following investigations is the FIRST to be advised?
• Blood complete picture
• Plain x-ray abdomen
• Barium meal
• MRI
• Upper Gl endoscopy*
10. The level of intestinal obstruction can be determined by following steps EXCEPT:
• Appropriate history of the events over last few days
• Thorough clinical examination
• Abdominal x-rays
• Repeated measurement of abdominal girth*
• Radiological examination by contrast studies
11. Which of the following types of gall stones are most common?
• Calcium oxalate stone
• Phosphate stones
• c.) Cholesterol stone*
• Pigment stones
• Mixed variety
12. Obstructive jaundice may be associated with following clinical findings EXCEPT:
• Generalized pruritis
• Enlarged liver
• Anemia*
• Clay coloured stools and dark coloured urine
• Deranged clotting profile
13. A 50 years old man developed obstructive jaundice that is progressive in nature and painless.
He has a smooth globular mass in RHC. Which of the following statements INCORRECT about
obstructive jaundice in this patient?
• It is due obstruction somewhere in the biliary tract.
• Liver enzymes (ALT, AST) are expected to be markedly raised. *
• It is associated with clay coloured stools.
• Treatment of choice is surgery to relieve obstruction.
• Most probable cause is carcinoma head of pancreas.
14. Laparoscopic cholecystectomy has become the gold standard procedures for symptomatic gall
stones. It is now preferred by majority of the patients due to following advantages, EXCEPT:
• It permits rapid recovery and early return to work
• (b.) The cost of the procedure is low*
• It is associated with minimal requirements of analgesics in postoperative period
• It has good cosmetic outcome
• It offers the chance of simultaneous inspection of abdominal cavity
15. Gall stones may be associated with following conditions EXCEPT:
• Cholangitis
• Diverticulitis*
• Intestinal obstruction
• Pancreatitis
• Cholecystitis
16. Paralytic ileus may develop from following factors EXCEPT:
• Surgery with intra-abdominal manipulation
• Spinal injury
• Hypertension*
• Peritoneal sepsis
• Shock
17. An aged patient has presented with an intra-abdominal mass does not move with respiration.
With which of the following structures it may be most likely associated?
• Liver
• Gall bladder
• Spleen
• Stomach
• Retroperitoneal tissues*
18. Which of the following is NOT a feature of acute appendicitis?
• Pain at the McBurney's point
• Positive psoas sign
• Oedema and ischemia of wall of the appendix
• Rovesing sign
• Positive Grey Tumer sign*
19. CORRECT about this condition Acute appendicitis
• Is most common during the 5th decade of life
• May cause pain on forced extension of hip joint*
• Typically presents with high temperature
• Can be excluded if the patient has diarrhea
• May lead to gross haematuria
20. Hypersplenism is not associated with
• Anemia
• Gastrointestinal bleeding and purpura
• Portal hypertension
• Enlarged palpable spleen
• Haematuria*
21. What is INCORRECT about haemorrhoids?
• They are caused by varicosity of anal veins*
• They are usually associated with pruritis
• They may cause severe pain
• They are prone to get infected
• They may resolve with conservative treatment
22. True for DIRECT inguinal hernia:
• It is frequent in females
• It is usually congenital
• Passes through posterior wall of inguinal canal medial to the deep inguinal ring*
• It is more common in children
• It often strangulates
23. Higher incidence of Incisional hernia in patients with following co-existing factors, EXCEPT:
• Postop wound infection
• Anaemia and malnutrition*
• The use of absorbable sutures for closing abdominal wounds
• Obesity
• Faulty surgical techniques
24. A chronic alcohol drinker presents with upper abdominal pain. Examination reveals epigastric
fullness; the area is tender. Which investigation will most likely help in diagnosis?
• Serum amylase / serum lipase level This is pancreatitis*
• Amylase-creatinine clearance ratio
• Upper Gl endoscopy
• Blood complete picture
• HIDA scan
25. The following organs are part of lymphatic system EXCEPT:
• Spleen
• Liver
• Thymus
• Tonsils
• Bone marrow
26. What percentage of lymph from breast is drained by the cervical lymph nodes
• Less than 40 percent
• 40-60 percent
• 60-80 percent
• 100 percent
27. These nodes do not directly receive lymph from breast*
• INCORRECT about FNAC of breast
• It is a safe investigation
• It is usually reliable
• It is the least invasive investigation for diagnosis of breast cancer
• It can differentiate invasive carcinoma from carcinoma in-situ*
• It is part of the 'Tripple assessment' for breast cancer
28. Carcinoma of the breast is the most common cancer in females all over the world. Which of
the following types of this neoplasm has the worst prognosis?
• Inflammatory carcinoma*
• Invasive ductal carcinoma
• Non-invasive ductal carcinoma
• Scirrhous carcinoma
• Lobular carcinoma
29. A young female patient presents with blood stained discharge from nipple of right breast.
Which of the following statements is INAPPROPRIATE in this regard?
• It may be associated with carcinoma of breast
• May be caused by duct papillopma
• Mastectomy is the treatment of choice*
• It is usually on one side
• It should always be investigated
30. The TNM classification system of breast cancer is based on:
• Clinical evaluation*
• Per-operative evaluation
• Liver and whole body scanning
• Histological findings
• Ultrasound examination of breast
31. The fibrocystic disease of the breast:
• It is a variant of normal cyclical changes that develop in breasts during each menstrual
cycle*
• It is always bilateral
• It usually persists in the postmenopausal years
• It may be precancerous
• It is usually associated with marked pain in breast

Trauma
1. What is the leading cause of trauma during first 30 years of life?
• Traffic accidents*
• Sports injuries
• Falls from height
• Acts of violence
• Blast injuries
2. A young man is brought to ER with a stab wound of chest at right mid-clavicular line. He has
dyspnoea. On examination, there is subcutaneous emphysema of right chest wall, absent
breath sounds, and trachea shifted to left side. What is the most probable diagnosis?
• Lung contusion
• Massive haemthorax
• Haemo-pneumothorax
• Tension pneumothorax*
• Chest wall laceration
3. Which of the following abdominal organs is least frequently injured during blunt abdominal
trauma?
• Liver
• Spleen
• Abdominal aorta*
• Kidneys
• Pancreas
4. Unilateral dilated pupil is an important sign of head injury. What is the underlying cause?
• Damage to the optic nerve
• Compression of 3rd cranial nerve*
• Cerebral Anoxia
• Increased intracranial pressure
• Compression of 6th cranial nerve
5. In a case of head injury, the 'Contre coup' injury is result of:
• Brain oedema
• Acceleration injury
• Deceleration injury*
• Mannitol infusion
• Colloid infusion
6. A man presents to emergency with blunt injury of abdomen after RTA. He has got tender
tense abdomen. Which the following investigations do you need to exclude abdominal organ
injury?
• Upper Gl endoscopy
• Gastrographin study
• Serum amylase
• IVU
• Ultrasound abdomen (FAST scan)*
7. A 'transverse' fracture of shaft of tibia is usually caused by a force that causes:
• Direct impact at the site of fracture*
• Twisting of bone
• Axial compression
• Crush injury
• Blunt injury
8. Fracture of a long bone is NOT associated with one of the following complications:
• Osteomyelitis
• Hypocalcemia*
• Fat embolism
• Nonunion
• Malunion

SKIN
1. A full thickness burn wound:
• Looks reddish in colour
• Skin grafting is likely to be successful if performed within the first week after burn injury
• Skin grafting should be delayed until scar tissue is developed
• Skin grafts will minimize scarring*
• Skin grafting in itself, is rarely necessary
2. Mark the INCORRECT statement about patients with major burns?
• They have normal caloric requirements*
• They are in a negative nitrogen balance
• They do not generally become anaemic
• They usually do not develop gastrointestinal complications
• They are less prone to develop septicemia

Wounds
1. Which of the following statements is best description of wound debridement?
• Excising about one mm of skin from edge of wound
• Not excising skin but excising all damaged muscles
• Using vacuum assisted dressing techniques
• Some sort of amputation
• Laying open all layers of a wound and excision of devitalized tissue*
2. A clean incised skin wound:
• Commences epithelialization after 8-10 days
• Regains the full strength of a normal skin in about 10 weeks
• Regains its strength as a result of fibroblast activity*
• Cannot undergo contraction
• Does not lead to keloid formation
3. Following is true about keloid EXCEPT:
• It is an extensive overgrowth of scar tissue
• It is common in Africans
• It is common in certain area of the body
• It is a benign condition
• It is also called hypertrophic scar*
4. What is usually described as the best dressing for a wound?
• Cotton gauze
• Skin*
• Silver foil
• Aerosol spray
• Amnion
5. Severity of tissue damage in wounds caused by bullets principally depends on:
• Size of the bullet
• Shape of the bullet
• Size of the weapon used
• Velocity of the bullet*
• Weight of the bullet

Thyroid Head & Neck


1. What is CORRECT about toxic diffuse goiter in a young patient?
• It may be an early stage of endemic goiter
• TSH is usually raised in these cases
• Thyroid scan is strongly indicated
• Thyroid disease is equally common in both genders
• It is more common in females*
2. INCORRECT about FNAC of thyroid
• It is safe and reliable
• It is the least invasive investigation for diagnosis of breast cancer
• It can differentiate between follicular carcinoma and follicular adenoma*
• It is highly accurate
• Trucut biopsy is a better option
3. Following statements are true about a follicular carcinoma of thyroid, EXCEPT:
• It is more common in females
• It is more common over the age of 30 years
• It is associated with relatively good prognosis
• It is best treated by total thyroidectomy
• It usually metastasizes to the regional lymph nodes
4. 'Simple puberty goiter' is treated by
• Carbimazole
• Radio iodine
• Thyroxin replacement*
• Lugol's iodine
• Surgical excision
5. NOT TRUE about a follicular carcinoma of thyroid
• It is more common in females
• It is more common over the age of 30 years
• It is associated with relatively good prognosis
• It is best treated by total thyroidectomy
• It usually melaslasizes to the regional lymph notes*
6. NOT indicated in the evaluation of solitary thyroid nodule in a clinically Euthyroid patient:
• Thyroid hormone level
• Thyroid scan*
• FNAC
• Frozen section
• Thyroid ultrasound
• TRUE about latent tetany
7. Carpopedal spasm is diagnostic
• Parathyroid hormone level is increased
• It may be part of MEN syndrome
• Serum calcium is increased
• It is picked by Chevostic's sign and Trousseau's sign. *
8. A young female patient presents with a discharging ulcer at middle of the neck on left side.
The edges of the ulcer are undermined. What is the most probable diagnosis?
• Rodent ulcer
• Nonspecific ulcer
• Venous ulcer
• Tuberculosis
• Epithelioma
9. Ulcer with undermined edges
• Rodent ulcer
• Nonspecific ulcer
• Venous ulcer
• Tuberculosis*
• Epithelioma
10. Swelling front of neck that moves with protrusion of the tongue
• Solitary thyroid nodule
• Submandibulr gland
• Branchial cyst
• Cystic hygroma
• Thyroglossal cyst*
11. Cancer of the tongue is:
• Most commonly presents with bleeding from mouth
• Metastasizes through blood stream, and not through lymphatics
• Is strongly associated with the use of tobacco in any form*
• Is usually an adenocarcinoma
• Is more common in females
12. Cleft palate may be associated with following abnormalities:
• Chronic Otitis media
• Conductive deafness
• Dental anomalies
• Speech impairment
• All of these*
13. Cystic hygroma may be found at all of the following sites EXCEPT:
• Neck
• Mediastinum
• Head region
• Lower limbs
• Inguinal region
14. The parotid gland is a major salivary gland in the body. How much of the parotid tumours are
likely to be malignant?
• About 20%
• About 40%
• Upto 60%
• Upto 80%
• All parotid tumours are malignant in nature
15. Chances of malignancy in parotid gland tumours
• About 20%*
• About 40%
• Upto 60%
• Upto 80%
• All parotid tumours are malignant in nature
16. In Tuberculous cervical lymphadenitis lymph nodes are usually
• Single
• Matted*
• Rubbery
• Hard
• Soft
17. Chances of bilateral Warthin's tumour of parotid gland
• About 10%*
• About 20%
• Upto 30%
• Upto 40%
• Approximately 50%
18. What is the most common malignant tumour of Parotid gland:
• Mucoepidermoid Carcinoma
• Adenoid Cystic Carcinoma
• Squamous cell Carcinoma
• Mixed parotid tumour
• Adenocarcinoma
19. The parotid gland is a major salivary gland in the body. How much of the parotid tumours are
likely to be in deep lobe?
• About 1-2%
• About 10%
• Upto 20%
• Upto 40%
• Approximately 60%

Urinary tract
1. Following is true about cancer of prostate EXCEPT:
• It mostly affects men over 60 years of age
• Most common pathological type is adenocarcinoma
• The second most common tumour of prostate is sarcoma*
• It may give rise to osteoblastic metastases
• It is less responsive to usual therapeutic measures
2. The most common type of urinary stones?
• Phosphate stones
• Uric acid stones
• Oxalate stones*
• Cysteine stones
• Xanthine stones
3. A 40 years old man presents with gross haematuria that is painless. What is the most probable
cause?
• Hydronephrosis
• Vesical calculus
• A tumour in urinary bladder*
• Perinephric abscess
• Renal tuberculosis
4. Following are causes of a uretheral stricture EXCEPT:
• Perineal trauma
• Uretheral infections
• Retroperitoneal fibrosis*
• Congenital anomalies
• Long standing uretheral catheterization
5. The commonest type of malignancy in urinary bladder is:
• Transitional cell carcinoma*
• Adenocarcinoma
• Squamous cell carcinoma
• Lymphosarcoma
• Sarcoma
6. What is INCORRECT about a simple sebaceous cyst?
• It is attached to skin
• It can be moulded
• It has a punctum
• It can get infected
• Transillumination test is positive*
7. Management of Tetanus includes following measures, EXCEPT:
• Muscle relaxant drugs
• Tracheostomy in selected cases
• Drugs to stimulate heart*
• Isolation of the patients
• Anti-tetanus immunoglobulins
8. The wall of a true CYST is lined by:
• Endothelium
• Granulation tissue
• Epithelium*
• Debris
• Mesenchymal tissue
9. Which of the following tumours has a strong association with chronic smoking?
• Carcinoma oesophagus
• Carcinoma of breast
• Carcinoma of colon
• Bronchogenic carcinoma*
• Carcinoma stomach
10. Staphylococcal infections:
• Do not cause cellulitis
• Do not produce septicemia
• Do not give rise to fever
• Can be effectively treated using Aminoglycoside group of antibiotics
• May be air borne infections*
11. Which is the best term to describe an abnormal tract that connects two different epithelial
surfaces?
• Fistula*
• Sinus
• Tunnel
• Excretory duct
• Inter-calated duct
12. Risk of postoperative wound infections can be minimized by following measures EXCEPT:
• Good patient preparation
• Good surgical technique
• Medicated surgical gloves
• Antibiotic prophylaxis
• Sterilization of surgeon's hands*
13. Transfer of tissues between two genetically identical individuals is called:
• Allograft
• Autograft
• Isograft*
• Xenograft
• Full thickness graft
14. The bone infarct in acute osteomyelitis is called:
• Involucrum*
• Sequestrum
• Cloaca
• Foreign body
• Sinus
15. What is WRONG about hepatitis C?
• About one percent blood donors are HCV positive
• Acute hepatitis C proceeds to cirrhosis in 20% cases
• It can spread through 'fecal-oral' route
• Chances of developing hepatic malignancy are negligible*
• Deteriorating HCV encephalopathy is an indication for liver transplantation
16. Diabetic patients are high risk for surgical procedures because of:
• Poor wound healing
• Increased chances of wound infection
• Increased risk of cardiovascular complications
• Increased risk of CVA
• All of the above*

Surgery 2
1. Which of the following investigations is NOT required for anaesthesia fitness before elective
surgery in a middle aged patient?
• Blood complete picture
• Ultrasound abdomen*
• Blood sugar
• Chest x-ray
• ECG
2. Antibiotic prophylaxis before elective surgery is provided with:
• Single low dose of broad spectrum antibiotic
• Multiple low doses of broad spectrum antibiotic
• Single high dose of broad spectrum antibiotic*
• Multiple high doses of broad spectrum antibiotic
• Oral route
3. A young male presents with symptomatic varicose veins involving great saphenous vein of
right lower limb. What is the best option treatment?
• Elastic stocking alone
• Sclerosant injection throughout the length of vein
• Flush ligation and ligation of perforators*
• Endoscopic valve repair
• Sub-facial ligation
4. The bone densitometry of spine and hips is best done by following investigation:
• CT Scan
• Ultrasound
• C. MRI
• Dual-energy x-ray absorption (DEXA) scan*
• Laser
5. Metronidazole is active against:
• Anaerobes*
• All Gram positive bacteria
• All Gram negative bacteria
• Fungal infections
• Some viral infections
6. Following are reservoir of HIV infection EXCEPT:
• Lymphocytes
• Monocytes
• Macrophages
• RBC's*
• Neural cells
7. Which nerve is involved in Carpal tunnel syndrome?
• Ulnar
• Radial
• Median*
• Axillary nerve
• lateral cutaneous nerve
8. Which one of the following diseases is a chronic SPECIFIC infection?
• Carbuncle
• Tuberculosis*
• Gas gangrene
• Tetanus
• Cellulitis
9. A patient has undergone gency laparotomy for ileal perforation. On 5th post op day the
dressing on abdominal wound was soacked with pus. The bluish green discharge from the
wound indicates:
• Development of gas gangrene in wound edges
• Abdominal compartment syndrome
• Risk of developing Tetanus
• Pseudomonas infection*
• Candida albicans infection
10. Which of the following organs is a primary site for tuberculosis?
• Lungs*
• Bones
• Kidneys
• Liver
• Brain
11. In a subphrenic abscess:
• There is marked tenderness over the ribs in that region
• It is usually accompanied by considerable systemic effects
• It rarely produces abnormal signs in chest
• It is usually diagnosed by ultrasound examination*
• It is always diagnosed by barium meal study
12. Bed sores are one of the preventable complications in patients who remain bed ridden for
long time. What is CORRECT about bed sores?
• They can be prevented by sheepskin blankets
• They can often be avoided by changing patients position 4-5 times a day
• They are caused by local infection
• They occur only over the sacrum
• They may undergo spontaneous healing*
• Basal cell carcinoma:
13. Usually metastasize to regional lymph nodes
• Commoner than squamous cell carcinoma
• Commonest site is face and neck*
• Is histologically characterized by presence of epithelial pearls
• Is more common in middle east and far eastern regions
14. Delayed primary sutures are applied:
• In all post-surgical wounds.
• In case of simple lacerated wounds.
• In wound after midline laparotomy.
• In some infected wounds after wound infection has been controlled*
• In repair of recurrent hernia.
15. What is INCORRECT about Tetanus?
• It can develop within few days after injury.
• It can be prevented by active or passive immunization.
• Effective and intensive treatment can save the patient.
• Penicillin is the drug of choice.
16. Patient should be kept in isolation because it can spread from one person to other* Hepatitis
B can be transmitted through transfusion of contaminated blood. How much time is required
to develop infection?
• One week
• Six weeks*
• Three months
• Six months
• Nine months
17. Anaerobic infections of wounds are liable to occur in following settings EXCEPT:
• Wide spread tissue damage
• Immuno-compromised status
• Leaving the wounds open after exploration*
• Diabetic foot
• None of these
18. Which of the following is not a PHYSICAL means of sterilization?
• Steam
• Ethylene oxide gas*
• Autoclave
• Radiation
• Boiling
19. INCORRECT about Vicryl suture
• It is absorbable suture.
• It gives rise to formation of stitch granuloma. *
• It is used in intestinal anastomosis.
• It is synthetic in nature
20. BEST treatment for an injection abscess
• Drain the abscess, start appropriate antibiotics, and close the wound.
• Drain the abscess, start appropriate antibiotics, and pack the wound daily*
• Keep him on broad-spectrum antibiotics only
• Massage the area with antiseptic creams
21. Passive immunity is available for which of the following diseases?
• Polio
• Tetanus*
• Mumps
• Appendicitis
22. Instrument used for putting ETT
• Oesophagoscope
• Laryngoscope*
• Gastroscope
• It needs no instrument
• What you mean by SECONDARY sutures?
• Skin closure of a wound 6 hours after surgery
• Skin closure of a wound one day after surgery
• Suturing with double-knot technique
• Suturing a wound again after control of wound infection*
23. Regarding tension pneumothorax, which one of the following statements is true?
• Needs urgent decompression by wide bore cannula*
• Requires C.T scan for diagnosis
• Requires urgent X-ray chest for diagnosis
• Urgent thoracotomy should be done
• Videothoracoscopy is the treatment of choice
24. The ideal first test to detect free air under diaphragm is
• Erect chest x-ray*
• Erect Abdominal x-ray
• CT scan Abdomen
• Supine Abdominal x-ray
25. Optimal modality for diagnosing Acute Pancreatitis is
• Erect Abdominal Xray
• ERCP
• CT scan*
• Ultrasound
• Supine Abdominal X ray
26. On x-ray chest tension pneumothorax shows
• Mediastinum is displaced to same side
• Mediastinum is displaced to opposite side*
• There is haemopneumothorax
• Mediastinum is undisplaced
• There is free air under the diaphragm
27. years old man, after being hit by a car, presented to ER with hypotension. On examination,
there is tenderness and bruising over his left lateral chest. An ultrasound examination reveals
free fluid in the abdomen. What is the most likely organ injured?
• Liver
• Kidney
• Spleen*
• Pancreas
• Intestine
28. Flail chest is defined as:
• Multiple rib fractures with subsequent subcutaneous emphysema
• Excess fluid in pericardium
• Two or more ribs fractured at two points*
• Blood in the pleural space
• Haemopneumothorax
29. A 45 old male patient has suffered from myocardial infarction one month back and wants
surgery for inguinal hernia. Which one of the following is true?
• Surgery should never be done on this patient
• Surgery should be performed after 6 months*
• Surgery can be done on the next operation list
• Surgery should be performed after eighteen months
• Surgery should be performed after one month
30. A patient has developed fever on 4th post laparotomy day. What is NOT a possible cause at
this stage?
• Urinary tract infection
• Wound infection
• Subphrenic abscess*
• IV site infection
• Chest infection
31. The antibiotic of choice for clean wounds is
• aminoglycoside
• first generation cephalosporins*
• penicillin group
• sulpha group
• imipenim group
32. A patient has arrive in the emergency with road traffic accident and is in shock which fluid you
will prefer to resuscitate him
• aminovel
• rose water
• normal saline*
• ringer lactate
• plasma
33. Meckel's diverticulitis most often occurs in the:
• Proximal Jejunum
• Distal Jejunum
• Proximal ileum*
• Distal ileum
• Duodenum
34. A middle aged female presents to surgical emergency with severe right upper abdominal pain
for one hour which is radiating to right shoulder. On examination there is mild tenderness in
right hypochondrium. WBC count is normal. Most likely diagnosis is:
• Acute appendicitis
• Acute pancreatitis
• Biliary colic*
• Acute cholecystitis
• Perforated peptic ulcer
35. Commonest cause of pancreatitis is:
• Viral Infection.
• ERCP
• Gallstones*
• Drugs.
• Abdominal Trauma
36. Most important prognostic factor in
• breast cancer
• Size of tumour
• ER PR receptors
• Axillary nodes involvement*
• Family history
37. A 6 years old child develops sudden pain in the right testicle which is exacerbated by elevation
of scrotum. What is the most probable diagnosis?
• Viral orchitis
• Testicular torsion*
• Obstructed Hernia
• Varicocele
• Epididymo OrchitisText
38. A Post-operative patient develops DVT. What is the best treatment?
• Embolectomy
• IV Heparin*
• IVC filter.
• Exploration.
• Aspirin
39. Ideal tumor marker for prostate carcinoma is?
• Acid phosphatase
• Alkaline phosphatase
• Prostate specific antigen*
• LDH
• C.E.A

You might also like